A Corner of Mathematical Olympiad and Competition Book II: Phnom Penh, October 24, 2014 Prepare By: Keo Sodara
A Corner of Mathematical Olympiad and Competition Book II: Phnom Penh, October 24, 2014 Prepare By: Keo Sodara
A Corner of Mathematical Olympiad and Competition Book II: Phnom Penh, October 24, 2014 Prepare By: Keo Sodara
1993/1994-1995/1996, 1998/1999
And
2000/2001-2012/2013
Estonian Mathematical Olympiad ’93
(Selected Problems)
Problems
1. (9, II) Find all triples of pairwise different real numbers (a, b, c) ,
such that the system of equations
ax = b
bx = c
cx = a
has a solution.
6. (11, II) In a right triangle ABC , the medians drawn to its side AC
and to the hypotenuse are perpendicular to each other. Find the
area of the triangle if the side AB has length 1 .
7. (11, III) Find all functions f (n) having the following three prop-
erties:
a) for any natural number n , f (n) is also a natural number;
b) f (n + m) = f (n)f (m) holds for any natural numbers n and m ;
c) the equation f (f (n)) = (f (n))2 has some natural number n0 as
its solution.
8. (12, II) Prove that, for any natural number k > 2 , there exist k
pairwise different natural numbers n1 , n2 , . . . , nk such that
1 1 1 3
+ + ··· + = .
n1 n2 nk 17
9. (12, III) A rectangle is cut into five rectangular pieces having equal
areas. Prove that at least two of the pieces are identical.
A 1 B
2. (II, 10) Find the largest natural number n such that 999
| {z. . . 99} is
999 digits
divisible by 9n .
3. (II, 11) Prove that for any real numbers x and y , we have
| sin x| > | cos y| if and only if | sin y| > | cos x| .
7. (III, 10) Find the least natural number having exactly 100 different
natural divisors (including 1 and the number itself).
sin β
8. (III, 10) Let α, β, γ, δ be such that 0◦ < α, β, γ, δ < 90◦ , sin α =
sin γ
tan β tan δ
and sin δ = . Prove that tan α = .
tan γ cos γ
9. (III, 10) Three men decided to have a picnic together with their
wives. They arrived at the railway-station one by one, each of them
greeting by handshake all those already present, except his/her wife
or husband. Later one of the men asked all the others how many
companions each had shaken hands with upon arrival, and he got
five different answers. What would have been his own answer?
10. (III, 11) Compute the sum of all the five-digit natural numbers
that do not change when the order of their digits is reversed (we do
not accept 0 as the first digit of a natural number).
11. (III, 11) Prove that for any triangle the equality
cot α β
R 2 + cot 2
=
r 2 sin γ
holds, where α, β, γ are the angles of the triangle and R , r are the
radii of its circumcircle and inscribed circle, respectively.
AO BO CO
+ + = 1994 .
OA1 OB1 OC1
AO BO CO
Find the value of · · .
OA1 OB1 OC1
14. (III, 12) Prove that, for any natural number n , the number
n
n! = 1 · 2 · . . . · n has less than trailing zeros.
4
15. (III, 12) All three vertices of a regular triangle lie on the sides of
a unit square (some of them may coincide with the vertices of the
square). Determine the minimum and maximum values for the area
of such a triangle.
2nd round
3rd round
1. Does there exist a positive integer such that its last digit is non-
zero and that it becomes exactly two times bigger when the order
of its digits is reversed?
2. Three children wanted to make a table-game. For that purpose
they wished to enumerate the mn squares of an m×n game-board
by the numbers 1, . . . , mn in such way that the numbers 1 and mn
lie in the corners of the board and the squares with successive
numbers have a common edge. The children agreed to place the
initial square (with number 1) in one of the corners but each child
wanted to have the final square (with number mn ) in different
corner. For which numbers m and n is it possible to satisfy the
wish of any of the children?
3. John and Mary play the following game. First they choose integers
n > m > 0 and put n sweets on an empty table. Then they start
to make moves alternately. A move consists of choosing a non-
negative integer k 6 m and taking k sweets away from the table
(if k = 0 , nothing happens in fact). In doing so no value for k
can be chosen more than once (by none of the players) or can be
greater than the number of sweets at the table at the moment of
choice. The game is over when one of the players can make no
more moves.
John and Mary decided that at the beginning Mary chooses the
numbers m and n and then John determines whether the per-
former of the last move wins or looses. Can Mary choose m and
n in such way that independently of John’s decision she will be
able to win?
4. Prove that 1n + 2n + . . . + 15n is divisible by 480 for any odd n > 5 .
5. Let p be a fixed prime number. Find all pairs (x, y) of positive
integers satisfying the equation p(x − y) = xy .
6. In a plane there are n triangles such that any three of them have
a common vertex and four of them have vertex in common. Find
the greatest possible value for n .
7. In the spase there are n tetrahedra such that any two of them have
two common vertices and no three of them have three vertices in
common. Find the greatest possible value for n .
x2 + y 2 + 6
1. The numbers x , y and are positive integers. Prove
xy
x2 + y 2 + 6
that is a perfect cube.
xy
2. Let a , b , c be the sides of a triangle and α , β , γ the opposite
angles of the sides respectively. Prove that if the inradius of the
triangle is r then a sin α + b sin β + c sin γ > 9r .
3. Find all functions f : R → R satisfying the following conditions for
all x ∈ R :
Open contests
10th grade
Solution. The given condition is equivalent to a2 = b(b1998 −1). If b > 2 then b and b1998 −1
are both positive and relatively prime, hence both perfect squares. But since b1998 is also
a perfect square, we get a contradiction. So b 6 1. The cases b = 1, b = 0 and b = −1
all give a = 0. At last note that b 6 −2 is impossible because it implies a2 < 0.
2. Find all values of a such that absolute value of one of the roots of the equation
x2 + (a − 2)x − 2a2 + 5a − 3 = 0
(a − 2)2 − 4 · (−2a2 + 5a − 3)
p
−(a − 2) ±
x= =
2
√
2−a± a2 − 4a + 4 + 8a2 − 20a + 12
= =
2
√
9a2 − 24a + 16 2 − a ± (3a − 4)2
p
2−a±
= = ,
2 2
so
2 − a − (3a − 4) 2 − a + (3a − 4)
x1 = = −2a + 3 , x2 = =a−1.
2 2
Assume |x1 | = 2|x2 |, i.e. | − 2a + 3| = 2 · |a − 1|. The numbers a − 1 and −2a + 3 are
3
negative iff a < 1 and a > respectively. Thus in the case a < 1 the condition reduces
2
3
to −2a + 3 = 2(1 − a), and in the case a > the condition reduces to 2a − 3 = 2(a − 1).
2
3
Hence both cases are impossible. So 1 6 a 6 , which gives −2a + 3 = 2(a − 1) and
2
5
a= .
4
Now assume |x2 | = 2|x1 |, i.e. |a − 1| = 2 · | − 2a + 3|. In the case a < 1 this condition
5
reduces to 1 − a = 2(−2a + 3), which gives a = , but this is not sound with the case
3
3
assumption a < 1. In the case 1 6 a 6 the condition reduces to a − 1 = 2(−2a + 3)
2
7 3
which gives a = , and in the case a > the condition reduces to a − 1 = 2(2a − 3)
5 2
5
which gives a = .
3
1
3. The incircle of the triangle ABC , with the center I , touches the sides AB , AC and BC
in the points K , L and M respectively. Points P and Q are taken on the sides AC and
BC respectively, such that |AP | = |CL| and |BQ| = |CM |. Prove that the difference of
areas of the figures AP IQB and CP IQ is equal to the area of the quadrangle CLIM .
Solution. We have SAP I = SCLI since |AP | = |CL| and both triangles have altitude LI .
Analogously we have SALI = SCP I since |AL| = |CP |. But △ALI ∼ = △AKI because
the sides are respectively equal. Hence
Thus we have
SAP IQB − SCP IQ = SAP IK + SBM IK − SCP I − SCQI = SCLI + SCM I = SCLIM .
4. 32 stones, with pairwise different weights, and lever scales without weights are given. How
to determine by 35 scaling, which stone is the heaviest and which is the second by weight?
Solution. At first we determine the heaviest stone by 31 scalings and find at the same
time 5 stones, one of which must be the second by weight. This can be achieved by pairing
all the 32 stones and comparing the stones in each pair; after that pairing the heavier
stones and comparing again the stones in each pair, etc. This takes 16 + 8 + 4 + 2 + 1 = 31
scalings. By the last scaling we find the heaviest stone and the second by weight may be
only one of these five which have been in pair with the heaviest.
To complete the solution we must determine by 4 scaling, which of these 5 stones is the
heaviest. It can be done linearly.
5. Let C be an interior point of line segment AB . Equilateral triangles ADC and CEB
are constructed to the same side from AB . Find all points which can be the midpoint of
the segment DE .
Answer: the midline of the equilateral triangle constructed on the segment AB which is
parallel to AB (endpoints excluded).
E
Fq
D
A C B
Figure 1
Solution. Let F be the midpoint of the segment DE . Lengthen the segments AD and
BE till intersecting in point G (see figure 1). The location of the point G does not depend
on the point P and the quadrangle ECDG is a parallelogram with the point F as the
midpoint of its diagonals. While the point C moves along AB from A to B , point F
moves from the midpoint of AG to the midpoint of BG along the segment connecting
these points. According to the condition of the problem, the endpoints must be excluded.
2
11th grade
4mn
(m − n)2 = .
m+n−1
k(k + 1) k(k − 1)
Answer: the pairs (k, −k) and , , (where k is an arbitrary integer),
2 2
(1, 0) and (0, 1) excluded.
Solution. By multiplying both sides of the equation with the denominator of the right
side, we get (m − n)2 (m + n − 1) = 4mn, which gives (m + n)2 = (m − n)2 (m + n). Hence
m + n = 0 or m + n = (m − n)2 . The first case gives n = −m, i.e. all the pairs (k, −k),
where k is integer, are suitable. In the second case take m − n = k , then m + n = k2
k2 + k k2 − k
and m = , n= . To ensure the denominator of the fraction in the problem
2 2
is not zero the condition m + n 6= 1 must be added, so k 6= 1 and k 6= −1.
1
Answer: 1999 .
2
Solution. One gets the answer directly using the fact that for any non-zero real number
x,
2
1
1 x2 x x2 1 1
f (x) + f = + 2 = + 2· =
x 1 + x2
1+ 1 1+x 2 x 1+ 1
2
x x
x 2 1
= + =1.
1 + x2 1 + x2
3. For the given triangle ABC , prove that a point X on the side AB satisfies the condition
−−→ −−→ −−→ −−→ −→ −−→
XA · XB + XC · XC = CA · CB iff X is the basepoint of the altitude or median of the
triangle ABC (~v · ~u denotes the scalar product of vectors ~v and ~u ).
C
O
-
~
A X B
Figure 2
3
−→ −−→ −−→ −−→ −−→ −−→
Solution. Since CA = XA − XC and CB = XB − XC (see figure 2), the condition in the
−−→ −−→ −−→ −−→ −−→ −−→ −−→ −−→
problem is equivalent to the condition XA· XB + XC · XC = = (XA− XC)·(XB − XC).
−−→ −−→ −−→
Transforming this, we get the equation XC · (XA + XB) = 0.
−−→ −−→ −−→ −−→ −−→
This condition holds iff XA + XB = 0 or XC ⊥ XA + XB . The first case holds iff X is
the midpoint of the side AB , the second case holds iff X is on the altitude.
n(n + 1)
Solution. The area of the side wall of the staircase is 1 + 2 + . . . + n = dm2 , the
2
area of each plate is 3 dm2 . Thus one of the numbers n and n + 1 must be divisible by
3, i.e. it isn’t possible to plate the side wall of the staircase if n ≡ 1 (mod 3).
Figure 3
n=6 n=9
Figure 4
Now we give a method how to construct plating in case n = 3k + 6 from plating in case
n = 3k . Thus the wall can be plated for any n = 3k , k > 1. For this leave unplated the
lower ribbon of the wall with height 6 dm and plate the upper part as in case n = 3k .
The ribbon can be plated as shown in figure 5.
4
...
| {z }
k rectangles
Figure 5
At last we show how to get the plating in the case n = 3k + 2 from the plating in the
case n = 3k . This implies the wall can be plated for any n = 3k + 2, k > 1. For this leave
unplated the lower ribbon with height 2 dm and plate the upper part as in case n = 3k .
The ribbon can be covered as shown in figure 6.
...
| {z }
k rectangles
Figure 6
Solution: The arrangement of grains of oat described in the problem implies that on each
white square of the chessboard there are 22n grains, while on each black square there
are 22n+1 grains, where n = 0, 1, 2, . . . , 31. Therefore the number of grains on any black
square is congruent to 1 modulo 3 and the number of grains on any white square is
congruent to 2 modulo 3. Since the knight moves always from a square of one colour to a
square of the other colour the number of grains it eats with each two consecutive move is
divisible by 3. But knight makes an even number of moves, because the initial and final
square are of the same colour. Hence the assertion of the problem holds.
12th grade
1. Let a, b, c and d be non-negative integers. Prove that the numbers 2a 7b and 2c 7d give
the same remainder when divided by 15 iff the numbers 3a 5b and 3c 5d give the same
remainder when divided by 16.
′ ′
Solution. First we show that if |a′ − a| = |b′ − b| = 2 then 2a 7b ≡ 2a 7b (mod 15) and
′ ′
3a 5b ≡ 3a 5b (mod 16). Indeed we can assume that a′ = a + 2. If b′ = b + 2, we obtain
′ ′
2a 7b = 2a 7b · 22 72 = 2a 7b · (2 · 7)2 ≡ 2a 7b · (−1)2 = 2a 7b (mod 15)
5
and
′ ′
3a 5b = 3a 5b · 32 52 = 3a 5b · (3 · 5)2 ≡ 3a 5b · (−1)2 = 3a 5b (mod 16) .
If b′ = b − 2, we can use the same relations noting that 74 ≡ 1 (mod 15) and
54 ≡ 1 (mod 16).
Now we prove that for every pair of non-negative integers (a, b) there exists a pair (a′ , b′ )
′ ′ ′ ′
such that 2a 7b ≡ 2a 7b (mod 15), 3a 5b ≡ 3a 5b (mod 16), a′ ∈ {0, 1, 2, 3} and b′ ∈ {0, 1}.
We conclude that both of the exponents can be changed by a number divisible by 4 without
changing the remainder of dividing by the required number. Thus we can consider only
the case where a, b ∈ {0, 1, 2, 3}. If b 6 1, take a′ = a and b′ = b; if b > 1 then b′ = b − 2
and a′ can be chosen from the set {0, 1, 2, 3} so that it differs from the number a exactly
by 2.
′ ′
It remains to prove that the remainders of the numbers 2a 7b when divided by 15 and the
′ ′
remainders of the numbers 3a 5b when divided by 16 are pairwise different if the numbers
a′ and b′ come from the abovementioned sets. This can be seen from the following tables.
Z1 p
ln x + 1 + x2 dx.
−1
Answer: 0.
thus f (x) is defined for every x. We will show that the function f (x) is odd, i.e.
f (−x) = −f (x). Indeed,
√ √ !
q ( 1+x 2 − x)( 1+x2 + x)
f (−x) = ln(−x + 1+(−x)2 ) = ln √ =
1+x2 + x
! !
1 + x2 − x2 1
= ln √ = ln √ =
1 + x2 + x x + 1 + x2
p
= − ln(x + 1 + x2 ) = −f (x) .
6
Now let
Z1 Z0 Z1
I= f (x) dx, I1 = f (x) dx, I2 = f (x) dx .
−1 −1 0
Z0 Z1 Z1
I1 = − f (−t) dt = f (−t) dt = − f (t) dt = −I2 .
1 0 0
Consequently
I = I1 + I2 = −I2 + I2 = 0 .
3. Prove that the line segment, joining the orthocenter and the intersection point of the me-
dians of the acute-angled triangle ABC is parallel to the side AB iff tan 6 A · tan 6 B = 3.
Solution: Let the basepoints of the altitudes drawn from the vertices A and C be D and
E respectively and let the orthocenter be H (see figure 7). Note first that H lies on the
line parallel to AB and passing through the intersection point of the medians if and only if
|CE| |CE|
= 3, thus it is enough to prove that = tan 6 A · tan 6 B . As 6 AHE = 6 CHD ,
|EH| |EH|
we also have 6 EAH = 6 BCE , i.e. the right triangles CEB and AEH are similar and
|CE| |AE| |CE|
tan 6 B = = . Noting that tan 6 A = , we obtain the necessary relation.
|EB| |EH| |AE|
C
D
·
H
·
A E B
Figure 7
4. Let us put pieces on some squares of 2n × 2n chessboard in such a way that on every
horizontal and vertical line there is an odd number of pieces. Prove that the whole number
of pieces on the black squares is even.
Solution: Enumerate all the horizontal and vertical lines by numbers 1, . . . , 2n and assume
that the square (1, 1) is black (this does not restrict generality). Let A be the number
of pieces on the squares with even horizontal and vertical line number, B the number of
pieces on the squares with odd horizontal and vertical line number and C the number of
pieces on the squares with even horizontal and odd vertical line number. Then A + C is
the number of pieces with even horizontal line number and B + C the number of pieces
with odd vertical line number. As the number of such horizontal and vertival lines is the
same, these numbers have the same parity and thus the number of pieces on the black
squares A + B = (A + C) + (B + C) − 2C is even.
5. The numbers 0, 1, 2, . . . , 9 are written (in some order) on the circumference. Prove that
a) there are three consecutive numbers with the sum being at least 15;
b) it is not necessarily the case that there exist three consecutive numbers with the sum
more than 15.
7
Solution: a) The sum of numbers on the circumference is 45 and thus adding the ten
triple sums gives 3 · 45 = 135. Note that in every two neighbouring triples there are two
common numbers and the third one is different, so their sums must be different. Hence,
if the largest sum would be 14, we should have the sums 13 and 14 altering. But then
the sum of every six-tuple should be 27 which is not possible, because the sums of two
neighbouring six-tuples must also be different.
b) Write on the circumference the numbers in the following order: 3, 8, 1, 5, 9, 0, 6, 7, 2,
4. Now it is elementary to check that no triple gives the sum greater than 15.
1. John knows n > 3 positive real numbers and he writes them all on the blackboard (every
number may occur more than once). Mary writes under every number the arithmetic mean
of the other n − 1 numbers and then deletes the initial numbers. She repeats the process
for 1998 times. After that Mary notices that there are exactly the initial numbers on the
blackboard. How many different real numbers does John know?
Solution. Let M be the greatest of the initial numbers. For arbitrary b1 , . . . , bn−1
n−1
z }| {
b1 + . . . + bn−1 M + ... + M
6 =M
n−1 n−1
and the equality holds if and only if b1 = . . . = bn−1 = M . If there were at least two
numbers less than M on the blackboard then all the next numbers would also be less
than M . And then of course on the 1998th time there cannot be M on the blackboard.
If only one of the numbers on the blackboard is less than M then the next time there
would be already two numbers less than M . Therefore all the initial numbers were equal
and John knows only one positive real number.
2. Two different points X and Y are chosen in the plane. Find all the points Z in this plane
for which the triangle XY Z is isosceles.
s
c1 c2
Xq qY
Answer: All the points on the circumferences c1 and c2 except the five points lying on
the line XY (see the figure).
8
Solution. If the points X , Y and Z are the vertices of an isosceles triangle then
|XZ| = |XY |, |Y Z| = |XY | or |XZ| = |Y Z|. The points Z such that |XZ| = |XY |
lie on the circumference c1 . The points Z such that |Y Z| = |XY | are situated on the
circumference c2 . The points Z such that |XZ| = |Y Z| are situated on the midperpen-
dicular s of the segment XY . It is clear that the points determine a triangle if and only
if they do not lie on the same line.
3. Prove that for every integer k the following assertions are equivalent (if one is true then
the other is also true and vice versa):
a) exist nonnegative integers a and b so that k = a2 + b2 + ab,
b) exist nonnegative integers c and d so that k = c2 + d2 − cd.
a2 + b2 + ab = k = c2 + d2 − cd.
Let now k be expressed in the form k = c2 + d2 − cd. Without loss of generality we can
assume that c > d. Denoting c = n, d = m on the left side and a = n − m, b = m on
the right side of the relation (2) we obtain
c2 + d2 − cd = k = a2 + b2 + ab.
9
4. Find all the four-digit numbers n such that multiplying n by we obtain the number
2
which consists of the same digits as n but in the opposite order.
Solution. Let n = abcd. Since 4,5 · abcd < 10000 then a = 1 or a = 2. The digit d has
9
to be even. At the same time multiplying it by the last digit of the product must be
2
1 or 2. Consequently a = 2 and d = 6 or a = 1 and d = 8. Let us examine these cases
separately.
9
1) If a = 2 and d = 6 then · abcd > 9000 which contradicts d = 6.
2
2) If a = 1 and d = 8 then we obtain
9
(1000 + 100b + 10c + 8) · = 8000 + 100c + 10b + 1
2
3465 + 55c = 440b
63 + c = 8b.
This equation has two solutions: c = 1, b = 8 and c = 9, b = 9. These values will give us
the numbers 1818 and 1998.
9
where A, B , C , D , E , F are positive integers, all three fractions are reduced and their
denominators are pairwise relatively prime. The pupil adds the fractions writing the least
common multiple of the denominators of the summands as the denominator of the result.
Prove that the fraction that the pupil writes is reduced.
Solution. As the denominators of the fractions are pairwise relatively prime, their least
ADF + CBF + EBD
common multiple is BDF and so the resulting fraction is in form .
BDF
Let’s suppose antithetically that this fraction is reducable by some number greater than
one. Then there exists a prime number p which divides both the numerator and the
denominator of this fraction. As the product BDF is divisible by the prime number p
so one of terms if divisible by it: without loss of generality we can assume that this term
is B . But then two summand in sum ADF + CBF + EBD are divisible by p and as
the sum is divisible by p by assumption, the summand ADF is divisible by p as well.
Therefore either A, D or F is divisible by p. In the first case we get a contradiction with
A
the fraction being reduced, in other cases with assumption that the denominators of
B
the initial fractions were pairwise relatively prime.
Comment. Analogical assertion can be proved in case where there are more than three
fractions as summands.
1
1+
1
1+
1
1+
1+n
Solution. The expression given in the problem is not defined when n = −1 or n = −2.
n+2 n+2
The value of this expression is 1 + what is an integer iff is an integer i.e.
2n + 3 2n + 3
the number 2n + 3 divides the number n + 2. We will show that no such integer exists.
As n 6= −1 and n 6= −2 then |3 + 2n| 6= 1. If there exits an integer n for which the
n+2
fraction could be reduced with some number d > 1 then both 2n + 3 and n + 2
2n + 3
would be divisible by d, so 2 · (n + 2) = 2n + 4 would be divisible by d as well and
(2n + 4) − (2n + 3) = 1 would be divisible by d—a contradiction. Therefore the fraction
n+2
is reduced for all integers n 6= −1, −2 and the value of given expression can’t be
2n + 3
an integer for any integer n.
3. On the plane there are two non-intersecting circles with equal radii and with centres O1
and O2 , line s going through these centres, and their common tangent t. The third circle
osculates these two circles in points K and L respectively, line s in point M and line t
in point P . The point of tangency of line t and the first circle is N .
a) Find the length of the segment O1 O2 .
b) Prove that the points M , K and N lie on the same line.
√
Answer: a) 2 2R.
10
R
Solution. a) The radius of the third circle is obviously ; let its centre be O3 (see
2
figure 8) and |O1 O2 | = c. Considering that the triangle O1 M O3 is a right triangle with
R 3R R c R 2 c 2 3R 2
the hypotenuse R + = and legs and , we get + = , from
2 2 2 √ 2 2 2 2
where R2 + c2 = (3R)2 , c2 = 8R2 and c = 2 2R.
A1
Np Pp t A9 A2
pO3
K p pL A8 A3
p p p s
O1 M O2
A7 A4
A6 A5
Figure 8 Figure 9
4. For which values of n (n > 3) is it possible to draw on a plane such a closed broken line
consisting of n links that every link has exactly one point in common with every other
link so that this point is an end point or an inner point for both links, and no point on
the plane is an end point for more than two links?
Solution. When n = 2k + 1 is odd we can get the necessary construction in the following
way. Let A1 , A2 , . . . , A2k+1 be the vertices of a regular (2k+1)-gon. We draw a segment
from every vertex Ai to vertices Ai+k (or A(i+k)−(2k+1) , if i + k > 2k + 1) and Ai−k (or
A(i−k)+(2k+1) , if i − k < 1). We can see that a closed broken line is obtained. Indeed, from
one side every vertex is connected to exactly two other vertices, from the other side it is
possible to reach any vertex from any other vertex, moving by links. To be convinced in
the latter assertion it is enough to notice that starting from the vertex Ai we can reach
the vertex Ai+1 using two links and hence continuing in the same way any other vertex.
In addition we have to prove that every link has exactly one common point with every
other link (either end point or inner point). Without loss of generality let’s consider the
link A1 Ak+1 . All the other links Ai Aj (i < j ) can be divided into three classes:
1) i = 1, j = k + 2; in this case these links have a common end point A1 ;
2) i = k , j = 2k + 1; in this case these links have a common end point Ak ;
3) 1 < i < k + 1, k + 1 < j < 2k + 1; in this case the vertices Ai and Aj lie on different
sides of the line A1 Ak and because of convexity of initial regular (2k+1)-gon the
links A1 Ak and Ai Aj have a common inner point.
Now let’s show that no broken line having an even number of links does not satis-
fy the requested conditions. Let’s denote the vertices of the broken line in the or-
der of passing by B1 , B2 , . . . , Bn . Let’s consider the line B1 B2 . As every segment
B3 B4 , B4 B5 , . . . , Bn−1 Bn has to intersect it, the vertices B3 , B5 , . . . , Bn−1 lie on one side
and the vertices B4 , B6 , . . . , Bn on the other side of that line. Therefore the segments
B2 B3 and Bn B1 can’t have common points.
11
5. Two palmists were asked several questions about the life of mr. X, each of which had to
22
be answered with yes“ or no“. The palmist A answered correctly to of questions
” ” 43 4
which were answered uncorrectly by the palmist B , palmist B answered correctly to
7
of questions which were answered uncorrectly by the palmist A. To how many questions
did the palmists A and B give the same answer, when the palmist A answered correctly
51% of all questions?
Answer: 50%.
Solution. As the palmist A answered uncorrectly to 49% of all questions there were
4
·49 = 28 per cent questions that were answered uncorrectly by A but correctly by B and
7
21% questions that both answered uncorrectly. On the other side, as A answered correctly
22
to of questions answered uncorrectly by B , then the part of questions answered
43
21
uncorrectly by both palmists was among questions answered uncorreclty by palmist
43
B . As both palmists answered uncorrectly 21% of all questions, the palmist B answered
uncorrectly 43% of all questions and there were 22% of questions that were answered
uncorrectly by B but correctly by A. So there were 28 + 22 = 50 pro cent of questions
that were answered differently by A and B and so their opinion coincided in another half
of questions.
[x] = ax + 1
1
Answer: If a < −1 then x = − ; if a = 0 then every such real number x is suitable that
a
3 2
1 6 x < 2; if a = 2 then x = − or x = −1; if a > 2 then x = − ; if a = −1 or a = 1
2 a
then the equation has no solutions.
Solution. We will use the inequalities x − 1 < [x] 6 x holding for all real numbers x.
1) If a = 0 then [x] = 1 that is 1 6 x < 2.
2) Since [x] 6 x < x + 1, the equation has no solutions if a = 1.
3) Let a > 2. We will obtain the inequalities (a − 1) x > −2 and (a − 1) x 6 −1 that is
2 1
− <x6− .
a−1 a−1
2 1
Since − > −2 and − < 0, we get two possibilities if a = 2: [x] = −2 or
a−1 a−1
3
[x] = −1, the corresponding values of x are x = − and x = −1 (these are really
2
2
solutions). If a > 2 then the only possibility is [x] = −1 and x = − which satisfies the
a
equation.
12
4) Let a 6 −1. Then a − 1 < 0 and we obtain the inequalities
2 1
>x> .
1−a 1−a
1 2
Since > 0 and 6 1 then consequently [x] = 0. The possibility a = −1 gives
1−a 1−a
1
us a contradiction, if a < −1 then the solution is x = − .
a
3. A n × m-table filled with positive integers was written on the paper. John wrote after
each row the greatest common divisor of the numbers of this row and below each column
the least common multiple of the numbers of this column. Let a be the least common
multiple of the column of the greatest common divisors and let b be the greatest common
divisor of the row of the least common multiples. Prove that b is divisible by a.
Solution. Let cij denote the element of the ith row and the jth column. We fix the row
i and the column j arbitrarily. Then the least common multiplier of the numbers of the
jth column is divisible by cij and cij is divisible by the greatest common divisor of the
numbers of the ith row. Thus the number written below the jth column is divisible by the
number written after the ith row. Since j was chosen arbitrarily, the number written after
the ith row is the common factor of all the numbers written below the columns. Thus the
greatest common factor b of all the numbers written below the columns is divisible by
the number written after the ith row. Since i was chosen arbitrarily, the number b is the
common multiple for all the numbers written after the rows. Thus b is divisible by the
least common multiple a of all the numbers written after the rows.
4. On the conference of linguists there were n > 3 participants who could speak altogether
14 different languages. It is known that for every three linguists existed a language that
was spoken by all three. But every language was spoken by no more than a half of linguists.
Find the minimal possible value of n.
Solution. If n 6 5 then we choose an arbitrary triple and we get a language that is spoken
by at least 3 that is more than a half of the participants, contradicting the conditions of
the problem. If n = 6 or n = 7 then we choose arbitrary 6v linguists. It is possible to
form 20 different triples of the linguists. By pigeon-hole principle there exists a language
that is the same for at least two triples and therefore is spoken by at least 4 linguists.
This is again a contradiction because 4 is more than a half of 6 or 7. If n = 8 then it is
possible to find the construction satisfying the conditions of the problem:
1 A C E G I K M
2 A C E H J L N
3 A D F G I L N
4 A D F H J K M
5 B C F G J K N
6 B C F H I L M
7 B D E G J L M
8 B D E H I K N
(here the letters A, . . . , N denote the languages and the numbers 1, . . . , 8 denote the
participants).
13
Comment: Constructing the example for the case n = 8 we can use the following condi-
tions that have to be satisfied (why?):
a) Every language is spoken by exactly 4 linguists;
b) Every linguist can speak exactly 7 languages;
c) For every three linguists there is one and only one language spoken by all of them.
5. On the side BC of the triangle ABC a point D different from B and C is chosen so
that the bisectors of the angles ACB and ADB intersect on the side AB . Let D ′ be the
symmetrical point to D with respect to the line AB . Prove that the points C , A and D ′
are on the same line.
Solution. Let E be the intersection point of the bisectors of the angles ACB and ADB
and let 6 ACB = δ1 , 6 ADB = δ2 and 6 CBA = α.
D′
δ1 δ2
α
C B
D
By the property of the bisector for the triangles ABC and ABD we get
Consequently
sin α sin α
= ,
sin(α + δ1 ) sin(α + δ2 )
hence
sin(α + δ1 ) = sin(α + δ2 ).
14
1) α + δ1 = α + δ2 , that is δ1 = δ2 . This is not possible because the point D is in the
interior of the segment BC .
2) α + δ1 + α + δ2 = π , that is δ1 + δ2 + 2α = π . Since the triangles ADB and AD ′ B
are congruent then 6 AD ′ B = 6 ADB = δ2 and 6 ABD ′ = 6 ABD = α and the point A
is on the side CD ′ of the triangle BCD ′ .
1. Let a be an integer, which square divided by n gives the remainder 1. Which remainder
can be obtained dividing the number a by n, if
a) n = 16;
b) n = 3k , where k is a positive integer?
Answer: a) 1, 7, 9 or 15; b) 1 or 3k − 1.
Solution. a) As the square of an even number is an even number and gives an even remain-
der dividing by 16 the number a must be an odd number. Examining three possible cases
we find that only 1, 7, 9 and 15 fit. It is possible to reduce the number of examinations
needed by noticing that the squares of the numbers i and 16 − i give the same remainder
by dividing by 16.
b) It’s clear that the squares of the remainders 1 and n − 1 give the remainder 1 modulo
n. Let’s assume that the number a2 gives the remainder 1 by dividing by n = 3k and
let’s prove that number a gives the remainder 1 or 3k − 1 by dividing by 3k . From the
assumption we get that 3k divides the number a2 − 1 = (a − 1)(a + 1), hence there exist
natural numbers i and j that i + j = k where 3i divides the number a − 1 and 3j divides
the number a + 1. If both i and j were positive then both numbers a − 1 and a + 1 would
be divisible by 3, but that’s impossible. Therefore one of the nest cases must occur: i = 0
and j = k , or j = 0 and i = k . Hence one of the numbers a − 1 and a + 1 is divisible by
3k , or in another way, a gives the remainder 1 or 3k − 1 by dividing by 3k .
Comment. The arguments in the part b) will still hold if number 3 is replaced by any
other odd prime number.
Solution. Let’s define α(x) = P (−x) and β(x) = Q(−x). Then α and β are polynomials
which coefficients all have same sign. Hence in the product γ of the polynomials α and
β all coefficients have the same sign as well (all coefficients are positive if the signs of
coefficients in α and β are same and negative if the signs of coefficients in α and β are
different). Therefore
3. Two right triangles are given, of which the incircle of the first triangle is the circumcircle
of the second triangle. Let the areas of the triangles be S and S ′ respectively. Prove that
S √
> 3 + 2 2.
S′
15
Solution. Let the lenghts of the legs of the outer triangle be a and b, the lenght of the
hypotenuse be c and the radius of its incircle be r (see figure 10). Then the length of
the hypotenuse of the inner triangle is 2r and the altitude h drawn to hypotenuse is not
1 ab a+b+c
greater than r . Hence S ′ = · 2rh = rh 6 r 2 . We know that S = = r , from
2 2 2
what using the inequality between arithmetical mean and geometrical mean we get
p √ √
ab = (a + b + c) · r = ((a + b) + a2 + b2 ) · r > (2 ab + 2ab) · r =
√ √
= (2 + 2) ab · r ,
√ √
e.g. ab > (2 + 2) · r . Squaring both sides of the equation and dividing by 2 we get
√ 1 1 S √
S > (3 + 2 2) · r 2 . Bearing in mind that ′ > 2 we see that ′ > 3 + 2 2.
S r S
c
· h
a r r
r
b
Figure 10
4. On n cells of an infinite squared board there is one piece on each cell. If one of the four
neighbours of the cell containing the piece A contains the piece B and the cell behind it is
empty, the piece A can be moved over the piece B to the empty cell behind it. Does there
exist such a combination of pieces from which it is possible in finite number of moves to
obtain the situation where the final combination of pieces is the same as in the beginning
but moved by one cell in any direction, if
a) n = 1999;
b) n = 2000;
c) n = 1998?
16
1 2 ... 500 1 2 1 2 1 ···
3 4 3 4 3 ···
w w w w ... w w ···
1 2 1 2 1
w w w w ... w w .. .. .. .. .. ..
. . . . . .
Figure 11 Figure 12
5. Inside the square ABCD there is the square A′ B ′ C ′ D ′ so that the segments AA′ , BB ′ ,
CC ′ and DD ′ do not intersect each other neither the sides of the smaller square (the
sides of the larger and the smaller square do not need to be parallel). Prove that the sum
of areas of the quadrangles AA′ B ′ B and CC ′ D ′ D is equal to the sum of areas of the
quadrangles BB ′ C ′ C and DD ′ A′ A.
Solution. When the centres of smaller and larger square coincide then the quadrangles
AA′ B ′ B , BB ′ C ′ C , CC ′ D ′ D and DD ′ A′ A are congruent and the assertion of the problem
holds. Therefore it is enough to show that the sum of areas of the quadrangles AA′ B ′ B
and CC ′ D ′ D does not change by any parallel displacement of smaller square (with this
displacement we alwyas can make the centres of the squares coincide).
A B
A′
D′
B′
C′
D C
Figure 13
Without loss of generality we can assume that the vertex A′ of the smaller square is not
further off the side AB than the vertex B ′ (otherwise we can change the labels of the
vertices A and B ; C and D ; A′ and B ′ ; C ′ and D ′ ). Then the diagonal A′ B lies inside
the quadrangle AA′ B ′ B and divides it into two triangles ABA′ and A′ B ′ B (see figure 13).
Also the vertex C ′ lies not further off the side CD than the vertex D ′ , because of what the
diagonal C ′ D lies inside the quardangle CC ′ D ′ D and divides it into two triangles CDC ′
and C ′ D ′ D . The areas of triangles ABA′ and CDC ′ does not change by any parallel
displacement of smaller square parallel to the side AB of the larger square (because their
basis AB and CD and the altitudes drawn on them don’t change) and the sum of the
areas of these triangles doesn’t change by any parallel displacement of smaller square
parallel to the side BC of the larger square (because their basis and the sum of their
altitudes don’t change). As every parallel displacement can be done by two displacement
perpendicular to each other so the sum of the areas of the triangles ABA′ and CDC ′
doesn’t change by any parallel displacement of smaller square inside the larger square.
Analogically we can see that the sum of areas of triangles A′ B ′ B and C ′ D ′ D doesn’t
change by parallel displacement parallel to any side of smaller square (because their basis
A′ B ′ and C ′ D ′ and the sum of their altitudes don’t change). So the sum of the areas of
the quardangles AA′ B ′ B and CC ′ D ′ D does not change by any parallel displacement of
the smaller square inside the larger square.
17
Estonian math
ompetitions 2000/2001 2. Find the largest real number K having the following property: for any positive
real numbers a; b;
satisfying the inequality a b
6 K , the inequality ab
6 K
also holds.
+ +
We thank the IMO
ommunity for many of these problems whi
h have been taken
from various materials distributed at the re
ent IMO-s. 3. Prove that, for any integer n > , the number | {z: : : } is divisible by , but is
0 11 1 3
n
3 digits n
not divisible by +1 . 3
n
Autumn Open Contest: O
tober 2000
4. The terms of the sequen
e a1; a2; a3; : : : satisfy the
ondition a a 1 a 2
for any n > . Find the sum of the rst terms of this sequen
e, if the sum
n = n n
of the rst terms is and the sum of the rst terms is .
3 2000
Juniors (up to 10th grade)
1997 2002 2002 1997
1. How many positive integers less than and not
ontaining digits other 5. On a plane n points are given, no three of them
ollinear. At most how many line
than and are there? segments it is possible to draw between these points in su
h a way that the line
20002001
segments form no triangle with verti
es at the given points?
0 2
2. Find the two last digits of the number :::
1! + 2! + 3! + .
+ 2000!
3. Consider points C1 , C2 on the side AB of a triangle ABC , points A1 , A2 on Solutions of Autumn Open Contest
the side BC and points B1 , B2 on the side CA su
h that these points divide the
orresponding sides to three equal parts. It is known that all the points A1 , A2 , J1. .
B1 , B2 , C1 and C2 are
on
y
li
. Prove that triangle ABC is equilateral. Answer: 136
The set of integers under
onsideration
onsists of all integers with up to digits
4. Real numbers x and y satisfy the system of equations
ontaining only digits and , all -digit integers of the form and the
7
integer . There are 1 integers with exa
tly k digits and , and 3
0 2 8 20000
k
integers of the form . So the required number of integers is
8 20002000 2 0 2 2
> x
<x+y + y
>
> = 10
20000
: (2
0 + 21 + : : : + 26) + 8 + 1 = (27 1) + 9 = 136 :
>
> x(x + y)
>
:
y
= 20
J2. Answer: 13 .
Find the sum of all possible values of the expression x y . The produ
t : : : has , and as fa
tors, therefore being divisible by
. Hen
e the last two digits of n are zeros for any n > and it su
es to
1 2 10 2 5 10
+
nd two last digits of . The two last digits of the summands are
100 ! 10
5. Let m m for any odd integer m and m m for any even integer m .
= +3 =
01 02 06 24 20
:::
, , , , , , , and , yielding as the answer.
20
1! + 2! +
40 20
+ 9!
80 13
a) Find all integers k su
h that k .
2
b) Prove that, for every odd integer K , there exist pre
isely three dierent
= 1
C
integers k su
h that k K .
) How many dierent integers k with the property k K exist for an even
=
B1 A2
integer K ?
= r r
B2 r
A1
r
r r
Seniors (grades 11 and 12) A C1 C2 B
Figure 1
1. Points A , B , C , D , E and F are given on a
ir
le in su
h a way that the three
hords AB , CD and EF interse
t in one point. Express angle EF A in terms of J3. Label the points on the sides of the triangle so that jAC1j jC1C2j jC2Bj ,
angles ABC and CDE (nd all possibilities). jBA1 j jA1 A2j jA2 C j and jCB1 j jB1 B2 j jB2 Aj (see Fig. 1). Then we have
= =
= = = =
1 2
6 BA C 6 BA C 6 BCA and 6 BC A 6 BC A 6 BAC . Sin
e points
1 2 2 1 2 1 1 2 In
ases (a) and (b) similar arguments give 6 EF A 6 CDE 6 ABC and
A1 , A2 , C1 and C2 are
on
y
li
, we get 6 BA2 C1 Æ 6 AC2A1 6 BC2A1 , 6 CDE , respe
tively.
= = = = =
6 EF A = 6 ABC
whi
h gives 6 BCA 6 BAC . The equality 6 BAC 6 CBA follows by symme-
= 180 =
try.
= =
E F
J4. Answer: 10 . A C A C A C A C
By Viete's theorem, the possible values of x y are in
luded in the set of roots
of the quadrati
equation E F
+
F E
a2 10a + 20 = 0 : D B D F
B D B D E
B
This equation has two dierent roots be
ause D 2 > . Viete's (a) (b) (
) (d)
formulae give to be the sum of these roots. It remains to
he
k that is not
= 10 4 20 0
10 11
among the roots (as y x x y y from the rst equation, x y 6 enables
=
+
+ = 11 Figure 2
us to nd the
orresponding values for x and y ).
11 ( + )
Consider
ase (d). Sin
e EF A and ADE are opposite angles of a
y
li
quadri-
J5. Answer: a) , and ;
) .
1 2 8 5
lateral ADEF and 6 ADE 6 ADC 6 CDE 6 ABC 6 CDE , we have
= + = +
a), b) Observe that if m is odd, then both m and m are even. Hen
e if 6 EF A = 180Æ 6 ADE = 180Æ 6 ABC 6 CDE :
K k is odd, then k K , and k and k are not both odd. This gives the p
following three possibilities.
= = 2
S2. Answer: 3 3 .
1) If both k and k are even, then k k k K . Let a b+
6 K. By the AM-GM inequality we have
2) If k is odd and k is even, then k k K .
= 2 = 4 = 8 +
k
3) If k is even and k is odd, then k k k K .
a + b +
3 3 K2
= 3 = (2 3) = 4 3
K = 2 = 2 ( 3) = 2 (2 3) = 4 6
ab
6 6 K K :
The numbers K , K and K are pairwise distin
t sin
e modulo they
=
3 3 27
are
ongruent to , and , respe
tively. For a), K gives k 2 f ; ; g .
8 4 3 4 6 4
Hen
e if K 6 , or equivalently K 6 p , the required
ondition is satis-
0 1 2 = 1 1 2 8 2
) Let now K be even. If k is even, then we get the same three possibilities for 1 3 3
k as above. If k is odd, then k is even and k
an be either even or odd.
ed. However, if K > p and a b
K , then a b
K and
27
4) If k is even, then k k k K K .
5) If k is odd, then k k K .
= 2 = 4 = 4( 3) = 4 12 3 3 = = = + + =
k K
3
K2
> K , so the
ondition is not satised.
= 3 = 2 3 = 2( 3) 3 = 2 9
Sin
e K is even, the numbers K , K , K and K are
ongruent to ab
K
=
, , and , respe
tively, modulo . Moreover, K is
ongruent to either
8 4 3 4 6 4 12
27
or modulo . Hen
e these ve numbers are pairwise distin
t. S3. We use indu
tion on n .
0 5 2 4 8 2 9
3 7 8
S1. Angle EF A is equal to either 6 ABC 6 CDE , or 6 ABC 6 CDE , or The proposition holds for n sin
e is divisible by 0 and is not
divisible by 1 .
Base: = 0 1 3 = 1
6 CDE 6 ABC , or Æ 6 ABC 6 CDE .
Answer: +
3 = 3
180
Given the
hords AB and CD , the
hord EF
an be drawn in four essentially Step:Observe the equality
dierent ways point E
an lie on the
ir
le between points D and A , between 11 : : : 1 = 11 : : : 1 1 00 : : : 0 1 00 : : : 0 1 :
points A and C , between points C and B or between points B and D (see | {z } | {z } | {z } | {z }
Fig. 2). 3n+1 3n 3n 1 3n 1
Let us nd 6 EF A for
ase (
). Sin
e EF C and CDE are angles subtended The rst fa
tor here is divisibleby but not by +1 by the indu
tion hypothesis,
n n
by the same
hord EC , we have 6 EF C 6 CDE ; similarly 6 CF A 6 ABC . and the se
ond fa
tor is divisible by but not by . Sin
e is prime, this implies
3 3
Hen
e that the produ
t is divisible by +1 , but not by +2 .
= = 3 9 3
n n
3 3
6 EF A = 6 CF A + 6 EF C = 6 ABC + 6 CDE : S4. Answer : .
2012
3 4
Denote a1 p and a2 q . It is easy to see that
= = Spring Open Contest: Mar
h 2001
a p if k ; ; ; : : :;
a q if k ; ; ; : : :;
k = = 1 7 13
a q p if k ; ; ; : : :;
k = = 2 8 14 Juniors (up to 10th grade)
p if k ; ; ; : : :;
k = = 3 9 15
a k =
q if k ; ; ; : : :;
= 4 10 16
1. Eight students, Anne, Mary, Cathy and Tina, Anthony, Mark, Carl and Tom have
a
a p q if k ; ; ; : : :.
k = = 5 11 17
to work in four pairs, one boy and one girl in ea
h pair. They know ea
h other,
k = = 6 12 18
with only these ex
eptions: Anthony knows neither Anne nor Mary; Mark doesn't
Observe that the sum of any six
onse
utive members of the sequen
e is equal to know Mary and both Carl and Tom know neither Cathy nor Tina. How many
zero. Denoting S a1 : : : a , we get ways are there to divide the students into pairs, so that ea
h boy
ould work with
a girl he knows?
k = + +
S p if k ; ; ; : : :;
k
S p q if k ; ; ; : : :;
k = = 1 7 13
S q if k ; ; ; : : :;
k = + = 2 8 14
2. In a triangle ABC , the lengths of the sides are
onse
utive integers and median
q p if k ; ; ; : : :;
k = 2 = 3 9 15
drawn from A is perpendi
ular to the bise
tor drawn from B . Find the lengths
S
S q p if k ; ; ; : : :;
k = 2 = 4 10 16
of the sides of triangle ABC .
if k ; ; ; : : :.
k = = 5 11 17
S k = 0 = 6 12 18
3. In a s
hool lo
ker room there are lo
kers
Hen
e q p S1997 and q p S2002 , whi
h give q and in three rows. The lo
kers in ea
h row are la-
60
K
with S2000 p q . belled from left to right with numbers to
= = 2002 2 = = 1997 = 5
p L
in the top row, to in the middle row and M
= 2007 = + = 2012 1 20
to in the bottom row.
21 40
n2 2
S5. for even n and n for odd n . 1 41 60
Kate's, Lisa's and Mary's lo
kers are lo
ated as shown in the gure. Ea
h of the
Answer:
Divide the points into two subsets with
ardinalities as
lose to ea
h other as
4 4
possible, and draw a line segment between any two points from dierent subsets. three lo
ker numbers is divisible by the number of Mary's house, whi
h is not . 1
Then ea
h
losed line formed by these line segments
ontains an even number a) What is the number of Mary's house?
of links and hen
e the line segments do not form any triangles2 with verti
es at b) What
ould be the numbers on the girls' lo
kers?
the given points. The number of line segments is n 2 n for even n and
4. Integers a , b ,
and d satisfy ja
bdj jad b
j . Prove that either
jaj jbj or j
j jdj .
= + = + = 1
2 4
n2
n
n for odd n .
= = 1 = = 1
1 + 1 1
5. A
onvex hexagon is
onstru
ted from n pie
es, ea
h of whi
h
=
Now prove that there
annot be more line segments. Consider any
olle
tion of line
2 2 4
segments satisfying the
onditions of the problem. Let m be the maximal number is an equilateral triangle (one example is given in the gure).
of line segments in
ident to one point, and let X be any point in
ident to m line a) Prove that the hexagon is equiangular.
segments. Let A be the set of the other endpoints of these m line segments, and b) Find all possible values of n .
B be the set of the other n m points (in
luding X ). Ea
h point of A
an be
joined only to points of B be
ause any two joined points from A together with
X would form a triangle. Hen
e ea
h of the m points of A o
urs as an endpoint Seniors (11th and 12th grade)
for at most n m line segments. On the other hand, ea
h of the n m points
from B o
urs as an endpoint for at most m line segments by the
hoi
e of m . 1. The serial numbers of lottery ti
kets are -digit integers. It is known that the
So there is at most m n m n m m m n m segment-endpoint pairs, serial number of a winning ti
ket has seven distin
t digits and is divisible by ea
h
7
and sin
e every line segment has two endpoints, we have at most m n m line
( )+( ) = 2 ( )
segments. It remains to noti
e that this expression a
hieves its maximum when
( )
of its digits.
the dieren
e of m and n m is as small as possible, i.e. if m n m for even a) Prove that the serial numbers of all winning ti
kets
onsist of the same digits.
n and if jm n m j for odd n . b) Find the largest possible serial number of a winning ti
ket.
=
( ) = 1
5 6
2. Let us
all a
onvex hexagon ABCDEF if 6 A 6 C 6 E 6 B 6 D 6 F . From the gure we obtain L K and M L K . Sin
e
K , L and M are all divisible by the number of Mary's house n , the dieren
es
boring + + = + + = + 21 = + 14 = + 35
a) Is every
y
li
hexagon boring? and M L are also divisible by n . It follows that the only
b) Is every boring hexagon
y
li
? L K
possible value of n is . Now, sin
e 6 K 6 , we obtain K or K ,
= 21 = 14
3. Find all real-valued fun
tions f x dened for all real numbers whi
h satisfy the and hen
e L and M or L and M .
7 1 20 = 7 = 14
ondition f x f x2 for ea
h real x .
( ) = 28 = 42 = 35 = 49
2001 + (0) = 2001
J4. If the numbers a
bd and ad b
have the same sign, then a
bd ad b
and a
bd ad b
a b
d . Hen
e a b or
d . If the numbers
+ + + = +
4. For some < x; y < , two of the three expressions 2 x 2 y , 2 x y a
bd and ad b
have distin
t signs, then
0 = + = ( )( ) = =
and have equal values and the third one is dierent.
0 sin + sin sin ( + )
a
bd ad b
a b
d
and a b or
d . In both
ases jaj jbj or j
j jdj . If jaj jbj , then
+ + 0 = + + + = ( + )( + )
1
a) Whi
h of the three expressions has a dierent value? ja
bdj is divisible by jaj , therefore jaj jbj . In
ase j
j jdj , we
= = = = =
b) Give an example of x and y for whi
h su
h a situation o
urs. similarly obtain j
j jdj .
1 = +
= = 1
= = 1 =
5. There are small boxes numbered from to , and one large box. John puts J5. b) all integers n > .
some balls in some (or all) of the small boxes, and starts relo
ating them by the
10 1 10
Answer: 6
following rules: a) Let a vertex of the hexagon be the vertex of k triangles. Then the interior
during ea
h move, John removes all balls from any small box numbered n
angle at this vertex must be k Æ . Sin
e the interior angles of a
onvex hexagon
are less than Æ , none of them
an be larger than Æ . Sin
e the sum of the
60
where the number of the balls equals n ; angles is Æ Æ , it follows that all the angles are equal to Æ .
180 120
he adds these balls into boxes to n (one ball into ea
h box) and puts
720 = 6 120 120
the remaining ball into the large box.
1 1
b) There must be at least pie
es, be
ause there is at least one triangle on ea
h
side of the hexagon, and sin
e the hexagon has no angles equal to Æ , ea
h
6
He
ontinues this way until he
annot make another move a
ording to these rules. triangle
an lie on only one side of the hexagon.
60
Find the largest possible total number of balls in the small boxes at the beginning
of the game, for whi
h it is possible to put all balls in the large box by the end of
the game.
Solutions of Spring Open Contest
n=6 n=7 n=8
J1. Answer : . 4 Figure 3
It is
lear that Carl and Tom
an only work with Anne and Mary: we obtain two The
onstru
tions for n , n ja n are given in Figure 3. We
an
ways to form two pairs. Now, Anthony and Mark have to work with Cathy and
ontinue the same way, adding more large triangles in the middle.
= 6 = 7 = 8
Tina, whom they both know: there are also two ways to form the two remaining
pairs. Hen
e, altogether there are four ways to form the pairs. S1. Answer: b) .
9867312
J2. , and . a) Obviously, the serial number
annot
ontain and must be even. Now, we
annot have among the digits, be
ause every even number divisible by ends
Answer: 2 3 4 0
Let D be the midpointof BC , then the medianis AD . Sin
e the bise
tor of 6 B is with a . If the serial number didn't have as one of its digits, it would
ontain
5 5
also an altitude in the triangle ABD , that triangle is equilateral, i.e. jBDj jBAj and should be divisible by , but the sum of the remaining seven digits is ,
0 9
and in the original triangle ABC we have jBC j jABj . Sin
e the lengths of
=
ontradi
tion. Hen
e is one of the digits. Now, the serial number is divisible by
3 3 31
the sides of triangle ABC are
onse
utive integers, the dieren
e jBC j jABj is
= 2
and the sum of its digits is between and . The only possible sum is
9
either or . In the rst
ase jABj , jBC j and the length of the side and the serial number
onsists of the digits ; ; ; ; ; ; .
9 32 39 36
AC must be either or , whi
h is impossible. In the se
ond
ase we obtain
1 2 = 1 = 2
1 2 3 6 7 8 9
jAB j , jBC j and jAC j .
0 3
b) Any -digit number
onsisting of these digits is divisible by , , and .
Now we must nd the largest possible even number among these whi
h is divisible
= 2 = 4 = 3 7 1 3 6 9
J3. Answer: a) ; b) ; ; or ; ; .
7 7 28 42 14 35 49 by 7 . This number is
8 = 56 . 9867312
7 8
S2. Answer: a) yes, b) no. Sin
e x sin
x and
x =
os
x sin
x , then
3
= sin
+
=
os
in both
ases 2 x 2 y 2 x 2 x .
2 2 2
B sin + sin = sin +
os = 1
A ? Let 2 x 2 y 2 x y . We shall prove that both of these are equal to .
Applying the formula for x y and squaring, we get
sin + sin = sin ( + ) 1
sin( + )
C sin
2 x + sin2 y = sin2 x
os2 y + sin2 y
os2 x + 2 sin x sin y
os x
os y ;
D sin
2 x (1
os
2y) + sin2 y (1
os
2x) 2 sin x sin y
os x
os y = 0 ;
2 x sin2 y x sin y
os x
os y = 0 :
E F 2 sin 2 sin
Figure 4 Figure 5 Sin
e < x; y < , neither x nor y equals . Hen
e we must have
, i.e. x y and x y equals to either
0 sin sin 0
x y x y
or , when
e 2 x y . So the only expression that
an have a dierent
sin sin
os
os = 0
os( + ) = 0 sin( + ) 1
a) Let ABCDEF be a
y
li
hexagon. Sin
e the quadrilaterals ABDF , CDF B
and EF BD are also
y
li
(see Fig. 4), we obtain value is 2 x y .
1 sin ( + ) = 1
sin ( + )
6 BDF = 6 A; 6 DF B = 6 C; 6 F BD = 6 E: b) Taking < x < and y x , we get
0 = +
Now, 6 A 6 C 6 E , and hen
e 6 A 6 C 6 E .
2 2
Thus 6 B 6 D 6 F 6 A 6 C 6 E , whi
h proves that the 2 x + sin2 y = sin2 x + sin2 x +
( ) + ( ) + ( ) = + + = 2
2 x +
os2x = 1 :
hexagon ABCDEF is boring.
+ + = 4 2 = 2 = + + sin = sin
2
b) Let us
ompress a regular hexagon along its two opposite sides (see Fig. 5). Sin
e < y < x y x < , we have 2 x y 6 .
The new hexagon is boring sin
e all its angles are equal, but it is not
y
li
sin
e + = +2
3
sin ( + ) = 1
three of its verti
es lie on one
ir
le and the rest on another
ir
le. 2 2 2
x2 2 S5. .
S3. and f x x .
Answer: 41
fx
( 2001)
We shall rst show that the total number of balls in the small boxes
annot ex
eed
. John
an empty box number only on
e, sin
e no balls are put into it during
Answer. ( ) = ( ) =
2001 2001
the relo
ations. He
an also empty box only on
e, sin
e at most one ball is put
41 10
Sin
e for any real number y there is an x y f su
h that y = x f ,
2001
(0)
= 2001 + (0)
into it (from the tenth box). Also, boxes , and
an be emptied only on
e.
9
2
Box number
an be emptied at most twi
e (at most balls will be added to it
8 7 6
the equality f y y f
( ) = 2001 holds for any real y . Taking y we
2001
(0)
= 0
from boxes with bigger numbers). Box
an also be emptied at most twi
e, box
5 5
at most times, box at most times and box at most times. John
an
4
f 2
get f (0) = and hen
e f
(0)
or f . Therefore the fun
tion
(0) = 0 (0) = 2001
therefore put no more than balls in the large box.
3 4 2
41
7 1 21
We shall now nd a way to pla
e balls in the smallboxes, so that all boxes
ould
2001
2 2
must be either f y y or f y y
( ) = . It is easy to
he
k that both
( ) =
( 2001)
be emptied. No balls are added to box , therefore it must
ontain balls. One
41
of these satisfy the given
onditions. ball will be added to box , therefore it must
ontain balls in the beginning.
2001 2001 10 10
Similarly there must be , and balls in boxes , and respe
tively. Sin
e
9 8
S4. a) 2 x y
an be the only expression with a dierent value; b) balls are added to box , it must
ontain balls in the beginning in order to
6 4 2 8 7 6
be emptied twi
e. Box must
ontain one ball, box three balls, boxes and
Answer: sin ( + ) 5 5 5
hoose < x < arbitrarily and take y x .
0
2
= +
2 must
ontain ball. The number of balls in the small boxes is now . It is easy
4 3 2 1
a) Let 2 x y . We shall prove that in this
ase also 2 x 2 y . to
he
k that if John always empties the box with the smallest possible number,
1 41
all balls will nally be in the large box.
sin ( + ) = 1 sin + sin = 1
From the equation 2 x y we have either x y or x y .
sin ( + ) = 1 + =
2
+ =
3
2
9 10
Final Round of National Olympiad: Mar
h 2001 (1) A is a word;
(2) if w is a word, then ww and ww are also words, where w is obtained from
w by repla
ing all letters A with B and all letters B with A ( xy denotes the
9th grade
on
atenation of x and y );
1. John had to solve a math problem in the
lass. While
leaning the bla
kboard, he (3) all words are
reated by rules (1) and (2).
a
identally erased a part of his problem as well: the text that remained on board Prove that any two words with the same number of letters dier exa
tly in half of
was x , where marks an erased digit. Show that John
an their letters.
still solve his problem, knowing that x is an integer.
37 (72 + 3 ) = 14 45
2. Dividing a three-digit number by the number obtained from it by swapping its 11th grade
rst and last digit we get as the quotient and the sum of digits of the original
number as the remainder. Find all three-digit numbers with this property.
3
1. The angles of a
onvex n -gon are ; ; : : : ; n . Find all possible values of n
and the
orresponding values of .
2
3. A
ir
le of radius is tangent to two adja
ent sides of a square and interse
ts
its two remaining sides at the endpoints of a diameter of the
ir
le. Find the side
10
length of the square. 2. A student wrote a
orre
t addition operation BA DC FE to the bla
kboard, su
h
+ =
4. It is known that the equation jx j jx j : : : jx j a has exa
tly that both summands are irredu
ible fra
tions and F is the least
ommon multiple
one solution. Find a . of B and D . After that, the student redu
ed the obtained sum EF
orre
tly by
1 + 2 + + 2001 =
5. A table
onsisting of rows and
olumnsis lled with integers ; ; : : :; an integer d . Prove that d is a
ommon divisor of B and D .
in su
h a way that ea
h of these integers o
urs in the table exa
tly times and
9 2001 1 2 2001
the integers in any
olumn dier by no more than . Find the maximum possible
9
3. Points D , E and F are taken on the sides BC , CA , AB of a triangle ABC ,
value of the minimal
olumn sum (sum of the numbers in one
olumn).
3
respe
tively, so that the segments AD , BE and CF have a
ommon point O .
Prove that jjOD
AOj jAE j jAF j
j jEC j jF B j .
= +
10th grade 4. Let x and y be non-negative real numbers su
h that x y . Prove that
x2y2 x2 y2 6 .
+ = 2
1. A
onvex n -gon has exa
tly three obtuse interior angles. Find all possible values
( + ) 2
of n . 5. ConsiderÆall trapezoids in a
oordinate plane with interior angles of Æ , Æ , Æ
and , su
h that their bases are parallel to one of the
oordinate axes and
90 90 45
2. Find the minimum value of n su
h that, among any n integers, there are three all verti
es have integer
oordinates. Dene the of su
h a trapezoid as the
135
whose sum is divisible by . total number of points with integer
oordinates inside and on the boundary of the
size
trapezoid.
3
3. There are three squares in the pi
ture. Find the sum of D
a) How many pairwise non-
ongruent su
h trapezoids of size are there?
angles ADC and BDC . b) Find all positive integers not greater than that do not appear as sizes of
2001
any su
h trapezoid.
50
A B C
4. We
all a triple of positive integers a; b;
( ) harmoni
if a b
. Prove that,
1
+
1
=
1
12th grade
for any given positive integer
, the number of harmoni
triples a; b;
is equal
to the number of positive divisors of
2 .
( )
1. Solve the system of equations
5. A tribe
alled Ababab uses only letters A and B, and they
reate words a
ording
x y
to the following rules: x :
sin =
sin y =
11 12
2. Find the maximum value of k for whi
h one
an
hoose k integers out of 3) If
, then 6 a b 6 363 whi
h implies a > , a
ontradi
-
; ; : : :; n so that none of the
hosen integers is divisible by any other
hosen tion.
= 3 300 32 = 200 + 7 10
integer.
1 2 2
p
9-3. .
3. Let I and r be the midpoint and radius of the in
ir
le of a right-angled triangle
Answer: 10 + 5 2
ABC with the right angle at C . Rays AI and BI interse
t the sides BC and
AC at points D and E , respe
tively. Prove that 6B C
jAE j jBDj r .
1 1 1
+ =
4. Prove that, for any integer a > , there is a prime p su
h that a a2 : : : a 1 p
O
q
is
omposite.
1 1+ + + +
5. Consider a table, lled with real numbers in su
h a way that ea
h number -
A
in the table is equal to the absolute value of the dieren
e of the sum of numbers
3 3
in its row and the sum of numbers in its
olumn. Figure 6
a) Prove that any number in this table
an be expressed as a sum or a dieren
e Introdu
e a
oordinate system where the sides of the square tangent to the
ir
le
of some two numbers in the table. are on the
oordinate axesthen the
entre of the
ir
le is O ; (see Fig. 6).
b) Show that there exists su
h a table with numbers in it not all equal to 0. Let the side of the square be a (evidently a > ) and the interse
tion points of
(10 10)
the
ir
le with its two other sides be A and B . As AB is the diameter of the
10
ir
le, the
ommon point C a; a of these two sides lies on the
ir
le. Sin
e CO
p
is a radius,pwe obtain p a 2 a 2 , giving a
( )
Solutions of Final Round and
.
( 10) + ( 10) = 10 10 = 5 2
a
9-1. x . = 10 + 5 2
9-4. .
Answer: = 1271
From the given equality we obtain x) = . To nd the number
y = 24 + x , note that
Answer: 1001000
Note that if x is a solution of the equation, x is also a solution. For
111(24 + 14 45
uniqueness we have x x , or x . In this
ase
2002
< 1445 < 222000 = 111 2000 ;
= 2002 = 1001
111 1000 = 111000
a 1000 + 999 + : : : + 2 + 1 + 0 + 1 + 2 + : : : + 999 + 1000 =
therefore y is a 4-digit number, with as its rst digit. Evidently y must end =
with . Let y ab , where 6 a; b 6 . Writing out the multipli
ation we
1
= (1000 + 1) + (999 + 2) + : : : + (2 + 999) + (1 + 1000) =
see that b ends with , hen
e b and there is a
arry of at least from
5 = 1 5 0 9
= 1000 1001 = 1001000 :
the third position. Sin
e there is no
arry to the rst position, we have a 6 .
+ 5 4 = 9 1
If the
arry from the third position were more than , we would have a > , a
2
ontradi
tion. Hen
e a and x .
1 8
Although this is not required in the problem, it
an be veried that
is indeed the only solution of the equation for a .
Remark.
= 2 = 1295 24 = 1271
x = 1001 = 1001000
9-2. Answer: 441 and . 882
9-5. 24.
We look for a number ab
su
h that ab
ba a b
, or a
b . Answer:
The numbers 1
an be in the same
olumn only with numbers 2, 3 and 4. As there
Hen
e 6
6 , and we have 3
ases.
= 3 +( + + ) 32 = 100 + 7
1
1) If
, then 6 a
3
whi
h implies 6 a 6 . If a , are altogether of these, the 1-s
an be at most in four
olumns. If all 1-s are
b6 in the same
olumn, the minimal
olumn sum is 9. If the 1-s are in two
olumns,
4 9
then b and b . If a , then b and b is not an
= 1 100 32 = 100 + 7 163 4 5 = 4
integer.
128 = 100 + 7 = 4 = 5 160 = 100 + 7
one of these must
ontain at least 5 of them and the sum of this
olumn is at most
2) If
, then 6 a whi
h implies 6 a 6 . If a , . If the 1-s are in four
olumns, then the sum of all numbers in
these
olumns is , hen
e the minimal
olumn sum is at most
5 1 + 4 4 = 21
b6
then b and b is not an integer. If a , then b , giving
= 2 200 32 = 200 + 7 263 7 8 = 7 9 (1 + 2 + 3 + 4) = 90
h 90 i
b = 8.
224 = 200 + 7 = 8 256 = 200 + 7
4
. If the 1-s are in three
olumns, we should have 3-s and 4-s in these
= 22
13 14
olumns to obtain the largest
olumn sum. In this
ase the sum of numbers in the right-angled triangles, we have
three
olumns is and the minimal
olumn sum is at most 24.
From the table below we see that this value is indeed attainable.
9 (1 + 3 + 4) = 72
6 ADF = 6 ADC 6 F DG = 6 ADC 6 DF G =
1 1 1 2 2 6 7 ... 2001
2
6 ADC 6 BDC ;
1 1 1 2 2 6 7 ... 2001 =
1 1 1 2 2 6 7 ... 2001
2
3 3 3 2 2 6 7 ... 2001 that implies 6 ADC 6 BDC 6 ADF . The segments AF and DF are
3 3 3 2 5 6 7 ... 2001
+ = +
transformed into ea
h other by a Æ rotation around F . Hen
e AF D is an
2
3 3 3 5 5 6 7 ... 2001 90
4 4 4 5 5 6 7 ... 2001 isos
eles right-angled triangle with 6 ADF , yielding 6 ADC 6 BDC . 3
4 4 4 5 5 6 7 ... 2001
= + =
4 4
4 4 4 5 5 6 7 ... 2001 Remark: There are also solutions using the
osine theorem or the identity
10-1. Answer: The possible values of n are , and . 4 5 6
tan( +
) =
.
tan + tan
The sum of the angles of a n -gon is n . Sin
e three of these angles are 1 tan tan
10-4. As a and b are non-zero integers, we have
( 2)
greater than and less than , and the remaining n angles are greater than
2
3
0 and less or equal to , we obtain n < n < n .
2
( 3) 0 + 3
2
( 2) ( 3)
2
+3
1
a b
+ () a ab b
1
=
1 +
=
1
() ab a b
()
= ( + )
Dividing by and transforming yields < n < . As n is an integer, we have
7
7 () ab a
b
= 0 () ab a
b
2
2 () + =
6 n 6 , and it is easy to
he
k that all these three values are indeed possible. () a
b
2:
2
( )( ) =
4 6
10-2. Answer: n .= 5 Now let a b
. If a and b are positive, then a
> and b
> . On
1
+
1
=
1
0 0
The sum of any three integers
ongruent to , and modulo is divisible the other hand, if a
> and b
> , then a and b are positive. Hen
e the
by . Also, the sum of any three integers
ongruent to ea
h other modulo is
0 1 2 3
harmoni
triples a; b;
are in one-to-one
orresponden
e with pairs of positive
0 0
divisible by . Consequently, among any ve numbers there are three whose sum
3 3
integers r; s , where rs
2 , and there are as many su
h harmoni
triples as
( )
is divisible by . On the other hand, among the numbers , , and there are
3
there are positive divisors of
2 .
( ) =
no three with a sum divisible by .
3 1 3 4 6
3
10-5. We use indu
tion on the length of a word. Let u1 and u2 be any dierent words
D of the same length, and suppose the
laim holds for all shorter words. As there is
only one word of length 1, u1 and u2 are
onstru
ted by rule (2). This implies
that there exist words v1 and v2 so that u1 v1v1 or u1 v1v1 and u2 v2v2
B or u2 v2v2 . Note that v1 and v2 are of the same length. If v1 v2 v , then
= = =
A C
one of the words u1 and u2 is vv and the other vv , diering exa
tly in half of
= = =
their letters. If v1 6 v2 , then v1 and v2 dier exa
tly in half of their letters by
the indu
tion hypothesis, and it remains to show that the latter halves of u1 and
=
F G
u2 also dier exa
tly in half of their letters. If these halves are v1 and v2 or v1
Figure 7 and v2 , this is obviously true. The words v1 and v2 , as well as v2 and v1 , dier
exa
tly in the letters where v1 and v2
oin
idediering therefore also exa
tly
10-3. Answer:
3
. in half of their letters. Hen
e in any
ase u1 and u2 dier in half of their letters.
Consider points F and G as shown on Fig. 7. As BCD and DGF are
ongruent
4
11-1. Answer: The only possibilities are n , and n , . = 3 =
6
= 4 =
5
15 16
Obviously n > . As the sum of angles of the n -gon is n n n ,
3
( + )
= ( 2)
11-4. Denote = 1 x , then x = 1 and from x + y=2 we get y = 1+ . Now
x2 y2 (x2 + y2 ) )2 (1 + )2 ((1 )2 + (1 + )2 ) =
2
we have n nn . Be
ause of
onvexity, we have n nn <
=
2 (
( + 1)
2)
=
2 (
+1
2) =
=
(1
((1 )(1 + ))2 (2 + 22) =
yielding n < . If n , we obtain ; if n , then .
5 = 3 =
6
= 4 =
5
= 2(1 2)2 (1 + 2) = 2(1 4)(1 2) :
11-2. Let D0 and B0 0be the0 multipliers of the0 rst and the se
ond fra
tion, respe
tively. Sin
e x; y > ,4we have2 jj 6 that implies 6 2 6 and 6 4 6 .
Hen
e 6 .
0 1 0 1 1 0 1 1
Then E AD B C and F BD DB0 , with B0 and D0
oprime sin
e F
2(1 )(1 ) 2
is the least
ommon multiple of the denominators. If, for a prime p , p divides d
= + = =
with k > , then p divides both E and F . Suppose p does not divide B . From
k
11-5. Answer: a) ; b) , , , , , , , , ja .
7 1 2 3 4 6 8 10 16 28 32
k
F BD0 we obtain that p divides D0 , hen
e p also divides B 0 C E AD0 .
0
k
Consider a trapezoid of height h and the length of its shorter base a (see Fig. 9).
Therefore, p divides either B0 or C , and as B0 and D0 are
oprime, p divides
= =
The longer base of the trapezoid is of length a h and thus there is a total of +
C . From F DB 0 we get that D is divisible by p , hen
e p is a
ommon fa
tor
=
k
N (a; h) = (a +1) + (a +2) + : : : + (a + h +1) =
(2 + a h +2)(h +1)
of C and D ,
ontradi
ting the irredu
ibility of DC . We
on
lude that p divides k 2
B , and similarly also D . Sin
e this is true for any prime divisor p of d , then B points with integral
oordinates inside and on the border of this trapezoid.
and D are both divisible by d .
11-3. Draw a line parallel to BC through A and denote its interse
tion points with 6
rays BE and CF by L and M , respe
tively (see Fig. 8). From similar triangles q q q
a q q q q q q
jAE j jALj jAF j jAM j
q q q q q q q q q q
AEL and CEB we have
jEC j jBC j . Analogously jF B j jBC j . Moreover,
q q q q q q q q q q
= = q
h q q q q q q q q
from similar triangles AOL and DOB we get jjOD AOj jALj
q q q q q q q q q q
j jBDj , and analogously -
q q q q q q q q q q
= q q q q q q q q q q
jAOj jAM j Figure 9
jODj jDC j . Hen
e
=
jAOj jALj jAM j jALj jAM j jALj jAM j jAE j jAF j a) We have to nd the number of distin
t pairs a; h for whi
h N a; h .
Taking into a
ount that , we
onsider two
ases:
( ) ( ) = 2001
jBC j jBC j jEC j jF B j :
+ +
jODj jBDj jDC j jBC j
2001 = 3 23 29
1) If h k is even, then N a; h a k k where k > and
= = = + = +
+
= 2 ( ) = ( + + 1) (2 + 1) 2 +1 3
C
a k
+ >k
+ 1 >
k
+ 1. The fa
tor k
an be , or , yielding the
2 + 1
2 + 1 3 23 29
pairs ; , ; and ; .
2
(665 2) (75 22) (54 28)
D 2) If h k is odd, then N a; h a k k , where h N a; h
k > and a k > k . The fa
tor k
an be , ,
= 2 1 ( ) = (2 +2 + 1) ( )
L a
or , yielding the pairs ; , ; , ; and ; .
1 2 +2 +1 2 +3 1 3 23 1 2 + 3
O 29 (999 1)
a (330 5) (20 45) (5 57)
E 2 3 + 6
F b) For h ; ; ; : : :; we express the size of a trapezoid in a
terms of a (see the table); if h > , then N a; h > for any
= 1 2 3 7 3 4 + 10
a
a > . It is easy to
he
k that numbers , , , , , , ,
A B 7 ( ) 50 4 5 + 15
a
, and are the only ones that
annot be expressed by
1 1 2 3 4 6 8 10
5 6 + 21
M
any of the formulae in the table. a
16 28 32
6 7 + 28
Figure 8 12-1. Answer: The only solution is x y . a
= = 0
7 8 + 36
17 18
Clearly x y is a solution. We know that j xj 6 jxj , where equality holds 12-4. If a , then p gives the desired result:
i x (this
an be easily proved using derivatives). Now
= = 0 sin = 2 = 11
: : : + 210 = 211
= 0
1+ 2+4+ 1 = 2047 = 23 :
89
jxj > j xj = jyj > j sin yj = jxj ;
sin
If a > , then a > and there exists a prime2 p that divides a . Hen
e a
and at least one of the inequalities is stri
t if x 6 or y 6 . is
ongruent to modulo p and M a a : : : a 1 is divisible by p .
2 1 1 1
p
We also have M > a > p , implying that M is
omposite.
= 0 = 0 1 p = 1+ + + +
12-2. Answer: n.
p 1+ p
Let the
hosen integers be a1; : : : ; a and, for ea
h i ; : : :; k , let n be the 12-5. a) Let r1 , r2 , r3 be the sums of numbers in the rst, se
ond and third row, and
exponent of in the prime fa
torization of a , i.e. a b with b odd. Sin
e
1 ,
2 ,
3 be the sums of numbers in the rst, se
ond and third
olumn. Denote by
k = 1 i
ni
6 b 6 n , there are n possibilities for the numbers b . If k > n , then a the element in the i -th row and j -th
olumn, and noti
e that all the elements
2 i i = 2 i i
there exist indi
es i and j su
h that b b b and n > n . Then a of the table are non-negative.
1 i 2 1 i + 1 ij
b ni
is divisible by a b. Sin
e r1 r2 r3
1
2
3 , we have
i = j = i j i = 2
nj
j = 2
If k 6 n , then
hoose any k numbers in the set f n ; : : :; n g . None of them
+ + = + +
is divisible by another sin
e n < n . jr1
1j j r2 r3
2
3 j j r2
2 r3
3)j =
+ 1 2
2 2 ( + 1)
a11 = = ( + ) ( + ) = ( ) +(
jr2
2j jr3
3 j a22 a33 :
12-3. Let 6 IAE 6 BAI and 6 DBI 6 IBA , then 6 EIA 6 BID = =
(see Fig. 10). Applying the sine rule for triangle AEI and the equality As all the elements are non-negative, a22 and a33
annot both have minus sign
= = = = = = +
r jAI j , we obtain here and,
onsequently, a11 is equal to the sum or dieren
e of two numbers in
the table. The proof for all other elements of the table is similar.
= sin
jAE j jAI j r
sin( + )
=
sin 6 AEI =
sin sin 6 AEI
: b) The tables below satisfy the required
ondition for any real x > : 0
B x x x x
;
0 0
x 0 x x x x :
0 x 0 x 2x 2x
2
D I IMO Team Sele
tion Test: April 2001
r First Day
C E A 1. Consider on the
oordinate plane all re
tangles whose
Figure 10 (i) verti
es have integer
oordinates;
(ii) edges are parallel to
oordinate axes;
From triangle BDI , we similarly get (iii) area is , where k ; ; : : :.
2
k
= 0 1 2
Is it possible to
olor all points with integer
oordinates in two
olors so that no
jBDj = jBI j r
: su
h re
tangle has all its verti
es of the same
olor?
sin( + ) sin 6 IDB sin sin 6 IDB
=
2. Point X is taken inside a regular n -gon of side length a . Let h1; h2; : : : ; h be
Sin
e 6 AEI =
os
sin and sin 6 IDB =
os , we have the distan
es from X to the lines dened by the sides of the n -gon. Prove that
n
sin
os sin
os sin( + ) 2
: :::+ > :
1 1 1 1 1 1
jAE j jBDj + = +
r sin( + ) r sin( + )
=
r sin( + )
=
r h1
+
h2
+
hn a
19 20
3. Let k be a xed real number. Find all fun
tions f : R ! R su
h that 3. If k then f x x or f x ; if k 6 then f x k or
.
Answer: = 1 ( ) = ( ) = 0 = 1 ( ) = 1
fx
f (x) + f (y) 2 = kf (x + y2 )
( ) = 0
Substituting y in the original equation we get
= 0
for all real numbers x and y . k( f x f 2:
1) ( ) = (0) (1)
If k 6 , then substituting x in (1) we get f or f k .
So the solutions in the
ase k 6 are tthe
onstant fun
tions f x and
= 1 = 0 (0) = 0 (0) = 1
Se
ond Day
fx k .
= 1 ( ) = 0
4. Consider all produ
ts by 2, 4, 6, ..., 2000 of the elements of the set
( ) = 1
If k , then from (1) we get f
= 1 . Substituting x in the original
(0) = 0 = 0
A
n 1 1 1
; ; ; : : :; ;
o
. Find the sum of all these produ
ts.
1 1
equation we get f y 2 f y2 , and furhter substituting y we nd that
or f .
= ( ) = ( ) = 1
2 3 4 2000 2001
f(1) = 1 (1) = 0
5. Find the exponent of in the representation of the number : :{z:: : : } as For any non-negative real number z there is a real number y su
h that y2 z , =
therefore from f y 2 f y2 we get that f z > for any z > . Also,
37 111 11
|
3372000 digits
produ
t of prime powers.
( ) = ( ) ( ) 0 0
substituting x y2 in the original equation we get f y2 f y 2 , so
= ( ) = ( )
f z 6 for any z 6 . Sin
e f y 2 f y2 f y 2 f y 2 , then we
6. Let C1 and C2 be the in
ir
le and the
ir
um
ir
le of the triangle ABC , respe
- must have f y f y , i.e. f is an odd fun
tion.
( ) 0 0 ( ) = ( ) = (( ) ) = ( )
tively. Prove that, for any point A0 on C2 , there exist points B0 and C 0 su
h that
Now let x be any real number and z > , then denoting pz y we get
( ) = ( )
C1 and C2 are the in
ir
le and the
ir
um
ir
le of triangle A0 B 0 C 0 , respe
tively. 0 =
f x z f x y2 f x f y 2 f x f y2 f x f z :
( + ) = ( +
) = ( )+ ( ) = (2) ( )+ ( ) = ( )+ ( )
Solutions of Sele
tion Test
Hen
e if a 6 b , then f b f a b a f a f b a > f a , i.e. f is
non-de
reasing.
( ) = ( + ( )) = ( ) + ( ) ( )
1. Answer: Yes. Sin
e f is an odd fun
tion, (2) holds also when x and z are both negative. Now
Color the points with integer
oordinates in three
olors so that on ea
h diagonal we show, using indu
tion on n , that f nx nf x for any real x and integer n .
y x k all points are of the same
olor and the
olors
hange in a
y
li
manner
Indeed, this holds for n and if f nx nf x then
( ) = ( )
when k in
reases. Sin
e for even m and for odd
= +
m m = 0 ( ) = ( )
m , it is easy to understand that all three
olors are present in verti
es of ea
h
2 1 (mod 3) 2 2 (mod 3)
f ((n +1)x) = f (nx + x) = f (nx)+ f (x) = nf (x)+ f (x) = (n +1)f (x) :
re
tangle under
onsideration. Now re
oloring the verti
es of some
olor with one
of the remaining two
olors, we obtain a
oloring with the required properties. Sin
e f is odd, we also have f ( nx) = f (nx) = nf (x) , i.e. f (nx) = nf (x)
holds for all integers n .
2. Let S be the area of the n -gon and r its inradius, then S n ar . On the other =
Earlier we proved that f (1) = 1 or f (1) = 0 . If f (1) = 0 then substituting
x = 1 in f (nx) = nf (x) we get that f (n) = 0 for all integers n , and sin
e f is
2
hand, S a h1 h2 : : : h . Using the AM-HM inequality, we get
=
1
( + + + n) non-de
reasing, we have f (x) = 0 for all real x . We show now that if f (1) = 1
2
then f (x) = x for all x . For integers we get it from f (nx) = nf (x) , substituting
x = 1 . For a rational number we have
n a
6 h1 h2 n : : : h
+ + + n
=
2S
na
= r: b
:::+
1 1 1
h1 h2 hn
+ +
a
a
a = f (a) = f b = b f ;
Comparing the lengths of
ir
umferen
es of the n -gon and its in
ir
le, we get b b
na > r . Hen
e
2
so f ab = ab . Assume that for some real number x we have f (x) 6= x , then
1
h1
+
1
h2
+ :::+
1
hn
> nr > a :2
f (x) = x + " where " 6= 0 . If " > 0 , then let r be a rational number su
h
21 22
that x < r < x " , and if " < , then let r be a rational number su
h that and note that
x > r > x " . In the rst
ase we get r < x " f x 6 f r r , in the se
ond
+ 0
37k+1 37k 37
ase r > x " f x > f r r , a
ontradi
tion.
+ + = ( ) ( ) =
1000 1 = 1000 1 =
+ = ( ) ( ) =
37k 37k + 37k 2 + : : : + 37k 36 :
4. Answer: 499
1001
. = 1000 1 1 + 1000 1000 1000
The value of The exponent of in the representation of number 37 is k by the
2001
k
indu
tion hypothesis. Hen
e it su
es to show that the exponent of in the
37 1000 1 + 1
representation of
37
1+
1
2
1+
1
3
::: 1+
2001
1
1
37k + 37k 2 + : : : + 37k 36
is equal to the sum of all produ
ts of the elements of set A by 1, 2, 3, ..., 2000, 1 + 1000 1000 1000
and the value of is . Sin
e then 37k 1 (mod 37) . Let 37k = 37q + 1 ,
then
1 1000 1 (mod 37) 1000 1000
1
1
1
1
::: 1
1
1
2 3 2001
37 + 100037 2 + : : : + 100037 36=
k k k
is equal to a similar sum where the produ
ts by 2, 4, 6, ..., 2000 are taken with
1 + 1000
2 36
a plus sign but the produ
ts by 1, 3, 5, ..., 1999 are taken with a minus sign. = 1 + (37q + 1) + (37q + 1) : : : + (37q + 1)
Denote the required sum by S , then q + 1) + (2 37q + 1) + : : : + (36 37q + 1) =
1 + (37
=
37 36
q
37 + 37 = 37
2 18 q + 37 37 (mod 372) :
2 S = 1+
1
2
1+
1
3
::: 1+
2001
1
+
2
1
1
:::
1
So 1 + 1000
37 k
+
37 2 : : :
1000
37 36 is divisible by
k
+ + 1000 but not by 2 ,
k
37 37
+ 1
2
1
3
1
2001
2 =
and the exponent of in the representation of 37 +1 is k .
37 1000
k
1 +2
=
3
:::
4 2002
+
1
:::
2 2000
2 = Hen
e the exponent of in the representation of 372000 is .
37 1000 1 2001
6. Let I and O be the in
enter and the
ir
um
enter of the triangle ABC , respe
-
2 3 2001 2 3 2001
; tively. We know by Euler's formula that jOI j2 R2 Rr , where r and R are
2002 1 1
= + 2 = 999
the radii of the in
ir
le and the
ir
um
ir
le, respe
tively.
2 2001 2001 = 2
and S . 1001
Assume now that there exists a point A0 on the
ir
le C2 su
h that it is impossible
= 499
2001 to
onstru
t the points B0 and C 0 as required in the problem. Let the tangents
5. . drawn from A0 to the
ir
le C1 tou
h C1 in B0 and C 0 , hen
e B0C 0 is not tan-
Answer: 2001
As and are relatively prime it is su
ient to nd the exponent of in the gent to the
ir
le C1 . Suppose the line B0C 0 and the
ir
le C1 have no points in
representation of the number
37 9 37
ommon0 0(the
ase of two
ommon points is similar). Let the distan
e between the
line B C and the
ir
le C1 be Æ > . 0
3372000 372000 Now start moving the points B0 and C 0 along the
ir
le C2 towards A0 in su
h
|999 : :{z
:: : : 99} = 10 1 = 1000 1 :
a way that the distan
es from the
ir
le C1 to the straight lines A0B0 and A0C 0
3372000 numbers remain equal (note that they are both equal to at the beginning) denote
this distan
e by . The distan
e Æ obviously de
reases, whereas the distan
e
0
We show by indu
tion on k that the exponent of 37 in the representation of in
reases, hen
e at some moment they must be
ome equal. Now we
an in
rease
37
1000 is k . In the
ase k we have
k
1 +1 = 0
the radius r by Æ > to make it the in
ir
le of the triangle A0B0C 0 . Hen
e
the triangle A0B0C 0 has
ir
umradius R and inradius r Æ , but the distan
e jOI j
= 0
370 3 37 ;
is the same as for the triangle ABC , hen
e Euler's formula for triangle A0B0 C 0
1000 1 = 999 = 3 +
i.e. the exponent of is . Suppose now that for some k our assertion holds,
37 1 be
omes violated.
23 24
Estonian math
ompetitions 2001/2002 3. For any positive integer n , denote by S (n) the sum of its positive divisors (in-
luding 1 and n ).
a) Prove that S (6n) 6 12S (n) for any n .
n does the equality S (6n) = 12S (n) hold?
We thank the IMO
ommunity for many of these problems whi
h have been taken
b) For whi
h
from various materials distributed at the re
ent IMO-s.
4. In a triangle ABC we have 6 B = 2 6 C and the angle bise
tor drawn from A
interse
ts BC in a point D su
h that jAB j = jCDj . Find 6 A .
Autumn Open Contest: O
tober 2001
5. Let b1 ; b2 ; : : :; b n be a rearrangement of positive real numbers a1 ; a2 ; : : :; a n.
Juniors (up to 10th grade) Prove that
a)
a1 +
1 a + 1 : : : a + 1 > 2 n
;
b1 2
b2 n
b
1
n
1. A gure
onsisting of ve equal-sized squares is pla
ed as 7
shown in a re
tangle of size 78 units. Find the side length
b) if equality holds here for an odd n then at least one of the numbers a i is .
of the squares.
2. Find the remainder modulo 13 of the sum
8 Solutions of Autumn Open Contest
1 + 2 + 3 + : : : + 2000
2001 2001 2001 2001
+ 2001 2001
: p
J1. Answer: 5 .
3. Find all triples (x; y; z) of real numbers satisfying the system of equations (where Let a be the required side length, then the proje
tions of ea
h side of any square
[r℄ frg
and denote the integer and fra
tional part of r , respe
tively): to the sides of the re
tangle are x and y where x2 + y2 = a2 . We have
8
< x + [y℄ + fz g = 200;2
> 8 = 2x + y + x + y = 3x + 2y
>
fxg + y + [z ℄ = 200;1 : and
:
[x℄ + fyg + z = 200;0
7 = 3x + y ;
4. Consider a point M inside triangle ABC su
h that triangles ABM , BCM and p p
CAM have equal areas. Prove that M is the interse
tion point of the medians of yielding y=1 x=2 a= x +y = 5
, and 2 2 .
triangle ABC .
J2. Answer: 0 .
5. For any integer n > 1
onsider all squares with verti
es in points having non- 1001
Arrange all terms of the sum ex
ept 13 2001
(whi
h is divisible by ) in pairs
negative integer
oordinates not greater than n .
(k ; (2002 k) )
2001
2002 2001
13 k + (2002 k)
. Sin
e is divisible by ,
2001 2001
is
a) How many su
h squares are there for n = 4?
ongruent to k +( k) = 0 2001
13 2001
modulo , i.e. the sum of ea
h pair is divisible
b) Find a general formula for the number R n of su
h squares for any n. by 13 0
and hen
e the required remainder is .
J3. Answer: x = 100;15 y = 100;95 z = 99;05
the only su
h triple is ; ; .
Seniors (grades 11 and 12) [r℄+frg = r
Adding all three equations and using x + y + z = 300;15 we have . Sub-
(y [y℄)+(z fzg) = 99;95
tra
ting from here the rst given equation, we obtain
1. The sum of two distin
t positive integers, obtainable from ea
h other by rear- or fyg + [z ℄ = 99;95 fyg = 0;95 [z℄ = 99
, yielding and . Similarly we get
rangement of digits,
onsists of 2001 equal digits. Find all possible values of the [x℄+ fzg = 100;05 fxg +[y℄ = 100;15 [x℄ = 100 fzg = 0;05 fxg = 0;15
and , i.e. , ,
digits of the sum. and [y℄ = 100 .
2. The side lengths of a triangle and the diameter of its in
ir
le, taken in some order, J4. It su
es to prove that if triangles ABM and BCM have equal areas then M
form an arithmeti
progression. Prove that the triangle is right-angled. lies on the median drawn from B. Let K and H be the perpendi
ular proje
tions
1 2
of A and C to BM , and let BM interse
t AC in a point P (see Figure 1). Then Note. R R = n + (n 1) + : : : + 1
Using the identities n n 1
2 2 2
and
jBM j jAK j = S =S = jBM j2 jCH j 1 + 2 + : : : + n = n(n + 1)(2
2 2
6
n + 1)2
2 ABM BC M
and hen
e jAK j = jCH j . If AC is perpendi
ular to BM , then K = H = P ,
we
an show by indu
tion that
jAP j = jP C j and BP is a median. If AC is not perpendi
ular to BM , then (n + 1) (n + 1) 1 : 2 2
R =
learly one of K and H lies inside triangle ABC and the other one outside of it. n
12
Hen
e 6 AKP = 90Æ = 6 6 AP K = 6 CP H , i.e. triangles AKP and
CHP and
CHP are
ongruent, whi
h again yields jAP j = jP C j and BP being a median. S1. Answer: 1 2 3 4 5 6 7 8
, , , , , , , .
Let A 0 1 2 3
be any of the digits , , , . Taking
B n = A (A+5) : : : (A+5)(A+5)A(A+6); m = A (A+5) : : : (A+5)(A+6)A(A+5);
| {z } | {z }
1997 digits 1997 digits
H r r r r
M r r r r r r r or
r r r r r r r r r
n =(A+1) : : : (A+1) A(A+2); m =(A+1) : : : (A+1)(A+2)A;
A P C r r r r r r r r r | {z } | {z }
K n=1 n=2 n=3
1999 digits 1999 digits
we have n and m obtainable from ea
h other by rearrangement of digits and
Figure 1 Figure 2
n + m =(2A+1) : : : (2A+1); n + m =(2A+2) : : : (2A+2) ;
| {z } | {z }
Answer: a) 50 ; b) 1 n + 2 (n 1)2 + : : : + (n 1) 22 + n 12 .
2
J5. 2001 digits 2001 digits
We rst study the possible squares for n = 1; 2; 3 .
1 8
For n = 1 we have a single square of side length 1 .
respe
tively. Hen
e to are possible digits.
m+n = 99 9
:::}
n = 2 we have 2 2 possible lo
ations
p for the square of side length 1
Suppose now that . Moving from right to left it is easy to see
For and new
| {z
2001 digits
possible squares of side lengths 2 and 2 , one of ea
h. that there
an be no
arries during the addition. Hen
e any digit A o
urs in
For n = 3 , we have 3 3 possible lo
ations for the square of side length
p 1 22
, n in these and only these positions where the digit 9 A o
urs in m . Sin
e n
possible lo
ations for ea
h of the squares of side lengths 2 and 2 and three new and m are obtainable from ea
h other by rearrangement of digits then any digit
types of squares, one of ea
h (see Figure 2). A o
urs in n the same number of times as 9 A 6= A , and the number of digits
in n, m and also n+m must be even a
ontradi
tion.
We see that for ea
h n we have 1=1 2
possible lo
ation for ea
h of the new
S2. Drawing a line through the in
enter of a triangle parallel to ea
h of its sides it is
squares (i.e. squares having all their verti
es at the edges of the grid) and for the
next values of n we have 2 ; 3 ; 4 ; :::
2 2 2
possible lo
ations for these squares. It
easy to see that the diameter of the in
ir
le is shorter than any of the sides. Let
x d>0
n and be the diameter of the in
ir
le and the dieren
e of the arithmeti
x + d , x + 2d and x +3d .
remains to noti
e that the number of the new squares is sin
e we
an pla
e
one of its verti
es either in a
orner of the grid or in one of the n 1 points on
progression, then the side lengths are
S of the triangle in two ways we get
Finding the area
the side of the grid, thereby determining the lo
ations of the other three verti
es.
q
n x
p = S = p p (x + d) p (x + 2d) p (x + 3d) :
Hen
e for any we have
2
R n = 1 n + 2 (n 1) + 3 (n 2) + : : : + (n 1) 2 + n 1
2 2 2 2 2
; s
R4 = 50 . p=
3(x + 2d) 3(x + 2d)x = 3(x + 2d)(x + 4d)(x + 2d)x
yielding Sin
e
2 4 we have
16 , yield-
ing 3x = x + 4d x = 2d and x + d = 3d x + 2d = 4d
. Hen
e the side lengths are ,
3 4
and x + 3d = 5d , i.e. the triangle is right-angled. Sin
e i1 b =a i2 and b2
i =a i1 , we
an omit a1
i and a2
i and use the indu
tion
S3. Answer: b) for n not divisible by 2 or 3 .
hypothesis.
1 1 1
n 6n If i3 6= i1 , we nd a4 =b3 = = , a = b 4 = a = a 1 et
. Sooner or
Considering the representations of and as produ
ts of primes we see that i i
a3 a1 5 i i i
any positive divisor of 6n is the produ
t of a positive divisor of 6 and a positive i +1 = i1 for some even k (sin
e a 1 6= 1 ). Similarly to the
i i 4 i
6n are numbers of the form d , 2d , 3d
later we must have
n.
k i
divisor of Hen
e the positive divisors of a 1 ; a 2 ; : : :; a k and use the indu
tion hypothesis.
6d
previous
ase we
an now omit
d n , and
i i i
and where is a positive divisor of
S (6n) 6 S (n) + 2S (n) + 3S (n) + 6S (n) = 12S (n) :
Spring Open Contest: February 2002
Here equality holds if and only if the abovementioned four series of divisors do
not interse
t, i.e. no divisor d of n
an be represented as 2d or 3d where d is
0 0 0
another divisor of n , or equivalently n is not divisible by 2 or 3 .
Juniors (up to 10th grade)
Answer: 72 .
S4.
Æ 1. Is it possible to arrange the integers 1 to 16
Let jAB j = jCD j = a , 6 C = and 6 A = 2 , then 6 CAD = 6 BAD = , a) on a straight line;
6 B = 2 and 6 BDA = + . Applying the sine rule in triangles ACD and ABD b) on a
ir
le
we have
so that the sum of any two adja
ent numbers is the square of an integer?
sin jADj sin 2
sin = a = sin( + ) ; 2. Does there exist a re
tangle with integer side lengths with the square of its diagonal
equal to 2002 ?
2 sin
os =Æ sin( + ) tan = tan 0 < ; < 90Æ
In a triangle ABC we have jAB j = jAC j and 6 BAC = . Let P 6= B be a point
yielding , or . Sin
e we have
= . 180 = 2 +2 + = 5
Now from = 36Æ 6 A = 2 = 72Æ we have and . 3.
on AB and Q a point on the altitude drawn from A su
h that jP Qj = jQC j .
S5. a) Using the AM-GM inequality we have: Find 6 QP C .
1
a1 + : : : a +
1 > 2
r
a1
:::2
r
a
n
=2 n
r
a1 : : : a n
=2 n
: 4. Dene a1 ; a2 ; : : : ; a ; : : : as follows:
b1 : : : b
n
b1 b1 n
b b
a1 = 0; a2 = 1; a = 5a n > 2.
n n n
a ;
b) If n = 1 then a1 +
1
= 2 and hen
e a1 = 1 . Suppose now that the
laim is true
n n 1 n 2 for
a1 For whi
h n is a divisible by: a) 5 ; b) 15 ?
1 = 2r a
n
for any odd integers less than n . The equality holds if and only if a +
i
b b i 5. For whi
h positive integers n is it possible to write n real numbers, not all equal
1 . If a = b for some i then a = b = 1 and we are done.
i i
0
for ea
h i , i.e. a =
to , on a
ir
le so that ea
h of these numbers is equal to the absolute value of
b i i i i i the dieren
e of its two neighbouring numbers?
a 6= b for all i then
onsider some i1 su
h that a 1 6= 1 . Then b 1 equals to
i
If
1
i i i i
some a2 i2 6= i1 , and 2
where a =b =
. Also, b 2 equals to some a 3 where Seniors (11th and 12th grade)
i 1 i
a i i i
1 =a
i1
i3 6= i2 . a3 =b2=
a , b and
of a right triangle form a geometri
progression, and ab
= 1 .
Hen
e .
i i
a2 i1
1. The sides
=i
i
If i3 1 then Find a, b,
.
+1 a + 1 = a + 1 a + 1 = a + a1 a + a1 =
a 2. Let a; b be any real numbers su
h that jaj 6= jbj . Prove that
i1
b1 i
i2
b i2 a a
i1
i2
i2
i3
i1
i2
i2
i1
ab
= (a + a ) a1 + a1 = 2a a2 = 2 : + b > 1 :
a
2
i1 i1 i1 a b
i1 i1 i1
5 6
Let ABCD be a rhombus with 6 DAB = 60 . Let K , L be points on its sides AD Sin
e jAB j = jAC j then the altitude drawn from A is also an angle bise
tor.
3.
Æ
and DC and M a point on the diagonal AC su
h that KDLM is a parallelogram. Note that jQB j = jQC j = jP Qj (see Figure 3), i.e. the triangles BQC , BQP
BKL is equilateral.
Prove that triangle and P QC are isos
eles. Denote 6 QBC = 6 QCB = , 6 QBP = 6 QP B =
and 6 QP C = 6 QCP = Æ , then 6 QCA =
. From triangle ABC we now have
4. Call a 10 -digit natural number magi
if it
onsists of 10 distin
t digits and is + 2 + 2
= 180Æ and from triangle P BC we have 2Æ +2 +2
= 180Æ . Hen
e
divisible by 99999 . How many su
h magi
numbers are there (not starting with
6 QP C = Æ = .
digit 0 )? 2
5. Find the maximum number of distin
t four-digit positive integers
onsisting only J4. Answer: a) for any odd n ; b) for n = 6k + 1 .
of digits 1 2
, and 3 su
h that any two of these numbers have equal digits in at a) From the equality a = 5a 1 a 2 we see that a is divisible by 5 if and
5 a1 = 0 5 a2 = 1
n n n n
most one position? only if a 2 is divisible by . Sin
e is divisible by but is not
5 5
n
divisible by , then a n is divisible by if and only if n is odd.
b) Taking n+1 instead of n in the given equality we have
Solutions of Spring Open Contest
an+1 = 5a n a n 1 = 5 (5a n 1 a n 2 ) an 1 = 24a n 1 5an 2 :
J1. Answer: a) yes; b) no.
From here we see that a is divisible by 3 if and only if a 2 is divisible by 3 .
4 = 16 < 16 + 1 6 = 36 > 16 + 15 9 16
n+1
a1 = 0 3 a2 = 1 and a3 = 5 1 0=5
n
2 2
Sin
e and then only
an be adja
ent to Sin
e is divisible by but are not divisible
(giving 16 + 9 = 25 = 5 2
). Hen
e it is impossible to arrange the numbers 1 to 16 by 3 , then a is divisible by 3 if and only if n = 3m + 1 for some m . Hen
e a
15 n = 3m + 1 , i.e.
n n
on a
ir
le in the required manner. A suitable arrangement on a straight line is: is divisible by if and only if n is both odd and of the form
n = 6k + 1 .
16; 9; 7; 2; 14; 11; 5; 4; 12; 13; 3; 6; 10; 15; 1; 8 :
J5. Answer: if and only if n is divisible by 3 .
J2. Answer: no. Sin
e ea
h number written on the
ir
le is equal to the absolute value of the dif-
We need to determine whether there exist positive integers a and b su
h that feren
e of two others, then all these numbers are non-negative. Let a be maximal
a2 + b = 2002
2
2002 7 among these numbers (then a > 0 ) and let b and
be the numbers adja
ent to
b >
> 0. b 6 a then also b
6 a , and the equality a = b
. Note that is divisible by and the square of any integer
0 1 2 4 7 a b it, with Sin
e
b=a
= 0 . Hen
e any number a on the
ir
le must have a
is
ongruent to , , or modulo . Hen
e and must both be divisible
7 a +b 2 2
49 2002 49 holds only if and
0 0
by , but then is divisible by , and is not divisible by .
and as its neighbours, and any number must have its both neighbours equal.
We see that the numbers on the
ir
le must be a , a , 0 , a , a , 0 , . . . (see Figure 4)
A and hen
e n must be a multiple of 3 .
P
22 ar
r
Æ
ar
Q 0r r
Æ r r
a a
r
0
B C Figure 4
On the other hand it is easy to
he
k that for any n = 3k and a > 0 the numbers
Figure 3
a; a; 0; a; a; 0; : : :; a; a; 0} satisfy the required
onditions.
6 QP C = . | {z
J3. Answer:
2 k triples a; a; 0
7 8
s p s p 3456
5 1 5+1 S4. Answer: .
S1. Answer:
2 1 , and
2 .
We
an write any ten-digit number ab
defghij as
b ab
defghij = 100000 ab
de + fghij =
Assume w.l.o.g. that a < b <
, then a= and
= bq for some q > 1. Hen
e
q
from ab
= 1 b =1
we have
3
and b=1 . From the Pythagorean Theorem we
= 99999 ab
de + ab
de + fghij :
1
+1=q q 1=0 x x 1
2
now have
q
2
, or
4
q2 . Sin
e the equation
2
has Hen
e ab
defghij 99999
is divisible by if and only if the sum ab
de + fghij is
p s p sp divisible by 99999 . Sin
e ea
h summand here is positive and less than 99999 , we
5+1 q=
5+1 1= 5 1 must have ab
de + fghij = 99999 , or equivalently
2 as its only positive solution, then
2 and
q 2 .
a+f = b+g =
+h= d+i = e+j = 9
S2. For any real y and x > 0 we have:
a) x > 1 , if x > 1 and y > 0 or x < 1 and y < 0 ;
y (sin
e the sum
ontains only digits 9 , no
arries
an o
ur on addition). We see
b) x < 1 , if x > 1 and y < 0 or x < 1 and y > 0 ;
y that magi
numbers are in one-to-one
orresponden
e with numbers of the form
) x = 1 , if x = 1 or y = 0 .
ab
de where a; b;
; d; e are ve distin
t digits su
h that a 6= 0 and the sum of
9 9 8 6 4 2 = 3456
y
no two of them is . There are su
h numbers ab
de .
We shall now
onsider the
ases where ab is positive, negative or equal to zero.
If ab > 0 , then a and b have the same sign and ja + bj > ja bj > 0 , hen
e S5. Answer: 9 .
a + b > 1 and a + b > 1 .
ab
Note that we
annot have more than 33 = 9 integers with the required property
a b a b sin
e the pairs of rst two digits of any two of them must be distin
t. A suitable
If ab < 0 , then a and b have opposite signs and ja bj > ja + bj > 0 , hen
e set of 9 integers is 1111 1222 1333 2123 2231 2312 3132 3213 3321
, , , , , , , , .
0 < aa + bb < 1 and aa + bb > 1 .
ab
Final Round of National Olympiad: Mar
h 2002
If ab = 0 , then a
+ b = 1 sin
e a + b 6= 0 .
ab
a b a b
9th grade
S3. The rhombus ABCD
onsists of two equilateral triangles ABD and BCD . We
shall prove that jKD j = jLC j (see Figure 5) then triangles KBD and LBC are
K and L are taken on the sides BC CD ABCD
jKB j = jLB j , 6 KBD = 6 LBC . Hen
e 6 KBL = 6 DBC = 60Æ ,
1. Points and of a square so that
ongruent and 6 AKB = 6 AKL . Find 6 KAL .
i.e. the triangle BKL is equilateral.
2. Do there exist distin
t non-zero digits a, b and
su
h that the two-digit number
D L C ab is divisible by
, the number b
is divisible by a and
a is divisible by b?
K 3. Let a1 ; a2 ; : : : ; a be pairwise distin
t real numbers and m be the number of
a + a (where i 6= j ). Find the least possible value of m .
n
M distin
t sums i j
4. Mary writes 5 numbers on the bla
kboard. On ea
h step John repla
es one of the
A B numbers on the bla
kboard by the number x + y z , where x , y and z are three
Figure 5 of the four other numbers on the bla
kboard. Can John make all ve numbers on
the bla
kboard equal, regardless of the numbers initially written by Mary?
To prove the equality jKD j = jLC j note that LM is parallel to AD and
6 LMC = 6 DAC = 6 DCA = 6 LCM . Hen
e the triangle MLC is isos
eles, 5. There were n> 1 aborigines living on an island, ea
h of them telling only the
jLC j = jLM j = jKDj .
i.e. truth or only lying, and ea
h having at least one friend among the others. The
9 10
new governor asked ea
h aborigine whether there are more truthful aborigines or 3. The tea
her writes a 2002 -digit number
onsisting only of digits 9 on the bla
k-
liars among his friends, or an equal number of both. Ea
h aborigine answered that board. The rst student fa
tors this number as ab with a>1 and b>1 and
there are more liars than truthful aborigines among his friends. The governor then
0
repla
es it on the bla
kboard by two numbers a and b su
h that ja a j = 2 and
0 0
ordered one of the aborigines to be exe
uted for being a liar and asked ea
h of jb b0 j = 2 . The se
ond student
hooses one of the numbers on the bla
kboard,
the remaining n 1 aborigines the same question again. This time ea
h aborigine fa
tors it as
d with
>1 and d>1 and repla
es the
hosen number by two
answered that there are more truthful aborigines than liars among his friends. numbers
0 and d0 su
h that j
0j = 2 and jd d0j = 2 . The third student again
Determine whether the exe
uted aborigine was truthful or a liar, and whether
hooses one of the numbers on the bla
kboard and repla
es it by two numbers
there are more truthful aborigines or liars remaining on the island. following a similar pro
edure, et
. Is it possible that after a
ertain number of
students have been to the bla
kboard all numbers written there are equal to 9 ?
10th grade
4. Let a1; a2; a3; a4 ; a5 be real numbers su
h that at least N of the sums a + a , i j
where i < j , are integers. Find the greatest value of N for whi
h it is possible
that not all of the sums a + a are integers.
i j
1. The greatest
ommon divisor d and the least
ommon multiple v of positive
integers m and n satisfy the equality 3m + n = 3v + d . Prove that m is divisible 5. John built a robot that moves along the border of a regular o
tagon, passing ea
h
by n. side of the o
tagon in exa
tly 1 minute. The robot begins its movement in some
vertex A of the o
tagon, and further on rea
hing ea
h vertex
an either
ontinue
2. Let ABC be a non-right triangle with its altitudes interse
ting in point H . Prove movement in the same dire
tion, or turn around and
ontinue in the opposite
that ABH is an a
ute triangle if and only if 6 ACB is obtuse. dire
tion. In how many dierent ways
an the robot move so that after n minutes
it will be in the vertex B opposite to A?
3. John takes seven positive integers a1 ; a2 ; : : : ; a7 and writes the numbers aa ,
+a i 6= j
i j
a i j and ja i aj
j for all on the bla
kboard. Find the greatest possible
number of distin
t odd integers on the bla
kboard.
12th grade
4. Find the maximum length of a broken line on the surfa
e of a unit
ube, su
h that
its links are the
ube's edges and diagonals of fa
es, the line does not interse
t itself 1. Peter, John, Kate and Mary are standing at the entran
e of a dark tunnel. They
and passes no more than on
e through any vertex of the
ube, and its endpoints have one tor
h and none of them dares to be in the tunnel without it, but the
are in two opposite verti
es of the
ube. tunnel is so narrow that at most two people
an move together. It takes 1 minute
for Peter, 2 minutes for John, 5 minutes for Kate and 10 minutes for Mary to
5. The tea
her writes numbers 1 at both ends of the bla
kboard. The rst stu- pass the tunnel. Find the minimum time in whi
h they
an all get through the
dent adds a 2 in the middle between them; ea
h next student adds the sum tunnel.
of ea
h two adja
ent numbers already on the bla
kboard between them (hen
e
there are numbers 1; 3; 2; 3; 1 on the bla
kboard after the se
ond student; 2. Does there exist an integer
ontaining only digits 2 and 0 , whi
h is a k -th power
1; 4; 3; 5; 2; 5; 3; 4; 1 after the third student et
.) Find the sum of all numbers of a positive integer with k > 2?
on the bla
kboard after the n -th student.
3. Prove that for positive real numbers a, b and
the inequality
11th grade
2(a + b +
) < (a + b +
)
4 4 4 2 2 2 2
holds if and only if there exists a triangle with side lengths a, b and
.
1. Determine all real numbers a su
h that the equation x8 + ax4 + 1 = 0 has four
real roots forming an arithmeti
progression. 4. All verti
es of a
onvex quadrilateral ABCD lie on a
ir
le ! . The rays AD ,
BC interse
t in point K and the rays AB , DC interse
t in point L . Prove that
2. Inside an equilateral triangle there is a point su
h that the distan
es from it to the
ir
um
ir
le of triangle AKL ! if and only if the
ir
um
ir
le
is tangent to
the sides of the triangle are 3 4 , and 5 . Find the area of the triangle. of triangle CKL is tangent to !.
11 12
5. There is a lottery at John's birthday party with a
ertain number of identi
al i.e. there are at least 2n 3 distin
t sums. Taking a =i we have 1+2 = 3
(n 1) + n = 2n 1
i
prizes, whereas ea
h of the guests
an win at most one prize. It is known that as the minimal sum and as the maximal sum, so there are
if there was one prize less than there a
tually is, then the number of possible exa
tly 2n 3 distin
t sums in this
ase.
distributions of the prizes among the guests would be 50% less than it a
tually
9-4. Answer: yes.
is, while if there was one prize more than there a
tually is, then the number of
50% Denote the numbers written by Mary by a , b ,
, d and e (not ne
essarily distin
t).
b x =
+d e
possible distributions of the prizes would be more than it a
tually is. Find
First John
an repla
e ea
h of a and by . Then he
an repla
e ea
h
d e+x x = e e + e e = e:
the number of possible distributions of the prizes.
of
and by and nally repla
e both numbers x by
Solutions of Final Round (a; b;
; d; e) ! (x; x;
; d; e) ! (x; x; e; e; e) ! (e; e; e; e; e) :
9-5. Answer: the exe
uted aborigine was truthful and after the exe
ution only liars
Answer: 45 .
9-1.
Æ remained on the island.
Let AM be the perpendi
ular drawn from A to KL (see Figure 6). Sin
e ABK First note that there was a truthful aborigine on the island before the exe
ution,
and AMK are
ongruent right triangles then jAM j = jAB j = jAD j , i.e. AML sin
e otherwise all friends of ea
h aborigine would have been liars, and hen
e their
and ADL are also
ongruent right triangles, and answers would have been true a
ontradi
tion.
6 KAL = 6 KAM + 6 LAM = 6 KAB + 6 LAD ; Suppose now there was a truthful aborigine on the island after the exe
ution.
Then both his answers must have been true but this is impossible sin
e the
when
e exe
ution of one aborigine
ould not
hange the dieren
e of the numbers of liars
26 KAL = 6 KAM + 6 LAM + 6 KAB + 6 LAD = 90Æ
and truthful aborigines among his friends from positive to negative.
Let m = dm and n = dn where g
d (m ; n ) = 1 . Then v = m n d and we have
0 0 0 0 0 0
6 KAL = 45Æ .
10-1.
and 3m0d + n0d0 = 3m0n0d + d , yielding 3m0 + n0 = 3m0n0 +1 and (3m0 1)(n0 1) = 0 .
Sin
e 3m 1 6= 0 , then n0 1 = 0 and hen
e n = d divides m .
D L C 10-2. If H is the ortho
enter of triangle ABC , then C is the ortho
enter of triangle
M ABH . We'll
onsider three possible
ases.
(1) If ABC is an a
ute triangle (see Figure 7), then H lies inside triangle ABC
K
and C lies outside triangle ABH , when
e ABH is an obtuse triangle.
(2) If ABC is an obtuse triangle with 6 ACB a
ute, then assume w.l.o.g. that
A B 6 B is obtuse (see Figure 8). Then C and H lie on opposite sides of AB . Hen
e
Figure 6 C is outside triangle ABH , and ABH is an obtuse triangle.
(3) If 6 ACBis obtuse (see Figure 9), then C lies inside triangle ABH , when
e
9-2. Answer: no.
ABH is an a
ute triangle.
Note that if a, b and
satisfy the required
onditions and one of them is even,
a b
then all three are even. Then
2 2
, and
2 also satisfy the required
onditions.
C H H
Hen
e we
an assume w.l.o.g. that a, b and
are all odd. Also note that none
of these numbers
an be 5 , sin
e then the other two should also be 5 . Hen
e it
H B C
su
es to
onsider 1 , 3 , 7 and 9 and one of a , b and
must be 3 or 9 let
this be a . Then b
is divisible by 3 , whi
h gives fb;
g = f3; 9g , a
ontradi
tion.
9-3. Answer: 2n 3. A B A C A B
We
an assume w.l.o.g. that a1 < a2 < : : : < a . Then n Figure 7 Figure 8 Figure 9
a1 + a2 < a1 + a3 < : : : < a1 + a < a2 + a < : : : < a 1 + a ;
n n n n 10-3. Answer: 30 .
13 14
If there are m odd integers among a i, then the maximum number of odd integers Answer: a =
82
written on the bla
kboard is
11-1.
9 .
t=x 4
t x =t 4
m(m 1)
+ 2 m(7 m) = 1 (m m + 28m 4m ) =
Substituting we get a quadrati
equation in , and the equation 0
f (m) = 2 2
2 2
has at most two roots whi
h in this
ase have equal absolute values. Hen
e the
four roots of the equation x + ax + 1 = 0 8 4
x have to be of the form 0 and
= 32 m(9 m) = 32 814 29 m x
2
1 . Assuming w.l.o.g. that x >x 1 0 we see that these roots form an arithmeti
progression if and only if x = 3x t + at +1
0 . Sin
e the roots of the equation
2
are
x 4
81x 4
81x = 1
1
x =
1 8
a = 82x =
82 4 4
and the maximum value of f (m) is f (4) = f (5) = 30 . 9 9
then 0 and 0 , we have 0 , yielding 0 and 0 .
a1 = 2 , a2 = 4 ,
It remains to
he
k that these numbers
an all be distin
t, e.g. for
a3 = 6 a4 = 25 = 52 , a5 = 125 = 53 , a6 = 15625 = 56 , a7 = 9765625 = 510 . 36 + 25p3
and 11-2. Answer:
4 .
Consider a triangle ABC P with a point jP Aj = 3 jP B j = 4 inside su
h that ,
and jP C j = 5 60Æ
. Rotating the triangle by C A B B around , we map to and
s
to some B0 (see Figure 12). P P0 Then jP 0B j = jP Aj = 3
maps to , where ,
s
jP 0B 0 j = jP B j = 4 jP 0C j = jP C j = 5
and CP P 0 . Sin
e is an equilateral
triangle then jP P 0j = 5 jP B j + jP 0B j = 4 + 3 = 5 = jP P 0j
. Hen
e
2 2 2 2 2 2
,
s
and 6 P BP 0 = 90Æ AP B
. From triangles BP 0 B 0 and we have
s 6 ABP + 6 BAP = 6 ABP + 6 B 0 BP 0 = 120Æ 90Æ = 30Æ
Figure 10 Figure 11 and 6 AP B = 180Æ 30Æ = 150Æ AP B . The
osine law in now gives
p
p jAB j = jAP j + jBP j 2 jAP j jBP j
os 6 AP B = 25 + 12 3
2 2 2
3+4 2 p
36 + 25p3
10-4. Answer: .
1 S=
3
p 4 jABj = 4
The links of su
h a broken line are the edges of the
ube (of length ) and the 2
2
and the area is .
diagonals of its fa
es (of length ). Sin
e the line passes ea
h vertex at most
on
e, it
an have at most 7 links. Coloring the verti
es as shown on Figure 10 we
see that opposite verti
es are of dierent
olour and the endpoints of the diagonal B0 C
of ea
h fa
e are of the same
olour hen
e an odd number of links have to be
4 5
P0
edges of the
ube. Also, it is
lear from this
olouring that no more than three
onse
utive links
an be diagonals (sin
e the
ube has only 4 verti
es of ea
h
5
olour). It is now easy to
he
k that a broken line with
p 1 edge and 6 diagonals is
impossible, hen
e its length
annot ex
eed 3+4 2 ; a suitable broken line of this 3 4 3
length is shown on Figure 11. P
10-5. Answer: 3 +1
n
.
B A
Let S be the sum of the numbers on the bla
kboard after the n -th student. We Figure 12
= 3 +1
. Indeed, S0 = 2 = 3 + 1 and ea
h
n
shall prove by indu
tion that Sn
n0
k -th student is
ounted in exa
tly two 11-3. Answer: no.
2002 999 : : : 9 3 4 = ab
number that is on the bla
kboard after the
(k+1) 1 The initial -digit number is
ongruent to modulo . If N
3 4 3
of the sums written by the -th student, with the ex
eption of the two -s
written by the tea
her whi
h are
ounted only on
e hen
e and N is
ongruent to modulo then one of a and b is
ongruent to and
the other is
ongruent to 1 modulo 4 and the same is true for a0 and b0 . Hen
e
S = S + 2S 2 = 3(3 + 1) 2 = 3
k k +1
+1: at all times there is a number on the bla
kboard whi
h is
ongruent to 3 modulo
4 9 1 4
k +1 k k
, while is
ongruent to modulo .
15 16
11-4. Answer: 6 . Note that it is possible to get everyone through the tunnel in 17 minutes:
If there are four integers and one non-integer among a i then N =6 . To prove 1) Peter and John go through the tunnel (2 minutes);
the maximality we denote the fra
tional part of x by fxg and note that: 2) Peter brings ba
k the tor
h (1 minute);
(a) iffag =6 fbg and
is any real number then at most one of
+ a and
+ b 3) Kate and Mary go through the tunnel (10 minutes);
4) John brings ba
k the tor
h (2 minutes);
is an integer;
if a = b then a + b is an integer if and only if fag = 0 or fag = 0;5 ;
5) Peter and John go through the tunnel (2 minutes).
(b)
if fag 6= fbg and a + b is an integer then neither fag nor fbg is 0 or 0;5 .
It remains to show the minimality of this total. Clearly they have to go through
(
)
the tunnel an odd number of times and bring ba
k the tor
h at least twi
e, hen
e
Considering now the possible partitions of the set fa1; a2 ; a3 ; a4 ; a5 g to subsets they have to go through the tunnel at least 3 times in one dire
tion and 2 times
(of elements with equal fra
tional parts) and keeping in mind the above remarks 7
6
in another dire
tion. If they go through the tunnel or more times then the total
(a), (b) and (
) we see that the only
ase when there
an be more than integer 10+2+5 1 = 17
+a fa1 g = fa2g = fa3g = fa4 g = fa5g , and in this
ase all
time
annot be less than minutes. If they go through the tunnel
sums a i j is when 5 times then ea
h pass in the initial dire
tion takes at least 2 minutes and one
these sums are integers. of these (with Mary) takes 10 minutes. If Peter brings ba
k the tor
h both times
11-5. Answer: 2 (2k 1 k 1
1) for n = 2k , and 0 for n = 2k + 1 .
then Kate and Mary
annot go through the tunnel together and they need at least
10 + 5 + 2 + 2 1 = 17 minutes. If someone else brings ba
k the tor
h at one time
Colour the verti
es of the o
tagon alternately bla
k and white. Sin
e A and B then they need at least 10 + 2 + 2 + 2 + 1 = 17 minutes.
are of the same
olour and in ea
h minute the robot moves from a vertex of one
olour to a vertex of the opposite
olour, then n must be even. 12-2. Answer: no.
We now label the verti
es by 1 8 A = 1 and B = 5 , and denote to so that by Consider an integer N
ontaining only digits 2 and 0 and ending in t zeroes
a = (a ; a ; : : :; a )
(k ) (k ) (k ) (k )
the numbers of possibilities, for the robot to rea
h ( t>0 ), then
1; 2; : : : ; 8 A=1 k
1 2 8
verti
es from in minutes. Using indu
tion by m we shall
N = 2 : : : 2 10 = 1 : : : 1 2 5 ;
m>1
t t+1 t
prove that for any
a = (2 + 2 ; 0; 2 ; 0; 2 2 ; 0; 22 ; 0) :
where the dotted part in 2 : : :2
an
ontain both 2 -s and 0 -s (in 1 : : :1 both 1 -s
0 1 : : :1 2 5
(2m) 2m 2 m 1 2m 2 2m 2 m 1 m 2
and -s respe
tively). Sin
e is not divisible by or then in the
ase
Obviously we have a = (2; 0; 1; 0; 0; 0; 1; 0) (2)
. Suppose now that the
laim is true
when N =n k
both t+1 and t have to be multiples of k , yielding k = 1 .
form=k 2 =s and denote
k 1
, then 12-3. The given inequality is equivalent to
= (s + s; 0; s ; 0; s s; 0; s ; 0) :
a(2 k) 2 2 2 2 a4 + b4 +
4 2a b 2b
2
a
2 2 2 2 2 2
<0:
Sin
e the robot
an move to any vertex M from either of its neighbouring verti
es, Transforming the left side of this inequality we have
we have
a4 + b +
2a b 2b
2
a = (a + b
)
4 4 2 2 2 2 2 2 2 2 2 2
4a b =
2 2
a(2 k +1)
= (0; 2s + s; 0; 2s 2 2
s; 0; 2s2 s; 0; 2s2 + s) = (a + b
2ab)(a + b
+ 2ab) =
2 2 2 2 2 2
= (a b)
) (a + b)
=
2 2 2 2
and similarly
= (a b +
)(a b
)(a + b +
)(a + b
) :
a(2 k +2)
= (4s + 2s; 0; 4s ; 0; 4s 2 2 2
2s; 0; 4s ; 0) :
2
Hen
e the given inequality is equivalent to
Sin
e 4s = 2 2
2s = 2 2k
and
k
, we have proved the
laim for m = k + 1 . Hen
e (a + b +
)(a + b
)(b +
a)(
+ a b) > 0 : (1)
the number of possibilities to rea
h B =5 from A =1 in n = 2k minutes is
22k 2
2
= 2 (2k 1
1) k 1 k 1
. Here the rst term is positive and at most one of the other three
an be negative
(e.g. a + b
< 0 and b +
a < 0 would give 2b < 0 a
ontradi
tion). Hen
e
12-1. Answer: 17 minutes. (1) holds if and only if a , b and
satisfy the triangle inequalities.
17 18
!1 !2 be the
ir
um
ir
les of triangles AKL CKL (see Figure 13). n
12-4. Let and and
We have possible distributions of k prizes among n guests, and
Suppose that ! and ! 2 are tangent to ea
h other in point C , and let l 2 be their k
ommon tangent. Then
n
6 KLC = 6 KCl2 = 6 BCl2 = 6 BDC : k+1
= nk+1k n
k
: (2)
Hen
e KL k BD and 6 ADB = 6 AKL therefore then angle between AB and
! in A is equal to the angle between AL and the tangent to ! Let n be the number of guests and m the a
tual number of prizes, then we
the tangent to 1
n
= 2
n
n
= 3 n
2 m
A. A, B , L ! !1
m 1 m+1
in Sin
e the points are
ollinear then the tangents to and in have and . Substituting from (2) we
A
oin
ide, i.e. these
ir
les are tangent to ea
h other. m
n m+1
This argument
an be reversed to show that
in A implies ! and !2
! and !1
being tangent to ea
h other in
being tangent to ea
h other
C.
have
m
mn 1 = 2 mn 1 n m n
m+1 m
= 23 mn
and . Hen
e
n m + 1 = 2m n = 3m 1
, yielding 2(n m) = 3(m + 1)
, and . Plugging in
n = 3m 1 4m 2 = 3m + 3
here we have m=5 n = 14 , when
e and . It
14
remains to
al
ulate
5 = 2002 .
l2
Kq
IMO Team Sele
tion Test: May 2002
Dq
First Day
qC
q
!2 1. The prin
ess wishes to have a bra
elet with r rubies and s emeralds arranged
q in su
h order that there exist two jewels on the bra
elet su
h that starting with
q B L
A these and enumerating the jewels in the same dire
tion she would obtain identi
al
! sequen
es of jewels. Prove that it is possible to fulll the prin
ess's wish if and
l1 !1 only if r and s have a
ommon divisor.
2. Consider an isos
eles triangle KL1 L2 with jKL1j = jKL2 j , and let KA , L1 B1 ,
Figure 13
L2 B2
os 6 B1 AB2 <
3
Alternative solution. Let !1 and !2 be the
ir
um
ir
les of triangles AKL
1
ab
+
1+2
1
b
+
1+2
1
a
1+
1
a2 + b2
+
1+
1
b2 +
2
+
1+
1
2 + a2
:
ter away from O , the angle KOL decreases, so the segment
bounded by chord LM and arc LAM decreases, too. Hence it Applying now the AM-HM inequality to the whole expression, one obtains
suffices to show that, if \KOL = 75Æ , the area of this segment is
B
more than one third of the area of the circle. Figure 8 1
a 2 + b2
+
1
b 2 +
2
+
1
2 + a2
Æ ; then the area of the sector r
1 + 1+ 1 +
r be the radius and \KOL
150
Let LOM is 2
= 75 = 3
360 3 =
5
r 2
and the area of triangle LOM is
(1 + a2 + b2 ) + (1 + b2 +
2 ) + (1 +
2 + a2 )
12 9 9
:
SLOM = 2 SLOK = jLK j jOK j = r2 sin \KOL
os \KOL =
=
3 + 2( a 2 + b2 +
2 )
=
3 + 2 3
= 1
r2 sin 2\KOL = r2 :
1 1
=
2 4
Solution 2. Applying the AM-HM inequality, one obtains
Hence the area of the segment bounded by chord LM and arc LAM is 1
+
1
+
1
9
=
1 + 2 ab 1 + 2 b
1+ 2
a 3 + 2 ab + 2b
+ 2
a
5
r 2 1
r2 =
5 1
r2 : =
9
=
9
:
12 4 12 4 a2 + b2 +
2 + 2ab + 2b
+ 2
a ( a + b +
)2
1 1 5 1 4 1
As > 3 implies < , we have > = . Consequently, the area of the The Jensen inequality for the square function establishes
4 12 12 4 12 3
segment under consideration is indeed greater than one third of the area of the circle.
a + b +
)2
a b 2 +
2
2
( + 3
= = 1 :
10. Let K , L, M be the basepoints of the altitudes drawn from the vertices A, B , C 9 3 3
! ! ! !
of triangle ABC , respectively. Prove that AK + BL + CM = 0 if and only if ABC is Thus
equilateral. (Grade 12.)
! ! !
Solution. If triangle ABC is equilateral then vectors AK , BL and CM have equal lengths
1
ab
+
1
b
+
1
a
a b
: 9
2
1
! ! ! !
1 + 2 1 + 2 1+ 2 ( + + )
and the sizes of the angles between them are equal to 120Æ . So AK + BL + CM = 0 .
! ! ! !
Assume now that AK +BL+CM = 0 . Let a, b,
be the lengths of the sides BC , CA, AB , Solution 3. Multiplying both sides of the desired inequality by (1+2ab)(1+2b
)(1+2
a),
1
! 2S , jBL
respectively, and let S be the area of triangle ABC . Then jAK j =
!j = 2S and one gets the equivalent inequality
8
a b
! S ! ! !
jCM j =
. Rotating vectors AK , BL and CM counterclockwise by 90Æ, they become
2 1 + ab + b
+ a
a b
:2 2 2
4
parallel to the corresponding sides of the triangle and, if we draw them one by one onto
the plane, every vector starting from the endpoint of the previous, we obtain a triangle Note that
similar to ABC because the sum of the vectors is zero. Let k be the similarity coefficient;
r a2 + b2 +
2 p
! ! ! S !j ka implying a S
3
ab
;3 2 2 2
then jAK j = ka, jBLj = kb and jCM j = k
. Hence jAK
2 2 1 = =
= = = . 3 3
a k
13 14
giving ab
1; thus Solution 2. b) Note at first that the size of the angle between two arbitrary line segments
p
connecting two vertices is an integral multiple of , and so the angle can occur in case
a2 b2
2 = (ab
)2 ab
: n n
n
2
of even n only. So
must be odd since
and n are coprime, which gives
(mod n).
2 2
So indeed one has n n n
Thus
(i + ) =
i +
i + n), i.e. any diameter transforms to a diameter.
ab + b
+ a
p p (mod
ab
ab
a2 b2
2 :
1 + 4 2 2 2
q q
2 2 2
= Now consider two cases.
4
If line segment Ai Aj is a diameter then j Ak Ak
n Aj
12. Let n and
be coprime positive integers. For any integer i, denote by i0 the re-
i+ n) and Ai Aj ? Ak Al if and only Aj
(mod
if j k l j (mod n) (see figure 11).
2
mainder of division of product
i by n. Let A0 A1 : : : An 1 be a regular n-gon. Prove Ai Am
that Then
j
k =
(j k )
(l j ) = Ai
l
j (mod n), i.e. j 0 k0 l0 j 0 (mod n) Al Al
a) if Ai Aj k Ak Al then Ai Aj k Ak Al ;
0 0 0 0 and hence A A ? A A .
i j 0
k0
l 0 0
b) if Ai Aj ? Ak Al then Ai Aj ? Ak Al .
0 0 0 0
If line segment Ai Aj is not a diameter then Figure 11 Figure 12
consider the vertex Am such that Ai Am is
(Grade 12.) a diameter (see figure 12). Then Ai Aj ? Aj Am because the angle Ai Aj Am is supported
by the diameter Ai Am . As diameters transform to diameters, also angle Ai Aj Am is
Solution 1. a) Clearly Ai Aj k Ak Al iff i k l j (mod n) (see figure 9). Consequently, supported by a diameter giving Ai Aj ? Aj Am . Since Ai Aj ? Aj Am and Ai Aj ? Ak Al ,
0 0 0 0
0 0 0
Ai Aj k A k Al ) i k l j (mod n) ) we get Aj Am k Ak Al and, by a), also Aj Am k Ak Al . Hence Ai Aj ? Ak Al .
0 0 0 0 0 0 0 0
)
(i k)
(l j ) (mod n) )
)
i
k
l
j (mod n) ) IMO Team Selection Contest
) i0 k0 l0 j 0 (mod n) )
) Ai Aj k Ak Al :
0 0 0 0 First Day
Ak
1. Let k > 1 be a fixed natural number. Find all polynomials P (x) satisfying the
Aj Aj
condition P (xk ) = (P (x))k for all real numbers x.
Ai Ai A k+ n4 Answer: P (x) = 0 and P (x) = xn , where n is an arbitrary non-negative integer; in the
Al case of odd k also P (x) = xn .
Ak Al+ n4 Al
Solution. Let the degree of a polynomial P (x) be n > 0, then
Figure 9 Figure 10
P (x) = an xn + an 1 xn 1
+ : : : + a 1 x + a0 ;
_ _ _ _
b) Using a well-known theorem of geometry, we obtain
where an 6= 0.
Ai Aj ? Ak Al () Ai Ak Aj Al Ak Aj Al Ai () + = +
Let i be the largest index smaller than n for which ai 6
= 0 (suppose that such an index i
exists), then
() k i l j j k i l ( ) + ( ) ( ) + ( ) (mod n) ()
() k l i j n 2( + ) 0 (mod ) P (xk ) = an xkn + ai xki + ai 1 xki k
+ : : : + a1 xk + a0
where the arcs considered are taken on the circumcircle of the polygon. Thus and
AA ?A A ) k l i j
i j k l 2( + ) 0 (mod n) ) (P (x))k = (an xn + ai xi + ai 1 xi 1
+ : : : + a 1 x + a 0 )k :
)
k l i j 2 ( + ) n) )
0 (mod
Find next the coefficient of the term x(k 1)n+i in both polynomials. As i < n, we get
) k 0 l 0 i0 j 0
2( + ) n) )
0 (mod
kn > (k 1)n + i > ki, and the coefficient of this term in the polynomial P (x ) is
k
) A i Aj ? Ak Al : 0 0 0 0 therefore 0. On the other hand, we get the term an xn in (P (x))k iff we take the term
15 16
an xn from k 1 factors and ai xi from one factor; therefore in the polynomial (P (x))k , Construction for n = 4k :
the coefficient of this term is kank 1 ai 6= 0. This contradiction shows that there is no such
index i and the polynomial P (x) has the form P (x) = an xn . Also notice that if the degree
; k
(0 4 ) 4k 1
n of the polynomial P (x) is 0, then also P (x) = a0 = an xn . 9 9
k ; k >
> (6 k ; k + 1)
1 6 2 >
>
From the equality P (xk ) = (P (x))k , we now get an xnk = ank xnk for all real x, i.e. an = ank
(2 1 2 ) 1
= k ; k + 3) =
(2 k ; k + 2)
3 2
k 5 (6 3 6
k 6
or an (akn 1 1) = 0. Hence an 2 f 1; 0; 1g for odd k (an = 0 is possible only if n = 0) and >
> ;
>
>
an 2 f0; 1g for even k. (1 4 ; k 2)
; 4 k 3 (4 k + 1; 8k 1) k
4 2
9 9
2. Let O be the circumcentre of the acute triangle ABC and let lines AO and BC inter- (4k 1; 4k + 2) 3 > (2; 8k 4 >
>
> 2)
=
sect at point K . On sides AB and AC , points L and M are chosen such that jKLj = jKB j k ; k + 4) = ; k
and jKM j = jKC j. Prove that segments LM and BC are parallel.
(4 3 4
k 7 (4 8 4)
; k 8
;
>
> >
>
1
(2 k + 1; 6k) 4 k 1 (2k ; k + 2) 4k
2 6 4
Solution. Draw heights for triangles KBL and KCM from
A
q
the vertex K and let their bases be S and T , respectively.
Also lengthen the segment AK until it intersects the cir- Construction for n = 4k + 1:
cumcircle of ABC at point P (see Figure 13). As segment
L M (0 4 ; k + 1) 4k + 1 (5 k + 1; 7k + 1) 2k
AP is a diameter of the circumcircle of ABP C , the trian-
1 1
S O T 9 9
gles ABP and ACP are right-angled. Triangle ASK is k + 1; k) > k + 1; 8k + 1) 2k + 2 >
>
similar to triangle ABP and triangle AT K is similar to B
C
(
k + 2; k
1
>
=
(2
k + 2; 8k) 2k + 4
=
triangle ACP (their corresponding sides are parallel), so
K ( 1)
k 3 (2
k
>
> >
>
jAS j jAK j jAT j P k; 1) ; k k; 7k + 2) k;
jAB j jAP j jAC j . Hence triangle AST is similar to
= = (2 2 1 (3 4
9 9
triangle ABC and therefore ST k BC . As jLS j jSB j and Figure 13 (5k; 5k + 2) 2 > k; 6k + 1) 2k + 1 >
=
> (4
>
jMT j jT C j, we obtain LM k BC .
= k 1; 5k + 3) = k 1; 6k + 2) 2k + 3 =
(5
; k 4
1
(4
; k
>
> >
>
3. For which natural number n is it possible to draw n line segments between vertices (4 k + 2; 6k) 2 k 2 (3 k + 1; 7k) k
4 1
of a regular 2n-gon so that every vertex is an endpoint for exactly one segment and these
segments have pairwise different lengths? (There are actually many other constructions for both cases.)
Answer: n = 4k and n = 4k + 1, where k is an arbitrary positive integer.
Solution. Colour the vertices of the 2n-gon alternately black and white. Call the smallest Second Day
number of sides needed to pass when moving from one vertex to another the weight of
the segment with endpoints at these vertices. We see that segments with odd weigths 4. Denote
connect vertices with different colours, but segments with even weights connect vertices
with the same colour. X
m
k
Suppose that the required construction exists for a given n. As there is an equal number
f (m) = ( 1)
k
os :
k=1
m+1
2
of vertices of both colours and all segments with odd weights occupy an equal number
of vertices of both colours, also all segments with even weights must take occupy an For which positive integers m is f (m) rational?
equal number of vertices of both colours. Therefore the number of segments connecting
two white vertices equals the number of segments connecting two black vertices, and Answer: for all positive integers m.
the number of segments with even weights is an even number. Therefore there must be Solution Fix a positive integer m arbitrarily and take
an even number of even numbers among numbers 1; 2; : : : ; n, which is possible only if
n 0 (mod 4) or n 1 (mod 4). a =
os
2(2 m + 1)
6
= 0 :
We show next that such sets exist for n = 4k and n = 4k + 1. In the following tables,
the segments are grouped into blocks of parallel segments. In each row of a block, (x; y ) Using the formula
shows that a segment is drawn between vertices x and y , next comes the weight of this
1
segment and, after the end of a block, the number of segments in this block is shown.
os x
os y = (
os( x y ) +
os(x + y )) ;
2
17 18
we get the chain of equations Let now be k > l. Then blog2 n
+ 1 2(blog3 n
+ 1) or
X
m
k b n
2blog3 n
+ 1 ; (15)
f (m) a a=
log2
k
= ( 1)
os
k=1
m+1
2
this holds for n = 2 and n = 8. With immediate check we see that for 4 n 7 and
X 9 n < 27 the inequality (15) does not hold. If n 27 then
m
k k
= ( 1)
os
os =
k=1
2m + 1 2(2m + 1) log2 n log2 27 = log2 3(log3 n log3 27) < 2(log3 n 3) = 2 log3 n 6 ;
Xm k
1
k hence 2 log3 n log2 n > 6 log2 27 > 6 5 = 1. Therefore blog2 n
< b2 log3 n
2blog3 n
+ 1 for all n 27 . Hence (15) holds iff n = 2 or n = 8.
= ( 1)
os +
2 2m + 1 2(2m + 1)
k=1
k Therefore the only possible values for n are 1, 2, 3 and 8. For them, we get A(1) = 1 = 20 ,
+
os
2m + 1
+
2(2m + 1)
=
A(2) = 2 = 21 , A(3) = 4 = 22 and A(8) = 32 = 25 . Hence all these values suit.
1
Xm (2k 1) (2k + 1)
k 6. Call a convex polyhedron a footballoid if it has the following properties.
= ( 1)
os +
os =
2 2(2m + 1) 2(2m + 1)
k=1
1
(2m + 1)
(1) Any face is either a regular pentagon or a regular hexagon.
m
=
os + ( 1)
os =
2 m + 1) 2(2 2(2m + 1) (2) All neighbours of a pentagonal face are hexagonal (a neighbour of a face is a face that
=
1
2
( a+( 1)
m
0) =
1
2
a:
has a common edge with it).
Find all possibilities for the number of pentagonal and hexagonal faces of a footballoid.
1
Hence f (m) = for all m. Answer: there are 12 pentagonal and 20 hexagonal faces.
2
Solution. We show first that there exists a footballoid with 12 pentagonal and 20 hexag-
5. Find all natural numbers n for which the number of all positive divisors of the onal faces. Start with a regular icosahedron and abstract from every vertex a regular
number lcm (1; 2; : : : ; n) is equal to 2k for some non-negative integer k . 1
pyramid with lateral edge of the edge of the icosahedron. In such a way, we get 12
3
Answer: 1, 2, 3 and 8.
regular pentagons instead of 12 vertices of icosahedron and we get 20 regular hexagons
Solution. Let P be the set of all prime numbers. Let Æ (m) denote the number of positive instead of 20 faces of icosahedron. All neighbours of any pentagonal face are hexagonal.
divisors of natural number m and let A(n) = Æ (lcm (1; : : : ; n)). Denote by p . m the Now show that it is the only possibility. Let B be a footballoid. Consider an arbitrary
exponent of prime number p in the canonical representation of m. Notice that vertex of B ; let it belong to x pentagonal and y hexagonal faces. Then x + y 3 as every
Y vertex of a polyhedron belongs to at least 3 faces. As the sizes of the interior angles of
A(n) = Æ (lcm (1; : : : ; n)) = Æ pmax(p.1;:::;p.n) = pentagonal and hexagonal faces are 108Æ and 120Æ , respectively, we get x 108Æ + y 120Æ <
360 . Hence x + y 3 and x > 0. Therefore x + y = 3, which means that every vertex of
p2P Æ
Y Y
= Æ pblogp n
= b
( logp n
+ 1) : a footballoid belongs to exactly 3 faces, at least one of which is pentagonal. As these 3
p2P p2P faces are pairwise neighbours and pentagonal faces cannot be neighbours, every vertex
must belong to exactly one pentagonal and two hexagonal faces.
Therefore A(n) is a power of 2 iff all numbers in the form blogp n
+ 1, where p 2 P, are Consider an arbitrary hexagonal face. All its vertices belong to one pentagonal and one
powers of 2. Let blog2 n
+ 1 = 2k and blog3 n
+ 1 = 2l . As log2 n log3 n, we get k l. hexagonal face. Therefore the neighbours of a hexagonal face are alternately pentagonal
Consider two cases. and hexagonal, so there are exactly three of both kinds.
If k = l then Now cover every pentagonal face with a regular pentagonal pyramid, whose lateral
blog2 n
= blog3 n
; (14) edges are continuations of the (hexagonal) neighbours of this face. In this way, hexag-
onal faces become equilateral triangles and pentagonal faces are replaced with vertices
this holds for n = 1 and n = 3. With immediate check we see that, for n = 2 and in which five edges meet. As any two neighbours of a pentagonal face which are neigh-
4 n < 8, the equation (14) does not hold. If n 8 then bouring themselves meet under the same angle (two regular hexagons and one regular
pentagon can meet in one vertex in principle in only one way), also the triangles meet-
log2 n 3 = log2 n log2 8 = log2 3(log3 n log3 8) > log3 n log3 8 ; ing in a vertex of our new polyhedron meet under the same angle. Therefore the new
hence log2 n log3 n > 3 log3 8 > 3 2 = 1 . Therefore blog2 n
> blog3 n
for all n 8. polyhedron is a regular icosahedron. It has 12 vertices and 20 faces, so the footballoid
Hence (14) holds iff n = 1 or n = 3. B had to have 12 pentagonal and 20 hexagonal faces.
19 20
Estonian Math Competitions
2004/2005
The Gifted and Talented Development Centre
Tartu 2005
As rays BK and CL are opposite-directed, we have
|AB| + |AC|
|BC| = ⇐⇒ |BC| = |BK| + |CL| ⇐⇒ K = L .
2
3. On some square of an infinite squared plane, there is a cube which covers the square
exactly. The top face of the cube is white, the other faces are black. With one step, one
can turn the cube over any edge so that it starts covering a neighbouring square. Is it
possible to achieve a situation where the cube lies on the initial square with the white
Selected Problems from Open Contests face at the bottom, making exactly
a) 2004 steps; b) 2005 steps? (Juniors.)
1. In the final tournament of football championship, the teams are divided into groups
of four. Each team plays one game with every other team in its group. A win gives 3
points, a draw 1 point and a loss 0 points. From each group, two teams advance so that
each advancing team gets at least as many points as each non-advancing team. A
a) What is the smallest possible score of an advancing team?
b) What is the biggest possible score of a non-advancing team? (Juniors.) D E
Answer: a) 2; b) 6.
Solution. a) If one team in a group wins all matches and the other three draw all matches,
B KL C
then the second advancing team collects just 2 points. Let us prove that a team cannot
advance with less than 2 points. Indeed, if team A gets at most 1 point, it must lose at Figure 1 Figure 2 Figure 3
least two games. Therefore two teams get at least 3 points and outperform A, so that A
cannot be in top two. Answer: a) yes; b) no.
b) If one team in a group loses all matches and the other three win cyclically (X wins Solution 1. a) Turn the cube two steps forward, one step to the right, two steps backward,
Y , Y wins Z and Z wins X ), then three teams collect 6 points and one of them does one step to the left (see Figure 2). After these six steps, the cube gets back onto the initial
not advance. Let us prove that a team with more than 6 points must advance. If team square but the white face is now at the bottom. The rest 1998 steps are made in pairs:
C gets at least 7 points, it must win at least two games. As the losing teams can get at turn the cube onto arbitrary neighbouring square and then turn it back onto the initial.
most 6 points, team C must be in top two. b) We colour squares dark and light by diagonals so that the cube lies on a light-coloured
square at the beginning (see Figure 3). Since, from any square, the cube can move only
2. In triangle ABC let D, E be the midpoints of AB and AC , respectively. Prove that to squares of the opposite colour, the cube lies on a light-coloured square after any even
the intersection point of the bisectors of angles BDE and CED lies on AB if and only if number of steps and on a dark-coloured square after any odd number of steps. Thus
the length of BC is equal to the arithmetic mean of the lengths of AB and AC . (Juniors.) after 2005 steps, the cube lies on a square different from the initial.
Solution 2. a) Turn the cube two steps forward, one step to the left, two steps forward,
Solution. Let K and L be the intersection points of BC with the bisectors of two steps to the right, four steps backward, one step to the left. After these 12 steps, the
angles BDE and CED, respectively (see Figure 1). As DE k BC , we have cube is back on the initial square but the white face is now at the bottom. Repeating this
|AB| |AC|
∠BDK = ∠EDK = ∠BKD, and |BK| = |BD| = . Similarily |CL| = . cycle, we see that, after any odd number of repetitions, the white face of the cube is at
2 2 the bottom and, after any even number of repetitions, the white face is at the top. Since
Therefore
2004 = 12 · 167, we obtain the desired result by repeating this cycle for 167 times.
|AB| + |AC| b) Assume that the cube makes a circuit and gets back onto the initial square. Let a,
|BK| + |CL| = .
2 b, c, and d be the numbers of steps made during this circuit respectively to the right,
to the left, up, and down. Then a = b and c = d. Thus the cube makes altogether
1 2
K
a + b + c + d = 2(a + c) steps which is even number. Hence the cube cannot get back to Solution. Let ∠O1AG1 = α and ∠O1 G1 A = β H1 H2
the initial square after 2005 steps. (see Figure 4). If line G1 K touches circle C1 , then β P
∠AH1 G1 = 90◦ and ∠H1 G1 O1 = ∠AG1 O1 = β. β
G1 O1 O2 G2
Also, ∠G1 P A = 90◦ and ∠P AO1 = G1 AO1 = α.
a+b
4. Relatively prime positive integers a and b are chosen in such a way that is From the right-angled triangles AH1 G1 and P1 αα P2
a−b G1 P A, we get
also a positive integer. Prove that at least one of the numbers ab + 1 and 4ab + 1 is a A
perfect square. (Juniors.) α + 2β = 90◦ , Figure 4
a+b a m+1 β + 2α = 90◦ .
Solution 1. Let = m. Then a + b = ma − mb implying = . As a and b
a−b b m−1
are relatively prime, there exists an integer k such that m + 1 = ka and m − 1 = kb. By Solving the system gives α = β = 30◦ . Therefore ∠P AG1 = 60◦ and
multiplying these equalities, we get m2 − 1 = k 2 ab implying k 2 ab + 1 = m2 . Number k ∠P AG2 = 120◦ − 60◦ = 60◦ . Thus we have ∠P AG1 = ∠P AG2 . As circles C1 and
as a common divisor of numbers m − 1 and m + 1 must be a divisor of their difference C2 lie in equal angles and touch at P , their radii must be equal. Therefore AK is the
2. Hence k can only be 1 or 2 and we are done. symmetry axis. By symmetry, line G2 K is a tangent to C2 and touches the circle at H2 .
Solution 2. As
a+b a−b+b+b 2b 7. A king wants to connect n towns of his kingdom with one-directional airways so
= =1+ ,
a−b a−b a−b that, from each town, exactly two airlines depart. From each town, it must be possible
2b to fly to every other town with at most one change. Find the biggest n for which this
we see that must be an integer. Numbers b and a − b are relatively prime because plan is feasible. (Seniors.)
a−b
a and b are relatively prime. Hence 2 must be divisible by a − b. Therefore a − b = 1 or
Answer: 6.
a − b = 2. The former case implies 4ab + 1 = 4(b + 1)b + 1 = (2b + 1)2 , the latter case
implies ab + 1 = (b + 2)b + 1 = (b + 1)2 . Solution. From a fixed town, one can get directly to two towns and further to at most
four more towns. Thus the number of towns cannot exceed 1 + 2 + 4 = 7.
a √ Let us assume that a suitable airway plan for 7 towns exists. From each town, there
5. The teacher has chosen positive integers a and b such that · a2 + b2 is an integer. must be a unique way to every other town (either direct or with one change), otherwise
b
a) Silly-Sam claims that a is divisible by every prime factor of b. Prove that he is right. some town would have less than 6 possible final destinations. Without loss of generality
assume that there is a direct flight from town L1 to towns L2 and L3 , from town L2 to
b) Silly-Sam claims that actually b 6 a. Is he right this time? (Seniors.)
towns L4 and L5 , and from town L3 to towns L6 and L7 (see Figure 5).
Answer: b) no.
Solution. a) Let p be √ an arbitrary prime factor of b. If the observed expression is an L4
integer,√the number a a2 + b2 must be divisible by p. As p is prime, either a is divisible L5
L2
by p or a2 + b2 is divisible by p. In the latter case, squaring gives that a2 +b2 is divisible
by p2 . By the initial assumption, b2 is divisible by p2 , hence also a2 is divisible by p2 . L1 L6
Therefore a is divisible by p in both cases. L3
a √ L7
b) The teacher may choose a = 12 and b = 16. In this case, · a2 + b2 = 15 is an
b
integer. Therefore the inequality b 6 a might be wrong. Figure 5 Figure 6 Figure 7
6. Two circles C1 and C2 with centres O1 and O2 , respectively, are touching exter-
From L2 , there must be a way to all towns in list L1 , L3 , L6 and L7 . As the direct flights
nally at P . On their common tangent at P , point A is chosen, rays drawn from which
from L2 take to towns L4 and L5 , there must be a direct flight from L4 to two towns in
touch the circles C1 and C2 at points P1 and P2 both different from P . It is known that
the list and from L5 to the other two. Without loss of generality assume that there is a
∠P1 AP2 = 120◦ and angles P1 AP and P2 AP are both acute. Rays AP1 and AP2 intersect
direct flight from L4 to L3 . But now there can be a flight from L4 to none of L1 , L6 , L7
line O1 O2 at points G1 and G2 , respectively. The second intersection between ray AO1
because otherwise there would be two ways to get from town L4 to town L3 , L6 , L7 ,
and C1 is H1 , the second intersection between ray AO2 and C2 is H2 . Lines G1 H1 and
respectively.
AP intersect at K . Prove that if G1 K is a tangent to circle C1 , then line G2 K is tangent
to circle C2 with tangency point H2 . (Seniors.) A suitable airway plan for 6 towns exists, as shown on Figure 6 or Figure 7.
3 4
8. For which integers a does there exist two different finite sequences of positive inte- 2004 2005
gers i1 < i2 < · · · < ik and j1 < j2 < · · · < jl such that
(ai1 + 1)(ai2 + 1) · · · (aik + 1) = (aj1 + 1)(aj2 + 1) · · · (ajl + 1) ?
2004
2004
(Seniors.)
Answer: −1, 0, and 1.
Solution. In the case a = −1, both sides of the equation equal to zero whenever both Figure 8 Figure 9 Figure 10
sequences contain an odd number. In the case a = 0, both sides equal to one irrespective
of the sequences. In the case a = 1, all factors of the products are equal to 2, so the Answer: a) Kalle; b) Kalle; c) Juku.
products are equal whenever the sequences have the same length. Therefore suitable
Solution. a) To Juku’s first move, Kalle can reply with a move after which a part of shape
sequences exist in these three cases.
2 × 2 has been eaten up (see Figure 8). Irrespectively of Juku’s second move, Kalle can
Let us prove that if |a| > 1, then such sequences do not exist. Suppose the contrary, make one more move. After that, Juku has no move.
i.e. i1 , . . . , ik and j1 , . . . , jl are different sequences that lead to equal products. We
b) When Juku eats up some two pieces, Kalle replies by eating two pieces which lie
may assume that no integer is in both sequences or else the respective terms can be
symmetrically with respect to the midpoint of the cake (see Figure 9). This guarantees
cancelled in the products. After deletions, both sequences are still nonempty or else we
that, after every move by Kalle, pieces not yet eaten up are situated symmetrically to
get an equation between 1 and the product of integers not being equal to 1. We can also
the midpoint of the cake and, as it is not possible to eat two symmetric pieces at one
assume that i1 < j1 .
move by the same player, Kalle can always follow the strategy described. Hence Juku’s
Multiplying and then removing the parentheses on both sides gives an equation be- moves come first to the end.
tween sums of powers of a. Both sides contain term 1 and we can reduce that. This
c) On his first move, Juku can eat two pieces between which the midpoint of the cake
ends up in the equation of form
lies and later use the strategy of Kalle from part b) (see Figure 10).
ai1 +···+ik + · · · + ai1 = aj1 +···+jl + · · · + aj1 . Remark. This game is called Cram, its full analysis for the case where length and width
are both odd numbers seems to be quite complicated and is not completed yet.
The smallest exponent is i1 on the left-hand side and j1 on the right-hand side. As
j1 > i1 + 1, the right-hand side is divisible by ai1 +1 . On the left-hand side, all terms
10. Find all functions f : R → R satisfying
except the last one are divisible by ai1 +1 . Hence the left-hand side is not divisible by
ai1 +1 , a contradiction. f (x + f (y)) = x + f (f (y))
Comment. From the solution, we get that for each a (|a| > 1), any finite subset I of
for all real numbers x and y whereby f (2004) = 2005. (Seniors.)
natural numbers can be assigned a unique number
Y Answer: f (x) = x + 1 is the only such function.
f (I) = (ai + 1). Solution 1. Taking y = 0, we get the equality f (x + f (0)) = x + f (f (0)). Making the
i∈I substitution x + f (0) = z, we obtain f (z) = z − f (0) + f (f (0)) for every real number z.
The number determines the subset uniquely. This can be used in proving that the num- Hence f is a linear function f (x) = x + a. To find a, take x = 2004 in the last expression
ber of finite subsets of natural numbers is countable. and, by using the known value of the function, obtain a = 1. A quick checking shows
that the function f (x) = x + 1 satisfies the conditions of the problem.
Solution 2. Taking x = −f (f (y)), we see that f (y) = 0 for some y. Then f (x) = x + f (0).
9. Mother has baked a platecake and cut it into m×n square pieces of equal size. Kalle From the condition f (2004) = 2005, we find f (0) = 1. Thus f (x) = x + 1.
and Juku play the following game. Each player at his move eats two pieces having a
common side. Moves are made by turns, Juku starts. A player who cannot move loses.
Who wins the game if 11. Three rays are going out from point O in space, forming pairwise angles α, β and
γ with 0◦ < α 6 β 6 γ 6 180◦ . Prove that
a) m = 3, n = 3;
α β γ
b) m = 2004, n = 2004; sin + sin > sin .
2 2 2
c) m = 2004, n = 2005? (Seniors.) (Seniors.)
5 6
O Solution 1. Choose points A, B, and C on the three Solution. Let a1 , a2 , a3 , a4 be the smallest numbers
10 120 5 15
α α rays, respectively, so that |OA| = |OB| = |OC| = d and b1 , b2 , b3 , b4 be the biggest numbers of the first,
d d 12 24 3
2 2 and ∠BOC = α, ∠COA = β, ∠AOB = γ . These second, third and fourth rows, respectively. Obvi-
2 8
three points must be different and do not lie on the 4 ously b4 = a4 and b3 = a3 b4 . In the second row,
same line. From isosceles triangle BOC with vertex there exists a number which is the product of the
B C Figure 12
angle α and side length d (see Figure 11), we obtain biggest element in the third row and some other
α element in the second. Thus b2 > a2 b3 = a2 a3 a4 .
Figure 11 |BC| = 2d sin . Analogously from triangles COA
2 Finally for the first row, we obtain similarily b1 > a1 b2 > a1 a2 a3 a4 . All numbers in the
β γ
and AOB, find |CA| = 2d sin and |AB| = 2d sin . As triangle are greater than 1, otherwise we could find two equal numbers in it. Since all
2 2 numbers are different, we have a1 a2 a3 a4 > 2 · 3 · 4 · 5 = 120. Hence b1 > 120. Number
|BC| + |CA| > |AB|, 120 is achievable as follows from Figure 12.
we obtain
α β γ
Selected Problems from the Final Round of National
2d sin + 2d sin > 2d sin ,
2 2 2 Olympiad
giving the desired inequality.
Solution 2. At first, we show that γ 6 α + β. Consider the two of the given three rays
which form angle of size γ , and build two cones by moving the third ray around both 1. Rein solved a test on mathematics that consisted of questions on algebra, geometry
rays. On the plane defined by the axes, the first cone cuts angle α off from angle γ and and logic. After checking the results, it occurred that Rein had answered correctly 50%
the second cone cuts angle β. Assume γ > α+β, then the last two angles do not overlap, of questions on algebra, 70% of questions on geometry and 80% of questions on logic.
therefore the cones have no common points except the vertex O, a contradiction. Thus Thereby, Rein had answered correctly altogether 62% of questions on algebra and logic,
γ 6 α + β. On the other hand, α + β + γ 6 360◦ , giving and altogether 74% of questions on geometry and logic. What was the percentage of
correctly answered questions throughout all the test by Rein? (Grade 9.)
γ α β γ
6 + 6 180◦ − . Answer: 65%.
2 2 2 2
Solution: Let a, g, and l be the numbers of correctly answered questions on algebra,
Therefore geometry and logic, and A, G, and L be the total number of questions on algebra, ge-
γ
α β
α β β α α β ometry and logic, respectively. The conditions of the problem imply a = 0.5A, g = 0.7G,
sin 6 sin + = sin cos + sin cos < sin + sin l = 0.8L, a + l = 0.62(A + L), g + l = 0.74(G + L). After substituting to the fourth and
2 2 2 2 2 2 2 2 2
fifth equation, we obtain 0.5A + 0.8L = 0.62A + 0.62L, or equivalently 0.12A = 0.18L,
α β γ β giving
because , and are grater than 0◦ but do not exceed 90◦ , and also cos < 1 and
2 2 2 2 A = 1.5L,
α
cos < 1.
2 and 0.7G + 0.8L = 0.74G + 0.74L, or equivalently 0.04G = 0.06L, giving
G = 1.5L.
12. We call a number triangle amazing if all its elements are 21 84 7
different positive integers and, under every two neighbouring 4 12 Now
numbers, the quotient by division of the greater of two by the 3
smaller is written. In the figure, one amazing triangle with side a + g + l = 0.5A + 0.7G + 0.8L = 0.75L + 1.05L + 0.8L = 2.6L
length 3 is shown. Find the smallest number which can occur as the greatest element in and
an amazing triangle with side length 4. (Seniors.)
A + G + L = 1.5L + 1.5L + L = 4L.
Thus the percentage of correct answers was
a+g+l 2.6
= = 65%.
A+G+L 4
7 8
2. Represent the number
√
q
3
1342 167 + 2005
in the form where it contains only addition, subtraction, multiplication, division and
square roots. (Grade 10.)
√ Figure 13 Figure 14
Answer: 2 167 + 1.
Solution 1. First, isolate the terms divisible by 167:
√ √ √ Solution. Suppose all dark squares are covered. Since one hook cannot cover more than
1342 167 + 2005 = 1336 167 + 2004 + 6 167 + 1 = one dark square, in total at least nine hooks are needed. As only eight of these are avail-
√ √
= 8 · 167 167 + 12 · 167 + 6 167 + 1. able, one of the dark squares must remain uncovered. There are three fundamentally
different possibilities: the free square lies in the corner, in the middle of a side or in the
Second, represent the result in the form centre of the board. The corresponding tilings are shown in the Figure 14.
√ √ √ √
1342 167 + 2005 = (2 167)3 + 3 · (2 167)2 + 3 · (2 167) + 1 =
√ 4. Real numbers x and y satisfy the system of equalities
= (2 167 + 1)3 .
(
sin x + cos y = 1
Therefore .
cos x + sin y = −1
√ √
q
3
1342 167 + 2005 = 2 167 + 1.
Prove that cos 2x = cos 2y. (Grade 11.)
√
Solution 2. Search the answer in the form a 167 + b, where a and b are integers. Then Solution 1. After squaring both sides, we obtain
we must have
sin2 x + 2 sin x cos y + cos2 y = 1,
√ √
(a 167 + b)3 = 1342 167 + 2005, cos2 x + 2 cos x sin y + sin2 y = 1.
or equivalently After adding the equations and dividing by 2, we obtain
√ √ √
167a3 167 + 3 · 167a2 b + 3ab2 167 + b3 = 1342 167 + 2005. sin x cos y + sin y cos x = 0,
Thus a and b must satisfy the system or equivalently
(
167a3 + 3ab2 = 1342 sin(x + y) = 0.
.
501a2 b + b3 = 2005 Hence x + y = kπ, where k is integral. Therefore 2x = 2kπ − 2y, giving cos 2x = cos 2y.
Solution 2. After adding the equations, we obtain
The second equation can be rewritten in the form (501a2 +b2 )b = 2005. Since a and b dif-
fer from 0 and are integers, 501a2 + b2 must be a divisor of 2005 that is greater than 501. sin x + sin y + cos x + cos y = 0,
The only possibility is now 501a2 + b2 = 2005, giving b = √ 1, a = 2. Simple check
√ shows
that a = 2, b = 1 satisfy the first equation as well. Hence (2 167+1)3 = 1342 167+2005. which is equivalent to
x+y x−y x+y x−y
2 sin cos + 2 cos cos = 0,
3. A 5×5 board is covered by eight hooks (a three unit square figure, shown 2 2 2 2
in the picture) so that one unit square remains free. Determine all squares of implying
the board that can remain free after such covering. (Grade 10.)
x−y x+y x+y
Answer: All the squares that are colored dark in the Figure 13. cos sin + cos = 0.
2 2 2
9 10
x−y x−y π assumption that C is more important than A. Analogously, if C comes up first, then
If cos = 0, then = (2k − 1) · , giving 2x − 2y = (2k − 1) · 2π.
2 2 2 we get a contradiction with the assumption that B is more important than A. Thus our
x+y x+y x+y
Hence cos 2x = cos 2y. If sin + cos = 0, then tan = −1, original assumption was false.
2 2 2
x+y π π Remark. In graph theory, the relation “is more important than” of this problem is called
implying = kπ − and x + y = 2kπ − . Therefore
2 π 4
π
2 postdominance.
cos y = cos − − x = cos + x = − sin x, leading to sin x + cos y = 0 that
2 2
contradicts the first equation of the initial system. 7. In a fixed plane, consider a convex quadrilateral ABCD. Choose a point O in the
plane and let K , L, M , and N be the circumcentres of triangles AOB, BOC , COD, and
5. Let a, b, and n be integers such that a + b is divisible by n and a2 + b2 is divisible DOA, respectively. Prove that there exists exactly one point O in the plane such that
by n2 . Prove that am + bm is divisible by nm for all positive integers m. (Grade 11.) KLMN is a parallelogram. (Grade 11.)
Solution 1. We prove that a and b are divisible by n, then the claim immediately follows. Solution. If O is the point described in the problem, then D
As M
we must have KL ⊥ BO because K and L both lie on
the perpendicular bisector of BO. Similarly LM ⊥ CO, C
2ab = (a + b)2 − (a2 + b2 ), MN ⊥ DO, and NK ⊥ AO. Let O be the intersec-
tion point of the diagonals of ABCD (see Figure 15). O
2ab is divisible by n2 . Let p be any prime in the prime decomposition of n and let α be N
Then both KL and MN are perpendicular to BD, giving
its exponent. Then the exponent of p is at least 2α in the prime decomposition of 2ab, L
KL k MN . Similarly LM k NK . Therefore the oppo-
and at least 2α − 1 in the prime decomposition of ab. Therefore at least one of numbers
site sides of KLMN are parallel, meaning that KLMN
a and b must be divisible by pα . As a + b is divisible by n and hence by pα , also the other
is a parallelogram. On the other hand, if O is a point for A B
of the numbers a and b must be divisible by pα . Altogether, this means that both a and K
which KLMN is a parallelogram, we have KL k MN .
b are divisible by n.
Then also BO k DO, giving that O lies on the line BD. Figure 15
We can show similarily that O lies also on the line AC .
6. A post service of some country uses carriers to transport the mail; each carrier’s Therefore O is the intersection point of the diagonals.
task is to bring the mail from one city to a neighbouring city. It is known that it is
possible to send mail from any city to the capital P . For any two cities A and B, call
8. Does there exist an integer n > 1 such that
B more important than A, if every possible route of mail from A to the capital P goes
through B. n −1
22 −7
a) Prove that, for any three different cities A, B, and C , if B is more important than A
is not a perfect square? (Grade 11.)
and C is more important than B, then C is more important than A.
b) Prove that, for any three different cities A, B, and C , if both B and C are more Answer: Yes.
n
important than A, then either C is more important than B or B is more important Solution 1. Let us show that if n = 5, then the number 22 −1 − 7 is not a perfect square.
than C . (Grade 11.) Note that 210 = 1024 ≡ 1 (mod 11), giving 231 = 2 · (210 )3 ≡ 2 (mod 11). Hence the
remainder of division of 231 − 7 by 11 is 6. On the other hand, squares of integers can
Solution. a) Let t be any possible mail route from A to P . Since B is more important have remainders 0, 1, 4, 9, 5, and 3 in division by 11.
than A, the route t goes through B. The end part of t from B to P is a mail route from Solution 2. Computation gives 231 − 7 = 32768 · 65536 − 7 = 2147483641 but
B to P . Since C is more important than B, this route goes through C . Therefore, t goes 463402 = 2147395600 and 463412 = 463402 + 2 · 46340 + 1 = 2147488281. So
through C . 463402 < 231 − 7 < 463412 .
5
b) Assume that the claim doesn’t hold, that is, C is not more important than B and B Remark. The number 11 is the least modulus with respect to which 22 −1 − 7 is not a
is not more important than C . Then there exist a route from B to P not going through quadratic residue. There exist greater such numbers, e.g. 31.
C and a route from C to P not going through B. Consider any route from A to P .
Since B and C are more important than A, this route goes through both B and C . Start
moving from A along this route and find out which of the cities B and C comes up
first. If it is B, then continue along the route to P that doesn’t pass through C . So we
have found a route from A to P that doesn’t go through C , a contradiction with the
11 12
9. Punches in the buses of a certain bus company always cut exactly six
holes into the ticket. The possible locations of the holes form a 3 × 3 table Ai+1
as shown in the figure. Mr. Freerider wants to put together a collection
of tickets such that, for any combination of punch holes, he would have a Q
ticket with the same combination in his collection. The ticket can be viewed P
both from the front and from the back. Find the smallest number of tickets in such a Ai
collection. (Grade 12.)
Figure 16 Figure 17
Answer: 47.
Solution. Instead of holes, we can deal with non-holes — the locations that are not cut
through during punching. The number of possibilities to choose 3 locations for non- As P and Q are different, there exist such vertices of the polygon Ai and Ai+1 that the
holes from 9 locations is point Q is inside or on the side of Ai P Ai+1 , not coinciding with the vertex P (see Figure
17). But this implies ∠Ai QAi+1 > ∠Ai P Ai+1 , a contradiction. Hence there exists only
9
= 84. one point satisfying the conditions of the problem.
3
Remark. For n = 4, 6 one may find several different points satisfying the conditions of
One ticket can represent either one punch combination that is symmetric with respect the problem.
to the central axis parallel to the longer sides of the ticket or two different combinations
that are mirror images of each other with respect to this axis. In the case of symmetric
combinations, either all three non-holes lie in the second column (1 possibility) or one 11. A string having a small loop in one end is set over a horizontal pipe so
non-hole lies in the second column and other two lie in the same rows, one in the first that the ends hang loosely. After that, the other end is put through the loop,
column and the other in the second (3 · 3 = 9 possibilities). So there are 1 + 9 = 10 pulled as far as possible from the pipe and fixed in that position whereby α
symmetric combinations and 84 − 10 = 74 non-symmetric combinations. The number this end of the string is farther from the pipe than the loop. Let α be the
of tickets needed to cover these combinations is angle by which the string turns at the point where it passes through the loop
(see picture). Find α. (Grade 12.)
74
10 + = 47. π
2 Answer: .
3
Solution. Let O and r be the centre point and the radius of the pipe, respectively. Let
l be the length of the string, A and B the loose end and the end with the loop, re-
10. Consider a convex n-gon in the plane with n being odd. Prove that if one may find spectively, and let C and D be the first and the last tangent point with the surface of
a point in the plane from which all the sides of the n-gon are viewed at equal angles, the pipe (see Figure 18). At first, find the length of AO, denote by d(α). Obviously
then this point is unique. (We say that segment AB is viewed at angle γ from point O iff |BC| = |BD| = r cot α. As ∠COD = π − 2α, the string touches the pipe along the arc
∠AOB = γ .) (Grade 12.) CD at angle π + 2α and thus the length of the string along the arc CD is r(π + 2α). After
Solution. Draw the rays from the point described in the problem through the vertices of subtracting the lengths of segments BC and BD and the arc CD from the total length
of the string, we obtain
the polygon. The point can lie either inside or outside the polygon, therefore there are
two possibilities for the rays: they divide either all the plane or only an angle into equal |AB| = l − 2r cot α − r(π + 2α).
angles (see Figure 16). The latter case would imply that the outermost rays were both
We also have
incident to one vertex and all the others were incident to two vertices of the polygon,
r
giving that n is even. This contradiction shows that the point satisfying the conditions |BO| = .
of the problem lies inside the polygon. sin α
Assume now that there are two different points P and Q inside A1 A2 . . . An , from Altogether d(α) = |AB| + |BO|, or equivalently
which all the sides are viewed at equal angles. Then for every i = 1, 2, . . . , n (tak- r
d(α) = l − 2r cot α − r(π + 2α) + .
ing An+1 = A1 ), sin α
Now we find α for which the value of d(α) is the greatest. The derivative equals
2π 2π
∠Ai P Ai+1 = , ∠Ai QAi+1 = . 2r r cos α
n n d0 (α) = − 2r − ,
sin2 α sin2 α
13 14
be periodic modulo 2N . On the other hand, if ai and aj are congruent modulo 2N ,
they are also congruent modulo N . Therefore, the sequence is periodic modulo N , a
contradiction.
O b) We are going to prove that the sequence ai = 2i−1 is periodic modulo no even num-
bers and all odd numbers. Let first the modulus N be even. If the sequence were
periodic modulo N , then, using a similar argument as in a), we get that the sequence
r r
would be periodic modulo any factor of N . But the sequence 1, 0, 0, 0, . . . of remainders
modulo 2 is not periodic, so our sequence cannot be periodic modulo N . Now let the
α α modulus N be odd. As the set of possible remainders in division by N is finite, there
D C
exist two indices i and j with i < j such that ai = 2i−1 and aj = 2j−1 are congruent
B
modulo N . Then the difference 2j−1 − 2i−1 = 2i−1 (2j−i − 1) is divisible by N . Since N
A is odd, 2j−i − 1 must be divisible by N . Hence a1 = 1 and aj−i+1 = 2j−i are congruent
modulo N , implying also that a2 = 2a1 and aj−i+2 = 2aj−i+1 are congruent modulo N ,
Figure 18 that a3 = 2a2 and aj−i+3 = 2aj−i+2 are congruent modulo N etc, that is, the sequence is
periodic modulo N .
or equivalently,
13. A crymble is a solid consisting of four white and one black unit cubes
2 − 2 sin2 α − cos α 2 cos2 α − cos α
d0 (α) = r =r . as shown in the picture. Find the side length of the smallest cube that can
sin2 α sin2 α
be exactly filled up with crymbles. (Grade 12.)
The condition d0 (α) = 0 gives the equation 2 cos2 α − cos α = 0, implying cos α = 0 or Answer: 10.
1 π π π
cos α = . Thus α = or α = , bearing in mind that 0 < α 6 . In order to find Solution. Since a crymble consists of 5 unit cubes, the volume of the cube made up from
2 2 3 2
the maximal value, we consider d0 (α) in the neighbourhoods of α found out before. The crymbles and hence also the length of its side must be divisible by 5. A cube with the
denominator sin2 α is positive in these neighbourhoods, the numerator 2 cos2 α − cos α side length 5 cannot be filled up with crymbles. To prove this, colour 27 unit cubes as
1 shown in the Figure 19. One crymble cannot fill more than one coloured cube, therefore
is negative iff 0 < cos α < . Therefore d0 (α) is positive and d(α) is increasing in the at least 27 crymbles are needed. But their volume 27 · 5 = 135 is larger than the volume
2
π π π 53 = 125 of the cube.
interval 0 < α < , d0 (α) is negative and d(α) is decreasing in the interval < α < .
3 3 2 A cube with side length 10 can be filled up with crymbles. By putting together two
π
Hence the function d(α) obtains its maximal value at α = . crymbles, construct the solid that is in the Figure 20. From two such solids, make a
3
Remark. The value for α found out in the solution is attainable iff the ratio between the
√ 2 × 2 × 5 cuboid. From such cuboids, it is possible to put together a 10 × 10 × 10 cube.
5π + 2 3
length of the string and the diameter of the pipe is at least .
6
12. A sequence of natural numbers a1 , a2 , a3 , . . . is called periodic modulo n if there
exists a positive integer k such that, for any positive integer i, the terms ai and ai+k are
equal modulo n. Does there exist a strictly increasing sequence of natural numbers that
a) is not periodic modulo finitely many positive integers and is periodic modulo all the
other positive integers;
b) is not periodic modulo infinitely many positive integers and is periodic modulo in- Figure 19 Figure 20
finitely many positive integers? (Grade 12.)
Answer: a) no; b) yes.
Solution. a) Suppose that the sequence a1 , a2 , a3 , . . . is not periodic modulo finitely
many positive integers, let N be the largest of these. Since 2N > N , this sequence must
15 16
IMO Team Selection Contest c1
c1
First Day Q1
c2 O1
l
Q1
O2 Q2 P M
1. On a plane, a line l and two circles c1 and c2 of different radii are given such that l
touches both circles at point P . Point M 6= P on l is chosen so that the angle Q1 MQ2 is l
as large as possible where Q1 and Q2 are the tangency points of the tangent lines drawn Q2
c2
P M
from M to and
c1 c2 ,
respectively, differing from l. Find ∠P MQ1 + ∠P MQ2 .
Figure 21 Figure 22
Answer: π.
Solution. Consider first the case where c1 and c2 are on the same side from l (see Figure
21). Let O1 and O2 be the circumcentres and r1 and r2 the radii of c1 and c2 , respec- • each Automorian is loved by some Automorian;
tively. Without loss of generality, assume r1 > r2 . Denote ∠P MQ1 = α1 , ∠P MQ2 = α2 • if Automorian A loves Automorian B, then also all Automorians honouring A
1 1 love B;
and |P M| = d. As ∠P MO1 = ∠P MQ1 and ∠P MO2 = ∠P MQ2 , we see that
2 2 • if Automorian A honours Automorian B, then also all Automorians loving A hon-
α1 α2
∠Q1 MQ2 = α1 − α2 is maximal if and only if ∠O1 MO2 = − is maximal; the our B.
2 2
α1 α2
latter holds if and only if tan − is maximal because the angle is in the first Is it correct to claim that every Automorian honours and loves the same Automorian?
2 2
quadrant. The formula of tangent of difference gives
Answer: Yes.
α1 α2 r1 r2
tan − tan − Solution. Denote by f (A) the Automorian loved by A and by g(A) the Automorian
α1 α2
2 2 = d d .
tan − = α α r 1 r2
honoured by A. The conditions of the problem imply the following:
2 2 1 + tan
1
· tan
2
1+ ·
2 2 d d • for every Automorian A, there exists an Automorian C such that f (C) = A;
Representing the result in the form • for every Automorian C , f (g(C)) = f (C);
• for every Automorian C , g(f (C)) = g(C).
α α r 1 − r2 r − r
1 2
tan − = rr = 1 2
√ ,
2 2 d+
1 2 √ d r 1 r2 We will show that f (A) = g(A) for every A. Applying f to both sides of the third
d r1 r2 √ +
r1 r2 d condition, we get
we obtain that the value of the denominator of the last expression is minimal in the case f (g(f (C))) = f (g(C)).
√
d = r1 r2 . Now
Using the second condition in both sides, we get
√ √
α1 r1 r1 α2 r2 r2
tan = =√ , tan = =√ , f (f (C)) = f (C).
2 d r2 2 d r1
α1 α2 α1 α2 π Using the first condition, this implies
i.e. tan and tan are reciprocals of each other. Therefore + = and
2 2 2 2 2 f (A) = A
∠P MQ1 + ∠P MQ2 = α1 + α2 = π.
In the other case when c1 and c2 are on the different sides from l (see Figure 22), the for all A. Using the second condition again, we obtain the desired result:
maximal size of the angle Q1 MQ2 is π which is the greatest size an angle can have. In
this case, the point M lies on the common tangent to c1 and c2 intersecting l. Then f (A) = f (g(A)) = g(A).
∠P MQ1 + ∠P MQ2 = ∠Q1 MQ2 = π. Thus, every Automorian loves and honours himself.
2. On the planet Automory, there are infinitely many inhabitants. Every Automorian
3. Find all pairs (x, y) of positive integers satisfying the equation
loves exactly one Automorian and honours exactly one Automorian. Additionally, the
following can be noticed: (x + y)x = xy .
17 18
Answer: (2, 6) and (3, 6). We see that in both inequalities, equality actually holds. Consequently, x1 = x2 ,
Solution. We have xy = (x + y)x > xx implying y > x. Let y = nx where n > 1 is a 2
x10 = x20 = x30 , and − a = 4. From the latter, we find a = −6. Thus x10 x20 x30 = 8
rational number. From the equality given in the problem, we get 3
leading to x10 = x20 = x30 = 2 which gives b = 12.
(x + nx)x = xnx . On the other hand, taking a = −6, b = 12 gives 6x2 − 24x − 4a = 6(x − 2)2 and
x3 + ax2 + bx − 8 = (x − 2)3 satisfying the conditions of the problem.
1
Raise both sides to the power of and then divide them by x; we obtain
x 5. On a horizontal line, 2005 points are marked, each of which is either white or black.
1 + n = xn−1 . (1) For every point, one finds the sum of the number of white points on the right of it and
the number of black points on the left of it. Among the 2005 sums, exactly one number
p occurs an odd number of times. Find all possible values of this number.
On the right hand side, the exponent n − 1 can be represented as a reduced fraction ,
q
therefore the number Answer: 1002.
p√ Solution. It is easy to see that the sums computed for a white point V and a black point
xn−1 = x q =q
xp M immediately following V on its right are equal. Note also that the sums are equal
also if the two points of different colour lie in the opposite order. If one interchanges
is either natural or irrational. The left hand side of (1) cannot be irrational, thus it is
two consecutive points of different colour, only the two equal sums corresponding to
natural. Hence n is natural. By choice, n > 1.
these two points change giving rise to two new equal sums. Hence, for any k, such
If n = 2, (1) gives x = 3 implying y = 2x = 6. If n = 3, (1) gives x = 2 implying transitions preserve parity of the number of occurrences of k among the 2005 sums.
y = 3x = 6. Note that (1) implies x > 2, thus 1 + n > 2n−1 . The latter inequality does
Assume there are n white and 2005 − n black points on the line. By a sequence of
not hold in the case n > 4. Hence no more solutions exist.
transitions described, collect all white points to the left. Then, going from left to right,
the corresponding sums are
Second Day
n − 1, n − 2, . . . , 1, 0, 0, 1, . . . , 2003 − n, 2004 − n.
4. Find all pairs (a, b) of real numbers such that the roots of polynomials 6x2 −24x−4a According to the invariant discovered in the first paragraph, exactly one number must
and x3 + ax2 + bx − 8 are all non-negative real numbers. occur an odd number of times also in this sequence. As the middle numbers occur in
pairs, the single number occurring an odd number of times is either the leftmost n − 1
Answer: (−6, 12). or the rightmost 2004 − n. If the former case takes place, then n − 2 = 2004 − n leading
Solution. Let x1 , x2 be the roots of the first polynomial and x10 , x20 , x30 be the roots of the to n = 1003 and n − 1 = 1002. If the other case takes place, then n − 1 = 2003 − n, giving
2 n = 1002 and 2004 − n = 1002 just like in the first case.
other polynomial. Division of the first polynomial by 6 gives x2 − 4x − a whose roots
3
are x1 and x2 , too. By Viète’s formulae,
6. On a plane, line l and a circle having no common points are given. Let AB be the
2 diameter of the circle being perpendicular to l whereby B is nearer to l than A. Let C
x1 + x2 = 4, x1 x2 = − a be a point on the circle different from both A and B. Line AC intersects l at point D.
3
Points B and E, the latter obtained as the tangency point of a line drawn from D to the
and circle, lie on the same side from AC . Line EB intersects l at point F ; line F A intersects
the circle second time at point G. Prove that the point symmetric to G with respect to
x10 + x20 + x30 = −a, x10 x20 + x20 x30 + x30 x10 = b, x10 x20 x30 = 8. AB lies on F C .
Now Solution. See IMO-2004 Shortlist.
2 2
4 x1 + x2 2
4= = > x1 x2 = − a
2 2 3
and
2 2 p
− a = (x10 + x20 + x30 ) > 2 3 x10 x20 x30 = 4.
3 3
19 20
WE THANK:
Estonian Math Competitions
2005/2006
Tallinn City Government
Tartu City Government
Narva City Government
The Gifted and Talented Development Centre
Tartu 2006
D N C
Selected Problems from Open Contests
A M B
1. A farmer noticed that, during the last year, there were exactly as many calves born
as during the two preceding years together. Even better, the number of pigs born during Figure 1
the last year was one larger than the number of pigs born during the two preceding
years together. The farmer promised that if such a trend will continue then, after some
years, at least twice as many pigs as calves will be born in his cattle, even though this far divisible by 3, a2004 is divisible by 3 and so is a. But then a2004 is also divisible by 9
this target has not yet ever been reached. Will the farmer be able to keep his promise? making the sum of its digits divisible by 9. Since 2004 is not divisible by 9, the sum of
(Juniors.) the digits of a2004 can not be equal to 2004, a contradiction.
b) Assume there exists such a number a. We will use the fact that a natural number and
Answer: no.
its sum of digits give the same remainder when divided by 3. It follows that a2006 ≡
Solution. Let Fn and Gn be the numbers of calves and pigs born during the nth year,
2006 ≡ 2 mod 3. On the other hand, the number a2006 = (a1003 )2 is a perfect square that
respectively. We will prove that if the farmer’s promise has been false during the pre-
can not give the remainder 2 when divided by 3, a contradiction.
vious years, it will remain so after the nth year as well. From the problem statement,
we have Fn = Fn−1 + Fn−2 and Gn = Gn−1 + Gn−2 + 1. If the number of born pigs
was less than twice the number of born calves during the previous years, we must have 4. A 9 × 9 square is divided into unit squares. Is it possible to fill
Gn−1 6 2Fn−1 − 1 and Gn−2 6 2Fn−2 − 1. Consequently, each unit square with a number 1, 2, . . . , 9 in such a way that, when-
ever one places the tile so that it fully covers nine unit squares, the
Gn = Gn−1 + Gn−2 + 1 6 2Fn−1 − 1 + 2Fn−2 − 1 + 1 = tile will cover nine different numbers? (Juniors.)
= 2( Fn−1 + Fn−2 ) − 1 = 2Fn − 1
Answer: no.
or Gn 6 2Fn − 1, i.e. the promise will not be true during the nth year. Since the farmer Solution 1. Assume that the numbers can be written in the required way. Put the tile over
has kept the cattle for at least two years, we can claim by induction that the promise will the central square; w.l.o.g. we can assume that the numbers are placed like in Figure 2,
never come true. left. Next move the tile like in Figure 2, middle. Two upper left vacant squares can have
neither 6 nor 7. Thus we must have 8 and 9 there, in some order. Now place the tile like
2. Let ABCD be a parallelogram, M the midpoint of AB and N the intersection of CD in Figure 2, right. We can see that either way we must cover number 8 twice, hence the
and the angle bisector of ABC. Prove that CM and BN are perpendicular iff AN is the required configuration of numbers does not exist.
angle bisector of DAB. (Juniors.)
8, 9
Solution. The triangle CNB is isosceles since ∠CNB = ∠ MBN = ∠CBN (see Figure 1). 8, 9
Thus we have | NC| = | BC|. 1 2 3 4 5 1 2 3 4 5 1 2 3 4 5
Assume first CM ⊥ BN. Since BN is both bisector and altitude for triangle BMC, we 6 8 6 8 6 8
7 9 7 9 7 9
have | BM| = | BC|. Consequently | BM| = |CN |, implying that N is the midpoint of CD
and MN is a segment joining the midpoints of the sides of the parallelogram. Then we
must have | AM| = | DN | = | NC| and | AD | = | MN | = | BC|. Thus the sides of AMND Figure 2
are equal and we have a rhombus. Its diagonal AN bisects DAM.
Assume now that AN bisects DAB. Then ∠ DN A = ∠ BAN = ∠ DAN, which implies Solution 2. Put the tile over the central square; w.l.o.g. we can assume that the numbers
| DN | = | DA|. On the other hand, | DA| = |CB| = | NC|. Thus N is the midpoint of CD. are written like in Figure 3, left. We will analyse which number can be written into the
Since M is the midpoint of AB, we have that MBCN is a rhombus with the diagonals gray central square. Moving the tile one position left, we see that the central square can
CM and BN being perpendicular. not contain 1, 2, 3 or 4. Moving the tile one position right, we see that the central square
can not contain 5. Moving the tile one position down, we see that the central square can
3. Does there exist a natural number with the sum of digits of its kth power being not contain 6, 7, 8 or 9. Thus the required numbering is not possible.
equal to k, if a) k = 2004; b) k = 2006? (Juniors.)
5. Find all real numbers with the following property: the difference of its cube and
Answer: a) no; b) no. its square is equal to the square of the difference of its square and the number itself.
Solution. a) Assume there exists such a number a. Since the sum of digits of a2004 is (Juniors.)
1 2
1 2 3 4 5 1 2 3 4 5 1 2 3 4 5 1 2 3 4 5 A′ D′
6 8 6 8 6 8 6 8
7 9 7 9 7 9 7 9
A D
Figure 3 P
C
B
C′
B′
Figure 5
Figure 4 Solution 2. Independently of the location of P, the equalities
| PA| · | PA′ | = | PC| · | PC′ |, | PB| · | PB′ | = | PD | · | PD ′ |
Answer: 0, 1 and 2.
Solution 1. Let x be a number with this property. Then x3 − x2 = ( x2 − x )2 , which are valid. Multiplying these, we get
leads to the equation x4 − 3x3 + 2x2 = 0 or x2 ( x2 − 3x + 2) = 0. Hence x2 = 0 or
x2 − 3x + 2 = 0. Solving these quadratic equations, we get x0 = 0, x1 = 1, x2 = 2. | PA| · | PB| · | PA′ | · | PB′ | = | PC| · | PD | · | PC′ | · | PD ′ |.
Solution 2. Transform the equation x3 − x2 = ( x2 − x )2 to obtain x2 ( x − 1) = x2 ( x − 1)2 . Points A, B, C, D are concyclic if and only if
Therefore x2 ( x − 1)2 − x2 ( x − 1) = 0 or x2 ( x − 1)( x − 2) = 0. Clearly, the solutions of
the last equation are 0, 1, 2. | PA| · | PB| = | PC| · | PD |
6. A solid figure consisting of unit cubes is shown in the picture. Is it or, taking into account the previous equality, if and only if
possible to exactly fill a cube with these figures if the side length of the
| PA′ | · | PB′ | = | PC′ | · | PD ′ |,
cube is a) 15; b) 30? (Juniors.)
Answer: a) no; b) yes. which holds if and only if A′ , B′ , C′ , D ′ are concyclic.
Solution. a) Since the figure consists of four unit cubes, the number of unit cubes in every Note. As can be seen from solution 2, the assertion of the problem holds regardless of
solid composable from these figures is divisible by 4. Since the cube with side length 15 positions of the circles.
contains an odd number of unit cubes, this cube is not among these solids.
8. A computer outputs the values of the expression (n + 1) · 2n for n = 1, n = 2, n = 3,
b) From two figures, it is possible to assemble a cube with side length 2 (see Figure 4).
etc. What is the largest number of consecutive values that are perfect squares? (Juniors.)
From these cubes, it is possible to assemble a cube with side length 30.
Answer: 2.
7. Two non-intersecting circles, not lying inside each other, are drawn in the plane. Solution. Two consecutive values can be perfect squares, for example, for n = 7 and
Two lines pass through a point P which lies outside each circle. The first line intersects n = 8 we get 8 · 27 = (25 )2 and 9 · 28 = (3 · 24 )2 .
the first circle at A and A′ and the second circle at B and B′ ; here A and B are closer
Now prove that three consecutive values cannot be perfect squares. Assume that (n +
to P than A′ and B′ , respectively, and P lies on segment AB. Analogously, the second
1) · 2n and (n + 3) · 2n+2 are both perfect squares. If n is even then both 2n and 2n+2
line intersects the first circle at C and C′ and the second circle at D and D ′ . Prove that
are perfect squares. Therefore also n + 1 and n + 3 must be perfect squares, which is
the points A, B, C, D are concyclic if and only if the points A′ , B′ , C′ , D ′ are concyclic.
impossible. If n is odd, i.e. n = 2k + 1 for some k > 0, then (n + 1) · 2n = (2k + 2) ·
(Juniors.)
22k+1 = (k + 1) · 22k+2 and (n + 3) · 2n+2 = (2k + 4) · 22k+3 = (k + 2) · 22k+4 . Here, the
Solution 1. Since A, A′ , C′ , C are concyclic (see Figure 5), we have ∠ AA′ C′ + ∠ ACC′ = factors 22k+2 and 22k+4 are perfect squares, therefore also k + 1 and k + 2 must be perfect
180◦ , hence ∠ B′ A′ C′ = ∠ ACD. Analogously ∠C′ D ′ B′ = ∠ DBA. Points A, B, C, D are squares, which is impossible for non-negative k.
concyclic if and only if ∠ ACD = ∠ DBA, which is equivalent to ∠ B′ A′ C′ = ∠C′ D ′ B′ ,
the last equality holds if and only if points A′ , B′ , C′ , D ′ are concyclic. 9. Let a, b, c be positive integers. Prove that the inequality
( x − y ) a ( x − z)b (y − z)c > 0
3 4
starts all over. Find all possible values for p1 and q1 such that the teacher can hold Juku
at school forever. (Seniors.)
Answer: either p1 is any integer and q1 = 0 or p1 = 1 and q1 = −2.
Solution. If q1 = 0 then we have the equation x2 + p1 x = 0 with solutions − p1 and 0.
The teacher can write another equation x2 − p1 x = 0 with solutions p1 ja 0, then again
Figure 6 Figure 7 x2 + p1 x = 0, etc. Thus all pairs ( p1 , 0) satisfy the conditions of the problem.
If q1 = −1 then the product of the solutions must be −1 and the solutions −1 and 1
in some order. Since the equations x2 − x + 1 = 0 and x2 + x − 1 = 0 have no integer
holds for all reals x, y, z if and only if a, b, c are even. (Juniors.) solutions, no pairs of the form ( p1 , −1) satisfy the conditions of the problem.
Solution. If a, b, c are positive then the inequality holds. On the other hand, assume that If q1 = −2 then the product of the solutions must be −2 and the solutions are either
the inequality holds for all reals x, y, z. Choosing z < x < y and dividing the given 2 and −1 or 1 and −2. In the first case, the teacher can choose between the equations
inequality by a positive number ( x − z)b (y − z)c , we get the inequality ( x − y)a > 0, x2 + 2x − 1 = 0 and x2 − x + 2 = 0, none of them having integer solutions. In the second
from which we conclude due to x − y < 0 that a is even. Analogously, choosing y < case, the teacher can write the equation x2 + x − 2 = 0 with solutions 1 and −2. Thus
z < x, we conclude that c is even. Finally, choosing x < y < z, we get after dividing we see that the pair (1, −2) staisfies the conditions of the problem.
the inequality by a positive number ( x − y)a (y − z)c that ( x − z)b > 0 from which we Now let q1 be an integer not equal to 0, −1 nor −2. If x1 and x2 are the solutions of
conclude due to x − z < 0 that b is even. x2 + p1 x + q1 = 0, Viète formulae imply that x1 + x2 = − p1 , x1 x2 = q1 and
10. All the streets in a city run in one of two perpendicular directions, forming unit x12 + x22 = ( x1 + x2 )2 − 2x1 x2 = p12 − 2q1 < p12 + q12 .
squares. Organizers of a car race want to mark down a closed race track in the city in Thus the sum of squares of the coefficients of the equation strictly decreases for q1 6∈
such a way that it would not go through any of the crossings twice and that the track [−2; 0]. Since sum of squares is non-negative, we must sooner or later reach one of the
would turn 90◦ right or left at every crossing. Find all possible values of the length of two situations: the solutions are not integers or the constant term belongs to the interval
the track. (Seniors.) [−2; 0]. The latter case is impossible, since every equation x2 + px + q = 0 uniquely
Answer: all positive integers divisible by 4, except for 8. determines its predecessor x2 − ( p + q) x + pq = 0, hence the pairs ( p1 , 0) ja (1, −2) can
Solution. Define natural coordinates with the origin at some crossing and consider two only arise from equations with constant terms 0 or −2, respectively. Thus there are no
consecutive track fragments of length 1. One of them is parallel to x-axis and the other other pairs of numbers satisfying the conditions of the problem.
one to y-axis; moving along the first one, parity of the x-coordinate changes, and mov-
ing along the second one, parity of the y-coordinate changes. Moving along the track, 12. Let ABC be an acute triangle and choose points A1 , B1 and C1 on sides BC, CA
parities of x- and y-coordinates change alternately, but when returning to the initial and AB, respectively. Prove that if the quadrilaterals ABA1 B1 , BCB1 C1 and CAC1 A1 are
point, both parities must be the same as in the beginning. Since the pairs of parities cyclic then their circumcentres lie on the sides of ABC. (Seniors.)
repeat after every four track fragments, the length of the track must be divisible by 4. Solution. Since BCB1 C1 is cyclic (see Figure 8), we have ∠ BB1 C = ∠ BC1 C = α. Similar-
There exists a suitable track of length 4 going around one block. There is no track of ily, let ∠CC1 A = ∠CA1 A = β and ∠ AA1 B = ∠ AB1 B = γ. Considering the angles with
length 8. If there were one, it would have four fragments parallel to x-axis and four vertices at points A1 , B1 and C1 , we get the following system of equations:
fragments parallel to y-axis. Hence, we could not deviate more than 2 units in either
direction and the whole track should fit into 2 × 2 square. It is easy to see that the track β + γ = 180◦
can not contain three corners of the square, but then there will be less that 8 possible γ + α = 180◦
turning points left. α + β = 180◦ .
All the other positive integers divisible by 4 are attainable. Figure 6 shows a track of Adding the equalities and dividing by 2 gives α + β + γ = 270◦ , implying α = β = γ =
length 12 and we can increase this length repeatedly by 4 units using the operation in 90◦ . Thus the segments BC, CA and AB are the diameters of the circles and contain their
Figure 7. circumcentres.
11. After the schoolday is over, Juku must attend an extra math class. The teacher s1
13. Martin invented the following algorithm. Let two irreducible fractions and
writes a quadratic equation x2 + p1 x + q1 = 0 with integer coefficients on the blackboard t1
and Juku has to find its solutions. If they are not both integers, Juku may go home. If the s2
be given as inputs, with the numerators and denominators being positive integers.
solutions are integers, then the teacher writes a new equation x2 + p2 x + q2 = 0, where t2
Divide s1 and s2 by their greatest common divisor c and obtain a1 and a2 , respectively.
p2 and q2 are the solutions of the previous equation taken in some order, and everything
Similarily, divide t1 and t2 by their greatest common divisor d and obtain b1 and b2 ,
5 6
A smallest on the board and strictly less than the other number in the pair. The situation
with all numbers being equal to zero can this way only occur after B’s move.
B1
Now let m be odd and n even. If none of the numbers is yet zero, B can ensure that after
C1
his move the following conditions hold: the smallest number on the board is even, the
quantity of the smallest numbers is odd and the number of occurrencies of every other
number is even. Indeed, if A changes the smallest number then B can change it again,
but if A changes some other number then B can change another number equal to the
B A1 C
one A changed. It goes on until some number becomes zero, afterwards B can divide
all the remaining numbers into pairs and use the strategy described above.
Finally, let m and n be odd. Then after A’s first move, there is a position described in
the previous paragraph on the board. Thus A can use the strategy of B and win.
Figure 8 15. Kati cut two equal regular n-gons out of paper. To the vertices of both n-gons, she
wrote the numbers 1 to n in some order. Then she stabbed a needle through the centres
a1 b2 + a2 b1 of these n-gons so that they could be rotated with respect to each other. Kati noticed
respectively. After that, form a new fraction , reduce it, and multiply the that there is a position where the numbers at each pair of aligned vertices are different.
t1 b2
numerator of the result by c. Martin claims that this algorithm always finds the sum of Prove that the n-gons can be rotated to a position where at least two pairs of aligned
the original fractions as an irreducible fraction. Is his claim correct? (Seniors.) vertices contain equal numbers. (Seniors.)
Answer: yes. Solution. Assume that the lower n-gon is fixed and move the upper n-gon. Let the initial
Solution. Since position of n-gons be the one found by Kati. For each vertex of the upper n-gon, there
is an angle by which rotating clockwise the upper n-gon brings this vertex atop of the
s 1 t2 s 2 t1 vertex of the lower n-gon with the same number. There are n different vertices, but only
c· · + ·
c · (a1 b2 + a2 b1 ) c d c d s 1 t2 + s 2 t1 s s n − 1 different rotation angles since the angle 0◦ is excluded by conditions. Hence for
= = = 1 + 2,
t1 b2 t2 t1 t2 t1 t2 two vertices of the upper n-gon, the rotation angles are equal.
t1 ·
d
16. A real-valued function f satisfies for all reals x and y the equality
the resulting fraction has correct value. We still need to prove that it is irreducible. For
that, it is enough to show that the numbers c and t1 b2 are relatively prime. f ( xy) = f ( x )y + x f (y).
Suppose there exists a prime p dividing both c and t1 b2 . Since c = gcd(s1 , s2 ), we have
Prove that this function satisfies for all reals x and y 6= 0 the equality
that p also divides both s1 and s2 . Consequently, t1 and t2 can not be divisible by p,
s1 s2
because the fractions and are irreducible. Thus p does not divide t1 t2 , implying x f ( x )y − x f (y )
t1 t2 f = .
y y2
t2
that p can not divide t1 b2 = t1 · either, a contradiction.
d (Seniors.)
f ( xy) − x f (y)
14. Two players A and B play the following game. Initially, there are m equal positive Solution. From the given expression we obtain f ( x ) = , this equality
integers n written on a blackboard. A begins and the players move alternately. The y
x
player to move chooses one of the non-zero numbers on the board. If this number k is holds for any reals x and y 6= 0. Taking at place of x, we get
y
the smallest among all positive integers on the board, the player replaces it with k − 1;
if not, the player replaces it with the smallest positive number on the board. The player x x
f · y − f (y )
who first turns all the numbers into zeroes, wins. Who wins if both players use their x y y f ( x )y − x f (y )
best strategies? (Seniors.) f = = .
y y y2
Answer: A wins if mn is odd; B wins if mn is even. Note. It is possible to prove (for example, using reduction to the Cauchy equation), that
Solution. If the quantity m of numbers is even then B has the following winning strategy. the only continuous functions satisfying the given conditions are
B divides all the numbers into pairs and if A makes a move and changes some number, (
B changes the other number in the pair, ensuring that after his move all pairs contain ax ln | x |, if x 6= 0,
f (x) =
equal numbers. This is possible, since after A’s move the number A wrote must be the 0, if x = 0.
7 8
F
is 7 − 1 · 3 = 4. This means that the term a361 is divisible by both 17 and 118, i.e. it is
divisible by 2006.
Note 1. Since 17 and 118 are relatively prime, the existence of the suitable n follows from
O
the Chinese Remainder Theorem: the remainders of n upon division by 17 and 118 must
be 4 and 7, respectively.
E D Note 2. The least number satisfying the given conditions corresponds to n = 87, in this
A
case a87 = 627878 = 313 · 2006.
B C 19. Let n > 2 be a fixed integer and let ai,j (1 6 i < j 6 n) be some positive integers.
For a sequence x1 , . . . , xn of reals, let K ( x1 , . . . , xn ) be the product of all expressions
G ( xi − x j )ai,j where 1 6 i < j 6 n. Prove that if the inequality K ( x1 , . . . , xn ) > 0 holds
independently of the choice of the sequence x1 , . . . , xn then all integers ai,j are even.
(Seniors.)
Figure 9 Solution 1. Suppose the contrary: some of the numbers ai,j are odd. Let l be the smallest
index for which there are odd numbers among the numbers ai,l (1 6 i < l); also let k
17. Four points A, B, C, D are chosen on a circle in such a way that arcs AB, BC, and be the largest index for which ak,l is odd. Then ak,l is the only odd number among the
CD are of the same length and the arc DA is longer than these three. Line AD and the numbers ai,j where k 6 i < j 6 l. Now choose x1 , . . . , xn as follows:
line tangent to the circle at B intersect at E. Let F be the other endpoint of the diameter x1 > x2 > . . . > x k −1 > x l > x k +1 > . . . > x l −1 > x k > x l +1 > . . . > x n ;
starting at C of the circle. Prove that triangle DEF is equilateral. (Seniors.)
i.e., choose some n numbers in decreasing order and swap the positions of the k-th and
Solution 1. Let O be the centre of the circle and let G be the point where BE intersects
the l-th number. Then in the given expression, the factors ( xi − x j )ai,j , where i < k or
the tangent drawn from D to the circle. Since the total length of arcs DA and AB is
j > l, are positive, since the bases of the power are positive. All the remaining factors
larger than the total length of arcs BC and CD, the points G and E lie on different sides
( xi − x j )ai,j where k 6 i < j 6 l have even exponents with the exception of ( xk − xl )ak,l ,
from B (see Figure 9). Points B and D are symmetric with respect to the line CF, hence
1 which has negative base and odd exponent. So the whole product is negative.
G lies on the line CF and ∠ BGF = ∠ DGF. Now ∠ BED = (∠ BOD − ∠ AOB) = Solution 2. Assume that ak,l is odd for some indices k and l. Fix x1 , . . . , xk−1 , xk+1 , . . . , xn
2
1 in such a way that they are pairwise different and consider the product K ( x1 , . . . , xn ) as
∠COD = ∠CFD, therefore ∠GED = ∠GFD. Consequently G, E, F, D are concyclic. a polynomial of one variable K ( xk ). The root xl of this polynomial has odd multiplicity.
2
Since ∠EGF = ∠ DGF, chords EF and DF of the circumcircle of quadrangle GEFD are Therefore the graph of K ( xk ) intersects the x-axis at xl and we can choose xk such that
equal. K ( xk ) is negative, a contradiction.
Solution 2. Since the lengths of the arcs BA and BC are equal, the lines EB and AC
are parallel. Therefore ∠ BED = ∠CAD = ∠ BDA. Thus triangle BED is equilateral.
Further, ∠CBF = 90◦ . By symmetry, the lines BC and AD are parallel, hence BF is Selected Problems from the Final Round of National
perpendicular to AD. Segment BF is the altitude of the equilateral triangle BED, it Olympiad
bisects its base ED. This segment is also the altitude of the triangle DEF and it bisects
its base. This is possible only when the triangle DEF is equilateral.
Note. The assertion of the problem holds also in the case when the arc DA is shorter 1. Find all pairs of positive integers (a, b) such that
than the other three, the point G then lies on the other side.
ab = gcd(a, b) + lcm(a, b).
18. In the sequence (an ) with general term an = n3 − (2n + 1)2 , does there exist a term
that is divisible by 2006? (Seniors.) (Grade 9.)
Answer: yes. Answer: (2, 2).
Solution. First, a4 = 43 − 92 = −17 and a7 = 73 − 152 = 118. Since n3 − (2n + 1)2 Solution. As the left-hand side and summand lcm(a, b) on the right-hand side are both
is a polynomial, a4+17k is divisible by 17 and a7+118l is divisible by 118 for all natural divisible by a, also gcd(a, b) has to be divisible by a. On the other hand, gcd(a, b) 6 a as
numbers k and l. Since 119 is divisible by 17, adding 118 decreases the remainder by 1 a is positive. Thus gcd(a, b) = a. Analogously we obtain that gcd(a, b) = b. Therefore
on division by 17. Therefore, when 361 = 7 + 118 · 3 is divided by 17, the remainder a = b and the equation has the form a2 = a + a or a(a − 2) = 0. The only positive
solution of the equation is a = 2 and thus also b = 2.
9 10
C
√ 2 2
3c 1 3c c 2 3c c
h: = tan 30◦ = √ , therefore h = . Hence b2 = h2 + = + = c2 ,
2 3 2 2 4 4
a
b b
a giving b = c. We now have that CDE is an equilateral triangle and its angles are 60◦ .
M h |CH | 1 | DH | 1
Solution 3. As = sin 30◦ = and = , segment CD satisfies the Angle
30 ◦ 30 ◦ |CA| 2 | DA| 2
A c D c H c E c B Bisector Property. Hence CD bisects angle ACH whose size is 60◦ and ∠ DCH = 30◦ .
2 2 Then the vertex angle of the isosceles triangle CDE is 60◦ and the base angles are also
60◦ .
Solution 4. Let the medians of triangle CDE meet at M. By the Ray Property, | DM| =
C′ 1 1 1
| AC| = | BC| = | EM|. We obtain |CH | = | AC| as in the previous solution and thus
3 3 2
Figure 10 1
|CM| = | AC|. So |CM| = | DM| = | EM|, i.e., the intersection point M of medians of
3
CDE is also the circumcentre of CDE. It follows that CDE is an equilateral triangle and
Solution 2. It is known that ab = gcd(a, b) lcm (a, b) for positive integers a and b. Thus all its angles are of size 60◦ .
we obtain the equality gcd(a, b) lcm (a, b) = gcd(a, b) + lcm(a, b) which is equivalent to Solution 5. Assume that ∠ DCE > 60◦ . Then ∠CED < 60◦ . Hence | DE| > |CD | giving
(gcd(a, b) − 1) (lcm(a, b) − 1) = 1. The last equality expresses 1 as a product of two non- also | AD | > |CD |. Then in triangle ACD, we have ∠ ACD > 30◦ , and in triangle
negative integers; this is only possible if both of them are equal to 1. Thus gcd(a, b) − BCE by symmetry, ∠ BCE > 30◦ . Adding the inequalities, we get ∠ ACB > 120◦ —
1 = 1 and lcm(a, b) − 1 = 1 or gcd(a, b) = 2 and lcm(a, b) = 2. Hence a = b = 2. a contradiction. Analogously we obtain a contradiction by starting with assumption
Solution 3. Let gcd(a, b) = d. We represent integers a and b as a = da′ and b = db′ ∠DCE < 60◦ . Hence ∠DCE = 60◦ and thus ∠CDE = ∠CED = 60◦ .
where a′ and b′ are relatively prime. Then lcm(a, b) = da′ b′ . The equation gets the
1 4. Consider a rectangular grid of 10 × 10 unit squares. We call a ship a figure made
form d2 a′ b′ = d + da′ b′ , or a′ b′ = . As a′ b′ is an integer, we must have d − 1 = 1
d−1 up of unit squares connected by common edges. We call a fleet a set of ships where no
and a′ b′ = 1. Hence d = 2 and a′ = b′ = 1. Therefore the solution of the equation is two ships contain squares that share a common vertex (i.e. all ships are vertex-disjoint).
a = da′ = 2 and b = db′ = 2. Find the least number of squares in a fleet to which no new ship can be added. (Grade
9.)
2. Let there be n > 2 real numbers such that none of them is greater than the arith-
metic mean of the other numbers. Prove that all the numbers are equal. (Grade 9.) Answer: 16.
Solution. Call a fleet full if no new ships can be added. We have to find the least number
Solution. Let a be the greatest among the numbers. Suppose, by contradiction, that not
of squares in a full fleet.
all the numbers are equal. Then there must be some numbers less than a. Considering
First we show that a full fleet covering 16 unit squares exists. Put on the grid 16 one-
numbers other than a, we therefore know that their arithmetic mean is less than a. But
square ships as shown in Figure 11. Note that then each square of the grid has a common
this contradicts the conditions of the problem. Hence all the numbers are equal.
vertex with one of those ships and thus no ship can be added.
3. Triangle ABC is isosceles with AC = BC and ∠C = 120◦ . Points D and E are Second we prove that there can not be fewer than 16 unit squares in a full fleet. Suppose
chosen on segment AB so that | AD | = | DE| = | EB|. Find the sizes of the angles of a full fleet is fixed. Consider the set of 16 unit squares painted gray in Figure 11. For
triangle CDE. (Grade 9.) each of these 16 squares, there is a square of the full fleet that shares (at least) a common
Answer: all the angles are 60◦ .
Solution 1. The base angle of the isosceles triangle ABC is (180◦ − 120◦ ) : 2 = 30◦ . Let
H be the foot of the altitude drawn from vertex C (see Figure 10). Reflect the triangle
ABC with respect to side AB, the point C going to C′ . As ∠CAC′ = 60◦ and | AC| =
| AC′ |, triangle ACC′ is equilateral and AH is its median. Moreover, point D divides the
median with ratio 2 : 1. Thus the medians of ACC′ meet at D, and CD is both a median
and an angle bisector to ACC′ . We obtain ∠ DCH = 30◦ and ∠ DCE = 60◦ . As CDE is
isosceles, ∠CDE = ∠CED = 60◦ .
Solution 2. Denote |CA| = |CB| = a, |CD | = |CE| = b and | AD | = | DE| = | EB| = c for
brevity. Let h be the altitude drawn from vertex C to side AB. In triangle CH A, we have Figure 11
11 12
diameter is also an integer. Find all possible side lengths of the triangle. (Grade 11.)
Answer: 3, 4 and 5.
Solution. Let p and m be the lengths of the sides of the triangle and let n be the length of
the hypotenuse, where p is a prime (Figure 14). Then p2 + m2 = n2 , implying p2 = (n −
m)(n + m). Since p is prime, the terms of the right-hand side must satisfy n − m = 1,
n + m = p2 .
The perimeter of the triangle is p + m + n. In order to find the incircle diameter d, we
note that the total length of the segments tangent to the incircle, originating from the
Figure 12 Figure 13 vertex of the right angle, equals the length of the incircle diameter, whereas the total
length of the four tangents originating from the other two vertices is 2n. Thus, d + 2n =
p + m + n. According to the assumption, p + m + n is divisible by d = p + m − n.
vertex with it. All these squares of the fleet must be different. Hence there are at least Substituting m + n and m − n from above, we get that p + p2 = p( p + 1) is divisible by
16 squares in the fleet. p − 1. Since p and p − 1 are coprime, it must be that p + 1 is divisible by p − 1. Hence,
p − 1 = 1 or p − 1 = 2. We see that p = 2 is impossible, since n − m and n + m cannot
5. Consider a rectangular grid of 10 × 10 unit squares. We call a ship a figure made
be of different parity. Thus, p = 3, m = 4 and n = 5. A triangle with side lengths 3, 4, 5
up of unit squares connected by common edges. We call a fleet a set of ships where no
is clearly a right triangle.
two ships contain squares that share a common vertex (i.e. all ships are vertex-disjoint).
Find the greatest natural number that, for each its representation as a sum of positive
integers, there exists a fleet such that the summands are exactly the numbers of squares
contained in individual ships. (Grade 10.) p m
Answer: 25.
Solution. First we prove that, for all n > 25, we can divide n into summands so that a
n
fleet with respective ship sizes can not be put on the grid. In particular, we prove that
one can not put more than 25 ships of size 1 on the grid. Let us divide the grid into Figure 14
squares of 2 × 2 (as in Figure 12), there are 25 of them. As each 2 × 2 square can contain
at most one ship of size 1, then the total number of such ships is at most 25.
Second we show that, for any representation of 25 as a sum of positive integers, there is a 8. The sequence ( Fn ) of Fibonacci numbers satisfies F1 = 1, F2 = 1 and Fn = Fn−1 +
fleet with respective ship sizes. Let us initially put 25 ships of size 1 on the grid as shown Fn−2 for all n > 3. Find all pairs of positive integers (m, n), such that Fm · Fn = mn.
in Figure 13. Then, starting from the upper left corner, shift along the line together (Grade 11.)
as many ships as the first summand of the representation tells; then shift together as
Answer: (1, 1), (1, 5), (4, 6), (5, 1), (5, 5) and (6, 4).
many ships as the second summand tells etc. The fleet obtained this way satisfies the
conditions of the problem. Solution. By induction on n, it is easy to prove that Fn > n for all n > 6 and Fn > 2n for
all n > 8. Thus, if m > 6 and n > 6, then Fm · Fn > m · n. W.l.o.g., we may now assume
6. Find the greatest possible value of sin(cos x ) + cos(sin x ) and determine all real m 6 5 (the remaining solutions can be obtained by exchanging m and n).
numbers x, for which this value is achieved. (Grade 11.)
• If m = 1, then 1 · Fn = 1 · n, or Fn = n. From above, the only solutions are n = 1
Answer: the greatest possible value is sin 1 + 1, which is achieved iff x = 2kπ, where k and n = 5 and the suitable pairs are (1, 1), (1, 5), (5, 1) and (5, 5).
is an arbitrary integer.
Solution. Since the value of cos x is in the interval [−1; 1] and since sin x is increasing • If m = 2, we get 1 · Fn = 2 · n, or Fn = 2n. Since there are no solutions for n < 8,
in this interval, the greatest possible value of the first addend is sin 1, and the value is there are no solutions at all.
achieved iff cos x = 1, or x = 2kπ, where k is an arbitrary integer. The greatest possible 3 3
• If m = 3, we get 2 · Fn = 3 · n, or Fn = n. Since n < 2n, there are no solutions, as
value of the second addend is 1, which is achieved iff sin x = 0, or x = lπ, where l is 2 2
an arbitrary integer. Both terms achieve the maximal value simultaneously iff x = 2kπ, in the previous case.
where k is an arbitrary integer; the value of the expression is then sin 1 + 1. 4
• If m = 4, then 3 · Fn = 4 · n, or Fn = n. Here, the only solution is n = 6, giving
7. In a right triangle, the length of one side is a prime and the lengths of the other 3
pairs (4, 6) and (6, 4).
side and the hypotenuse are integral. The ratio of the triangle perimeter and the incircle
13 14
• If m = 5, then 5 · Fn = 5 · n, or Fn = n; this case is analysed above. Solution. First, we prove that a suitable sequence of moves exists for even n. Divide the
board into blocks of 2 × 2 squares (see Figure 16) and place the pawn on the upper left
corner square. To move through the first block, take the following moves: down-right,
9. In a triangle ABC with circumcentre O and centroid M, lines OM and AM are
up, down-left, down. Repeat the same combination of moves until the pawn reaches the
perpendicular. Let AM intersect the circumcircle of ABC again at A′ . Let lines BA′ and
bottom-most block in a column of blocks. In the bottom-most block, move down-right,
AC intersect at D and let lines CA′ and AB intersect at E. Prove that the circumcentre
left, up-right, and right; the first column of blocks is passed. In the bottom-most block
of triangle ADE lies on the circumcircle of ABC. (Grade 11.)
of the second column, move down-right, left, up-right, up, and continue by moving
upwards block by block. By passing the columns alternatingly up and down, the pawn
A
visits each square exactly once.
H G
O
M
B C
F
A′
Figure 16 Figure 17
D
E
Next, we prove that a sequence does not exist for odd n. Colour the second, fourth,
Figure 15 n2 − n
sixth, etc. row dark (see Figure 17), then there are dark squares. Note that every
2
Solution. Let F, G and H be the base points of the medians drawn from vertices A, B diagonal move starts from a dark square or ends on a dark square. Since a suitable
and C, respectively (see Figure 15). Then, triangle A′ OA is isosceles with height OM sequence does not visit a square twice and does not contain two consecutive diagonal
and | A′ M| = | MA|. Since the centroid divides a median in ratio 2 : 1, we get | FM| = moves, each diagonal move corresponds to a different dark square. Thus, we can make
1 n2 − n n2 − n
| MA|, and | A′ F| = | FM|. On the other hand, | BF| = | FC|. Hence, A′ BMC is a at most diagonal moves and, consequently, at most + 1 non-diagonal
2 2 2
parallelogram. Parallel sides then imply that triangles ABD and AHC are similar with moves, or n2 − n + 1 moves altogether. For n > 3, this number is smaller than n2 − 1,
similarity ratio 2 — the ratio of the lengths of AB and AH. Analogously, triangles ACE the number of moves required to visit all squares.
and AGB are similar with the same ratio. Homothecy with centre A and ratio 2 brings
triangle ABC to triangle AED, while the circumcentre O of triangle ABC is transformed 11. We call a ship a figure made up of unit squares connected by common edges.
to the second intersection point of AO and the said circumcircle. Prove that if there is an odd number of possible different ships consisting of n unit
Note. The use of homothecy can be avoided by finding the second intersection point squares on a 10 × 10 board, then n is divisible by 4. (Grade 12.)
P of line AO and the circle and by proving that | AP| = 2| AO|, | DP| = 2|CO| and Solution. Let n be such that the number of possible different ships of n squares is odd.
| EP| = 2| BO|. Divide all ships in classes, such that all ships in the same class are precisely those that
can be obtained from one another by shifts, vertical and horizontal reflections. Then
10. A pawn is placed on a square of a n × n board. There are two types of legal
there must exist a class with an odd number of ships.
moves: (a) the pawn can be moved to a neighbouring square, which shares a common
side with the current square; or (b) the pawn can be moved to a neighbouring square, Let L be a ship in such a class. Assume that L is not symmetrical w.r.t. either the vertical
which shares a common vertex, but not a common side with the current square. Any or horizontal axis of symmetry of its tight bounding box. Then no ship in this class is
two consecutive moves must be of different type. Find all integers n > 2, for which it is symmetrical w.r.t. this axis. Thus, we can divide all ships in this class into pairs: a ship
possible to choose an initial square and a sequence of moves such that the pawn visits and its reflection from this axis; a contradiction with the odd cardinality of this class.
each square exactly once (it is not required that the pawn returns to the initial square). Therefore, L must be symmetrical w.r.t. both the vertical and the horizontal axis.
(Grade 11.) The side lengths of the rectangle bounding L must be even, for there is an even number
of ways to place a rectangle with an odd side on a board with an even side length
Answer: n = 2k, where k is an arbitrary positive integer. 10; again a contradiction in parity. Thus, the vertical and horizontal axes of symmetry
15 16
divide the squares of L into four disjoint reflection symmetrical sets. Since there is an b) Assume that for some n, there exist n suitable integers a1 < a2 < . . . < an . Then, on
equal number of squares in each set, the total number of squares of the ship is divisible the one hand,
by 4.
a1 + a2 + . . . + a n − 2 < a n − 2 + a n − 2 + . . . + a n − 2 = ( n − 2 ) a n − 2
12. Find the smallest possible distance of points P and Q on a xy-plane, if P lies on
but, on the other hand,
the line y = x and Q lies on the curve y = 2x . (Grade 12.)
1 + ln ln 2 a n −1 a n > ( n − 1) a n > ( n − 2) a n −2 .
Answer: √ .
2 ln 2 Thus, a1 + a2 + . . . + an−2 < an−1 an , and the sum of a1 , a2 , . . . , an−2 can not be divisible
Solution. We find the minimum point of h( x ) = 2x − x. Since h′ ( x ) = 2x ln 2 − 1, we get by the product of an−1 and an .
2x ln 2 − 1 = 0, and Solution 2. a) Choose arbitrary pairwise distinct numbers b1 , b2 , . . . , bn and let m be
1 ln ln 2 the least common multiple of all sums of (n − 2) terms. For each i = 1, 2, . . . , n, take
2x = and x=− . ai = mbi . Then the product of any two numbers ak al is ak al = (mbk ) · (mbl ), which is
ln 2 ln 2
divisible by the sum of the remaining numbers, since m2 is divisible by this sum.
Since h′ ( x ) is increasing, this is indeed a minimum. The value of h at this point is
14. Let O be the circumcentre of an acute triangle ABC and let A′ , B′ and C′ be the
1 ln ln 2 1 + ln ln 2 circumcentres of triangles BCO, CAO and ABO, respectively. Prove that the area of
h( x ) = + = .
ln 2 ln 2 ln 2 triangle ABC does not exceed the area of triangle A′ B′ C′ . (Grade 12.)
Here 1 + ln ln 2 = ln(e ln 2). Since 2 < e < 4, we get ln(e ln 2) > ln(2 ln 2) = ln ln 4 > Solution 1. First, we prove that for a fixed circumcircle, a triangle with maximal area
> ln ln e = 0. Thus, the value of h at the minimum is positive, so the graph of g( x ) = 2x is equilateral. Assume that a triangle KLM with maximal area has two sides of un-
is always higher than the graph of f ( x ) = x. Consider points A( x, f ( x )) and B( x, g( x )) equal lengths, say, KM and LM. Take a point M ′ on the circumcircle of KLM such that
and let C be the projection of B to the graph of f . Then, triangle ABC is a right isosceles |KM′ | = | LM′ | (Figure 18). Triangles KLM and KLM′ have a common base but the
| AB| h( x ) altitude of the first triangle is smaller, a contradiction.
triangle, since ∠ BAC = 45◦ . Consequently, | BC| = √ = √ and the distance | BC|
2 2
is minimal iff h( x ) is minimal. The sought distance is thus M
M′
M
Q
h( x ) 1 + ln ln 2
√ = √ . M′
2 2 ln 2
log2 e − log2 log2 e
Note. The answer can be expressed in many different ways, e.g., by √ .
2 X
P
13. Prove or disprove the following statements. K L K L L′
a) For every integer n > 3, there exist n pairwise distinct positive integers such that
the product of any two of them is divisible by the sum of the remaining n − 2 num- Figure 18 Figure 19
bers.
Next, we prove that for a fixed incircle, a triangle with minimal area is equilateral.
b) For some integer n > 3, there exist n pairwise distinct positive integers, such that Assume w.l.o.g. that in a triangle KLM with minimal area, ∠KLM > ∠KML. Con-
the sum of any n − 2 of them is divisible by the product of the remaining two num- sider a triangle KL′ M′ , where points L′ and M′ lie on lines KL and KM such that
bers. ∠KL′ M′ = ∠KM′ L′ and L′ M′ is tangent to the incircle (Figure 19). Let X be the in-
tersection point of lines LM and L′ M′ . Draw perpendicular segments LP and MQ from
(Grade 12.)
points L and M to line L′ M′ . Then | XM′ | > | XL′ | and | MQ| > | LP|, since | XQ| > | XP|
Answer: a) true; b) false. and right triangles MQX and LPX are similar. Thus, the area of triangle MM ′ X is
Solution 1. a) Take n numbers (n2 )!, 2(n2 )!, 3(n2 )!, . . . , n(n2 )!. The product of any two greater than the area of triangle LL′ X and consequently, the area of KLM is greater than
of these numbers is divisible by (n2 )!(n2 )!, whereas the sum of the remaining numbers the area of KL′ M′ , a contradiction.
is k(n2 )!, where k is some positive integer smaller than 1 + 2 + . . . + n, which in turn is Now, let R be the circumradius of ABC. Since the sides of triangle A′ B′ C′ are perpen-
smaller than n2 . dicular to OA, OB and OC (see Figure 20) and bisect these segments, point O is the
17 18
A
15. The Ababi alphabet consists of letters A and B, and the words in the Ababi lan-
C′
guage are precisely those that can be formed by the following two rules:
B′
1) A is a word.
2) If s is a word, then s ⊕ s and s ⊕ s̄ are words, where s̄ denotes a word that is obtained
β
O β by replacing all letters A in s with letters B, and vice versa; and x ⊕ y denotes the
αα concatenation of x and y.
The Ululu alphabet consists also of letters A and B and the words in the Ululu language
B C are precisely those that can be formed by the following two rules:
A′
1) A is a word.
Figure 20 2) If s is a word, then s ⊗ s and s ⊗ s̄ are words, where s̄ is defined as above and x ⊗ y
is a word obtained from words x and y of equal length by writing the letters of x
R and y alternatingly, starting from the first letter of x.
incentre of A′ B′ C′ and the inradius is . An equilateral triangle with circumradius R
2 R
and an equilateral triangle with inradius have equal area S by the property of the Prove that the two languages consist of the same words. (Grade 12.)
2
centroid. From above, we now get S ABC 6 S 6 S A′ B′ C ′ . Solution. Since each step doubles the length of a word, both languages contain only
Solution 2. Let R be the circumradius of ABC and let α, β and γ be the angles at the words of length 2n , where each such word has been obtained in exactly n steps.
vertices of the triangle. Then ∠ BOC = 2α, ∠COA = 2β, and ∠ AOB = 2γ. Thus, First, we show that each language contains 2n words that can be obtained in exactly n
steps. Indeed, in 0 steps, we obtain only the word A in both languages. Every k-step
R2 word gives two different (k + 1)-step words and any two different k-step words give
S ABC = SBOC + SCOA + S AOB = (sin 2α + sin 2β + sin 2γ).
2 different (k + 1)-step words, since the initial word is always a part of the new word.
Since triangle BOC is isosceles, the midperpendicular OA′ of side BC is also an an- Thus, the number of k + 1-step words is twice the number of k-step words. Now, it
gle bisector, and ∠ BOA′ = ∠COA′ = α. Similarly, ∠COB′ = ∠ AOB′ = β and suffices to prove that every Ababi word is an Ululu word.
R Any 0-step Ababi word is clearly an Ululu word. Assume that the claim holds for all
∠ AOC′ = ∠BOC′ = γ. Consider triangle B′ OC′ . The height drawn to side B′ C′ is , k-step Ababi words and consider a k + 1-step word t. Then, for some word s in the
2
R R2 Ababi language, t = s ⊕ s or t = s ⊕ s̄. By the induction hypothesis, there must exist a
so | B′ C′ | = (tan β + tan γ) and the area of this triangle is SB′ OC ′ = (tan β + tan γ). sequence of k operations by the Ululu rules that allows to construct the word s from the
2 8
R2 R2 word A.
Analogously, SC ′ OA′ = (tan γ + tan α) and S A′ OB′ = (tan α + tan β). Thus,
8 8
• If the word t is obtained in the Ababi language by the rule t = s ⊕ s, apply the
R2 aforementioned sequence of Ululu rules to the word AA. It is easy to see that after
S A′ B′ C ′ = SB′ OC ′ + SC ′ OA′ + S A′ OC ′ = (tan α + tan β + tan γ).
4 each step, the new word is of the form a ⊕ a, where a is the corresponding inter-
h πi mediate word in the construction process of s, since each Ululu operation has the
Function f ( x ) = tan x − 2 sin 2x is concave in the interval 0; , since the second same effect on the two equal halves of a word.
2
derivative
• If the word t is obtained by the rule t = s ⊕ s̄, apply the aforementioned sequence of
2 sin x
f ′′ ( x ) = + 8 sin 2x Ululu rules to the Ululu word AB. In this case, after each step, the obtained word is
cos3 x a ⊕ ā, where a is the corresponding intermediate word in the construction process
is non-negative in this interval. Jensen’s inequality now gives of s, since if the two halves of a word are “negations” of each other, any Ululu
operation preserves this property.
α+β+γ π 2π
f (α ) + f ( β) + f (γ ) > 3 f = 3 tan − 2 sin = 0, Consequently, t is a word in the Ululu language.
3 3 3
so tan α + tan β + tan γ > 2 sin 2α + 2 sin 2β + 2 sin 2γ. The final equality yields S A′ B′ C ′ >
S ABC .
19 20
WE THANK:
Estonian Math Competitions
2006/2007
The Gifted and Talented Development Centre
Tartu 2007
with side lengths a′ , b′ , c′ where a′ = b and b′ = c. Then
b a′ b′ c
Selected Problems from Open Contests = = = ,
a a b b
c2
1. Find all positive integers N with at most 4 digits such that the number obtained by hence b2 = ac. Second let K satisfy b2 = ac. Taking a′ = b, b′ = c and c′ = , we see
b
reversing the order of digits of N is divisible by N and differs from N. (Juniors.) that
Answer: 1089 and 2178. a′ b′ c′
Solution. Let N be any number satisfying the conditions. Let N ′ be the number with = = ,
a b c
digits in reversed order. Then N ′ = k · N for some integer k ≥ 2. Clearly, N must have
at least 2 digits. It cannot end with 0 as otherwise N ′ < N, which is impossible. Let i.e., triangle K ′ with side lengths a′ , b′ , c′ is just like required by the definition of disguis-
N = a . . . b and N ′ = b . . . a where a and b are digits. Then b ≥ ka ≥ 2a. On the other ability.
hand, a multiple of b ends with a. The pairs (a, b) that meet all conditions imposed by If a, b, c are integers then obviously b ≥ 2. A case study of possible values of b shows
the observations made so far are the following: that, for b = 2, . . . , 5, there exist no integers a and c such that a < b < c and ac = b2 and
c < a + b. For b = 6, we can take a = 4 and c = 9, giving perimeter 19. Thus we have
(1, 3), (1, 7), (1, 9), (2, 4), (2, 6), (2, 7), (2, 8), (2, 9), (3, 7), (3, 9), (4, 8), (4, 9). one more condition: a + b + c ≤ 19. As a ≥ 1 and c ≥ b + 1, this implies 2b + 2 ≤ 19
and b ≤ 8. So it suffices to check that for b = 7 and b = 8, no integers a and c such that
For pairs (1, 3), (2, 4), (2, 7), (2, 9), (3, 7), (3, 9), (4, 8), (4, 9), the conditions ka ≤ b and a < b < c and ac = b2 and c < a + b exist.
kb ≡ a (mod 10) are contradictory. Among the remaining pairs, (1, 7) gives k = 3, (b) Let K be a triangle satisfying the conditions of the problem. Then gcd(a, b, c) = 1
while the first digit of 3 · 1 . . . 7 cannot be 7. Analogously, (2, 6) gives k = 2, while the and b2 = ac. This implies that gcd(a, c) = 1 (as each common prime divisor of a and c
first digit of 2 · 2 . . . 6 cannot be 6. Hence only the pairs (1, 9) and (2, 8) are possible. would also divide b). Thus, both a and c are perfect squares.
As 91 is not divisible by 19 and 82 is not divisible by 28, no two-digit numbers satisfy
the conditions. For three-digit numbers, there must be either N = 1c9 and N ′ = 9c1 3. In a school tennis tournament with m ≥ 2 participants, each match consists of 4
with k = 9 or N = 2c8 and N ′ = 8c2 with k = 4. The first case leads to equality sets. A player who wins more than half of all sets during a match gets 2 points for
900 + 10c + 1 = 9 · (100 + 10c + 9) which gives 80c + 80 = 0, so no suitable c exists. The this match. A player who wins exactly half of all sets during the match gets 1 point,
second case fails analogously. and a player who wins less than half of all sets gets 0 points. During the tournament,
In the case of four-digit numbers, we have either N = 1cd9 and N ′ = 9dc1 with k = 9 each participant plays exactly one match against each remaining player. Find the least
or N = 2cd8 and N ′ = 8dc2 with k = 4. In the first case, the equality 9000 + 100d + number of participants m for which it is possible that some participant wins more sets
10c + 1 = 9 · (1000 + 100c + 10d + 9) leads to d = 89c + 8. The only solution in digits is than any other participant but obtains less points than any other participant. (Juniors.)
c = 0, d = 8. In the second case, the equality 8000 + 100d + 10c + 2 = 4 · (2000 + 100c + Answer: 6.
10d + 8) leads to 2d = 13c + 1. The only solution in digits is c = 1, d = 7.
Solution. Let m = 5. A participant who wins more sets than any other during the
2. Call a scalene triangle K disguisable if there exists a triangle K ′ similar to K with tournament must win more than half of all sets he plays. This implies that he must win
two shorter sides precisely as long as the two longer sides of K, respectively. Call a more sets than his opponent in at least one match, i.e., he must win at least one match.
disguisable triangle integral if the lengths of all its sides are integers. But in order to obtain less points than anyone else, he must lose more matches than
win. As each participant plays 4 matches, this special participant must win exactly one
(a) Find the side lengths of the integral disguisable triangle with the smallest possible match and lose at least two. Under such conditions, he can win at most 8 sets during
perimeter. the tournament but this is not more than half of the number 16 of all sets.
(b) Let K be an arbitrary integral disguisable triangle for which no smaller integral Thus, for m = 5, the described situation is impossible. If it were possible for some m
disguisable triangle similar to it exists. Prove that at least two side lengths of K are such that m < 5, we could obtain a suitable tournament table also for m = 5 by adding
perfect squares. an appropriate number of players whose matches all end in draw.
The following table shows a situation for m = 6 where all conditions are fulfilled:
(Juniors.)
Answer: (a) 4, 6, 9. Player Marks Sets won
Solution. (a) Let K be a triangle with side lengths a, b, c, where a < b < c. We show that 1. 4:0 4:0 1:3 1:3 1:3 4 11
K is disguisable iff b2 = ac. First let K be disguisable and let K ′ be a similar triangle to K 2. 0:4 2:2 3:1 2:2 3:1 6 10
1 2
n
3. 0:4 2:2 2:2 3:1 2:2 5 9 by exactly students. Thus k and n must be even.
4. 3:1 1:3 2:2 2:2 2:2 5 10 2
It remains to show that the situation is possible for arbitrary even k and n. For this,
5. 3:1 2:2 1:3 2:2 2:2 5 10 enumerate the questions by numbers from 1 to k and the students by numbers 1 to
6. 3:1 1:3 2:2 2:2 2:2 5 10 n. Let every student with an odd number answer correctly exactly the questions with
an odd number and every student with an even number answer correctly exactly the
4. Call a k-digit positive integer a hyperprime if all its segments consisting of 1, 2, . . . , questions with an even number. The requirements are fulfilled.
k consecutive digits are prime. Find all hyperprimes. (Juniors.)
6. Let an = 1 + 2 + . . . + n for every n ≥ 1; the numbers an are called triangular. Prove
Answer: 2, 3, 5, 7, 23, 37, 53, 73, 373. that if 2am = an then a2m−n is a perfect square. (Seniors.)
Solution. One-digit hyperprimes are precisely the one-digit primes 2, 3, 5, 7.
In a larger hyperprime, all digits must be prime. The last digit can be neither 2 nor Solution. We depict an as a set of points orga- (n − m) 2
5, and no two consecutive digits can be equal (this would form a composite segment). nized triangularly as shown in Fig. 1. From
Adding one digit to all one-digit primes, while following these requirements, we obtain two ends of the base, separate two triangles
|
numbers 23, 27, 37, 53, 57, 73. Among these, only 23, 37, 53, 73 are primes. both containing am points. For counting 2am
Note that all segments of any hyperprime are hyperprime. Thus all three-digit hyper- points, we count the points in the intersec-
primes can be obtained from two-digit hyperprimes by adding one digit to the end. tion of the two triangles twice, while leav-
Following the requirements above, we get hyperprime candidates 237, 373, 537, 737, ing the points in the upper rhomboid un- am
{z
among which only 373 is really a prime and a hyperprime. counted; for counting an points, every point
is taken into account once. Thus if 2am = an
Hyperprimes with more than 3 digits are impossible since their every segment of 3
then the intersection contains as many points
digits should be 373.
as the rhomboid. The former contains a2m−n
points while the latter contains (n − m)2 .
}
5. In an exam with k questions, n students are taking part. A student fails the exam
Remark. One can also prove the claim al- | {z }
if he answers correctly less than half of all questions. Call a question easy if more than a 2m −n
half of all students answer it correctly. For which pairs (k, n) of positive integers is it gebraically using the formula of the sum of
possible that arithmetic progression. Figure 1
(a) all students fail the exam although all questions are easy; 7. Three circles with centres A, B, C touch each other pairwise externally, and touch
(b) no student fails the exam although no question is easy? circle c from inside. Prove that if the centre of c coincides with the orthocentre of triangle
ABC, then ABC is equilateral. (Seniors.)
(Juniors.)
Solution 1. Let the tangent point of circles with centres A and B be C′ , the tangent point
Answer: (a) there are no such pairs; (b) all pairs (k, n) with both k and n even.
of circles with centres B and C be A′ and the tangent point of circles with centres C and
Solution. Let v be the total number of correct answers given by all students.
A be B′ (see Fig. ??). Let A′′ , B′′ , C′′ be the tangent points of circle c with the circles with
k
(a) If all students fail then each of them gives less than correct answers, i.e., v < centre A, B, C, respectively. Let H and I be the orthocentre and the incentre of ABC,
2 respectively. Assume that H is the centre of c.
k nk n
n· =
2 2
. If all questions are easy then, for each of them, more than correct answers
2 We prove first that triangle ABC is acute. Line H A′′ passes through A and contains both
n nk a radius of c and the altitude of ABC drawn from A. If angle BAC were not acute then
are given, i.e., v > k · = , contradiction. the orthocentre of ABC would be on ray AA′′ while the centre of c would be outside
2 2
k k this ray since | AA′′ | < | H A′′ |. Analogously, the other angles of ABC must be acute.
(b) If no student fails then each of them gives at least correct answers, i.e., v ≥ n · =
2 2 Draw tangents to c from points A′′ , B′′ , C′′ . As ABC is acute, point H lies inside it. Hence
nk k each of the three arcs of c with endpoints A′′ , B′′ , C′′ is less than 180◦ . Consequently,
; the equality holds iff each student gives exactly correct answers. On the other
2 n 2 these tangents intersect each other pairwise, forming a triangle DEF whose incircle is c.
hand, if no question is easy then no more than correct answers are given to each of
n nk 2 n As both BC and EF are perpendicular to H A′′ , they are parallel. Analogously, CA and
them, i.e., v ≤ k · = , whereby equality holds iff each question gets exactly correct FD are parallel, and AB and DE, too. Thus triangles ABC and DEF are similar.
2 2 2
nk Prove now that the orthocentre of DEF is I. Points A′ , B′ , C′ lie on the sides of ABC;
answers. These two inequalities can both hold only if v = . Consequently, each it is known that they are also the points where the incircle of ABC touches the sides.
2
k Thus I A′ , C′′ D and B′′ D are the radical axes of c and two circles touching it and each
student answers exactly questions correctly and each question is answered correctly
2
3 4
other. The radical axes meet at D. Thus ID, IE and IF are perpendicular to BC, CA, AB, these into (??), we obtain
respectively, and consequently also to the corresponding sides of DEF.
From all this, we get that the distance between the orthocentre and the incentre is the r cos ∠CAB + | AC| = r cos ∠CBA + | BC|. (2)
same in triangles ABC and DEF. As these triangles are similar but not equal, this can
happen only if the distance is zero, i.e., the orthocentre and incentre coincide. This As the opposite angle of a bigger side is bigger in every triangle, | AC| < | BC| would
implies that ABC is equilateral. imply ∠CBA < ∠CAB and cos ∠CBA > cos ∠CAB, leading to r cos ∠CAB + | AC| <
F r cos ∠CBA + | BC| which contradicts (2). Analogously, also | AC| > | BC| cannot be.
Consequently, | AC| = | BC|. In the same way, we get | AB| = | AC|, i.e., triangle ABC is
equilateral.
M Remark. The claim of the problem holds also without the assumption that the tangency
of c with the three circles is inner.
B ′′ A′′ B ′′ A′′
B A B A
8. Let b be an even positive integer for which there exists a natural number n such
C′ bn − 1
H H that n > 1 and is a perfect square. Prove that b is divisible by 8. (Seniors.)
A′ B′ b−1
C K C L bn − 1
Solution. As b is even, the perfect square is odd. Hence it is congruent to 8
b−1
D C ′′ E C ′′ modulo 1, i.e., the number
bn − 1
Figure 2 Figure 3 − 1 = b + b2 + . . . + b n − 1 = b 1 + b + . . . + b n − 2
b−1
Solution 2. Let r be the radius of c and let r A , r B , rC be the radii of circles with centre A,
B, C, respectively. Let h A be the length of the altitude of ABC drawn from A. Let A′ , B′ , is divisible by 8. As the factor 1 + b + . . . + bn−2 is odd, it is relatively prime to 8 and
C′ be the feet of altitudes of triangle ABC drawn from vertices A, B, C, respectively. hence b is divisible by 8.
From triangle H A′ C, we get | H A′ |2 = |CH |2 − |CA′ |2 ; from triangle AA′ C, we get Remark. One can also prove the claim by considering b modulo 8.
|CA′ |2 = | AC|2 − | AA′ |2 . Here, |CH | = r − rC , | AC| = r A + rC and | AA′ | = h A . Thus
9. The Fibonacci sequence is determined by conditions F0 = 0, F1 = 1, and Fk =
| H A′ |2 = (r − rC )2 − (r A + rC )2 + h2A = r2 − 2(r + r A )rC − r2A + h2A .
Fk−1 + Fk−2 for all k ≥ 2. Let n be a positive integer and let P( x ) = am x m + . . . + a1 x + a0
Analogously, we obtain be a polynomial that satisfies the following two conditions:
| H A′ |2 = r2 − 2(r + r A )r B − r2A + h2A . (1) P( Fn ) = Fn2 ; (2) P( Fk ) = P( Fk−1 ) + P( Fk−2 ) for all k ≥ 2.
These two equalities together give r B = rC . Analogously, r B = r A . Thus the radii of the Find the sum of the coefficients of P. (Seniors.)
circles drawn around A, B, C touching pairwise each other are equal. This can be only
Answer. Fn .
if the sides of ABC are all equal.
Solution. We are asked to find P(1). If n = 1 then P(1) = P( F1 ) = F12 = 1, giving
Solution 3. Let r A , r B , rC be defined as in Solution 2. Like in Solution 1, note that H lies
P(1) = F1 . If n ≥ 2 then using condition P( Fk ) = P( Fk−1 ) + P( Fk−2 ), 2 ≤ k ≤ n,
inside triangle ABC. As H is the centre of c, radii H A′′ and HB′′ are equal (see Fig. ??)
repeatedly, we get
which means that | H A| + r A = | HB| + r B . Adding rC to both sides of this equality, we
get P( Fn ) = P( Fn−1 ) + P( Fn−2 ) = 2P( Fn−2 ) + P( Fn−3 ) = 3P( Fn−3 ) + 2P( Fn−4 ) = . . .
| H A| + | AC| = | HB| + | BC|. (1) = Fn P( F1 ) + Fn−1 P( F0 ) = Fn P(1) + Fn−1 P(0).
Take triangle KLM whose midlines are the sides of ABC; then triangles KLM and ABC Using the given condition again for k = 2, we obtain
are similar. Thereby, AH is the perpendicular bisector of side LM as LM k BC and
| LA| = | AM|. Also, BH is the perpendicular bisector of side MK. Thus H is the circum- P(1) = P( F2 ) = P( F1 ) + P( F0 ) = P(1) + P(0),
1
centre of triangle KLM and ∠ MH A = ∠ MHL = ∠ MKL = ∠CAB. which gives P(0) = 0. Altogether, Fn2 = P( Fn ) = Fn P(1), showing that P(1) = Fn .
2
From the right triangle MAH, we get | H A| = r cos ∠ MH A = r cos ∠CAB where r is Remark. We could ask whether there exist polynomials for every n satisfying the con-
the radius of the circumcircle of KLM. Analogously, | HB| = r cos ∠CBA. Substituting ditions of the problem. Using the condition P( Fk ) = P( Fk−1 ) + P( Fk−2 ), 2 ≤ k ≤ n,
5 6
T
for finding the other values P( Fk ), 2 ≤ k ≤ n, we get P( Fk ) = Fn Fk , for all k such that
0 ≤ k ≤ n. Elsewhere, the values of the polynomial are not determined. One suitable
polynomial is P( x ) = Fn x.
10. Does there exist a natural number n such that n > 2 and the sum of squares of
some n consecutive integers is a perfect square? (Seniors.)
Answer: yes.
Solution 1. For n = 11, we can construct the following example: D1
A2
(−4)2 + (−3)2 + (−2)2 + (−1)2 + 02 + 12 + 22 + 32 + 42 + 52 + 62 = 112 . C1
B2
n
Solution 2. It is easy to prove by induction that 12 + 22 + . . . + n2 = (n + 1)(2n + 1). P
6
If n = 24 then all three factors in the last product are perfect squares (4, 25 and 49, B1
C2
respectively). Thus the product is also a perfect square.
l1
Remark 1. For n = 3, . . . , 10, no suitable examples exist because there is a number mod- A1
ulo which no sum of squares of n consecutive integers is a quadratic residue. l2
D2
n sum expression bad modulus
3 ( a − 1 )2 + . . . + ( a + 1 )2 3a2 + 2 3 Figure 4
4 ( a − 1 )2 + . . . + ( a + 2 )2 4a2 + 4a + 6 4
5 ( a − 2 )2 + . . . + ( a + 2 )2 5a2 + 10 25 Solution 1. Let r1 and r2 be the radii of c1 and c2 , respectively. Since quadrangle
6 ( a − 2 )2 + . . . + ( a + 3 )2 6a2 + 6a + 19 4 A1 A2 D1 D2 is cyclic, ∠ A2 A1 D1 = ∠ D1 D2 A2 . In addition, ∠ A1 PA2 = ∠ D2 PD1 (see
7 ( a − 3 )2 + . . . + ( a + 3 )2 7a2 + 28 49 Fig. 4). Thus triangles A1 A2 P and D2 D1 P are similar. Their incircles are c1 and c2 , hence
8 ( a − 3 )2 + . . . + ( a + 4 )2 8a2 + 8a + 44 16
| A P|
1 r
9 ( a − 4 )2 + . . . + ( a + 4 )2 9a2 + 60 9 = 1.
| D2 P | r2
10 ( a − 4 )2 + . . . + ( a + 5 )2 10a2 + 10a + 85 25
Since quadrangle B1 B2 C1 C2 is cyclic, ∠ B2 B1 C1 = ∠C1 C2 B2 , whence ∠TB1 A1 =
Remark 2. Using diophantine equation theory, it has been proven that the sum of squares ∠TC2 D2 . As also ∠TA1 B1 = ∠TD2 C2 , triangles A1 B1 T and D2 C2 T are similar. Their
of numbers from 1 to n is a perfect square only for n = 0, n = 1 and n = 24. Thus the incircles are c1 and c2 again, hence
choice n = 24 in Solution 2 is the only possibility to succeed.
Remark 3. Sloane’s Encyclopedia of Integer Sequences contains the sequence A001032 with | A1 T | r
= 1.
description “Numbers n such that the sum of squares of n consecutive positive integers | D2 T | r2
can be a perfect square” (i.e., in addition to the conditions of our problem, the numbers | A1 P | | A T|
whose squares are considered must be positive). The sequence starts with numbers 1, Now consider triangles A1 PT and D2 PT. We have = 1 and ∠TA1 P =
| D2 P | | D2 T |
2, 11, 23, 24, 26, 33, 47, 49, 50, 59, 73, 74, 88, 96, 97, 107, 121, 122, 146, 169, 177, 184, 191, ∠TD2 P. Thus these triangles are similar and
193, 194, 218, 239, 241, 242, 249, 289, 297, 299, 311, 312, 313, 337, 338, 347, 352, 361, 362,
376, 383, 393, 407, 409, 431, 443, 457, 458, 479, 481, 491. r 1 | A P| | PT |
= 1 = = 1.
r2 | D2 P | | PT |
11. Tangents l1 and l2 common to circles c1 and c2 intersect at point P, whereby tangent
points remain to different sides from P on both tangent lines. Through some point Solution 2. Let r1 and r2 be the radii of c1 and c2 , respectively. Let s be the line that passes
T, tangents p1 and p2 to circle c1 and tangents p3 and p4 to circle c2 are drawn. The through P and is perpendicular to the line joining the centres of c1 and c2 . Consider the
intersection points of l1 with lines p1 , p2 , p3 , p4 are A1 , B1 , C1 , D1 , respectively, whereby composition of two plane transformations: reflection w.r.t. s and homothety w.r.t. P
the order of points on l1 is: A1 , B1 , P, C1 , D1 . Analogously, the intersection points of l2 r2
with factor . This composite transformation takes c1 to c2 and P to P.
with lines p1 , p2 , p3 , p4 are A2 , B2 , C2 , D2 , respectively. Prove that if both quadrangles r1
A1 A2 D1 D2 and B1 B2 C1 C2 are cyclic then radii of c1 and c2 are equal. (Seniors.) Denote the image of any point X under this transformation by X ′ . As the transformation
respects all angles between lines, the equality of angles in cyclic quadrilateral B1 B2 C1 C2
7 8
√ √
implies ∠ B2′ B1′ P = ∠ B2 B1 P = ∠C1 C2 P. As lines B1′ B2′ and C2 C1 both touch circle c2 and 2+ 2 1 2+ 2 1
inradius and the perimeter gives r. Thus = r, whence r = √ =
intersect PC2 under the same angle, line B1′ B2′ coincides with line C2 C1 . Analogously, 2 2 2 2+ 2
√ √
A1′ A2′ coincides with line D2 D1 . 2− 2 2 1 √ 3 √
Hence the transformation takes the intersection point of lines A1 A2 and B1 B2 to the = 1− . Therefore r2 = 1 + − 2 = − 2, so r2 is not rational.
2 2 2 2
intersection point of lines C2 C1 and D2 D1 , i.e., T is taken to T. But if r1 6= r2 then the abc a2 b2 c 2
transformation obviously can have only one fixpoint. Consequently, r1 = r2 . Solution 2. (a) By the formula S = , we get R2 = . Thus it suffices to prove
4R 16S2
that S2 is rational. By Heron’s formula,
12. Find all positive integers n such that one can write an integer 1 to n2 into each unit
square of a n2 × n2 table in such a way that, in each row, each column and each n × n 1 2 2
S2 = p( p − a)( p − b)( p − c) = 2a (b + c2 ) − (b2 − c2 )2 − (a2 )2 .
block of unit squares, each number 1 to n2 occurs exactly once. (Seniors.) 16
The last expression clearly evaluates to a rational number.
Answer: 1.
(b) In part (a), it was proven that S2 is rational. For the triangle with side lengths 1, 1
Solution. In the case n = 1, the conditions can be fulfilled trivially. Assume n ≥ 2. √ !2 √
√ 2+ 2 6+4 2 3 √
Denote the unit square in the ith row and jth column by 1 2 ... n n +1 and 2, we get p2 = = = + 2, hence p2 is not rational and
(i, j). Let A be the n × n block containing both (1, 1) and 1
2 4 2
(n, n). Let B and C be the n × n blocks obtained from A by S2
2 neither is r2 = .
shifting it by one unit down and right, respectively (see . p2
Fig. 5). ..
The numbers in the bottommost row of B must be the n
same as the numbers in the topmost row of A in some or- Selected Problems from the Final Round of National
n +1
der since both blocks must contain each number 1, . . . , n2 Olympiad
exactly once. Analogously, the rightmost column of C
must contain the same numbers as the leftmost column Figure 5
of A in some order. Now, the number in (n + 1, n + 1) 1. Two medians drawn from vertices A and B of triangle ABC are perpendicular.
cannot occur in the topmost row of A since all these occur already in the row number Prove that side AB is the shortest side of ABC. (Grade 9.)
n + 1 left from the square under consideration. Analogously, this number cannot occur
in the leftmost column of A. As A contains all numbers 1 to n2 , this number must occur Solution. Let the medians intersect in point M and let the median drawn from vertex
elsewhere in A. But then it occurs twice in the n × n block that contains squares (2, 2) C intersect AB in point F (see Fig. 6). Then, F is the midpoint of the hypotenuse of
and (n + 1, n + 1) which is prohibited. the right triangle ABM and thus the midpoint of the circumcircle of ABM, so we get
| AB| = 2| FM|. Since M divides median CF in ratio 2 : 1, we have | AB| = |CM|. The
13. Consider triangles whose each side length squared is a rational number. Is it true largest angle of triangle AMC is the obtuse angle AMC, therefore AC is the longest side
that of this triangle. We get | AC| > | MC| = | AB|. The proof of | BC| > | AB| is analogous.
(a) the square of the circumradius of every such triangle is rational; Remark. One can also solve the problem using the Pythagorean theorem and the fact
(b) the square of the inradius of every such triangle is rational? that the centroid divides the medians in ratio 2 : 1.
(Seniors.) 2. Juhan wants to order by weight five balls of pairwise different weight, using only
Answer: (a) yes; (b) no. a balance scale. First, he labels the balls with numbers 1 to 5 and creates a list of weigh-
ings, such that each element in the list is a pair of two balls. Then, for every pair in
Solution 1. (a) Fix a triangle from the family under consideration. Let a, b, c be its side
the list, he weighs the two balls against each other. Can Juhan sort the balls by weight,
lengths, γ the size of the angle opposite to the last side and R the circumradius. The
using a list with less than 10 pairs? (Grade 9.)
a2 + b2 − c 2 ( a2 + b 2 − c 2 )2
cosine law gives cos γ = , whence cos2 γ = . As a2 , b2 , c2 are Answer: no.
2ab 4a2 b2
rational, also cos2 γ is rational. Therefore 1 − cos2 γ, i.e., sin2 γ, is rational. The sine law Solution. There are 10 possible pairs of 5 balls. Suppose w.l.o.g. that Juhan does not
c c2 weigh the pair (1, 2). Then, it is not possible to distinguish orderings 1, 2, 3, 4, 5 and
gives R = , hence R2 = . Thus R2 is rational.
2 sin γ 4 sin2 γ 2, 1, 3, 4, 5, since the remaining 9 weighings give the same result. Thus, Juhan’s list must
√
(b) Consider the right isosceles triangle with side lengths 1, 1, 2. Let r be its inradius. contain all 10 pairs.
1
The area of this triangle, computed via leg lengths, is ; computing the area via the 3. Two radii OA and OB of a circle c with midpoint O are perpendicular. Another
2
9 10
circle touches c in point Q and the radii in points C and D, respectively. Determine
∠ AQC. (Grade 10.)
Answer: 45◦ .
C 3 vertices 4 vertices 5 vertices 6 vertices 7 vertices
b a
2 2 B
8 vertices 9 vertices 10 vertices 12 vertices
D Q Figure 8
E
y x
M D union is an n-gon (not necessarily convex). Find all possible values of the number of
b a vertices n. (Grade 10.)
O′
2 2x 2y 2
Answer: all integers from 3 to 12, except 11.
A′ A
Solution. The n-gon must have at least 3 vertices. We show first that the number of
A B
O C vertices is at most 12. Indeed, each vertex of the n-gon is either a vertex of one of the
F
triangles or an intersection point of some two sides of the two triangles. There are 6
Figure 6 Figure 7 triangle vertices and 6 possible intersection points, since every side of the first triangle
can intersect at most two sides of the second triangle. Thus, n ≤ 12.
Solution. By symmetry, ∠ AQC = ∠ BQD (see Fig. 7). Since AQB is an internal angle
of a regular octagon, we have ∠ AQB = 135◦ . Let now O′ be the midpoint of the circle Next, suppose that the n-gon has 11 vertices. If 6 of those vertices are vertices of the two
through C, D and Q. The quadrilateral OCO′ D has three right angles: ∠COD = 90◦ by triangles, then neither triangle can contain a vertex of the other triangle. Thus, each side
assumption, while angles OCO′ and ODO′ are angles between a radius and a tangent. of each triangle intersects the other triangle either never or twice, so we cannot have an
1 odd number of intersection points. On the contrary, if the n-gon has 6 intersection points
Thus, CO′ D is also a right angle, so ∠CQD = ∠CO′ D = 45◦ and as vertices, every side of each triangle must intersect the second triangle twice, and thus
2
all vertices of one triangle must be outside the other triangle. Thus, all 6 triangle vertices
1 1 are also vertices of the n-gon, and n = 12.
∠ AQC = (∠ AQB − ∠CQD ) = (135◦ − 45◦ ) = 45◦ .
2 2 The remaining configurations from 3 to 12 are all possible (see Fig. 8).
Remark. One can also make use of the similarity of triangles QO′ D and QOA′ (where A′
6. The identifier of a book is an n-tuple of numbers 0, 1, . . . , 9, followed by a check-
is the other endpoint of the diameter of c drawn through A), yielding that angle AQA′
sum. The checksum is computed by a fixed rule that satisfies the following property:
subtends the diameter. Another approach is to apply the tangent chord property on
whenever one increases a single number in the n-tuple (without modifying the other
tangent AC and secant CD to obtain that triangles AQC and CQD are similar.
numbers), the checksum also increases. Find the smallest possible number of required
4. Prove that the sum of the squares of any three pairwise different positive odd in- checksums if all possible n-tuples are in use. (Grade 10.)
tegers can be represented as the sum of the squares of six (not necessarily different) Answer: 9n + 1.
positive integers. (Grade 10.) Solution. Consider the checksum of (0, 0, . . . , 0). Increasing the first number, we get n-
Solution. Let a > b > c be positive integers. Then tuples (1, 0, . . . , 0), . . . , (9, 0, . . . , 0) with 9 new checksums. Increasing the second num-
ber in the last n-tuple, we get (9, 1, . . . , 0), . . . , (9, 9, . . . , 0), and again obtain 9 new val-
a2 b2 b2 c 2 c 2 a2 ues. Continuing like this, we see that the number of different check values is at least
a2 + b2 + c 2 = + + + + + = 9n + 1.
2 2 2 2 2 2
2 2 2 2 2 It is easy to see that the sum of all numbers in the n-tuple is a valid checksum. On the
aa+b
− b b + c b − c a + c a − c 2
= + + + + + . other hand, the sum of n numbers 0, 1, . . . , 9 is at least 0 and at most 9n, so with this
2
2 2 2 2 2
rule, we have exactly 9n + 1 different checksums.
Since a, b and c are all odd, the latter is a sum of squares of six positive integers.
7. Find all real numbers a such that all solutions to the quadratic equation x2 − ax +
5. Two triangles are drawn on a plane in such a way that the area covered by their a = 0 are integers. (Grade 11.)
11 12
Answer: a = 0 and a = 4. ∠CAR = ∠CBQ = ∠PAB, so triangles ACR and ABP are similar. If AQ ⊥ BC, then AR
Solution. Let x and y be the solutions of the quadratic equation. Viète formulae give is the height of ABC and ∠ ARC = ∠ APB = 90◦ , so P = R and thus P lies on AQ. If P
x + y = xy = a. The case y = 1 gives a contradiction 1 + x = x, while y 6= 1 gives lies on AQ, then again P = R and ∠CRA = ∠CPA = 180◦ − ∠ BPA = 180◦ − ∠CRA, or
x=
y
. Thus, x is an integer if and only if y = 2 or y = 0. Now, y = 2 gives x = 2 ∠CRA = 90◦ , so AQ ⊥ BC.
y−1 A
Solution 2. First consider the case where AB
and a = x + y = 4; y = 0 gives x = 0 and a = x + y = 0.
is the diameter of the circle through A, B and
Remark. Oneqcan also proceed from the fact that the sum a as well as the difference
p P. Then ∠ APB = 90◦ and ∠QBA = 90◦ giv-
a2 − 4a = (a − 2)2 − 4 of the solutions is integral. ing that AQ is the diameter of circumcircle
of ABC. Thus AQ ⊥ BC and P lies on line
8. A 3-dimensional chess board consists of 4 × 4 × 4 unit cubes. A rook can step from AQ.
any unit cube K to any other unit cube that has a common face with K. A bishop can On the other hand, when the centre of
step from any unit cube K to any other unit cube that has a common edge with K, but the circle passing through points A and B
does not have a common face. One move of both a rook and a bishop consists of an moves away from the leg AB on the half-
arbitrary positive number of consecutive steps in the same direction. Find the average plane containing point C, points Q and P R
number of possible moves for either piece, where the average is taken over all possible will move toward points B and C, respec- B P C
starting cubes K. (Grade 11.) tively. When the centre of the circle moves
away from AB on the other halfplane, points
Answer: the rook has on average 9 moves, the bishop has 10.5. Q
Q and P will move toward points C and B,
Solution. The rook has always 3 pos- respectively. Therefore in either case point P Figure 10
R R sible moves in the direction of one neither lies on line AQ nor AQ is perpendic-
axis, regardless of the choice of the ular to BC.
starting cube K (see Fig. 9), and thus
9 possible moves on average. 10. Find all pairs (m, n) of positive integers such that mn − nm = 3. (Grade 11.)
In any of the 8 “middle” cubes of the Answer: (4, 1).
B chess board, the bishop has 5 possi-
Solution. First, m and n must have different parity, for otherwise the lhs is even.
B B ble moves in the direction of every
axis, and thus 15 moves in total. In • If m is odd and n even, then mn ≡ 1 (mod 4), so nm ≡ 2 (mod 4), which is possible
Figure 9 any of the 24 cubes that have a com- only for m = 1. But then mn = 1 and the lhs is smaller than 3. Thus, in this case
mon face with a middle cube, the there are no solutions.
bishop has 5 moves on the plane parallel to the common face, and 3 moves on each • If now m is even and n odd, then nm ≡ 1 (mod 8), so mn ≡ 4 (mod 8). Thus,
of the remaining two planes, or 11 moves in total. For the remaining 32 cubes on the n ≤ 2, and n odd gives n = 1, m = 4 as the only solution.
edge of the board, the bishop has 3 moves in the direction of every axis, and 9 moves in
total. Averaging over all cubes, we get the result.
11. Some circles of radius 2 are drawn on the plane. Prove that the numerical value of
Remark. The number of possible moves shows to some extent the strength of every piece. the total area covered by these circles is at least as big as the total length of arcs bounding
The solution to this problem implies that in 3-dimensional chess, a bishop is perhaps the area. (Grade 11.)
stronger than a rook (as opposed to regular chess). On the other hand, the bishop is
weakened by the fact that it can always reach only half of the cubes or squares. Solution 1. The boundary line of the area consists of circular arcs, each corresponding to
While a 3-dimensional 4 × 4 × 4 board and a regular 8 × 8 board have the same number rl
a circular sector. According to the formula S = , where r is the radius and l the length
of cells, a regular rook and bishop have 14 and 8,75 moves on average, respectively. 2
of arc, we obtain that in the case r = 2 the area of circular sector equals numerically the
length of the circular arc on its boundary.
9. A circle passing through the endpoints of the leg AB of an isosceles triangle ABC
intersects the base BC in point P. A line tangent to the circle in point B intersects the We prove that no two such sectors have common interior points. First, the sectors from
circumcircle of ABC in point Q. Prove that P lies on line AQ if and only if AQ and BC the same circle share only the centre of the circle. Now let one sector be part of circle w1
are perpendicular. (Grade 11.) with centre O1 and the other be part of circle w2 with centre O2 . Assuming that the sec-
tors have a common interior point C, we draw the radii O1 A and O2 B of circles w1 and
Solution 1. Let line AQ intersect base BC in point R (see Fig. 10). On one hand, ABC w2 through C, respectively. Obviously A and B lie on the boundary of the area covered
is isosceles, so ∠ ACR = ∠ ABP. On the other hand, the tangent chord property gives by circles, meaning that B and A do not lie in the interior of w1 and w2 , respectively.
13 14
Therefore |O1 B| ≥ |O1 A| and |O2 A| ≥ |O2 B|. We draw the mediator of segment AB x x n z
Since < 1, + 1 < 2 holds for all n. On the other hand, since > 1, there exists
(see Fig. 11). The inequalities imply that O1 and A lie on the one side of the mediator y y y
N
and O2 and B lie on the other side. Hence segments O1 A and O2 B cannot have common z
points, a contradiction. A
an integer N such that > 2, contradiction.
y
We have obtained that the total length of the bound- B
ary line of the area equals the sum of the lengths of 14. Does there exist an equilateral triangle
circular arcs. But the total area is greater or equal (a) on a plane; (b) in a 3-dimensional space;
than the sum of the areas of the sectors.
Solution 2. We shall use induction on the number of such that all its three vertices have integral coordinates? (Grade 12.)
O1
circles. For one circle, the circumference and the area Answer: (a) no; (b) yes.
O2
are equal (4π). We prove that whenever we add a −→ −→
circle, the area a covered by the intersection of the Solution 1. (a) Suppose that such a triangle ABC exists. Then vectors AB and AC have
−→
new circle with the old area is at most as big as the Figure 11 integral coordinates. Denote AB = ( x, y), then the vector of the height drawn from
√
perimeter p of that intersection. 3 −→
vertex C is (y, − x ). The coordinates of AC are then either
First, notice that a ≤ 4π. Assume now that also p ≤ 4π. We write p = 2πr, so r ≤ 2
2. Since a circle maximizes the area for a fixed perimeter, we see that the area of the √ √ √ !
−→ 1 3 x + 3y y − 3x
intersection is a ≤ πr2 ≤ 2πr = p as desired. AC = ( x, y) + (y, − x ) = ,
2 2 2 2
12. Consider a cylinder and a cone with a common base such that the volume of the or
part of the cylinder enclosed in the cone equals the volume of the part of the cylinder √ √ √ !
outside the cone. Find the ratio of the height of the cone to the height of the cylinder. −→ 1 3 x− 3y y + 3x
AC = ( x, y) − (y, − x ) = , .
(Grade 12.) 2 2 2 2
1
Answer: 1 + √ . In either case, the coordinates are integral only for x = y = 0.
3 (b) Triangle ABC with A = (1, 0, 0), B = (0, 1, 0) and C = (0, 0, 1) is equilateral.
Solution. Denote by v and V, h and H the volume and the height of the cylinder and the
Solution 2. (a) W.l.o.g. assume that one vertex of the triangle is A(0, 0). We also assume
cone, respectively, and denote by S the area of the common base. The vertex of the cone
w.l.o.g. that the ordinates of B and C are non-negative and these vertices do not lie on
must lie outside the cylinder, for otherwise the volume of the intersection would be at
1 the y-axis.
most of the total volume of the cylinder. Since all the coordinates are integral, the slopes of AB and AC are rational. Denote the
3 h
Denote = x. The part of the cone that lies outside the cylinder is a cone similar to slope angles of lines AB and AC by β and γ, respectively, then γ = β ± 60◦ . Denote
H H −h tan β = k. We have
the original cone with scale factor = 1 − x and volume (1 − x )3 V. The volume √ √ √
H
v tan β ± tan 60◦ k± 3 (k ± 3)(1 ± k 3)
of the part of the cylinder inside the cone is thus V − (1 − x )3 V = . From v = Sh and tan γ = = √ = .
√ 2 1 ∓ tan β tan 60◦ 1∓k 3 1 − 3k2
1 3 3± 3 √ √ √ √ √
V = SH we get ( x3 − 3x2 + 3x ) = x, so x = . Since h < H implies x < 1, the But (k ± 3)(1 ± k 3) = k ± k2 3 ± 3 + 3k = 4k ± (k2 + 1) 3. As k2 + 1 > 0, it is
3 √ 2 2
3− 3 impossible that tan γ and k would be simultaneously rational.
only possible solution is x = and we get the desired ratio.
2 Remark. There are several other solutions to this problem. One may w.l.o.g. denote
the vertices A(0, 0), B( x1 , y1 ) and C( x1 , y1 ), where x1 , y1 , x2 and y2 are relatively prime,
13. Let x, y, z be positive real numbers such that x n , yn and zn are side lengths of some derive the equalities x12 + y12 = x22 + y22 = 2( x1 x2 + y1 y2 ) and consider all cases of re-
triangle for all positive integers n. Prove that at least two of x, y and z are equal. (Grade mainders of x1 , y1 , x2 and y2 modulo 2.
12.) Another approach would be to notice that ( x1 + x2 )2 + (y1 + y2 )2 is divisible by 3 and
Solution. Assume that x, y, z are all different and assume w.l.o.g. x < y < z. For any n, use the fact that perfect squares have only remainders 0 and 1 modulo 3.
the triangle inequality implies x n + yn > zn , or The most straightforward way is to compute the area S of the triangle with vertices
n n x1 x2 x3
x z A( x1 , y1 ), B( x2 , y2 ) and C( x3 , y3 ) in two different ways: 2S = y1 y2 y3 and 2S =
+1 > . 1 1 1
y y
15 16
√
a2 3
(where a is the side of the triangle). Since all the coordinates as well as the side
2
length are integral, such a triangle cannot exist.
15. Let a, b, c be positive integers such that gcd(a, b, c) = 1 and the product of every
two of these integers is divisible by the third one.
(a) Prove that every one of these integers equals the least common multiple of the re-
maining two integers divided by the greatest common divisor of these two integers.
n = 4q n = 4q + 1 n = 4q + 2 n = 4q + 3
(b) Give an example of such integers a > 1, b > 1 and c > 1.
Figure 12
(Grade 12.)
Answer: (b) For example, a = 6, b = 10, c = 15. r 4
Solution 1. (a) Let d = gcd(a, b), a = a′ d and b = b′ d, where gcd(a′ , b′ ) = 1. Then
4
lcm(a, b) .
lcm(a, b) = a′ b′ d and = a′ b′ . We prove that a′ b′ = c. ..
gcd(a, b) .
..
On the one hand, since ab = a′ b′ d2 is divisible by c and gcd(d, c) = 1 because of 4
gcd(a, b, c) = 1, a′ b′ must be divisible by c. On the other hand, the conditions of the 4
problem imply that ca = ca′ d is divisible by b = b′ d, i.e. ca′ is divisible by b′ . As 4
gcd(a′ , b′ ) = 1, c is divisible by b′ . Analogously, c is divisible by a′ . Since a′ and b′ are 4 r
coprime, c is divisibly by a′ b′ . Altogether we have c = a′ b′ as desired. The claims about
4 4 ··· 4 r r 4 4 ··· 4
a and b are proven analogously.
(b) Let x, y and z be different numbers that are pairwise coprime, e.g. different primes. Figure 13
Having a = xy, b = yz and c = xz, the numbers a, b and c satisfy the conditions of the
problem. Indeed,
• If r = 1, we mark groups of 8 squares. Since there are q2 such groups, we again get
lcm(a, b) xyz A(4q + 1) ≤ 8q2 .
= = xz = c,
gcd(a, b) y • If r = 2, we mark as in the first case, but leave the last two rows empty. This way,
we mark 2q rows with n squares, or 2q(4q + 2) squares in total. Thus, A(4q + 2) ≤
analogously for other cases. For an example we may take x = 2, y = 3, z = 5 that yields 8q2 + 4q.
6, 15 and 10.
• If r = 3 and q > 0, we mark every fourth row and column and the middle squares
Solution 2. (a) Let p be some prime factor of at least one of a, b and c. Since gcd(a, b, c) =
of the remaining 3 × 3 groups. There are (q + 1)2 marked middle squares and q
1, we may assume w.l.o.g. that p does not divide c. At the same time, since ca is divisible
marked rows and columns, giving (q + 1)2 + 2(4q + 3)q − q2 marked squares in
by b and cb is divisible by a, the factor p must have the same exponent α in both a and
total. Thus, A(4q + 3) ≤ 8q2 + 8q + 1. For q = 0, we get n = 3 and A(3) = 0.
b. Similarly, the exponents of any prime factor p′ in the prime factorization of a, b and c
are α′ , α′ and 0 in some order. Since every exponent in such a triple equals the difference We now show that these bounds are tight.
of the maximum and the minimum of the remaining two, the result follows.
• For r = 0 or r = 1, we can divide a 4q × 4q grid into q2 squares of size 4 × 4. In
16. Some squares of an n × n grid are marked in such a way that in every 4 × 4 square, every such square, at least 8 squares must be marked, so A(n) ≥ 8q2 .
at least half of the squares are marked. Find the smallest possible number of marked • For r = 2 or r = 3, cut an r × r square from the lower left corner and divide the
squares in the grid. (Grade 12.) remaining squares into q L-shaped strips of width 4 (see Fig. 13, right). The ith strip
then contains 2i 4 × 4 squares. The last two squares intersect in the corner of the L
Answer: 8q2 for n = 4q or n = 4q + 1; 8q2 + 4q for n = 4q + 2; 8q2 + 8q + 1 for n = 4q + 3
and the intersection is a square of size (4 − r ) × (4 − r ). Every strip also contains
and q ≥ 1; 0 for n = 3.
an r × r square in the corner.
Solution. Let A(n) be the smallest possible number of marked squares. Write n = 4q + r,
where 0 ≤ r < 4. First, we bound A(n) from above. In order to bound A(n) from below, we add the smallest possible number of
marked squares in the 4 × 4 squares and the r × r squares and subtract the largest
• If r = 0, we mark half of the squares in groups of two rows as shown in Fig. 12, so possible number of marked squares in the (4 − r ) × (4 − r ) squares.
A(4q) ≤ 8q2 .
17 18
The case r = 2 gives Solution 2. Let ai be the number of times the ith switch changes its position during the
whole process. According to the conditions of the problem, each switch moves either
(2q + 2)q when it is moved directly by the operator or its left neighbour
A(n) ≥ 8 · (2 + 4 + . . . + 2q) + 0 · (q + 1) − 4 · q = 8 · − 4q = 8q2 + 4q. j moves
a k down. As all
2 switches are down at the beginning, the ith switch moves down i times. Thus a1 = 1
j a k 2
If r = 3 every 3 × 3 square contains at least one marked square, so i −1
and ai = i + for all i ≥ 2.
2
(2q + 2)q j k
a 1
A(n) ≥ 8 · (2 + 4 + . . . + 2q) + 1 · (q + 1) − 1 · q = 8 · + 1 = 8q2 + 8q + 1. We prove by induction that ai = 2(i − 1) for all i ≥ 2. As a2 = 2 +
1
= 2+ = 2,
2 2 2
this claim holds for i = 2. Assuming that it holds for i, we obtain
ja k
IMO team selection contest ai + 1 = i + 1 + i
= i+1+
2 (i − 1 )
= 2i,
2 2
First day i.e., the claim holds also for i + 1.
Altogehter, this shows that a1 is odd and ai is even for all i ≥ 2. Hence, after the process,
1. On the control board of a nuclear station, there are n electric switches (n > 0), all the first switch is up and all the others are down.
in one row. Each switch has two possible positions: up and down. The switches are Solution 3. Interpret the position of switches on the board as binary numbers so that the
connected to each other in such a way that, whenever a switch moves down from its ith switch from the left corresponds to the ith lowest binary digit: being down encodes
upper position, its right neighbour (if it exists) automatically changes position. At the 0 and being up encodes 1. Changing the ith switch then works like addition of 2i −1
beginning, all switches are down. The operator of the board first changes the position modulo 2n . The initial position encodes number 0 and the final position encodes 1 · 20 +
of the leftmost switch once, then the position of the second leftmost switch twice etc., 2 · 21 + . . . + n · 2n−1 modulo 2n .
until eventually he changes the position of the rightmost switch n times. How many
We prove by induction that 1 · 20 + 2 · 21 + . . . + n · 2n−1 ≡ 1 (mod 2n ). If n = 1 then
switches are up after all these operations?
this holds. Assume that the claim holds for n = k. Multiplying this congruence by 2
Answer: 1. gives
Solution 1. Enumerate the switches with numbers 1 to n from left to right. We prove first
that the result of two consecutive changes does not depend on the order of the changes. 1 · 21 + 2 · 22 + . . . + k · 2 k ≡ 2 (mod 2k+1 ).
Let x and y be the numbers of the switches changed, x < y.
Adding 20 + 21 + . . . + 2k to both sides gives
• If there exists a number z such that x ≤ z < y and switch number z is down then
changing the position of x can influence only switches from x to z, changing the 1 · 20 + 2 · 21 + 3 · 22 + . . . + ( k + 1 ) · 2 k ≡ 2 + 2 k + 1 − 1 ≡ 1 (mod 2k+1 ),
position of switch y can influence only this switch and switches right from y. Thus
i.e., the claim holds for n = k + 1.
the results of the changes are independent of each other.
Remark. In Solution 3, one could prove by induction a stronger claim: 1 · 20 + 2 · 21 +
• If no such z exists then changing switch number x causes a change of switch number
. . . + n · 2n−1 = (n − 1) · 2n + 1.
y. After that, switches x to y − 1 are all down while all switches in the right from
them are in the same position as if switch number y were changed. Thus after 2. Let D be the foot of the altitude of triangle ABC drawn from vertex A. Let E and
moving both x and y in either order, switches from x to y − 1 are down and the F be points symmetric to D w.r.t. lines AB and AC, respectively. Let R1 and R2 be the
switches with larger number are in the position as when switch y were moved circumradii of triangles BDE and CDF, respectively, and let r1 and r2 be the inradii of
twice. the same triangles. Prove that
We prove now that, after all operations, precisely the leftmost switch is up. This claim |S ABD − S ACD | ≥ | R1 r1 − R2 r2 |
holds trivially for n = 1. Assume the claim holding for n switches and consider a board
with n + 1 switches. According to what was proven above, the moves can be performed where SK denotes the area of figure K.
in arbitrary order. Therefore, first change switch number 2 once, then switch number 3
Solution 1. Consider first the case where D lies between points B and C (see Fig. 14). As
twice etc., until the last switch n times. By the induction hypothesis, switch number 2 1 1
is up and all the others are down. Each switch has to be moved once more; if we do it S ABD = · | AD | · | BD | and S ACD = · | AD | · |CD |, we have
2 2
from right to left then switches n + 1 to 3 go up, then moving switch 2 down brings all
them down and finally switch 1 is moved up. Thus 1 is the only switch remaining up. 1
S ABD − S ACD = · | AD | · (| BD | − |CD |).
2
19 20
Let G be the incentre of triangle BDE and let G ′ be the projection of G to line BD. Then Analogously we obtain
| GG′ | = r1 . By symmetry, ∠BEA = ∠BDA = 90◦ , hence quadrangle BEAD is cyclic
and line segment AB is its circumdiameter. Thus | AB| = 2R1 . As triangles ADB and 1 1 γ
S ACD = | AD |2 tan γ, R2 r 2 = | AD |2 tan γ tan .
| AB| | GB| 2 2 2
GG ′ B are similar, we have = , implying 2R1 r1 = | AD | · | GB|. Let H be the
| AD | | GG′ | From these equalities, we can conclude that S ABD − S ACD and R1 r1 − R2 r2 have the
incentre of triangle CDF; then analogously 2R2 r2 = | AD | · | HC|. Hence same sign since β and γ belong to the first quarter where tan is increasing. W.l.o.g.,
1 assume that both are non-negative (otherwise interchange B and C). Then β ≥ γ and
R1 r 1 − R2 r 2 = · | AD | · (| GB| − | HC|). the desired inequality is equivalent to S ABD − R1 r1 ≥ S ACD − R2 r2 . Now
2
1 1 1 β
Triangle ADG is isosceles because ∠ ADG = 90◦ − ∠ BDE = 90◦ − ∠ DAG. Thus S ABD − R1 r1 = | AD |2 tan β 1 − tan =
2 2 2 2
| AD | = | AG|. Analogously, | AD | = | AH |. Thus | AG| = | AH |. β β
2 tan tan
Subtracting equality | AD |2 + |CD |2 = | AC|2 from | AD |2 + |BD |2 = | AB|2 gives 2
| BD | −
1 2 β 2
= | AD |2 1 − tan = | AD |2 ,
|CD |2 = | AB|2 − | AC|2 which is equivalent to | BD | − |CD | · | BD | + |CD | = | AB| − 2 β 2 β
1 − tan2 1 + tan
| AC| · | AB| + | AC| . Consequently, 2 2
| BD | − |CD | · | BC| = | GB| − | HC| · | AB| + | AC| . whence
As | BC| < | AB| + | AC|, we must have | BD | − |CD | ≥ | GB| − | HC|, which gives the 1
desired inequality. S ABD − R1 r1 = | AD |2 1 −
β
E 1 + tan
E 2
A A
and, analogously,
F
F
K 1
S ACD − R2 r2 = | AD |2 1 − γ
.
G L
H G
1 + tan
2
H H′
By β ≥ γ and tan being increasing, the inequality S ABD − R1 r1 ≥ S ACD − R2 r2 follows.
B G′ D C B C D C′
bn − 1
Figure 14 Figure 15 3. Let n be a natural number, n ≥ 2. Prove that if is a prime power for some
b−1
If D does not lie between B and C (see Fig. 15) then assume w.l.o.g. that it is on ray BC. positive integer b then n is prime.
Reflect line segment AC w.r.t. line AD; points C and H transform to some points C′ and bn − 1
H ′ , respectively. Now apply the solution above for triangle ABC′ . The desired claim Solution. Clearly b ≥ 2. Assume that = pl where p is prime, then n ≥ 2 implies
b−1
follows then by using |C′ D | = |CD | and | H ′ C′ | = | HC|. l ≥ 1. If n = xy where both x and y are greater than 1 then consider the representation
Solution 2. Denote ∠ BAD = β and ∠CAD = γ. Then
bxy − 1 bxy − 1 by − 1 by − 1
1 1 = y · = (1 + b y + . . . + b y ( x − 1) ) · .
S ABD = · | AD | · | BD | = | AD |2 tan β. b−1 b −1 b−1 b−1
2 2
As the product is a power of p, both factors must be powers of p. As x > 1 and y > 1,
As in Solution 1, show that quadrangle BEAD is cyclic. Let K be the point of intersec- both factors are multiples of p. Then by − 1 is a multiple of p. Thus all addends in the
| AB| | AD | first factor are congruent to 1 modulo p which implies that the first factor is congruent
tion of its diagonals. As R1 = , we get R1 = . Furthermore, r1 = | GK | and
2 2 cos β to x modulo p. Hence x is divisible by p. As x was an arbitrary non-trivial factor of n,
∠BDE ∠BAD β β β this shows that n = pm for a positive integer m.
∠GDK = = = . Thus r1 = | DK | tan = | AD | sin β tan . Conse-
2 2 2 2 2 Now consider the representation
quently,
m m 2
| AD | β 1 β bp − 1 bp − 1 bp − 1 bp − 1
R1 r 1 = · | AD | sin β tan = | AD |2 tan β tan . = p m −1 ·...· p · .
2 cos β 2 2 2 b−1 b −1 b −1 b−1
21 22
bp − 1 Taking y = a in the initial relation, we get f ( x ) = a + f ( x + 1), and so g( x − 1) − g( x ) =
Each factor is both greater than 1 and a power of p. As is a positive integral
b−1 a − 1 for all x. Since a 6= 1, g is unbounded and by continuity, takes all real values, so
power of p, the numerator is divisible by p, i.e., b p ≡ 1 (mod p). By Fermat’s little f (z) = 1 + z for all z.
theorem, b p ≡ b (mod p). Thus b ≡ 1 (mod p) and b − 1 is divisible by p. But then Let now a = 1, i.e., f (1) = 0. Then x = 0 yields f ( f (y)) = y for all reals y. Taking now
the numerator b p − 1 must be divisible by p2 , i.e., b p ≡ 1 (mod p2 ). If m ≥ 2 then the
2 y = f (1 − x ) in the initial relation, we get f ( x + f ( f (1 − x ))) = f (1 − x ) + f ( x + 1),
bp − 1 or 0 = f (1 − x ) + f ( x + 1). Finally, taking y = 1 − x yields f ( x + f (1 − x )) = 1 −
representation above contains factor p = 1 + b p + . . . + b p( p−1) . On one hand,
b −1 x + f ( x + 1), so f ( x + f (1 − x )) = 1 − x − f (1 − x ). Let h( x ) = x + f (1 − x ), then
this is congruent to p modulo p2 as all addends are congruent to 1. On the other hand, f (h( x )) = 1 − h( x ) holds for all x. Replacing x with − x and taking y = 1 in the initial
this factor is a power of p while being greater than p, hence it is a multiple of p2 . This relation, we get f (− x ) = 1 + f (1 − x ), so h( x + 1) − h( x ) = 2. Again, h is continuous
contradiction shows that m = 1, qed. and must take all real values, so f (z) = 1 − z for all z.
Remark 1. Fermat’s little theorem can easily be avoided in the solution. Cutting this It is straightforward to verify that both solutions indeed satisfy the initial relation.
out from the solution above, it still shows that if m ≥ 2 then b p − 1 is not divisible by
bp − 1 6. Consider a 10 × 10 grid. On every move, we colour 4 unit squares that lie in the
p2 . Continuing from this, we see that is divisible by p but not by p2 . Hence this
b−1 intersection of some two rows and two columns. A move is allowed if at least one of the
factor must be p. Now 4 squares is previously uncoloured. What is the largest possible number of moves that
bp − 1 can be taken to colour the whole grid?
= 1 + b + . . . + b p−1 > b p−1 ≥ 2 p−1 ≥ p
b−1 Answer: 81.
gives a contradiction. Solution. By always choosing the first line, the first column and a square of the remaining
Remark 2. In the special case b = 2, l = 1, the problem reduces to the well-known fact 9 × 9 grid as the lower right square, the whole grid can be coloured in 81 moves.
that a Mersenne’s number Mn can be prime only if n is prime. We now prove that it is not possible to make more than 81 moves. Consider a sequence
of moves. Select for each move one square that is chosen for the first time during this
Second day move and colour the remaining squares already before starting the sequence. Then,
take all squares that were not selected and colour them in advance, i.e., already before
starting the sequence of moves. Since all selected squares must be different, every move
4. In square ABCD, points E and F are chosen in the in- D F C in the sequence now colours exactly one square.
terior of sides BC and CD, respectively. The line drawn Next, consider a bipartite graph with the 10 rows and 10 columns as vertices. Every time
from F perpendicular to AE passes through the intersec- a square is coloured, draw an edge between the row and the column corresponding to
tion point G of AE and diagonal BD. A point K is chosen this square. We claim that the graph is connected before we start the sequence of moves.
on FG such that | AK | = | EF|. Find ∠EKF. Indeed, suppose that during some move, we pick rows (a, b) and columns (c, d), such
that only the square (b, d) is coloured for the first time, i.e., we add the edge (b, d). But
Answer: 135◦ . K
then b is already connected with d through b − c − a − d, so the number of connected
Solution. Since AGFD is a cyclic quadrilateral (see Fig. 16), E components does not decrease. Since the graph of a fully coloured grid is connected, it
∠GAF = ∠GDF = 45◦ and ∠GFA = ∠GDA = 45◦ , so G
must also be connected in the beginning. But a connected graph with 20 vertices must
triangle AGF is isosceles and | GA| = | GF|. Now, right A B have at least 19 edges, so we can add only 100 − 19 = 81 new edges, and hence any
triangles AGK and FGE are congruent, and | GK | = | GE|,
sequence can have at most 81 moves.
so triangle GKE is also isosceles. Finally, ∠GKE = 45◦ and Figure 16
∠EKF = 180◦ − ∠GKE = 135◦ .
5. Find all continuous functions f : R → R such that for all reals x and y
f ( x + f (y)) = y + f ( x + 1).
Answer: f ( x ) = 1 + x and f ( x ) = 1 − x.
Solution. Taking y = − f ( x + 1), we see that there is a value a such that f (a) = 0. We
consider two cases.
Let first a 6= 1. Taking y = x + 1, we get f ( x + f ( x + 1)) = x + 1 + f ( x + 1). Let
g( x ) = x + f ( x + 1), then f ( g( x )) = 1 + g( x ) for all x. Since f is continuous, so is g.
23 24
Estonian Math Competitions
2007/2008
Tartu 2008
WE THANK:
Problem authors: Juhan Aru, Oleg Koshik, Toomas Krips, Kaie Kubjas, Härmel Nestra,
Hendrik Nigul, Uve Nummert, Reimo Palm, Oleksandr Rybak (Ukraine),
Laur Tooming, Jan Willemson, Indrek Zolk
Translation: Emilia Käsper, Ago-Erik Riet
Edition: Reimo Palm, Indrek Zolk
Estonian Mathematical Olympiad
1
Selected Problems from Open Contests
If the first boomerang covers (1, 1) with its end-square (Fig. 4) then we can assume
without loss of generality that it is positioned as (1, 1) − (1, n) − (n, n). The boomerang
covering (2, 1) must now be positioned as (2, 1) − (n + 1, 1) − (n + 1, n). Those two
have an empty rectangle in between that is too small to fit any boomerangs.
OC-2. Do there exist four different integers a, b, c, d, all greater than one, satisfying
gcd( a, b) = gcd(c, d) and a) ab = cd; b) ac = bd? (Juniors.)
2
Answer: a) yes; b) no.
Solution 1. a) Let x, y, z and w be arbitrary different pairwise co-prime positive integers.
Let a = xy, b = zw, c = xz and d = yw. All these numbers are greater than 1. Then
gcd( a, b) = gcd( xy, zw) = 1 and also gcd(c, d) = 1, whereas ab = cd = xyzw.
b) Assume for contradiction that such a, b, c, d exist. Let s = gcd( a, b) = gcd(c, d). Write
a = a′ s, b = b′ s, c = c′ s, d = d′ s, then gcd( a′ , b′ ) = 1 and gcd(c′ , d′ ) = 1. The equation
ac = bd becomes a′ s · c′ s = b′ s · d′ s, equivalently a′ c′ = b′ d′ . Thus d′ divides a′ c′ and
hence, since c′ and d′ are co-prime, d′ divides a′ . Analogously, since a′ divides b′ d′ and
b′ and a′ are co-prime, a′ divides d′ . It follows that a′ = d′ and a = d. This contradicts
the assumption that all of a, b, c, d are different.
Solution 2. b) Assume for contradiction that such a, b, c, d exist. Write the equation as
a d
= .
b c
When we put equal fractions into lowest terms, we get equal fractions (in lowest terms)
with equal numerators and also equal denominators. The number we divide by is the
greatest common divisor of the numerator and the denominator. Since we are given
gcd( a, b) = gcd(c, d) the denominators and numerators will be divided through by the
same number, that is, the numerators and denominators must be equal to begin with.
Thus a = d and b = c, contradicting the assumption that a, b, c, d are different.
OC-3. How many 5-digit natural numbers are there such that after deleting any one
digit, the remaining 4-digit number is divisible by 7? (Juniors.)
Answer: 8.
Solution. Let M = abcde be a number with the required property. By deleting a and b
we get A = bcde and B = acde, respectively. Since they are divisible by 7, so is their
difference B − A = 1000( a − b), hence a − b is divisible by 7, hence a and b are congruent
modulo 7. Analogously, we have that b and c, that c and d, and finally that d and e are
congruent modulo 7. Thus all the digits are congruent modulo 7.
If M has digits that are at least 7, we can subtract 7 from each such digit to obtain a new
number M′ . It is easy to see that M satisfies the condition in the problem if and only if
M′ does. Since all digits give the same remainder, we are left to consider xxxxx where
0 6 x 6 6. By deleting a digit we get xxxx = x · 1111 that is divisible by 7 only if x = 0.
Indeed, 1111 and 7 are co-prime. Thus every digit of M is either 0 or 7. The first two
digits must be 7 (since the number has 5 digits and any number we get by deleting a
digit has 4 digits), the last three digits can be any of 0 or 7 independently. Thus there
are 2 · 2 · 2 = 8 suitable numbers.
3
and write on the board the numbers
N N N
, , ..., .
1 2 n
1 2 n
These are different positive integers whose inverses are , , ..., respectively. We
N N N
1
see that the differences of consecutive numbers are all equal to − .
N
OC-5. A squaric is a square that has been divided into 8 equal tri-
angles by perpendicular bisectors of its sides and its diagonals. Each
of those lines divides the squaric into two parts; we can take one of
the parts and reflect it over a second dividing line that is perpendicu-
lar to the original line (equivalently, we rotate along the line by 180◦
in space). Every triangle has been coloured by one of four colours and there are two
triangles of each colour. Show that regardless of the initial colouring, the squaric can be
taken to an end position where at every side of the square both triangles have the same
colour. (Juniors.)
Solution. Denote the positions of triangles by numbers 1 to 8 and axes of reflection by
letters x, u, y, v as seen in Fig. 5. Additionally, let A, B, C, D be the colours used.
y D C C D B C C B D D D D
v u C B B C B D D B C B B C
⇒ ⇒ ⇒ ⇒ ⇒
D B D B D C D C B C B C
6 5 A A A A A A A A A A A A
7 4
x D C B B B B
8 3 D B D C C D
1 2 ⇒ ⇒
C B C D C D
A A A A A A
Figure 5 Figure 6
Assume without loss of generality that the triangle in position 1 has colour A. If the
other triangle of colour A is not in position 2, we take it there by reflections that leave
the triangle in position 1 fixed.
Now the triangles at the bottom side of the squaric have the same colour.
Assume without loss of generality that the triangle at position 3 has colour B. If the
other triangle of colour B is not located at position 4, we take it there as follows, always
leaving the triangle in position 1 unmoved:
4
Position of colour B triangle Axes of reflection
5 y, v, y
6 v, y
7 y
8 v, y, v, y
Now the bottom and right sides of the squaric have triangles of suitable colours.
Finally assume without loss of generality that the triangle in position 5 has colour C.
If the other triangle coloured C lies in position 6, we are done, since then the triangles
at positions 7 and 8 must have colour D. If the other triangle of colour C does not lie
in position 6, we use the following reflections to solve the squaric, always leaving the
triangle in position 1 unmoved (cf. Fig. 6):
5
from vertex A of the triangle ABC goes through the circumcentre of ABC, so it is also
the perpendicular bisector of BC. It is possible only if | AB| = | AC |.
Remark. If ABC is a right triangle, the validity of the claim depends on definitions used.
By the usual high-school definition a right triangle does not satisfy the assumptions of
the problem: if the right angle is at A, then S = D and we cannot talk about line DS; if
the right angle is at B there is no intersection point T; if the right angle is at C, the lines
DS and AB would coincide but coinciding lines are not considered parallel at school.
OC-8. Wolf and Fox play the following game on a board with a finite number of unit
squares. In the beginning, all squares are white and empty. First, Wolf picks up a game
piece from a pile, and either places it on a white square, paints this square gray and
removes all the other pieces from the board, or places it on an empty gray unit square.
Then, Fox makes a move by the same rules, only her colour is red and not gray. The
players continue taking turns and the last player to make a move wins (assume there is
an infinite supply of game pieces). Who wins if both play optimally? (Seniors.)
Answer: Wolf.
Solution 1. Let Wolf have the following strategy. If there are white squares on the board
he will place a piece on one (and colour it gray), otherwise on an empty gray square.
Let us prove this is a winning strategy. Since Wolf starts occupying white squares and
colours them at every move, then after each of his moves there are more gray than red
squares. After every move of Fox there are at least as many gray squares than red. This
is true until there are no more white squares. The square coloured last contains a piece,
all the other squares are empty and the game continues so that each player places pieces
on empty squares.
If the last white square is coloured by Wolf there will be more gray squares than red.
Although the square coloured last contains a piece and he cannot move there anymore,
there are at least as many gray squares left as red. This means Wolf can move after every
move of Fox. If the last square is coloured by Fox, it will contain a piece and she cannot
move there afterwards. Therefore Wolf will have more empty gray squares than Fox has
empty red ones. In both cases Wolf gets to make the last move.
Solution 2. Wolf can, after the first move of each player, colour a white square on his
second move, and after that copy moves of Fox.
Solution 3. It is clear that the game always ends, so somebody has a winning strategy.
Suppose Fox has a winning strategy. At the first move no player has a choice (they
colour a white square). At the second move Wolf has two choices.
• He colours a new white square. Then there are two gray squares, one containing a
piece, and one empty red square on the board. By assumption, Fox has a winning
strategy.
• He places a piece on the gray square. Fox can only colour a new white square. After
Fox moves there are two red squares, one containing a piece and one empty gray
square on the board. By symmetry now Wolf has a winning strategy.
Contradiction.
OC-9. The teacher gives every student a triple of positive integers. First, every stu-
dent has to reduce the second and third number by dividing them by their greatest com-
6
mon divisor, then reduce the first and third number of the resulting triple by dividing
them by their greatest common divisor, and finally, reduce the first and second number
of the new triple by dividing them by their greatest common divisor. Then, everybody
has to multiply the numbers in the final triple and tell the result to the teacher. It is
known that the initial triples only differ by the order of numbers. Find the greatest
possible number of different correct answers that the students could get. (Seniors.)
Answer: 3.
Solution. Let ( a, b, c) be the initial triple and
c b
d1 = gcd(b, c), d2 = gcd a, , d3 = gcd a, .
d1 d1
b c a b c
After the first and second division, we get triples a, , and , , , re-
d1 d1 d2 d1 d1 d2
spectively.
a b b c
Let us prove that gcd , = d3 . Since and are co-prime, their divisors d2
d2 d1 d1 d1
a b
and d3 are co-prime. Since d3 divides a, it hence divides . Since d3 divides , it
d2 d1
a b a a b
also divides gcd , . On the other hand, since divides a, clearly gcd ,
d2 d1 d2 d2 d1
divides
d3 whichproves the claim. Therefore the triple after the third division is
a b c abc
, , and the correct answer is 2 2 2 .
d2 d3 d1 d3 d1 d2 d1 d2 d3
If we swap b and c in the initial triple, d1 is left unchanged and d2 and d3 are swapped
which leaves the final answer unchanged. Therefore the answer depends only on what
we choose as the first component of the triple. Thus there can not be more than 3 differ-
ent answers.
We get three different correct answers if we pick a triple ( p2 qr, pq2 r, pqr2 ) where p, q
and r are pairwise different primes. Indeed this triple changes as
giving the answer p2 . By changing cyclically the order of the components in the triple,
answers q2 and r2 are obtained.
OC-10. In a square grid of dimension m × n where m, n > 5, every square has been
coloured black or white. At each step, we can pick some horizontal or vertical strip of
width 1 and odd length that contains squares of both colours, and colour all squares in
this strip by the colour occurring less in the strip. Show that by these steps we can give
all squares the same colour. (Seniors.)
Solution 1. Let the grid have m rows and n columns. At first we shall show that the
squares in each row can be given the same colour. If n is odd we cover a row by one
strip and colour all the squares by the colour that occurs less in that row. If n is even we
cover all squares in the row except for the first one by a strip (of odd length) and give
them one colour. If the first square is coloured differently from the rest of the row we
7
cover the squares 1, 2 and 3 by a strip and give them the same colour, then we cover the
squares 1, 2, 3, 4 and 5 by a strip and colour them by the colour of squares 4 and 5. Now
all the squares in that row have the same colour.
Let us now do the same construction for columns. After that the squares in each column
have the same colour, but also all the colours are equal since after the first stage all
columns looked identical.
Solution 2. At first, prove that a rectangle with dimensions 1 × 5 that contains squares of
both colours can be coloured by each colour. Now we can make the first row monochro-
matic by sequentially choosing the 1 × 5 blocks (or not choosing if a block is already
monochromatic). Then we can similarly give every column the colour of its first square.
Remark. One can find other solutions, precisely by dividing the grid into at most four
sub-grids, with each dimension odd and at least 3, and making the sub-grids and even-
tually the whole grid monochromatic.
OC-11. We are given different positive integers a1 , a2 , . . . , an where n > 3 and every
integer except the first and last one is the harmonic mean of its neighbours. Show that
none of the given integers is less than n − 1. (Seniors.)
1 1 1
Solution. As given, the numbers , , ..., form an arithmetic progression. By
a1 a2 an
1 1 1
symmetry we may assume that > > . . . > , equivalently a1 < a2 < . . . < an .
a1 a2 an
Since a1 , a2 , . . . , an are positive integers,
a − a1 n−2
1 1 1 1 1
> − = ( n − 2) − = ( n − 2) · 2 > .
a2 a2 a n a1 a2 a1 a2 a1 a2
1 n−2
Hence > , and multiplying both sides by a1 a2 gives a1 > n − 2. Since a1 is an
a2 a1 a2
integer, a1 > n − 1. The numbers a2 , . . . , an are greater than a1 and thus greater than
n − 1.
OC-12. Two circles are drawn inside a parallelogram ABCD so that one circle is tan-
gent to sides AB and AD and the other is tangent to sides CB and CD. The circles touch
each other externally at point K. Prove that K lies on the diagonal AC. (Seniors.)
D C
Solution 1. Let O1 and O2 be the centres of
the first and the second circle, respectively
(Fig. 8). Consider the triangles O1 AK and
O1
K O2 O2 CK. Their angles AO1 K and CO2 K are
equal since their sides O1 K and O2 K lie on the
same line and the sides O1 A and O2 C are par-
A B 1 1
allel since ∠O1 AB = ∠ DAB = ∠ BCD =
2 2
Figure 8 ∠O2 CD. As ∠ DAB = ∠ BCD, we have
|O1 A| |O2 C |
= . Therefore the triangles O1 AK and O2 CK are similar. Thus ∠O1 KA =
|O1 K | |O2 K |
∠O2 KC, from which it follows that the points A, K and C are collinear.
Solution 2. Consider the homothety with centre K that takes one of the circles onto the
8
other one. Then the line AB is taken to the line CD and the line AD is taken to the line
CB. Thus the intersection point A of the lines AB and AD goes to the intersection point
C of the lines CD and CB. It follows that the points A, K and C are collinear.
lcm( p a , pb ) pmax(a,b)
a b
= min ( a,b )
= p| a−b| .
gcd( p , p ) p
Therefore for each prime factor one may consider the behaviour of the sequence of ex-
ponents. Thus explore the properties of the sequence (bn ) defined by bn = |bn−1 − bn−2 |
for all n > 2. It is easy to see that either all terms of the sequence are even or there
is a cycle (even, odd, odd). Thus bn+3 and bn have the same parity. Also observe that
bn+3 6 bn for all n.
Consider now the prime factors appearing in given terms. For prime factor 7 one has
b560 = 1 and b1600 = 0. Since 3 | 1601 − 560, previous observations imply that b1601 = 1.
Then b1602 = |1 − 0| = 1. Now the fact that 3 | 2007 − 1602 leads to b2007 = 1. Hence the
exponent of 7 in a2007 is equal to 1.
For the prime factor 5, we have b560 = 1 and b1600 = 2. Analogously with previous case
we obtain b1601 = 1 and also b1602 = 1. Hence as before b2007 = 1, thus the exponent of
5 in a2007 is 1.
9
The prime factor 2 remains. For this b560 = 4 and b1600 = 6. Examine the possible
values of b1601 . Since it must have the same parity as b560 and may not be greater than
it, the only candidates are 0, 2 and 4. Suppose b1601 = 0. Then both b1601 and b1600 are
divisible by 6. Taking into account the definition of the sequence (bn ) implies that 6
divides also all previous terms, including b560 . This leads to contradiction, thus b1601 is
not 0. Suppose that b1601 = 2. Since consequent terms are even, all terms of the sequence
must be even. Dividing all terms by 2 leads to sequence, that still satisfies the definition,
thus all previously considered observations must be valid. The term b560 transforms to
2 and the term b1601 to 1, that means they have different parity. This is a contradiction
analogously to the cases of previous primes.
The last remaining possibility is b1601 = 4 (it is easy to see that a corresponding sequence
exists). Now performing calculations we obtain b1602 = 6 − 4 = 2, b1603 = 4 − 2 = 2,
b1604 = 2 − 2 = 0, b1605 = 2 − 0 = 2 and further the cycle (2, 0, 2) repeats, therewith the
value of terms with the number divisible by 3 is 2. Thus b2007 = 2, hence the exponent
of 2 in a2007 is 2.
Since the terms a560 and a1600 have no other prime factors, taking preceding into account
implies that the term a2007 neither has other prime factors. Hence the only solution is
a2007 = 7 · 5 · 22 = 140.
FR-1. On a railway connecting cities A and B, trains run at full speed except for two
railway segments, where poor track conditions force them to slow down. If any one of
those two segments were repaired, the average speed of a train between A and B would
increase by a third. How much would the average speed between A and B increase if
both segments were repaired? (Grade 9.)
Answer: 2 times.
Solution. Let the train journey between A and B take time t when neither segment is
repaired. If the first segment was repaired, the average speed would increase by a third,
4 3
in other words, times, so the journey would take time t. Thus, repairing the first
3 4
1 1
segment would save t time. Similarly, repairing the second segment would save t.
4 4
1
Repairing both segments would save t and the average speed would increase 2 times.
2
FR-2. Find all possible values of abc · ( a + b + c), given that bca = ( a + b + c)3 and
b 6= 0. (Grade 9.)
Answer: 2008.
Solution. There exist five three-digit cubes: 125 = 53 , 216 = 63 , 343 = 73 , 512 = 83 and
729 = 93 . Of these, only 512 satisfies bca = ( a + b + c)3 . Thus, a = 2, b = 5, c = 1 and
abc · ( a + b + c) = 251 · (2 + 5 + 1) = 2008.
10
O3
O2
C
B A
O1 B
O3 O1
A
O2 C D
O4
Figure 9 Figure 10
FR-3. a) Circles c1 and c2 touch externally at point A, circles c2 and c3 touch externally
at point B, and circles c3 and c1 touch externally at point C. Suppose that triangle ABC
is equilateral. Are the radii of c1 , c2 and c3 necessarily equal?
b) Circles c1 and c2 touch externally at point A, circles c2 and c3 touch externally at point
B, circles c3 and c4 touch externally at point C, and circles c4 and c1 touch externally at
point D. Suppose that ABCD is a square. Are the radii of c1 , c2 , c3 and c4 necessarily
equal? (Grade 9.)
Answer: a) yes; b) no.
Solution. a) Let O1 , O2 and O3 be the midpoints of c1 , c2 and c3 , respectively (Fig. 9).
Triangles O1 CA, O2 AB and O3 BC are isosceles, as each triangle has two radii of the
same circle as its two sides. Let ∠O1 CA = ∠O1 AC = α, ∠O2 AB = ∠O2 BA = β and
∠O3 BC = ∠O3 CB = γ. Suppose that triangle ABC is equilateral, so ∠ ABC = ∠ BCA =
∠CAB = 60◦ . As ∠O1 AC + ∠CAB + ∠O2 AB = 180◦ , we have α + β = 120◦ . Similarly,
β + γ = 120◦ and γ + α = 120◦ . The last three equations together give α = β = γ = 60◦ .
Thus, triangles O1 CA, O2 AB, O3 BC are equilateral and as ABC is also equilateral, they
are in fact equal.
b) Choose the midpoints of the three circles as O1 (6; 0), O2 (0; 3), O3 (−6; 0), O4 (0; −3)
(Fig. 10). Then O1O2O3O4 is a rhombus and points A(2; 2), B(−2; 2), C (−2; −2),
D (2; −2) on the sides of the rhombus form a square. Take each vertex of the rhom-
bus to be the midpoint of a circle drawn through the two closest vertices of the square.
Then these four circles touch externally at A, B, C, D, yet they do not all have equal
radii (e.g., |O1 A| 6= |O2 A|).
11
the same colour as the floor sector, we are done. If, on the other hand, the two sectors
are of different colour, then the stage sector symmetrically opposite to the original sector
satisfies our conditions. This stage sector is completely embraced by the floor sector
symmetrically opposite the original floor sector, however, when turning 180◦ , the stage
sectors change colour n + 1 times, while the floor sectors change colour only n times, so
the two sectors symmetrically opposite to the original sectors are of the same colour.
FR-5. Circles c1 and c2 with midpoints O1 and O2 intersect at point P. Circle c2 inter-
sects O1O2 at point A. Prove that there exists a circle touching c1 at P and O1O2 at A iff
∠O1 PO2 = 90◦ . (Grade 10.)
Solution. Assume first there exists a circle c touching c1 at P and O1O2 at A (Fig. 11). Let
O be the midpoint of c, then line O1O passes through P. Consider triangles OPO2 and
OAO2 . Clearly |OP| = |OA| and |O2 P| = |O2 A| as the radii of circles c and c2 . Also,
the triangles share a third side OO2 , so they are equal. As ∠OAO1 = 90◦ , we must also
have ∠OPO2 = 90◦ .
Assume now ∠O1 PO2 = 90◦ . Then line O2 P is perpendicular to radius O1 P and thus
touches c1 at P. As |O2 P| = |O2 A|, the line drawn through P perpendicular to O2 P
and the line drawn through A perpendicular to O2 A intersect at a point O such that
|OP| = |OA|. A circle with midpoint O and radius OP then touches c1 at P and O1O2
at A.
P
O C D
O1 A O2 A B
D′
Figure 11 Figure 12
FR-6. Do there exist 5 different points in the plane such that all triangles with vertices
at these points are right triangles and
a) no four of the chosen points lie on the same line;
b) no three of the chosen points lie on the same line?
(Grade 10.)
Answer: a) yes; b) no.
Solution 1. a) Choose four vertices of a square and the intersection point of its diagonals.
b) Consider a set of points in the plane such that all triangles with vertices in those
points are right triangles and no three points lie on the same line. Choose some two
points A and B; all the remaining points then lie either on the circle with diameter AB,
or on either line perpendicular with AB drawn through endpoint A or B (Fig. 12). At
most four points (including A and B) can lie on the circle, since any two of such three
12
points must be the two endpoints of some diameter. Also, in addition to A and B, there
can be at most one point on either perpendicular.
Suppose now that C and D are two points satisfying our conditions such that C lies on
the circle and D lies on one of the two lines, say, on the perpendicular drawn through
B. If C and D lie on the same side of AB, then ∠ ACD > ∠ ACB = 90◦ , and ACD is
not a right triangle. If, on the other hand, C and D lie on opposite sides of AB, then
∠ DBC > ∠ DBA = 90◦ , so DBC is not a right triangle. Thus, either all points lie on the
circle, or they all lie on the two perpendiculars. In either case, there can be at most 4
such points.
Solution 2. b) Assume by contradiction that it is possible to choose 5 points satisfying
the conditions. Since each three points form the vertices of a right triangle, there are 10
right triangles with vertices in these 5 points. Thus, there exists a point O that is the
vertex of at least two right angles. Let OAB and OXY be the two triangles with right
angles at O.
Now, if either X or Y was lying on line OA, the other point would have to lie on AB.
But then we would have three points on the same line, since at most one of X and Y
can coincide with A or B. Analogously, neither X nor Y can lie on OB. Now if X (resp.
Y) and B lie on opposite sides of line OA, then XOB (resp. YOB) is an obtuse triangle.
Similarly, X and A (or Y and A) cannot lie on opposite sides of OB. Thus, both X and Y
must lie within the right angle AOB, but then XOY is not a right triangle.
FR-7. Call a rectangle splittable if it can be divided into two or more square parts such
that the side of each square is of integral length and there is a unique square with small-
est side length. Find the dimensions of the splittable rectangle with the least possible
area. (Grade 11.)
Answer: 5 × 7.
Solution. The unique smallest square of the partition cannot lie
on the side of the rectangle, for it would have a larger square
on either side and the area between the two squares could only
be filled by squares no larger than the smallest square. Anal-
ogously, the smallest square cannot lie in the corner. Now, the
distance between the smallest square and any side of the triangle
must be at least one unit longer than the side length of the small- Figure 13
est square, for otherwise the area between the smallest square
and the side could not be filled. Thus, the length of each side of the rectangle is at least
1 + 2 + 2 = 5 and each square on a side must have side length at least 2. Thus, if the
rectangle has a side of length 5, on this side we must have a square with side length at
least 3. But then the distance between the smallest square and this side is at least 3. The
same holds for the opposite side of length 5, so the length of the longer side must be at
least 1 + 3 + 3 = 7. It is possible to partition a 5 × 7 rectangle in the desired way (see
Fig. 13). The area of this rectangle is 35, which is indeed the smallest possible area, since
any rectangle with shorter side length greater than 5 has area at least 6 · 6 = 36.
FR-8. Circles c1 and c2 with respective diameters AB and CD of different length touch
externally at point K. An external tangent common to both circles touches c1 at A and
c2 at C. Line BD intersects c1 again at point L and c2 at point M. Prove that triangles
13
A C
C A
O1 K O1 K O2
O2
M L
D B
L M
B D
Figure 14 Figure 15
FR-9. Let a, b, c be real numbers. Prove that a2 + 4b2 + 8c2 > 3ab + 4bc + 2ca. When
does equality hold? (Grade 11.)
Answer: Equality holds iff a = 2b = 4c.
Solution. Bringing all terms to the lhs, we get
FR-10. Does there exist a convex hexagon ABCDEF such that the circumcircles of
triangles ABC, CDE and EFA intersect at a common point inside the hexagon? (Grade
11.)
Answer: no.
14
B
less than 180◦ , so the sum of the three angles cannot reach
E
3 · 180◦ , contradiction.
Figure 16
FR-11. Find the least possible value of (1 + u2 )(1 + v2 ),
where u and v are real numbers satisfying u + v = 1. (Grade
12.)
25
Answer: .
16
1 1
Solution 1. Write u = + x and v = − x. Then
2 2
1 2 1 2 1
2 2 2
(1 + u )(1 + v ) = 1 + +x 1+ −x = 1+ +x +x ·
2 2 4
2
1 5 25 5 2 25 3 2
· 1 + + x2 − x = + x2 − x2 = + x + x4 − x2 = + x + x4 .
4 4 16 2 16 2
3
Since x2 and x4 are both non-negative, the obtained sum is minimal when x = 0. The
2
25
latter gives (1 + u2 )(1 + v2 ) = .
16
Solution 2. As u + v = 1, we get
(1 + u2 )(1 + v2 ) = 1 + u2 + v2 + u2 v2 =
= 1 + (u + v)2 − 2uv + u2 v2 = 2 − 2(uv) + (uv)2 .
Let s = uv. For a fixed sum u + v = 1, the product s = uv is maximal when u = v. Thus,
1 2 1
we can bound s 6 = . Now, we need to minimize 2 − 2s + s2 = (s − 1)2 + 1,
2 4i
1 1
which is decreasing in −∞; and obtains the minimum at s = .
4 4
Solution 3. Notice that u = 1, v = 0 gives (1 + u2 )(1 + v2 ) = 2, while for any u > 1
or v > 1 (or equivalently, v < 0 or u < 0), (1 + u2 )(1 + v2 ) > 2. Thus, we may
restrict to the case u, v ∈ [0, 1]. Now consider a triangle ABC such that its side BC and
altitude AH (Fig. 17) have unit length and H divides BC to parts of length u and v. Then
1 1 1
u + v = 1 and the law of sines gives · | AB| · | AC | · sin ∠ BAC = · | BC | · | AH | = ,
2 2 2
1
so (1 + u2 )(1 + v2 ) = | AB|2 | AC |2 = . The value sin ∠ BAC is maximal when
sin2 ∠ BAC
H is the midpoint of BC. Indeed, let c be the circumcircle of ABC in the case when
H is the midpoint. For any other configuration, A lies outside this circle c and thus
the angle BAC is smaller (note that the angle BAC is always acute as BC cannot be the
5
longest side of ABC). Now if H is the midpoint of BC, we get | AB|2 = | BC |2 = , and
4
15
25
| AB|2 | BC |2 = .
16
Remark. One may find other solutions, precisely determining the minima of g(u) =
(1 + u2 )(1 + (1 − u)2 ) using derivatives, or writing out Jensen’s inequality for a convex
function l ( x ) = ln(1 + x2 ).
A
A A′ D
O1 O
O2
B
E
B u H v C C
Figure 17 Figure 18
FR-13. All natural numbers that are less than a fixed positive integer n and relatively
prime to it are added one-by-one in increasing order. How many intermediate sums
(starting from the lonely first addend and including the final sum) are divisible by n, if
a) n is an odd prime number?
b) n is the square of an odd prime number?
(Grade 12.)
Answer: a) 1; b) 1.
Solution. a) Let n = p where p is an odd prime. The addends are 1, 2, . . . , p − 1, thus the
intermediate sums have form 1 + . . . + k where 1 6 k 6 p − 1. Suppose p | 1 + . . . + k.
k ( k + 1)
Then p | k (k + 1) as 1 + . . . + k = . Thus either p | k or p | k + 1. This is possible
2
only for k = p − 1.
b) Let n = p2 where p is an odd prime. Any number less than p2 is added if and only
if it is not divisible by p. Divide the addends into p groups, each consisting of p − 1
16
members:
1, 2, . . . . . . , p − 1,
p + 1, p + 2, . . . . . . , p + p − 1,
.......................................................
( p − 1) p + 1, ( p − 1) p + 2, . . . . . . , ( p − 1) p + p − 1.
Let an intermediate sum be divisible by p2 ; then it is divisible by p, too. As the rows
are equivalent modulo p, we can use part a) of the problem to deduce that the last
intermediate sum of every row is divisible by p and the others are not. Hence, in the
whole intermediate sum under consideration, the last row cannot occur partially, i.e.,
our intermediate sum consists of whole rows of addends.
p ( p − 1)
The sum of the elements of the first row is . The sum of the numbers of each
2
following row is by p( p − 1) larger than that of the row preceding it. Thus the row sums
p ( p − 1) p ( p − 1) p ( p − 1)
are 1 · ,3· ,5· etc.. The sum of the numbers of the first i rows
2 2 2
p ( p − 1) p ( p − 1)
is (1 + 3 + . . . + (2i − 1)) · = i2 · .
2 2
p ( p − 1)
If p2 | i2 · then p | i2 · ( p − 1), implying p | i. Hence i = p, i.e., the sum is the
2
final sum.
Remark. It is easy to show that the entire sum of ϕ(n) addends is divisible by n for all
integers n > 2. If n is neither a prime nor the square of a prime then there can be more
intermediate sums divisible by n. For example, if n = 16 then the intermediate sum
1 + 3 + 5 + 7 containing only half of the addends is divisible by 16. If n = 27 or n = 39
then two intermediate sums in addition to the final sum are divisible by n, etc.
FR-14. Consider a point X on line l and a point A outside the line. Prove that if there
exists a point Z1 on l such that the three side lengths of triangle AXZ1 are all rational,
then there exist two other points Z2 and Z3 on l such that the side lengths of triangles
AXZ2 and AXZ3 are also all rational. (Grade 12.)
Solution. We consider three separate cases.
• If AXZ1 is equilateral, i.e., | AX | = | AZ1 | and ∠XAZ1 = 60◦ , then take Z2 on
the extension of Z1 X across X such that | XZ2 | = 0.6 | AX |, and take Z3 to be the
reflection of Z2 across the perpendicular bisector of XZ1 (Fig. 19). The law of
cosines implies | AZ2 | = | AX |2 + | XZ2 |2 − 2 · | AX | · | XZ2 | · cos 120◦ = 1.96 | AX |2 ,
so | AZ2 | = 1.4 | AX | and the side lengths of AXZ2 as well as AXZ3 are rational.
Let now AXZ1 be not equilateral.
A A
Z2 X Z1 Z3 Z3 X Z1 Z2 X Z3 Z1 Z2
17
• Assume | XZ1 | 6= | AX | and | XZ1 | 6= | AZ1 |. Choose Z2 on ray XZ1 such that
∠Z2 AX = ∠ AZ1 X, and choose Z3 on ray Z1 X such that ∠Z3 AZ1 = ∠ AXZ1
(Fig. 20). Points Z2 and Z1 differ since ∠ AZ1 X 6= ∠Z1 AX, and points Z3 and X
differ since ∠ AXZ1 6= ∠XAZ1 . Triangles Z2 XA and Z3 AZ1 are similar to triangle
| AX | | Z A|
AXZ1 with similarity ratios and 1 , so their side lengths are rational, and
| Z1 X | | Z1 X |
| XZ3 | is rational, too.
• Assume w.l.o.g. | XZ1 | = | AZ1 | (Fig. 21). Choose Z2 as before, then Z2 differs from
X and Z1 and the side lengths of AXZ2 are rational. Take Z3 to be the reflection of
Z1 across the perpendicular bisector of XZ2 . Then Z3 differs from Z1 , as AXZ1 is
not an isosceles right triangle. Triangle AXZ3 is equal to triangle AZ2 Z1 , and the
latter has rational side lengths.
FR-15. A finite number of thin straight pins are attached to a vertical wall such that
no two pins touch each other. If a pin is detached, it slides straight down the wall,
keeping its original angle to the floor. Prove that there exists a pin that can slide freely
down to the floor without being stopped by any of the other pins. (Grade 12.)
Solution 1. If there exists a vertical pin that can slide freely, we are done. Assume now
that no vertical pin can slide down freely. Draw a horizontal line l where the wall meets
the floor and project the endpoints of each pin onto l. If there are no pin points between
some left endpoint and l, colour the projection point on l blue. Similarly, if some right
endpoint is the lowermost pin point on its projection line, colour the corresponding
projection point yellow. Clearly, the leftmost projection point on l is coloured blue, while
the rightmost point is yellow. Thus, moving on l left-to-right, some two consecutive
coloured points must be blue and yellow, respectively. We claim that these points are
the two endpoints of the same pin, and thus this pin can slide down. Indeed, on the
segment between the blue and the yellow point, any lowest pin point above l must
belong to the same pin as the left (blue) and the right (yellow) endpoint.
Solution 2. We prove by induction on the number of pins. The claim clearly holds for
one pin. Assume there is more than one pin, and consider three cases.
1. There exists a pin p such that below every point of p, there is a point of some other
pin. Remove p, then by the induction assumption, some pin v can slide down freely.
Now, put p back. Then p cannot be the only pin stopping v, since below every point
of p, there is a point of another pin, and at least one of those pin points should also
be stopping v.
2. There exists a pin p which cannot slide down freely such that all pins stopping p
lie entirely below p. Remove p and all the remaining pins that do not lie below
p. Then, there must exist a pin v that can slide down freely, but then v can also
slide down in the original configuration, since the only pins possibly stopping it
are those below p.
3. If the previous two cases do not hold, then each pin has some points that have no
other pin points below them, and either the pin can slide down or one of the pins
stopping it does not lie entirely below this pin. Let p be the pin with the rightmost
point amongst all pins (if there is more than one such pin, choose the one with the
topmost such point). Remove p. By the induction assumption, there now exists a
18
pin v that can slide down. Put p back. If v can still slide down, we are done. In the
other case, the only pin stopping it is p. Since v must have some free points with
no other pin points below, the left endpoint of v must reach further left than the left
endpoint of p. We claim that now p can slide down. Indeed, any pin points below
p that also lie below v cannot be stopping p, as they would also be stopping v. But
any other pin below p can also not be stopping p, as p has the rightmost endpoint,
so any pins stopping p should lie completely below p.
Remark 1. One can find other solutions, precisely using a directed graph with pins as
its vertices and an edge from vertex a to vertex b if pin b is stopping pin a: it suffices to
prove that this graph does not contain any directed cycles.
Remark 2. The claim does not always hold when the pins are not straight. For example,
two half-circle pins can be placed to mutually stop each other.
19
each group) have competed against each other at least once. For rounds 1, . . . , k make
up two new equal teams by taking one team corresponding to each group and putting
them together. Assume without loss of generality that in round k one team consists of
s
the programmers of the first group with numbers 1, . . . , and of the second group with
2
t
numbers s + 1, . . . , s + . In the new round swap and make up a team of programmers
2
s t
with numbers 1, . . . , and s + + 1, . . . , s + t. We can check that every two program-
2 2
mers (also from different groups) have now been in different teams at least once.
The other part can be done like in Solution 1.
Answer: b) No.
Solution 1. a) Let E be the intersection point of the bisectors from B and D. By the
bisector property,
| AB| | AE| | AD |
= = . (1)
| BC | | EC | | DC |
By Ptolemy’s theorem, | AB| · |CD | + | AD | · | BC | = | AC | · | BD |. Using this in (1), we
obtain
2 · | BC | · | AD | = | AC | · | BD |, (2)
20
The desired claim follows now by multiplying the corresponding sides of (4) and (5)
and finding the square root.
b) Let ABCD be a rectangle where | AB| > | BC |. Clearly | AG | = | BF | = |CG | = | DF | =
1 1
| AC | = | BD |, implying the rhs of the implication of part a) (lhs of the converse). But
2 2
| AB| | AD |
> 1 > shows that the bisectors of angles at B and D do not intersect on
| BC | | DC |
diagonal AC. Hence the converse implication is false.
Solution 2. a) Denote the interior angles of ABCD by ∠ A, ∠ B, ∠C, ∠ D. In triangle DAB,
cosine law gives
In triangle BCD, taking into account that ∠C = 180◦ − ∠ A, cosine law gives
−→ −→ −→
On the other hand, 2 AG = ( AB + AD ) implies
16 · | AG |2 · |CG |2 =
= (| AB|2 + | AD |2 )(|CB|2 + |CD |2 ) − 4 · | AB| · | AD | · |CB| · |CD | · cos2 ∠ A −
− 2 (| AB|2 + | AD |2 ) · |CB| · |CD | − (|CB|2 + |CD |2 ) · | AB| · | AD | · cos ∠ A.
If the bisectors of angles by B and D intersect on diagonal AC, the bisector property
| AB| | AD |
gives = or | AB| · |CD | = | AD | · |CB|. Thus
|CB| |CD |
21
TS-3. Let n be a positive integer and x, y positive real numbers such that x n + yn = 1.
Prove the inequality
! !
n n
1 + x2k 1 + y2k 1
∑ 1 + x4k ∑ 1 + y4k < (1 − x)(1 − y) .
k =1 k =1
1 + x2k 1
Solution. Note first that < . Indeed,
1 + x4k xk
Second day
TS-5. Points A and B are fixed on a circle c1 . Circle c2 , whose centre lies on c1 ,
touches line AB at B. Another line through A intersects c2 at points D and E, where D
lies between A and E. Line BD intersects c1 again at F. Prove that line EB is tangent to
c1 if and only if D is the midpoint of the segment BF.
22
Solution 1. Let K be the second intersection point of the line AD and the circle c1 (Fig. 22).
The triangles KFD and BAD are similar since the corresponding angles are equal. The
triangle BAD is similar to the triangle EAB since, by tangent-secant theorem, ∠ ABD =
∠ BED and they have a common angle at the vertex A. Let O be the centre of the circle
c2 . Since AB is the tangent to the circle c2 at point B, AB ⊥ BO. It follows that AO is a
diameter of the circle c1 since O is on the circle c1 by assumption. Hence also OK ⊥ AK
from which it follows that OK is an altitude of the isosceles triangle ODE. Thus | DK | =
|KE|.
The line EB is tangent to the circle c1 at B if and only if ∠EBK = A
∠ BAD. Since ∠ ABD = ∠ BED, the last equality is equivalent to F
the triangles EKB and BAD being similar. By the same equality
| AB| D
of angles, the two triangles are similar if and only if =
| BE| B
| DB|
. Since EAB and KFD are similar triangles, the last equality
|KE| K
| FD | | DB| O
is equivalent to = . This is equivalent to | FD | =
| DK | |KE|
| DB| since the denominators are equal.
Solution 2. As in the first solution we show that | DK | = |KE|. E
Let | AD | = x, | DK | = |KE| = y, | BE| = z, | DB| = u, | FD | = v,
| AB| = w. By the property of intersecting chords, uv = xy. Figure 22
2
Since AB is a tangent, w = x ( x + 2y). The triangles ABD and
AEB are similar since ∠ ABD = ∠ BED and at vertex A they
u z uw
have a common angle. Thus = and hence z = .
x w x
The condition that the line EB is tangent to the circle c1 is equivalent to
z2 = y( x + 2y). We shall show that the last condition is equivalent to u = v:
u2 w2
z2 = y( x + 2y) ⇔ = y( x + 2y) ⇔ u2 x ( x + 2y) = x2 y( x + 2y) ⇔
x2
⇔ u2 = xy ⇔ u2 = uv ⇔ u = v.
TS-6. A string of parentheses is any word that can be composed by the following rules.
1) () is a string of parentheses.
2) If s is a string of parentheses then (s) is a string of parentheses.
3) If s and t are strings of parentheses then st is a string of parentheses.
The midcode of a string of parentheses is the tuple of natural numbers obtained by find-
ing, for all pairs of opening and its corresponding closing parenthesis, the number of
characters remaining to the left from the medium position between these parentheses,
and writing all these numbers in non-decreasing order. For example, the midcode of
(()) is (2, 2) and the midcode of ()() is (1, 3). Prove that midcodes of arbitrary two
different strings of parentheses are different.
Solution. We can assume that the two strings have equal lengths because otherwise their
midcodes differ by length. We prove the desired claim by induction on the length. In the
23
case of length 2, the claim holds trivially. Let s and t be two longer strings of parentheses.
Consider, for both of them, the longest prefix that forms a string of parentheses itself.
The first and the last character of such prefix form a pair of opening and corresponding
closing parenthesis.
If the prefixes of s and t under consideration have different lengths 2k and 2l, respec-
tively, where assume w.l.o.g. that k < l, then consider the first k numbers in the mid-
codes of both strings. Let the opening parentheses occur at positions a1 , . . . , ak and the
corresponding closing parentheses occur at positions b1 , . . . , bk in word s. The number
a + bi − 1
in midcode that corresponds to the ith pair of parentheses is i . As the first 2k
2
characters of s form a string of parentheses, numbers a1 , . . . , ak , b1 , . . . , bk are precisely
1, . . . , 2k in some order. Thus the sum of k smallest members of the midcode of s is
k
a i + bi − 1 1 + 2 + . . . + 2k k
∑ 2
=
2
− .
2
i =1
In the midcode of t, the sum of k smallest members is larger since, otherwise, the sum
of position indices of some k pairs of parentheses would be 1 + 2 + . . . + 2k. This would
imply that the corresponding closing parenthesis for each opening parenthesis among
those at positions 1, 2, . . . , 2k occurs within the same positions, leading to l 6 k, a con-
tradiction.
If both prefixes under consideration have length 2k then, for both cases, the part of the
word between the first and the last character of the prefix forms a string of parentheses,
as does the part of the word remaining after the prefix (provided they are non-empty).
As s and t differ, either the first mentioned parts of the words or the second mentioned
parts differ.
In the former case, the induction hypotheses implies that their midcodes also differ. In
the midcodes of s and t, these midcodes are represented by numbers that are by 1 larger,
whereby all these numbers are less than 2k. In addition, both midcodes contain k (from
the pair of parentheses embracing the prefix) and the remaining numbers are larger than
2k. Thus the midcodes of s and t differ.
In the latter case, the induction hypothesis again implies that the midcodes of the parts
after the prefix are different. In the midcodes of s and t, these midcodes are represented
by numbers that are by 2k larger. All other numbers in the midcode are less than 2k.
Hence the midcodes differ.
Remark. A tuple of positive integers x1 , . . . , xn is a midcode of some string of parentheses
k n
iff it is monotone, ∑ xi > k 2
for every k = 1, . . . , n, and ∑ xi = n2 .
i =1 i =1
24
Estonian Math Competitions
2008/2009
Tartu 2009
WE THANK:
University of Tartu
Problem authors: Juhan Aru, Maksim Ivanov, Oleg Koshik, Toomas Krips,
Kaie Kubjas, Härmel Nestra, Uve Nummert, Oleksandr Rybak (Ukraine),
Laur Tooming, Jan Willemson, Indrek Zolk
Translators: Emilia Käsper, Härmel Nestra, Ago-Erik Riet
Editor: Indrek Zolk
*
This booklet contains problems that occurred in the open contests, the final round of
national olympiad and the team selection contest. For the open contests and the final
round, selection has been made to include only problems that have not been taken from
other competitions or problem sources and seem to be interesting enough. The team
selection contest is presented entirely.
1
Selected Problems from Open Contests
OC-1. The feet of the altitudes drawn from vertices A and B of an acute triangle ABC
are K and L, respectively. Prove that if | BK | = |KL| then the triangle ABC is isosceles.
(Juniors.) C
L
Solution 1. From | BK | = |KL| we have that BKL is an isosceles
π K
triangle and ∠KBL = ∠KLB (Fig. 1). Now ∠KLC = − ∠KLB,
2
π
∠KCL = ∠BCL = − ∠KBL. Thus ∠KLC = ∠KCL. So |KC| = A B
2
|KL| = | BK |. As the altitude AK is now also a median, the
triangle ABC is isosceles and | AB| = | AC|. Figure 1
π
Solution 2. Similarly to Solution 1 ∠KBL = ∠KLB. As ∠ AKB = ∠ ALB = (Fig. 1), the
2
points A, B, K and L are concyclic. Now ∠KAB = ∠KLB = ∠KBL = ∠KAL whence the
altitude AK is also the bisector of angle CAB. Thus ABC is isosceles.
Remark. The claim can be proven for any triangle.
1 1 1
OC-2. A computer program adds numbers , , , etc., and represents every inter-
1! 2! 3!
mediate sum as a fraction in lowest terms. Prove that for every positive integer k there
is a fraction among the results whose denominator is divisible by at least k different
primes. (Seniors.)
n! n! n!
Solution 1. For every positive integer n, denote an = + + . . . + , then the n-th
1! 2! n!
an
intermediate sum is .
n!
Let k be a fixed positive integer. Let t be such that there are at least 2k − 1 primes
less than or equal to t; then t! is divisible by at least 2k − 1 primes. If there are at least k
at
different prime divisors of the denominator of the fraction we get by writing in lowest
t!
terms, we have what we were looking for. Otherwise, at is divisible by all but at most
k − 1 prime divisors of t! – there are at least k of those. Note that at+1 = (t + 1) · at + 1.
Thus at and at+1 are coprime. So at+1 is not divisible by the prime divisors of t! we
at +1
considered. Thus, writing in lowest terms, the denominator will have at least k
( t + 1) !
prime divisors, so in this case we also have the fraction we were looking for.
Solution 2. Define an as in Solution 1; note that, for every n > 1, an = n · an−1 + 1. Let
n be a number divisible by k different primes. Then the n-th intermediate sum is of
n · an −1 + 1
the form . The numerator is obviously coprime with n whence we cannot
n!
reduce the fraction by dividing numerator and denominator by any prime divisor of n
– there are k of those by construction. The number n! in the denominator is divisible
by all of those primes, thus the denominator is divisible by at least k primes even after
writing the fraction in lowest terms.
2
OC-3. Three circles in a plane have the sides of a triangle as their diameters. Prove
that there is a point that is in the interior of all three circles. (Seniors.)
C
Solution 1. Let the triangle ABC be given. Assume w.l.o.g. that
the largest angle is at C (Fig. 2). As the angles at A and B are
acute, the foot of the altitude drawn from C – let this be F – lies
A
between A and B. Since CFA is a right angle, the circle with
F B
diameter CA goes through point F, thus the altitude CF is a
Figure 2 chord to the circle. Analogously, CF is a chord to the circle with
diameter CB. Hence all the points on the altitude CF except C
and F lie inside both of the circles. Since point F is in the interior of the triangle with
diameter AB, there are interior points of the segment FC that are in the interior of that
circle.
Solution 2. Given the triangle ABC, let the angles at A, B, C have C
π α β π π
, we have ∠ AIB = π −
2 2
+ > π− = , i.e. AIB A B
2 2 2 2 2
is an obtuse-angled triangle with the obtuse angle at I. This
Figure 3
shows that I is inside the circle with diameter AB. Analogously
we show that I is inside both of the other circles.
OC-4. There are three ants at vertex A1 of the regular n-gon A1 A2 . . . An initially. Each
minute some two of them move simultaneously to a neighbouring vertex in different
directions (one clockwise and the other counter-clockwise), while the third ant stands
still. For which n can it happen that after some time all the ants meet at a vertex different
from A1 ? (Seniors.)
Answer: if n is divisible by 3.
Solution. Note that the sum of numbers of the vertices on which the ants sit is an invari-
ant modulo n. As initially this sum is 3, it will always be 3 modulo n.
We now show that if 3 does not divide n then it is not possible for the ants to meet at a
different vertex. Suppose that the ants are all at k after a move. Then 3k is congruent to
3 modulo n, or 3k − 3 = 3(k − 1) is divisible by n. This is possible only if k = 1.
Finally show that if n is divisible by 3 then it is possible to meet at another vertex. Let
n
the first and second ants move times, ending up in A n3 +1 , resp. A 2n +1 . Then the third
3 3
n
and second ants move times, taking the third ant to A n +1 and the second ant to A n +1 .
3 3 3
After that all the ants sit at A n3 +1 .
OC-5. A unit square is removed from the corner of the n × n grid where
n > 2. Prove that the remainder can be covered by copies of the figures
consisting of 3 or 5 unit squares depicted in the drawing. Every square
must be covered once and the figures must not go over the bounds of the grid. (Seniors.)
Solution. Assume w.l.o.g. that the unit square removed is the one in the bottom right
3
corner. Further let us write “n × n grid” for the grid with the bottom right corner square
removed.
The cases n = 2, 3, 4 can be done by trial (Figures 4, 5 and 6).
We show how to extend the construction for n to a construction for n + 3; by repeating
this procedure, we get a solution for any positive integer n.
Let n be odd; consider an (n + 3) × (n + 3) grid. We cover the n × n grid in its bottom
right corner, then cover the (n − 1) × 3 band on the left and the 3 × (n − 1) band above
by 2 × 3 rectangles formed by two 3-square figures (Fig. 7). We are left with a 4 × 4 grid
in the top left corner which we can already cover.
Let n be even; consider an (n + 3) × (n + 3) grid. Cover the n × n grid in its lower right
corner, then cover the (n − 2) × 3 band on the left and the 3 × (n − 2) band above by
2 × 3 rectangles. The remaining part in the top left corner can be covered like shown in
Fig. 8.
OC-6. Find all posivite integers n for which there are exactly 2n pairs of integers (a, b)
where 1 6 a < b 6 n and b is divisible by a. (Seniors.)
Answer: 15.
Solution. Denote the number of pairs corresponding to an integer n by g(n). Obviously
g(1) = 0. Let n > 1. The pairs whose second component is at most n − 1 have been
counted for n − 1. We have to add pairs with the second component n and the first
component a proper divisor of n. Denoting the number of proper divisors of n by d(n),
we have
g ( n ) = g ( n − 1 ) + d ( n ). (1)
4
D C c D
γ C
A α β B A B
c
U V U
Figure 9 Figure 10
OC-7.
a) An altitude of a triangle is also a tangent to its circumcircle. Prove that some angle
of the triangle is larger than 90◦ but smaller than 135◦ .
b) Some two altitudes of the triangle are both tangents to its circumcircle. Find the
angles of the triangle. (Seniors.)
180◦ = α + β + γ = α + 2γ − 90◦ ,
OC-8. Find all functions f from positive real numbers to positive real numbers such
that the curve y = c · f ( x ) is symmetric with respect to the line y = x for every positive
real number c. (Seniors.)
5
a
Answer: f ( x ) = where a is an arbitrary positive real number.
x
Solution. The symmetry of the curve y = g( x ) with respect to the line y = x is equivalent
to the condition that g( g( x )) = x for each positive real number x. Indeed, the point
( x, g( x )) lies on the curve y = g( x ). By symmetry, the point ( g( x ), x ) is also on the
curve – but this is equivalent to g( g( x )) = x.
Using this for the curve y = c · f ( x ), we have for every c > 0 and x > 0 that
c · f (c · f ( x )) = x. (2)
1 1 f (1 )
Taking c = in the identity (2) we have · f (1) = x, i.e. f ( x ) = for every
f (x) f (x) x
positive real x.
a
It remains to check that every function of form f ( x ) = where a > 0 satisfies the
x
ca
conditions of the problem. Consider the function g where g( x ) = c · f ( x ) = and
x
c > 0. Now
ca ca
g( g( x )) = g = ca = x,
x x
which means that the curve y = c · f ( x ) is symmetric with respect to the line y = x.
Remark. There are other possibilities to determine all possible functions. E.g. taking
x = c = 1 in the identity (2), we have f ( f (1)) = 1, yielding by the substitution x = f (1)
in (2) that for every positive real number c it holds that c · f (c) = f (1).
FR-1. In triangle ABC, points F and E are chosen on sides AC and BC, respectively,
such that 2|CF| = | FA| and 2|CE| = | EB|. Outside triangle ABC, points K and L are
chosen on rays AE and BF, respectively, such that 2|KE| = | EA| and 2| LF| = | FB|.
Prove that ABKL is a parallelogram. (Grade 9.)
Solution 1. Note that △ ABF ∼ △CLF and △ ABE ∼ △KCE with similarity ratio 2
(Fig. 11). Indeed, ∠ BFA = ∠ LFC and ∠ BEA = ∠CEK (opposite angles) and we know
that | AF| = 2|CF|, | BE| = 2|CE|, | BF| = 2| LF| and | AE| = 2|KE|. Thus, LC and CK
are parallel to AB and 2| LC| = 2|CK | = | AB|. Consequently, points L, C and K are
collinear, | LK | = | AB| and LK k AB, implying that ABKL is a parallelogram.
C K
Solution 2. The assumptions of the problem directly yield: L
FR-2. Call a positive integer m magic if the sum of its digits equals the product of its
digits.
a) Prove that for all n = 1, 2, . . . , 10, there exists a magic number consisting of precisely
n digits.
b) Prove that there exist infinitely many magic numbers. (Grade 9.)
Solution 1. a) The numbers 1, 22, 123, 1124, 11125, 111126, 1111127, 11111128, 111111129
and 1111111144 are magic.
b) Given any positive integer for which the product of its digits is larger than the sum,
we can construct a magic number by appending a suitable number of 1-digits: each
appended 1 increases the sum by 1 without changing the product. Now, for any n > 0,
the product of the digits of 22 . . . 2} equals 2n > 2n, the sum of the digits, so for any n,
| {z
n
we can construct a magic number with at least n digits.
Solution 2. b) Let m > 1 be a magic number. We can always construct a larger magic
number m′ by appending 2 to m, followed by a suitable number of 1-digits, as append-
ing a 2 increases the sum of the digits by 2 and the product of the digits by at least 2,
while appending a 1 increases the sum by 1 without changing the product.
FR-3. Juku and Miku play a game on a rhombus of side length n consisting
of two equilateral triangles divided into equilateral triangular tiles with side
length 1 (n = 3 in the figure). Each player has one token. At the beginning
of the game, the tokens lie on the topmost and bottommost tile, respectively.
Players alternate moves by sliding their token one step to an adjacent tile
(tiles are adjacent if they share a side). A player wins the game by capturing
his opponent’s token (moving his own token to the same tile where the opponent’s
token lies); or by reaching his opponent’s starting tile. Suppose Juku makes the first
move. Does either of the players (who?) have a winning strategy? (Grade 9.)
Answer: Juku has a winning strategy.
Solution. We show that Miku cannot capture Juku’s token. Colour the tiles with the
triangle “pointing upwards” black, and the remaining tiles white, then any two adjacent
tiles are of different colour and every move takes the token from a black tile to a white
tile or vice versa. As the starting tiles are of different colour, after every two moves, the
tokens again lie on tiles of different colour and no Miku’s move can result in capturing
Juku’s token.
7
Consequently, Juku can safely take the shortest route to his opponent’s starting tile.
Since the shortest route has equal length for Juku and Miku, Juku is bound to reach his
goal first.
FR-4. Find all pairs of positive integers (m, n) such that in an m × n rectangular grid,
the number of unit squares touching at least one side of the rectangle equals the number
of remaining unit squares. (Grade 9.)
Answer: (5, 12), (6, 8), (8, 6), (12, 5).
Solution. We can exclude m = 1 or n = 1, as then all unit squares touch a side.
Assume now m > 2, n > 2. There are 2m + 2n − 4 unit squares touching a side, so the
number of remaining squares is mn − 2m − 2n + 4. We require 2m + 2n − 4 = mn −
2m − 2n + 4, which is equivalent to (m − 4)(n − 4) = 8. As m − 4 > −2 and n − 4 > −2,
we can eliminate {−1, −8} and {−2, −4} as possible factorings of 8. The remaining
possibilities {1, 8} and {2, 4} yield four solutions for (m, n).
(Grade 10.)
Solution. Note that
180 x+y+z x y z y z
= = + + = 1+ + . (3)
x x x x x x x
y 1 z 180
a) Assume = x and are rational. By (3), is a sum of three rationals and thus
x y x x
itself rational. Hence, x is rational. The proof for y and z is analogous.
x y y z z x
b) Assume w.l.o.g. (and thus also ) is rational and , (and thus also , ) are
y x z x y z
180
irrational. The equation (3) then represents as a sum of one irrational and two
x
180 x
rational numbers. Hence, is irrational, and so is x. As is rational, y must also be
x y
irrational.
x+y
Assume now z is rational, then x + y = 180 − z is also rational. Now as a ratio
y
x+y x
of a rational and an irrational number is irrational, yet = + 1 as a sum of two
y y
rationals must be rational, contradiction.
FR-6. Find all triples of positive integers ( x, y, z) satisfying 99x + 100y + 101z = 2009.
(Grade 10.)
Answer: (1, 9, 10), (2, 7, 11), (3, 5, 12), (4, 3, 13), (5, 1, 14).
8
Solution. We bound the lhs from both sides:
FR-8. Mari and Jüri play a game on an 2 × n rectangular grid (n > 1) whose sides of
length 2 are glued together to form a cylinder. Alternating moves, each player cuts out
a unit square of the grid. A player loses if his/her move causes the grid to lose circular
connection (two unit squares that only touch at a corner are considered to be discon-
nected). Suppose Mari makes the first move. Which player has a winning strategy?
(Grade 10.)
Answer: Mari, if n is odd; Jüri, if n is even.
Solution 1. Consider the grid immediately before one of the players is forced to make
a losing move. Divide missing squares into horizontal blocks of consecutive squares
in one row of the grid. Clearly, no upper and lower block can overlap, as this would
disconnect the grid. If the whole upper row is missing, then the lower row has no
9
missing squares, and vice versa. In this case, there have been n moves, so Mari wins if
n is odd and Jüri wins if n is even.
Assume now no horizontal missing block covers the whole cylinder. Notice that if a
block is missing in the upper row, then its neighbouring column cannot have a missing
square in the lower row, and vice versa. Thus, there is always a full column between
two missing blocks. On the other hand, if there was more than one column between
some two missing blocks, it would be possible to make another move without losing
by extending one of the missing blocks. Finally, if two consecutive missing blocks were
both in the same row, it would be possible to make a move by removing the square
between them.
We conclude that before the last move, the missing blocks are in the upper and lower
row, alternatingly, implying that the number of missing blocks is even. As there is
exactly one full column between two neighbouring missing blocks, the number of full
columns f is also even. The remaining columns have exactly one missing square, so the
number of missing squares is m = n − f . If n is odd, m is odd and Mari wins; if n is
even, m is even and Jüri wins.
Solution 2. Consider the planes of symmetry of the cylinder that intersect the midpoints
of its bases. Choose a plane p such that at least one of its intersections with the cylinder
surface coincides with a grid line.
If n is even, both intersections coincide with the grid and reflecting over p divides the
unit squares into symmetric pairs. Jüri’s strategy is to cut out a square symmetric to
the square Mari chose. We claim that if Mari’s move does not disconnect the grid, then
neither does Jüri’s follow-up move. Indeed, if Mari removes a square not adjacent to
p, then Mari’s move does not affect the grid around Jüri’s square. Thus, the moves
are mirror images of each other and have identical impact. If Mari removes a square
adjacent to p, then before Jüri’s move, the grid around his square has one additional
missing adjacent square compared to the grid around Mari’s square before her move.
Thus, Jüri’s move can only disconnect the grid along the line between his square and
Mari’s. However, since the two removed squares are in the same row, and the two other
squares in the same column must be present (for otherwise Mari’s move would have
disconnected the grid), the disconnect does not happen. It follows that the grid can
only disconnect after Mari’s move, so Jüri has a winning strategy.
If n is odd, reflecting over p divides all squares into symmetric pairs save for two
squares that p intersects in the middle. Let Mari first choose one of those two squares.
Then Jüri cannot choose the other square without losing, as this would disconnect the
square. Thus, Mari can now use the strategy of choosing symmetric squares to win the
game.
FR-9. The teacher asks Arno to choose some of the positive factors of 200910 such that
no chosen factor divides another chosen factor. At most how many factors can Arno
choose? (Grade 10.)
Answer: 11.
Solution. Since 2009 = 72 · 41, where 7 and 41 are primes, we can represent all factors
of 200910 in the form 7n · 41m , where 0 6 n 6 20 and 0 6 m 6 10. Since there are 11
possible choices for m, Arno can choose at most 11 factors. Otherwise, by the Pigeonhole
principle, two of the chosen factors would have the same exponent m, and the one with
10
a smaller exponent n would divide the other.
Now, we show that none of the 11 factors of the form 720−m · 41m , m = 0, 1, . . . , 10,
is divisible by another. Assume by contradiction that for some two m1 and m2 , 720−m1 ·
41m1 divides 720−m2 · 41m2 . But then 20 − m1 6 20 − m2 and m1 6 m2 , implying m1 = m2 ,
contradiction.
FR-10. Let n > 18 be a positive integer such that n − 1 and n + 1 are both primes.
Prove that n has at least 8 different positive factors. (Grade 11.)
Solution 1. First, since of the three consecutive integers n − 1, n, n + 1, two are primes,
the third, n, must be divisible by both 2 and 3. Thus, 1, 2, 3 and 6 are factors of n.
Next, n > 18 = 3 · 6. If n = 5 · 6 = 30, we get four additional factors 5, 10, 15 and 30.
We can excldue n = 4 · 6 = 24 and n = 6 · 6 = 36, as 24 + 1 and 36 − 1 are not prime.
n n n n n
Finally, if n > 6 · 6, then 6 < , so n has four additional factors , , , larger than 6.
6 1 2 3 6
Solution 2. As above, note that n is divisible by 2 and 3. We can express the number
of factors via the exponents in the canonical representation of n. If n has a third prime
factor, then its canonical representation contains at least 3 primes with exponents at
least 1, so the number of factors is at least (1 + 1) · (1 + 1) · (1 + 1) = 8. Assume now
n = 2a 3b . If a = 1, then b > 2, and vice versa, so the number of different factors in this
case is at least (1 + 1) · (3 + 1) = 8. Finally, if a, b > 1, the number of different factors is
at least (2 + 1) · (2 + 1) = 9.
FR-11. Find all real numbers k that satisfy 0 6 a + b − kab 6 1 for all real numbers a
and b such that 0 6 a 6 1 and 0 6 b 6 1. (Grade 11.)
Answer: 1 6 k 6 2.
Solution. First, we show that if 1 6 k 6 2, then k satisfies the condition. Let 0 6 a, b 6 1,
then a2 6 a and b2 6 b. We get
Next, we show that the condition does not hold for k < 1 and k > 2. Take a = b = 1,
then a + b − kab = 2 − k. Now k < 1 yields a − b − kab = 2 − k > 1 and k > 2 yields
a − b − kab = 2 − k < 0.
11
as each of the n numbers on the diagonal occurs an equal 2n times on the lhs, and each
n ( n2 + 1 )
of the n2 numbers occurs twice on the rhs. Solving for D gives D = .
2
FR-13. Call a point in a plane rational (resp. irrational) if both its coordinates are ratio-
nal (resp. irrational).
a) Does every point on the plane lie on some line defined by two rational points?
b) Does every point on the plane lie on some line defined by two irrational points?
(Grade 11.)
Answer: a) no; b) yes.
Solution. a) Two different rational points A( x1 , y1 ) and B( x2 , y2 ) define a line given by
( x − x1 )(y2 − y1 ) = (y − y1 )( x2 − x1 ),
Points A and B are irrational, since each of their coordinates is a sum of a rational and
an irrational number. It is also easy to verify that point ( x, y) lies on line AB:
x − ( x + α1 ) − α1 − β1 y − (y + β1 )
= = −1 = = .
( x + α2 ) − ( x + α1 ) α2 − α1 β2 − β1 (y + β 2 ) − ( y + β 1 )
FR-14. Four equally sized spheres are placed in a larger sphere such that each of the
small spheres touches the remaining three small spheres and the large sphere. Is the
total volume of the four small spheres equal to, larger or smaller than the remaining
volume of the large sphere? (Grade 12.)
12
Answer: smaller.
Solution 1. Assume the total volume of the small spheres is at least one half of the volume
of the large sphere. Let the radii of the small and big sphere be r and R, respectively.
Since the ratio of the volumes equals the ratio of the radii in cubic power, we have
R R
2 · 4 · r3 > R3 , or r > . The strict inequality r > is clearly impossible, as the centre
2 2
of the large sphere would have to lie in each of the smaller spheres simultaneously. If
R
r = , any two small spheres should touch at the centre of the large sphere (i.e., the 6
2
points at which the small spheres touch should all coincide), which is also impossible.
We conclude that the total volume of the small spheres is less than one half of the volume
of the large sphere.
Solution 2. As above, we show that the total volume of the small spheres is less than
half of the volume of the large sphere. Let the radius of the small spheres be 1, then
the radius of the large sphere can be written as 1 + a, where a is the distance from the
centre to the vertex in a regular tetrahedron of side length 2. Consider a triangle with
vertices in the centre of this tetrahedron, and at two of its vertices (i.e., at the centres
of two small spheres). The triangle has side lengths 2, a and a. The triangle inequality
gives 2a = a + a > 2, so a > 1.
4
The volume of half the large sphere is V = · π · (1 + a)3 , while the total volume of the
3
4
small spheres is v = 4 · · π · 1. As a > 1, (1 + a)3 > 8, and V > 2v.
3 √
6
Remark. The exact value of a in Solution 2 is .
2
n n +1 2n n n n 3n +1 − 1
3 +3 +...+3 = 3 · (1 + 3 + . . . + 3 ) = 3 · .
2
3n +1 − 1
The factors 3n and are relatively prime: the former can have only 3 as its prime
2
factor while the latter is not divisible by 3. As the product is a perfect square, both
factors are perfect squares. The factor 3n gives now that n is even.
Suppose n is not divisible by 4. As n is even, n ≡ 2 (mod 4) whence n + 1 ≡ 3 (mod 4).
Note that 34 = 81; from 16 | 80 we get 34 ≡ 1 (mod 16) whence 3n+1 ≡ 33 ≡ 11
3n +1 − 1
(mod 16) and 3n+1 − 1 ≡ 10 (mod 16), so ≡ 5 (mod 8). But a perfect square
2
is not congruent to 5 modulo 8.
Remark. From the assumption n ≡ 2 (mod 4), an analogous contradiction can also be
derived using module 5.
The solutions can also be written down in easier terms, omitting the application of the
geometric progression sum formula and just studying the behaviour of the sums of
powers of 3 modulo 8 (or modulo 5).
13
FR-16. Find all real numbers a such that polynomial x3 + ax − 2(a + 4) has exactly
two distinct real roots. (Grade 12.)
Answer: −12, −3.
Solution 1. Note that x3 + ax − 2(a + 4) = ( x − 2) · x2 + 2x + (a + 4) . Hence 2 is a
root of our polynomial irrespective of a. Consider two cases.
If x = 2 is a single root then the quadratic polynomial x2 + 2x + (a + 4) must have
exactly one real root, i.e., its discriminant equals zero. Thus 4 − 4(a + 4) = 0, giving
a = −3.
If x = 2 is a double root then x = 2 must be a root of x2 + 2x + (a + 4), hence 22 + 2 ·
2 + (a + 4) = 0, giving a = −12. As x2 + 2x − 8 6= ( x − 2)2 , the other root is different
from 2 indeed.
Hence the only possibilities are a = −3 and a = −12.
Solution 2. As a polynomial with real coefficients cannot have one imaginary root and
other roots real, the given cubic polynomial has exactly two distinct roots, i.e., one of
its roots must be a double root. This root is also a root of its derivative 3x2 + a. Letting
a = −3x2 in the original polynomial, we get the equation −2x3 + 6x2 − 8 = 0 for roots
common to the polynomial and the derivative. As −2x3 + 6x2 − 8 = −2( x − 2)2 ( x + 1),
the possible common roots are x = 2 and x = −1. From 3x2 + a = 0, we now establish
the corresponding possibilities a = −12 and a = −3.
FR-17. Prove that the ratio of the lengths of the two diagonals of a parallelogram
equals the ratio of its side lengths iff the angles at the intersection of the diagonals are
equal to the interior angles of the parallelogram. (Grade 12.)
| BC|
Solution 1. Since the diagonals of a parallelogram bisect each other, condition =
|CD |
|CA| | BC| |CP|
is equivalent to = (Fig. 13). The Sine Law in triangles BCD and CPD
| BD | |CD | | PD |
| BC| sin ∠ BDC |CP| sin ∠ PDC sin ∠ BDC | BC| |CP|
give = and = = . Thus, =
|CD | sin ∠CBD | PD | sin ∠ PCD sin ∠ PCD |CD | | PD |
iff sin ∠CBD = sin ∠ PCD. From triangle BCD, we observe ∠CBD + ∠ PCD < 180◦ ,
hence sin ∠CBD = sin ∠ PCD iff ∠CBD = ∠ PCD. Since triangles CPD and BCD share
a common angle ∠ PDC = ∠ BDC, the latter condition translates to ∠CPD = ∠ BCD.
| BC| |CA|
To conclude, we have that condition = is equivalent to ∠CPD = ∠ BCD.
|CD | | BD |
| DC| |CA|
Analogously, = is equivalent to ∠CPB = ∠ BCD.
|CB| | BD |
D C
Solution 2. Let the diagonals AC and BD of parallelo-
P gram ABCD intersect at P and let a = | AB| = |CD |,
b = | BC| = | AD | and d = | BD |, e = | AC|. We show
a d
A B that = iff ∠ BCD = ∠CPD (analogously, we can
b e
a e
Figure 13 then show that = iff ∠ BCD = ∠CPB).
b d
14
b2 e2
In parallelogram ABCD, we have 2(a2 + b2 ) = d2 + e2 , or 2a2 1 + 2 = d2 1 + 2 (by
a d
a d
the Cosine Law in triangles ABC and BCD). Thus, condition = is equivalent to
b e
a d
condition 2a2 = d2 . The two equations = and 2a2 = d2 are in turn equivalent to
b e
2b 2a d
= = , i.e., triangle BCD with side lengths b, a and d is similar to triangle CPD
e d a
e d
with side lengths , and a, or ∠ BCD = ∠CPD.
2 2
Remark. To construct a parallelogram where the ratio of diagonal lengths equals the
pone side a and an adjacent angle α. Then the other side of the
ratio of side lengths, fix
parallelogram is b = a 1 + cos2 α − a cos α.
FR-18. Some unit squares are removed from a rectangular grid in such a way that
whenever a unit square is removed, all unit squares in the rectangular area obtained by
extending the left side of the unit square to the top of the grid and the bottom side of the
square to the right side of the grid are also removed. Finally, a count is written in each of
the remaining unit squares indicating the total number of remaining squares above that
square in the same column and to the right of that square in the same row. Prove that
there are at least as many even counts as odd counts written in the remaining squares.
(Grade 12.)
Solution. Consider a configuration of the rectangular grid after removing some squares.
We prove that if we can legally remove exactly two more adjacent squares, then after
updating the counts, the number of even and odd counts both decrease by 1.
Indeed, if we can remove two squares in the same row, the columns of these squares
must be empty above that row, and full below that row. In this row, each count decreases
by 2, so the parities do not change. In the two columns below those two squares, each
row contains one odd and an one even number (the numbers of squares remaining
above are equal and the numbers of squares remaining to the right differ by one). After
removing the two squares, both counts change by one, so the total number of even and
odd counts remains the same. Finally, the counts in the two removed squares are 0 and
1, so removing those squares decreases both the number of even and odd counts by 1.
Analogously, removing two squares in the same column decreases the number of even
and odd counts by 1.
Now, starting from the given configuration, keep removing pairs of adjacent unit
squares until it is no longer possible. In the end, we must have one of two configu-
rations:
In both configurations, there are at least as many even counts as odd counts. Compared
to the original configuration, we decreased the even and odd counts by an equal num-
15
ber, so also in the original configuration, there had to be at least as many even counts as
odd counts.
TS-1. For arbitrary pairwise distinct positive real numbers a, b, c, prove the inequality
( a2 − b 2 )3 + ( b 2 − c 2 )3 + ( c 2 − a2 )3
> 8abc.
( a − b )3 + ( b − c )3 + ( c − a )3
( a2 − b 2 )3 + ( b 2 − c 2 )3 + ( c 2 − a2 )3
Thus = (a + b)(b + c)(c + a). Hence the given in-
( a − b )3 + ( b − c )3 + ( c − a )3
equality is equivalent to √ (a + b)(b + c)(c + a) > 8abc. This inequality holds due to
AM-GM (use a + b > 2 ab and note that a 6= b; similarly for other pairs).
TS-2. Call a finite set of positive integers independent if its elements are pairwise co-
prime, and nice if the arithmetic mean of the elements of every non-empty subset of it is
an integer.
a) Prove that for any positive integer n there is an n-element set of positive integers
which is both independent and nice.
b) Is there an infinite set of positive integers whose every independent subset is nice
and which has an n-element independent subset for every positive integer n?
Answer. b) No.
Solution 1. a) Let A = {n! + 1, 2 · n! + 1, . . . , n · n! + 1}. For any two elements k · n! + 1
and l · n! + 1 where 1 6 k < l 6 n,
16
From arbitrary m elements k1 · n! + 1, k2 · n! + 1, . . . , km · n! + 1 of A where 1 6 m 6 n,
we get
TS-3. Find all natural numbers n for which there exists a convex polyhedron satisfying
the following conditions:
(i) Each face is a regular polygon.
(ii) Among the faces, there are polygons with at most two different numbers of edges.
17
(iii) There are two faces with common edge that are both n-gons.
Answer: 3, 4, 5, 6, 8, 10.
Solution. For n = 3, 4, 5, the well-known regular polyhedrons satisfy the conditions.
If we remove a regular pyramid around each vertex of a regular tetrahedron, cube,
or dodekahedron, the faces of these polyhedrons become 6-gons, 8-gons and 10-gons,
respectively. Clearly one can make the cuts so that all these faces are regular polygons.
At place of vertices that are cut off, equilateral triangles appear. Hence also 6, 8, 10
satisfy the conditions of the problem.
M
C
Show now that there are no other answers. Suppose we
have a polyhedron satisfying the conditions for some n.
Consider two neighbouring n-gons; let AB be the edge sep-
B
arating them (Fig. 14). Suppose A belongs to exactly k other
N ( n − 2) π π
faces. Then 2π > 2 · +k· or, equivalently,
n 3
12
A n < . Thus k > 2 implies n < 6 and k = 1 implies
k
n < 12.
All we have to show is that n = 7, 9, 11 are impossible; so
Figure 14
assume n is odd and 6 < n < 12. Then vertex A, as well as
vertex B, belongs to exactly one face other than the two n-gons. Let m be the number of
vertices of these faces.
Let N be one of the two n-gons under consideration and let M be the face meeting
them at B. Let C be the other endpoint of the edge separating M and N . Let γ be
the sum of all interior angles of the faces other than M and N that meet at C. Then
( n − 2) π
clearly γ > (if only one face meets M and N at C then it must be an n-gon,
n
and if there are more faces then the sum of their angles at C is greater). We obtain the
inequalities
20 π 5π ( m − 2) π ( n − 2) π ( n − 2) π 5π
π> π = 2π − − > 2π − − >γ> > .
21 3 7 m n n 7
From π > γ we see that there are at most 2 other faces meeting M and N at C. If
there were 2 of them, they both would have no more than 5 vertices; therefore they
would not be n-gons and thus they both would be polygons of the second kind. But for
this situation, π > γ gives that they both must be triangles which is contradictory to
5π
γ> . Consequently, there is only one face meeting M and N at C and this must be
7
an n-gon.
Continuing the same way, we find that the faces surrounding N are alternately n-gons
and m-gons. As n is odd by assumption, we get n = m. Thus three n-gons meet at A,
implying n < 6. This completes the solution.
Remark. The condition (iii) of the problem is necessary. Without it, a required poly-
hedron would exist for arbitrary n: a right prism whose bases are regular n-gons and
lateral faces are squares would do.
18
Second day
TS-4. Points A′ , B′ , C′ are chosen on the sides BC, CA, AB of triangle ABC, respec-
| BA′ | |CB′ | | AC′ |
tively, so that = = . The line which is parallel to line B′ C′ and goes
| A′ C| | B′ A| |C′ B|
through point A′ intersects the lines AC and AB at P and Q, respectively. Prove that
| PQ|
> 2.
| B′ C′ |
| BA′ | |CB′ | | AC′ |
Solution 1. Denote = = = k.
| A′ C| | B′ A| |C′ B|
If k = 1 then B′ C′ is the segment connecting midpoints of sides AB and AC of the tri-
angle ABC, so it is parallel to the side BC. Thus the lines PQ and BC coincide, meaning
| PQ| | BC|
P = C and Q = B. Thus ′ ′ = ′ ′ = 2.
|B C | |B C |
| AB′′ |
Now assume that k 6= 1 (Fig. 15). Let B′′ be the point on the side AC such that ′′ =k
| B C|
(in other words, B′′ is symmetric to B′ w.r.t. the perpendicular bisector of AC).
By the intercept theorem, B′′ C′ k BC and A′ B′′ k AB. Thus BC′ B′′ A′ is a parallelogram;
also △ PA′ C ∼ △ B′ C′ B′′ since the corresponding sides are collinear (the triangles exist
| A′ P| | A′ C| | A′ C| 1
since k 6= 1). Hence ′ ′ = ′ ′′ = ′
= . Let C′′ be a point on the side AB
|B C | |C B | | BA | k
| BC′′ | | A′ Q| | A′ B|
such that ′′ = k. By analogy, we have that ′ ′ = = k. It follows that
|C A| |B C | |CA′ |
| PQ| | A′ P| + | A′ Q| 1
′ ′
= ′ ′
= + k > 2.
|B C | |B C | k
C C
P P
A′ A′
B ′′
B′
A′′ B′
Q Q
A C ′′ C′ B A C′ B
Figure 15 Figure 16
Solution 2. From the intercept theorem,
Let d(X, l ) be the distance of the point X from the line l and S∆ the area of the triangle
∆. Considering lines perpendicular to B′ C′ through points P, resp. A (Fig. 16), we have
| PB′ | d( P, B′ C′ ) d( A′ , B′ C′ ) S△ A ′ B ′ C ′
′
= ′ ′
= ′ ′
= .
| AB | d( A, B C ) d( A, B C ) S△ AB′ C ′
19
Thus
| PQ| | PB′ | S△ A ′ B ′ C ′
′ ′
> 2 ⇐⇒ ′
> 1 ⇐⇒ > 1 ⇐⇒ S△ A′ B′ C ′ > S△ AB′ C ′ .
|B C | | AB | S△ AB′ C ′
x + (1 − x ) 2
1 1
Since from AM-GM, x (1 − x ) 6 = , we have S△ AB′ C ′ 6 · S△ ABC .
2 4 4
1 1
Analogously S△ A′ BC ′ 6 · S△ ABC and S△ A′ B′ C 6 · S△ ABC . To summarize,
4 4
S△ A′ B′ C ′ = S△ ABC − (S△ AB′ C ′ + S△ A′ BC ′ + S△ A′ B′ C )
3 1
> S△ ABC − · S△ ABC = · S△ ABC > S△ AB′ C ′ .
4 4
| BA′ | |CB′ | | AC′ | | BA′ | |CB′ | | AC′ |
Solution 3. As = ′ = , we also have = = , let this
| A′ C| | B A| |C′ B| | BC| |CA| | AB|
ratio be x. Then
−−→′ −→ −→′ −→
AC = x · AB, AB = (1 − x ) · AC,
−−→′ −→ −→ −→ −→
AA = AB + BA′ = AB + x · BC =
−→ −→ −→ −→ −→
= AB + x · ( AC − AB) = (1 − x ) · AB + x · AC.
| PQ| | AP|
Let ′ ′
= v; we need to show that v > 2. Since PQ k B′ C′ , we obtain =
|B C | | AB′ |
| AQ| −→ −→ −→ −→ −−→ −→
′
= v. Hence AP = v · AB′ = v · (1 − x ) · AC and AQ = v · AC′ = v · x · AB. As P,
| AC |
−→ −→ −−→ −→ −→ −→
A′ , Q are collinear, we have PA′ = z · PQ or AA′ − AP = z · ( AQ − AP ). Substituting
−−→ −→ −→
the formulae for AA′ , AP, AQ, we have
−→ −→ −→ −→ −→
(1 − x ) · AB + x · AC − v · (1 − x ) · AC = z · v · x · AB − z · v · (1 − x ) · AC.
−→ −→
Since AB and AC are not collinear, the identity can only hold if their coefficients on both
sides are the same. We have the system of equations
1 − x = zvx,
x − v(1 − x ) = −zv(1 − x ).
20
1−x
From the first equation zv = , and substituting this into the second we get x −
x
(1 − x )2
1−x +x (1 − x )2 + x 2
v (1 − x ) = − · (1 − x ). Thus v = x = . Since (1 − x )2 +
x 1−x (1 − x ) x
x > 2(1 − x ) x, the last equation gives v > 2, just as required.
2
| PQ| | AP|
Solution 4. Let ′ ′ = v; we have to show v > 2. As PQ k B′ C′ , we have =
|B C | | AB′ |
| AQ|
= v. W.l.o.g., assume | AP| > | AC| and | AQ| 6 | AB|. Then
| AC′ |
and, analogously,
TS-5. A strip consists of n squares which are numerated in their order by integers
1, 2, 3, . . . , n. In the beginning, one square is empty while each remaining square con-
tains one piece. Whenever a square contains a piece and its some neighbouring square
contains another piece while the square immediately following the neighbouring square
is empty, one may raise the first piece over the second one to the empty square, remov-
ing the second piece from the strip.
Find all possibilites which square can be initially empty, if it is possible to reach a state
where the strip contains only one piece and a) n = 2008; b) n = 2009.
Answer: a) 2, 5, 2004, 2007; b) none.
Solution. Interpret the move so that the piece at distance 2 from the empty square is
removed from the strip and the piece from the intermediate square moves to the empty
square. In the following, assume n > 5.
First show that, under this interpretation, the location of any piece, as long as it is on the
board, does not change more than by one square. For that, we show that after a move in
some direction, no piece can make next move in the same direction. Perform induction
on the distance between the piece and the end of the strip from which the piece went
farther on the move – assume that the claim holds for smaller distances. Clearly there
are two possibilities for the state after the move: (1) there are no pieces between the
piece that moved and the end of the strip under consideration; (2) there are other pieces
in that area but the nearest among them is separated from the piece that moved by at
21
least two empty squares. But for moving in the same direction, there must be a piece on
the square where the piece came from. By the induction hypothesis, the pieces between
the piece that moved and the end of the strip under consideration do not get farther by
more than one square. Hence the piece that moved cannot move further in the same
direction.
Note that whenever we divide the squares into two non-empty groups then, in order
to achieve the desired final state, one must first obtain a state where some pieces from
different groups are on neighbouring squares – otherwise one could not do a move that
takes off the last piece of one group. Thus if there are three consecutive empty squares,
in both sides of which there are pieces, then the required final state is impossible to
achieve because the pieces from the two groups do not move nearer to each other by
more than one square and there always remains an empty square between them.
Analogously, if there is a lonely piece separated by at least two empty squares from the
nearest piece then the required final state is impossible to achieve since the other pieces
do not reach its neighbouring square and it cannot move itself.
Independently on the location of the empty square, the state after the first move of the
game has two consecutive empty squares and other squares containing a piece. Call
these two squares central (Fig. 17). We show that, from this state, it is possible to reach
the desired final state if and only if there are 2 pieces at one side and an even number of
pieces at the other side of the central squares.
Suppose that this condition is fulfilled; w.l.o.g. there are 2 pieces on the left and 2k pieces
on the right. Show by induction on k that the desired final state can be achieved. If k = 0
then there are only 2 pieces on the board and they are on consecutive squares, so one
move reaches the goal. If k > 0 then first move with the piece on square 2 to the right
and with the piece on square 5 to the left. In the resulting state, pieces are on squares
3 and 4, the following two squares are empty and 2(k − 1) consecutive squares contain
pieces. By the induction hypothesis (it holds for the substrip starting from square 3) it
is possible to achieve the desired final state from this state.
Suppose now there being more than 2 pieces on both sides of the central squares. In
order to achieve the desired final state, the pieces from both sides must get together. It
is possible only if the rightmost piece of the left-hand side moves to the right and the
leftmost piece of the right-hand side moves to the left. The pieces immediately following
these pieces are removed and the remaining pieces do not move nearer by more than
one square. Thus an empty square remains on both sides of the central squares where
no remaining pieces can ever come. Hence when one of the pieces on the central squares
makes a move, the other one is removed and three consecutive squares become empty,
whereby there are still pieces on both sides from them. Thus the desired final state
cannot be achieved.
initial state
central squares
Figure 17
22
Also if there is 1 piece on one side from the central squares then the desired final state
is unreachable. If there is 0 pieces on one side then there will be one piece after one
move, consequently the final state is unreachable again. It remains to study the case
with 2 pieces on one side – w.l.o.g., on the left – and an odd number 2k + 1 of pieces
on the other side. Show by induction on k that the required final state is unreachable
even if the strip had more empty squares to the left from square 1. If k = 0 then both
possible moves (from 2 to the right and from 1 to the left) leads to a stub configuration
with 2 lonely pieces. Let k > 0. The move from 1 to the left gives a state where three
consecutive squares are empty, thus uninteresting. The moves from 2 to the right and
from 5 to the left remain, which both are certainly performed during the play. Suppose
w.l.o.g. that the very first move is from 2 to the right because the possible moves before
it concern the pieces on the right only and they can be performed also if the move from 2
to the right is already done. Hence in the state after two moves, squares 3 and 4 contain
pieces, the next two squares are empty and then 2(k − 1) + 1 consecutive squares with
pieces follow. By the induction hypothesis, the desired state is unreachable.
We have got that the desired state is possible only for even n since only then there can
be 2 pieces on one side and an even number of pieces on the other side from two central
squares. In the case of even n, the state where squares 3 and 4 are empty can arise
if, initially, 2 or 5 is empty. Analogously, the symmetric case is handled. Hence, for
n = 2008, the squares 2, 5, 2004, 2007 can be initially empty.
TS-6. For any positive integer n, let c(n) be the largest divisor of n not greater than
√
n and let s(n) be the least integer x such that n < x and the product nx
is divisibleby
n
an integer y where n < y < x. Prove that, for every n, s(n) = (c(n) + 1) · +1 .
c (n)
n
Solution. Take y = c(n) · + 1 . Then
c (n)
n n
n = c (n) · < y < ( c ( n ) + 1) · +1
c (n) c (n)
while
n n n
y | c (n) · · ( c ( n ) + 1) · + 1 = n · ( c ( n ) + 1) · +1 .
c (n) c (n) c (n)
n
This implies s(n) 6 (c(n) + 1) +1 .
c (n)
n
For establishing minimality, choose x such that n < x < (c(n) + 1)( + 1). It has to
c (n)
be proven that no integer strictly between n and x divides nx.
y
Let y be an arbitrary integer strictly between n and x. Let d = gcd(y, n) and y′ = ,
d
n
n′ = .
d
23
n
As c(n) and are the central divisors of n, we have
c (n)
n n
c (n) + 6 d+ . (4)
c (n) d
1
(Indeed, consider function f (z) = z + ; it is decreasing between 0 and 1. Then c(n) +
z
n √ c (n) n √ min(d, n ) min(d, nd ) c (n)
= n · f ( √ ) and d + = n · f ( √ d ) as well as √ 6 √ 6 1.)
c (n) n d n n n
Adding n + 1 to both sides of (4) and factorizing, we get
n n
( c ( n ) + 1) · + 1 6 ( d + 1) · +1 . (5)
c (n) d
Hence x is strictly between two consecutive multiples dy′ and (d + 1)y′ of y′ , therefore
y′ ∤ x.
If y | nx were the case then reducing by d would lead to y′ | n′ x. As y′ is relatively prime
to n′ , this would imply y′ | x. This contradiction completes the solution.
Remark. The table for small values of functions c and s is
n 1 2 3 4 5 6 7 8 9 10 11 12
c (n) 1 1 1 2 1 2 1 2 3 2 1 3
s (n) 4 6 8 9 12 12 16 15 16 18 24 20
n 13 14 15 16 17 18 19 20 21 22 23 24
c (n) 1 2 3 4 1 3 1 4 3 2 1 4
s (n) 28 24 24 25 36 28 40 30 32 36 48 35
24
Estonian Math Competitions
2009/2010
University of Tartu
Problem authors: Juhan Aru, Maksim Ivanov, Urve Kangro, Oleg Koshik,
Mihkel Kree, Toomas Krips, Härmel Nestra, Hendrik Nigul, Uve Nummert,
Laur Tooming, Jan Willemson, Indrek Zolk
Translators: Urve Kangro, Härmel Nestra
Editors: Uve Nummert, Reimo Palm
*
This booklet contains problems that occurred in the open contests, the fi-
nal round of national olympiad and the team selection contest. For the open
contests and the final round, selection has been made to include only prob-
lems that have not been taken from other competitions or problem sources
and seem interesting enough. The team selection contest is presented entirely.
3
Selected Problems from Open Contests
OC-1. Find all positive integers n such that 1 + 22 + 33 + 4n is a perfect
square. (Juniors.)
Answer: n = 1.
Solution 1. Let 1 + 22 + 33 + 4n = x2 . This implies 32 = x2 − 4n or, equiv-
alently, 25 = ( x − 2n )( x + 2n ). As the l.h.s. is a power of 2, the factors in the
r.h.s. are of the form x − 2n = 2a and x + 2n = 25− a where a is 0, 1 or 2. Sub-
tracting the first of the two equalities from the second gives 2n+1 = 25− a − 2a .
This leads to an integral n only if a = 2; then n = 1. A check shows that
1 + 22 + 33 + 41 = 62 indeed.
Solution 2. Observe that 1 + 22 + 33 + 4n = 32 + 4n = 25 + 22n = 25 ·
(1 + 22n−5 ). If n > 3, then 2n − 5 > 1; hence 22n−5 is an even integer and
1 + 22n−5 is therefore odd. Thus in the prime factorization of the number
given in the problem, the exponent of 2 is 5. As this is odd, the number cannot
be a perfect square. If n = 2 or n = 1, then 32 + 4n = 48 or 32 + 4n = 36,
respectively, where only the latter is a perfect square. Consequently, only
n = 1 is possible.
Solution 3. If n = 1, 2, 3, then the given number is 36, 48, 96, respectively,
where only the first is a perfect square. If n > 4, then 2 · 2n + 1 > 2 · 16 + 1 >
32, implying (2n )2 < (2n )2 + 32 < (2n )2 + 2 · 2n + 1 = (2n + 1)2 . As the
number under question is equal to (2n )2 + 32, it falls between two consecutive
perfect squares, hence cannot be a perfect square itself.
4
Solution 2. We use the same notation as in the Solution 1. Triangle BCD
is isosceles, hence ∠ DCB = ∠ DBC = β. As D is the circumcenter of ABC,
we have ∠ BDC = 2∠ BAC = 2β. The sizes of the angles of triangle BCD are
therefore β, β, and 2β; thus β + β + 2β = 180◦ , whence β = 45◦ . As ∠ BCA =
∠BDA , we have ∠ BCD = α + α = β, whence α = 2 β = 30◦ . Consequently,
2 2 3
◦
the sizes of the angles of quadrangle ABCD are ∠ DAB = ∠ ABD = 1802 −α =
75◦ , ∠ ABC = ∠ ABD + ∠CBD = 75◦ + 45◦ = 120◦ , ∠ BCD = β = 45◦ , and
∠CDA = ∠CDB + ∠BDA = 90◦ + 30◦ = 120◦ .
Remark. The convexity of the quadrangle actually follows from the other
constraints of the problem. Namely, consider the circle with center D, passing
through points A and B, and a point C on it. If A and B were on different sides
from line CD, we would have ∠ ADB > ∠ BCA > ∠ DCA, hence ∠ ADB and
∠DCA could not be equal.
OC-3. In the buffet of the kitchen, there are three candy boxes, each con-
taining the same number of candies. Every time when Juku goes into the
kitchen, he takes either three candies from one box or one candy from ev-
ery box. Prove that irrespectively of how Juku takes the candies, he always
retains the possibility to completely clean out all candy boxes. (Juniors.)
Solution. The difference of the numbers of candies in any two boxes can
only be a multiple of 3 because it is 0 in the beginning and, with every move,
it changes by either 0 or 3. Hence, starting from an arbitrary intermediate
state, Juku can clean out the boxes as follows: he takes one candy from each
box as many times as possible, after which one box is empty and the number
of candies in each of the other two is divisible by 3, and then empties the
other boxes by taking three candies from one box every time.
5
Dividing both sides by x ( a − x )y(b − y), one obtains
1 1 1 1
= + + .
( a − x )(b − y) x (b − y) ( a − x )y xy
If the price that every musketeer paid for the plot was 1, then the l.h.s. of the
last equality is precisely the price per area unit of the piece with area ( a − x ) ·
(b − y). Analogously, the r.h.s. equals the sum of the prices per area unit of
the other three pieces. Hence multiplying the sides of this equality by the
number of area units per acre, the claim of the problem follows.
OC-5. Let a be a fixed real number. Find all real numbers b such that, for
every real number x, at least one of the numbers x2 + ax + b and x2 − ax + b
is non-negative. (Juniors.)
Answer: b > 0.
Solution. Note that x2 + ax + b and x2 − ax + b sum up to 2x2 + 2b. If
b > 0, then it is non-negative for arbitrary real number x, implying that at
least one of the numbers added was non-negative. If b < 0, then taking x = 0
turns both summands negative.
Remark. This problem can be solved also in technical ways, by calculating
the negative and non-negative domains of the quadratic polynomials.
OC-6. Call a positive integer n prime-prone if there exist at least three prime
numbers from which we can get n by removing the last digit. Prove that
every two prime-prone positive integers differ from each other by at least 3.
(Juniors.)
Solution. As the prime numbers under consideration have at least two
digits, the last digit can be only 1, 3, 7, or 9. Thus n is prime-prone if and only
if, among numbers 10n + 1, 10n + 3, 10n + 7, and 10n + 9, at least three are
primes.
If n = 3k, then 10n + 3 = 30k + 3 and 10n + 9 = 30k + 9 are divisi-
ble by 3 and hence composite. If n = 3k + 2, then 10n + 1 = 30k + 21 and
10n + 7 = 30k + 27 are divisible by 3 and hence composite again. Conse-
quently, all prime-prone integers are congruent to 1, and hence to each other,
modulo 3. Thus they differ by a multiple of 3, i.e., by at least 3.
OC-7. Does there exist a prime number p such that both p3 + 2008 and p3 +
2010 are primes as well? (Seniors.)
Answer: no.
Solution. Let p be any prime number. If p is not divisible by 7, then p3
is congruent to either 1 or −1 modulo 7. Since 2008 ≡ −1 (mod 7) and
2010 ≡ 1 (mod 7), either of the numbers p3 + 2008 and p3 + 2010 is divisible
by 7 and hence composite. If p is divisible by 7, then p = 7 and p3 + 2010 =
73 + 2010 = 2353 = 13 · 181 is composite, too.
6
OC-8. In a regular n-gon, either 0 or 1 is written at each vertex. Using
non-intersecting diagonals, Juku divides this polygon into triangles. Then
he writes into each triangle the sum of the numbers at its vertices. Prove that
Juku can choose the diagonals in such a way that the maximal and minimal
number written into the triangles differ by at most 1. (Seniors.)
Solution. If all numbers written at the vertices of the polygon are equal,
then the claim holds trivially. Hence assume that there are both zeros and
ones among the numbers at the vertices. We prove by induction that, for
every convex polygon, the partition into triangles can be chosen in such a
way that Juku writes either 1 or 2 to each triangle.
If n = 3, then this claim holds since the sum of the numbers at the vertices
of a triangle can be neither 0 nor 3. If n = 4 (Fig. 2), then draw the diagonal
that connects the vertices where 0 and 1 are written, respectively, or, if such
a diagonal does not exist, then an arbitrary diagonal. In both cases, only
sums 1 and 2 can arise. If n > 5, then choose two consecutive vertices with
different labels and a third vertex P that is not neighbour to either of them
(Fig. 3). Irrespective of whether the label of P is 0 or 1, we can draw the
diagonal from it to one of the two consecutive vertices chosen before so that
the labels of its endpoints are different. Now the polygon is divided into two
convex polygons with smaller number of vertices so that both 0 and 1 occur
among their vertex labels. By the induction hypothesis, both polygons can be
partitioned into triangles with sum of labels of vertices either 1 or 2.
P
⋆ 1 1 0
0 ⋆ 0 1 0 1
Fig. 2 Fig. 3
7
C
C
C
O E
H A I B
A A B
B
c c c
Fig. 4 Fig. 5 Fig. 6
8
in such a way that it evaluates to 0. Adding ak − ak+1 to it, the desired
expression for n = k + 1 is obtained.
2. If ak 6= ak+1 , then consider integers a1 , . . . , ak−1 , | ak − ak+1 |. As | ak −
ak+1 | and ak + ak+1 have the same parity, the sum of these k numbers is
even. Also note that 1 6 | ak − ak+1 | 6 k. Thus these numbers satisfy the
conditions of the problem, so it is possible to choose signs in the expres-
sion a1 ± a2 ± . . . ± ak−1 ± | ak − ak+1 | in such a way that it evaluates to
0. As either | ak − ak+1 | = ak − ak+1 or | ak − ak+1 | = ak+1 − ak , this also
leads to a corresponding expression for numbers a1 , a2 , . . . , ak , ak+1 .
Solution 2. Prove by induction on i that, for each i and s such that 1 ≤ i ≤ n
and 1 ≤ s ≤ a1 + . . . + ai , it is possible to choose some of the numbers a1 , . . . ,
ai that sum up to s. If i = 1, then this claim holds since a1 = 1. Assume that
the claim holds for i = k − 1 and consider the case i = k. Let S = a1 + . . . + ak
and S′ = a1 + . . . + ak−1 . If 1 ≤ s ≤ S′ , then the desired statement holds by the
induction hypothesis. If S′ < s ≤ S, then 0 ≤ s − ak ≤ S′ (the first inequality
holds because s − ak ≥ s − S′ − 1 > 0, implied by ak ≤ k and S′ ≥ k − 1;
the second inequality follows from S = S′ + ak ). Therefore, to get the sum s,
we can choose the number ak , and if s − ak > 0, then add to it those numbers
among a1 , . . . , ak−1 whose sum is s − ak , using the induction hypothesis.
Let now a1 + a2 + . . . + an = 2T. Choose the numbers among a1 , a2 , . . . ,
an that sum up to T. This divides all the numbers into two groups with equal
sum. It remains to write minuses in front of every term of the group that does
not contain a1 .
Solution 3. Start choosing signs from right to left. Denote S1 = an and
define Sk+1 , k = 1, . . . , n − 1, as follows: if Sk > 0, then Sk+1 = Sk − an−k ,
otherwise Sk+1 = Sk + an−k . We show that then always |Sk | 6 n − k + 1. This
holds if k = 1. Assume therefore that it holds for k = m and prove it for
k = m + 1. If Sm > 0, then Sm+1 = Sm − an−m 6 (n − m + 1) − 1 = n − m
and Sm+1 = Sm − an−m > 0 − (n − m), hence |Sm+1 | 6 n − m. If Sm < 0, then
Sm+1 = Sm + an−m < 0 + n − m and Sm+1 = Sm + an−m > −(n − m + 1) +
1 = −(n − m), hence |Sm+1 | 6 n − m again.
Now |Sn | 6 1 since |Sk | 6 n − k + 1 for every k = 1, . . . , n. Thus Sn = 0
as the sum of all terms is even. If in this formal sum, the term a1 has minus
sign, turn all signs to the opposite one.
Remark. Solution 2 shows that the assumption ak 6 k for all k = 1, . . . , n
could be replaced with the more general assumption ak 6 1 + a1 + . . . + ak−1
for all k = 1, . . . , n.
One can also note that the lemma proved at the beginning of Solution 2
does not need the assumption that the sum of all numbers is even.
9
OC-11. The diagonals of trapezoid ABCD with bases AB and CD meet at
P. Prove the inequality SPAB + SPCD > SPBC + SPDA , where SXYZ denotes
the area of triangle XYZ. (Seniors.)
Solution 1. Let a = | AB| and b = |CD | and let h a and hb be the altitudes of
triangles PAB and PCD drawn from P (Fig. 7). Denote S1 = SPAB + SPCD and
S2 = SPBC + SPDA . Then S1 = 21 ( ah a + bhb ) and S1 + S2 = 12 ( a + b)(h a + hb ),
whence S2 = 12 ( ahb + bh a ). Since triangles PAB and PCD are similar, a > b
implies h a > hb and also a < b implies h a < hb (a 6= b because a and b are the
lengths of the bases of the trapezoid). Hence
1 1
S1 − S2 = ( ah a + bhb − ahb − bh a ) = ( a − b)(h a − hb ) > 0,
2 2
i.e., S1 > S2 .
Solution 2. Let M and N be the intersection A a B
points of the arms BC and DA of the trape- ha
zoid with the line being parallel to the bases
of the trapezoid and passing through point P. N M
Let l be the length of MN, let d be the length P
hb
of the midline of the trapezoid, and let h and
S be the height and the area of the trapezoid,
respectively. Let S′ = SPBC + SPDA . Then D b C
′ 1
S = dh and S = 2 lh whence it suffices to Fig. 7
show that l < d.
W.l.o.g., assume | AB| < |CD |. Comparing the heights of similar triangles
PAB and PCD shows that MN is closer to base AB than to base CD. Thus
MN is situated between the midline and the shorter base AB. Consequently,
MN is shorter than the midline.
Solution 3. Let a = | AB| and b = |CD |. Let h and S be the height and
the area of the trapezoid, respectively, and let h a and hb be the heights corre-
sponding to vertex P of triangles PAB and PCD, respectively. Similar trian-
gles PAB and PCD imply h a : hb = a : b. As h a + hb = h we get
a b
ha = · h, hb = · h.
a+b a+b
Now
1 1 a 2 + b2
SPAB + SPCD = ( ah a + bhb ) = · · h.
2 2 a+b
It suffices to show that SPAB + SPCD > S2 or, equivalently,
1 a 2 + b2 1 a+b
· ·h > · · h,
2 a+b 2 2
or, equivalently, 2( a2 + b2 ) > ( a + b)2 . But the last inequality is equivalent to
( a − b)2 > 0 (a 6= b since a and b are the lengths of the bases of the trapezoid).
10
OC-12. Call pure any positive integer n that does not occur in any integer
sequence c0 , c1 , c2 , . . . , where 0 < c0 < n and
( 1
2 c i −1 if ci−1 is even,
ci =
3ci−1 − 1 if ci−1 is odd,
for every i > 1. (For instance, 10 is not pure since it occurs in the sequence 5,
14, 7, 20, 10, . . . )
a) Is every positive multiple of 3 pure?
b) Prove that if an integer n > 1 is pure but not divisible by 3, then n + 1
is divisible by 6.
(Seniors.)
Answer: a) yes.
Solution. a) Note that 3ci−1 − 1 is never divisible by 3 and if 12 ci−1 is di-
visible by 3, then also ci−1 is divisible by 3. Thus, if some term ck = n is
divisible by 3, then, up to it, only dividing by 2 is used to build the terms
(i.e., ci = 21 ci−1 for every i such that 1 6 i 6 k) and, consequently, c0 > c1 >
. . . > ck = n. But this contradicts the condition c0 < n. Hence every positive
multiple of 3 is pure.
b) If n is not divisible by 3, then n = 3k + 1 or n = 6k + 2 or n = 6k + 5.
If n = 3k + 1, then taking c0 = 2k + 1 gives c1 = 6k + 2 and c2 = 3k + 1 = n.
Thereby k > 0 since n > 1, therefore c0 < n. Hence none of such numbers n
is pure. If n = 6k + 2, then taking c0 = 2k + 1 gives c1 = 6k + 2 = n, whereby
c0 < n. Hence also none of such numbers n is pure. Hence, among the
positive integers n > 1 not divisible by 3, only those of the form n = 6k + 5
can be pure.
Remark. Not every integer of the form n = 6k + 5 is pure. For example,
23 = 6 · 3 + 5 occurs in the sequence 21, 62, 31, 92, 46, 23, . . .
OC-13. Let a and b the lengths of the legs of a given right triangle. Prove
that angle ϕ, where 0 < ϕ < 90◦ , is an acute angle of this triangle if and only
if ( a cos ϕ + b sin ϕ)( a sin ϕ + b cos ϕ) = 2ab. (Seniors.)
Solution 1. The equality given in the problem is equivalent to
( a2 + b2 ) sin ϕ cos ϕ + ab(sin2 ϕ + cos2 ϕ) = 2ab
and hence also to
( a2 + b2 ) sin ϕ cos ϕ = ab. (1)
Let α and β be the
√ angles opposite to legs with
√ length a and b, respectively.
Then sin α = a/ a2 + b2 , sin β = cos α = b/ a2 + b2 , implying
( a2 + b2 ) sin α cos α = ab.
Comparing this to (1) shows the equivalence of the equality of the problem
and the equality sin ϕ cos ϕ = sin α cos α, i.e., equality sin 2ϕ = sin 2α. As
11
0 < α, β < 90◦ , this implies 2ϕ = 2α or 2ϕ = 180◦ − 2α, whence ϕ = α or
ϕ = 90◦ − α = β. Hence, ϕ satisfies the equality if and only if it equals one of
the acute angles of the right triangle.
Solution 2. Let ABC be the given trian- C
gle with right angle at vertex C. Let c be the
length of its hypothenuse and h be the height
corresponding to the hypothenuse. Let C ′ be
b h a
a point on the circumcircle of ABC such that
′
one acute angle of triangle ABC is ϕ (Fig. 8). A c
ϕ B
Let a′ and b′ be the lengths of the legs of tri-
angle ABC ′ and h′ be the height of the trian- b′ a′
′ h′
gle ABC corresponding to its hypothenuse.
Then ( a2 + b2 ) sin ϕ cos ϕ = c2 sin ϕ cos ϕ =
(c sin ϕ)(c cos ϕ) = a′ b′ = ch′ . Since the equal- C′
ity in the problem is equivalent to the equal-
ity (1) from the Solution 1, it is also equiva- Fig. 8
lent to ch′ = ab. But ab = ch, hence it is also
equivalent to h = h′ . This condition holds if and only if ABC ∼ ABC ′ or
ABC ∼ BAC ′ , i.e., ϕ equals one of the acute angles of triangle ABC.
FR-3. Juku drew a regular hexagon and chose three triangles with different
areas whose vertices were among the vertices of the hexagon. Prove that the
sum of the areas of the triangles is equal to the area of the hexagon. (Grade 9.)
Solution. Any triangle whose vertices are among the vertices of a regular
hexagon is one of the following:
• a triangle ∆1 whose vertices are three consecutive vertices of the hexa-
gon;
• a triangle ∆2 whose two vertices are adjacent vertices of the hexagon
and the third one is adjacent to none of the first two;
• a triangle ∆3 where any two vertices are not adjacent vertices of the
hexagon.
Since the areas of the chosen triangles are different, the triangles must be
equal to the triangles ∆1 , ∆2 , ∆3 . The hexagon can be divided into four parts
(Fig. 9): the triangle ∆3 surrounded by three triangles ∆1 . The area of the tri-
angle ∆2 (marked by a dotted line in Fig. 9) is twice the area of the triangle ∆1
beacuse they have the same base but the height of ∆2 is twice the height of ∆1 .
FR-4. Points A′ , B′ and C ′ are chosen correspondingly on the sides AB, BC,
| A′ B| | B′ C| |C′ A |
and CA of an equilateral triangle ABC so that | AB| = | BC| = |CA| = k. Find
all positive real numbers k for which the area of the triangle A′ B′ C ′ is exactly
half of the area of the√triangle ABC. (Grade 10.)
Answer: k = 12 ± 63 .
Solution. Let α be the angle at the vertex A (Fig. 10). The area of the triangle
AA′ C ′ is S AA′ C′ = 12 · | AA′ | · | AC ′ | · sin α = 12 · (1 − k)| AB| · k| AC | · sin α =
(1 − k)k S ABC . Similarly SBB′ A′ = (1 − k)k S ABC and SCC′ B′ = (1 − k)k S ABC .
Hence the triangles AA′ C ′ , BB′ A′ and CC ′ B′ are of equal area. Therefore
A
α
C′
A′
B B′ C
Fig. 9 Fig. 10
13
the area of the triangle A′ B′ C ′ is half of the area of the triangle ABC iff the
area of the triangle AA′ C ′ is one sixth of the area of the triangle
√
ABC, i.e.
(1 − k)k = 16 . The solutions of k2 − k + 16 = 0 are k1,2 = 12 ± 63 , both of them
are positive.
Remark. As seen from the solution, the result actually holds for an arbi-
trary triangle.
FR-5. Three players A, B and C play the following game. At the beginning
of the game, each player has a sheet of paper with the name of the player
written on it. Player A chooses one of the other players and replaces the name
on this player’s sheet with the name on his own sheet. Then player B makes
a similar move, then player C and after that the turn to move goes to player
A again. The game ends when all the sheets have the same name written on
them and the winner is the player whose name it is. Does any of the players
have a winning strategy (i.e., a strategy that allows a player to win no matter
what his opponents play)? (Grade 10.)
Answer: no.
Solution 1. Player B does not have a winning strategy, since on the first
move player A can write the name A on his sheet, after that the name B is not
on any of the sheets. Similarly player C does not have a winning strategy.
To prove that even player A does not have a winning strategy, we show
that players B and C have a joint strategy which guarantees that among the
names written on the sheets there are at least two different names. Namely,
if player A on his move writes a name on the sheet of player B, then B writes
a name on the sheet of player A, otherwise on the sheet of player C. Player C
always writes a name on the sheet of player B.
In the beginning both players B and C have names different from the name
on the sheet of player A. Hence A cannot win in one move. Independent of
which name A changes on his move, after B moves, the name on the sheet of
C differs from the name on the sheet of A, and after C moves, both B and C
have names on their sheets different from the one on the sheet of A, as in the
beginning. So the cycle repeats.
Solution 2. Denote the players starting from any player in the order of their
turns by X, Y, and Z. Show that the players Y and Z can together always keep
X from winning. Indeed, X can win only on his move because Y and Z can
always play so that their move does not result immediately in X winning. X
can win on his turn only if before his move he and somebody else have his
name on their sheets. The player Z cannot prevent this situation only if the
same situation occurred already before his move and his sheet has the name
of X on it. But after Y moves, then either X or Z has the same name on their
sheets as Y has, and so Y can always prevent both X and Z having the same
name on their sheets. Thus none of the three players has a winning strategy.
14
FR-6. A regular 2010-gon is divided into pieces of triangular shape. Find
the least possible number of pieces. (Grade 10.)
Answer: 2008.
Solution. All the interior angles of the 2010-gon can be built from the in-
ner angles of the triangular pieces. As the sum of the inner angles of the
2010-gon is 2008 · 180◦ and that of every triangle is 180◦ , there must be at
least 2008 triangles. On the other hand, each convex 2010-gon can be divided
into exactly 2008 triangles by choosing one vertex and cutting the figure into
pieces along the diagonals that start from this vertex.
FR-8. Find all pairs of integers (m, n) such that for all positive real numbers
x and y the inequality x m + yn > x n ym holds. (Grade 11.)
Answer: (0, 0).
Solution. If m = 0, then the inequality is 1 + yn > x n . This holds for all
positive real numbers x and y iff n = 0. Hence (0, 0) is a solution. Let now
both m and n be different from zero. If the pair (m, n) satisfies the condi-
tion, then substituting x and y by 1x and 1y we see that the pair (−m, −n) also
satisfies the condition. Hence we can assume without loss of generality that
m > n and m > 0. If m > n, then by taking x = 1 we get 1 + yn > ym , which
does not hold for y large enough. Hence m = n. By taking x = y = 4 we get
2 · 4m > 42m which does not hold for any positive integer m. Therefore there
are no more suitable pairs.
FR-9. Let D be the midpoint of side BC of triangle ABC. Prove that the
intersection point of medians of triangle ABD and that of triangle ACD are
equidistant from line AD. (Grade 11.)
Solution. Triangles ABD and ACD have equal area since | BD | = |CD | and
the altitudes drawn from A coincide (Fig. 11). As these triangles have a com-
mon side AD, also the altitudes drawn from vertices B and C, respectively,
must be equal. Thus B and C are equidistant from line AD. Since the point
15
A
M
N
D
B C
Fig. 11
16
to cover the rest of the strip whose lower and left sides have correspondingly
the lengths k + 2 and 2k.
• If k is divisible by 3, then cut the figure into two strips of sizes 2 × 2k
and k × 2 and cover both of them with 2 × 3 rectangles consisting of two
unit L-shapes (Fig. 13) and we are done.
• If k ≡ 1 (mod 3), then cut the figure into two strips of sizes 2 × (2k − 2)
and (k + 2) × 2 and cover both of them with 2 × 3 rectangles (Fig. 14).
This is possible because 2k − 2 and k + 2 are divisible by 3.
• If k ≡ 2 (mod 3), then cut the figure into two strips of sizes 2 × (2k − 4)
and (k − 2) × 2, and a corner part, which is a L-shape with k = 2. Both
strips can be covered by 2 × 3 rectangles since 2k − 4 and k − 2 are di-
visible by 3; the corner part can be covered by induction basis (Fig. 15).
FR-11. A ball bearing consists of two cylinders with the same axis and n
equal balls between them. The centers of all the balls are on the same plane
perpendicular to the axis of the cylinders and each ball touches both cylinders
and two adjacent balls. Let r be the radius of the balls and let R be the radius
of the outer cylinder. Prove that Rr < n+ π
π . (Grade 11.)
Solution 1. Consider the regular n-gon with vertices at the centers of the
balls (Fig. 16). Its edges are of length 2r and its perimeter is n · 2r. The radius
of the circumcircle of the n-gon is R − r and the length of the circumcircle is
2π ( R − r ). Since a chord of a circle is always shorter than the corresponding
arc of the circle, we have n · 2r < 2π ( R − r ) or nr + πr < πR, which implies
r π
R < n+π .
Solution 2. Consider the isosceles triangle with vertices at the centers of
two adjacent balls and at the closest point to them on the common axis of the
cylinders (Fig. 17). The two equal sides of the triangle are of length R − r, the
base is of length 2r and the vertex angle is 2π n . The altitude drawn onto the
base divides the triangle into two equal right triangles with the hypotenuse
r
R − r, one of the legs r and the opposite angle πn . Hence R− π π
r = sin n < n ,
r π
whence nr < πR − πr, which implies R < n+π .
Fig. 16 Fig. 17
17
FR-12. The sequence ( an ) is defined by a1 = 1 and an = n · ( a1 + . . . + an−1 )
for all n > 1. Find all indices n for which an is divisible by 1 · 2 · . . . · n.
(Grade 12.)
Answer: 1 and all positive even numbers.
Solution. For each n > 2 denote Sn = a1 + . . . + an−1 . Then an = Sn · n and
for all n > 2 we have Sn = Sn−1 + an−1 = Sn−1 + Sn−1 · (n − 1) = Sn−1 · n.
Hence Sn = Sn−1 · n = Sn−2 · (n − 1)n = . . . = S2 · 3 · . . . · n = n! 2 because
S2 = 1 = 12·2 . Consequently an = Sn · n = n! · n2 for all n > 2. Therefore, for
n > 2, an is divisible by n! iff n is even, and n = 1 also satisfies the condition.
FR-13. The lengths of the sides of a quadrilateral are a, b, c, d and its area
is S. Prove that a2 + b2 + c2 + d2 > 4S. For which quadrilaterals does the
equality hold? (Grade 12.)
Answer. The equality holds only for squares.
Solution. Without loss of generality we can assume that a, b, c and d are
the lengths of consecutive sides of the quadrilateral. A diagonal divides the
quadrilateral into two triangles. From one partition we get the inequality
2 + 2 > S, whence ab + cd > 2S, and from the other partition 2 + 2 > S,
ab cd bc da
FR-14. In a coordinate city there are n > 3 tramlines parallel to the x-axis
such that each line begins from x-coordinate 0 and ends at x-coordinate n.
Exactly one tram of length 1 is moving on each line: on the first line with
speed 1, on the second line with speed 2 etc, until on the last line with the
speed n. When a tram reaches the end of its line it instantly starts moving
back without turning around. In the morning all trams start moving at the
same time from the starting position where the x-coordinate of the back end
of the tram is 0. Prove that the trams’ projections onto the x-axis never cover
the whole interval from 0 to n. (Grade 12.)
Solution. The projections of the trams can cover the whole interval only
when one projection covers [0, 1], another [1, 2] etc. until [n − 1, n]. Consider
the moments when the projection of the slowest tram covers one of these
intervals. When the slowest tram moves by 1 unit, then the fastest and the
third fastest trams move correspondingly by n and n − 2 units. Together these
two trams move by 2n − 2 units which is exactly one to and fro cycle.
Denote the integer positions of the trams on the round trip by numbers 0
to 2n − 3, i.e the starting position is 0 and each next one until returning to the
18
starting point is greater by one. Call these numbers the position characteris-
tics. If the sum of the position characteristics of two trams is 2n − 2, then their
projections cover the same interval because one of them has moved the same
amount from the starting point as the other one still has to go to reach it. If
the sum of the position characteristics is 0, then they both are in the starting
positions, so they again cover the same interval. Hence, when the sum of the
position characteristics is divisible by 2n − 2, the projections cover the same
interval.
At the beginning the sum of the position characteristics of the fastest and
the third fastest tram is 0 and each time they together move by 2n − 2 units the
sum of their position characteristics stays divisible by 2n − 2. Consequently,
when the projection of the slowest tram covers an interval with integer end-
points, the projections of these two trams cover the same interval, hence at
least one of the intervals is not covered.
FR-15. Find the minimal distance between two points, one of which is on
the graph of function y = e x and the other on the graph of function y = ln x.
(Grade 12.) √
Answer: 2.
Solution. The graphs of functions y = e x and y = ln x are symmetrical
w.r.t. line y = x (Fig. 18). Hence the distance between points on these graphs
is minimal iff both points are at minimal distance from line y = x. The dis-
tance between the graph of the function y = e x and the graph of the function
y = x is minimal at the point where the tangent is parallel to y = x. Then,
y′ = 1 for the function y = e x , whence e x = 1, giving x = 0 and y = √1. The
minimal distance is therefore between points (0, 1) and (1, 0) and it is 2.
y = ex
y = ln x
1
1
y=x
Fig. 18
19
IMO Team Selection Contest
First day
a−b
TS-1. For arbitrary positive integers a, b, denote a ⊖ b = .
gcd( a, b)
Let n be a positive integer. Prove that the following conditions are equiv-
alent:
(i) gcd(n, n ⊖ m) = 1 for every positive integer m < n;
(ii) n = pk where p is a prime number and k is a non-negative integer.
Solution. Note at first that da ⊖ db = a ⊖ b for all positive integers a, b, and
d. Indeed,
da − db d · ( a − b) a−b
da ⊖ db = = = = a ⊖ b.
gcd(da, db) d · gcd( a, b) gcd( a, b)
Show now that if n is a prime power and m < n, then n ⊖ m is relatively
prime to n. Indeed, let n = pk where p is a prime number, and let m = pi s
where gcd( p, s) = 1. Then m < n implies i < k. Now
p k−i − s
n ⊖ m = p k−i ⊖ s = = p k−i − s
gcd( pk−i, s)
because gcd( pk−i , s) = 1 by the choice of s. Also, for the same reason,
gcd( p, pk−i − s) = 1, hence gcd(n, n ⊖ m) = gcd( pk , pk−i − s) = 1.
It remains to show that if n is not a prime power, then there exists a posi-
tive integer m such that m < n and the integers n ⊖ m and n share a common
prime factor. Since n is not a prime power, it has at least two different prime
factors. Let p and q be some prime factors of n, whereby p < q. Let n = pk t
where gcd( p, t) = 1. Take m = n − pk+1. As n is divisible by both pk and
q which are relatively prime, it is also divisible by their product pk q. Conse-
quently, pk+1 < pk q 6 n, i.e., 0 < m < n. Now
t − (t − p) p
n ⊖ m = n ⊖ ( n − p k +1 ) = t ⊖ ( t − p ) = = =p
gcd(t, t − p) gcd(t, p)
since gcd(t, p) = 1. We see that n ⊖ m and n have a common prime factor p.
TS-2. Let n be a positive integer. Find the largest integer N for which there
exists a set of n weights such that it is possible to determine the mass of all
bodies with masses of 1, 2, . . . , N using a balance scale (i.e. to determine
whether a body with unknown mass has a mass 1, 2, . . . , N, and which
namely).
n
Answer: N = 3 2−1 .
Solution. The possibility to determine mass m means the possibility to
place the weights on the two scalepans so that the difference of total masses
on the two scalepans is exactly m.
20
Every weight can be placed on either of the two pans or on neither of
the pans. For n weights this makes 3n different placements. Note that the
placement where none of the weights is on the scales does not determine
any mass. Also, for each placement there is a symmetric placement with
all the weights on the two pans swapped, which determines the same mass.
n
Therefore with n weights it is possible to determine at most 3 2−1 different
masses.
We show by induction that it is possible to determine all masses from 1
3 n −1
to 2 using n weights with masses 1, 3, . . . , 3n−1 . For n = 1 it is obvious.
Assume that the claim holds for n = k. By the induction assumption we
k
can determine all masses from 1 to 3 2−1 by weights 1, 3, . . . , 3k−1 . Using the
weight with mass 3k , we can determine the mass 3k , and using it together
k
with the other weights also the masses 3k + 1, . . . , 3k + 3 2−1 and 3k − 1, . . . ,
k k 3 k −1 3 k −1 3 k +1 − 1
3k − 3 2−1 . Since 3k − 3 2−1 = 2 + 1 and 3k + 2 = 2 , the claim is
also true for n = k + 1.
TS-3. Let the angles of a triangle be α, β, and γ, the perimeter 2p and the
radius of the circumcircle R. Prove the inequality
2
9R
cot α + cot β + cot γ > 3
2 2 2
−1 .
p2
When is the equality achieved?
Answer: the equality holds for equilateral triangles.
Solution. Let the opposite sides of the angles α, β, and γ be correspond-
ingly a, b, and c. Since cot2 α = 1/ sin2 α − 1 and from the law of sines
1/ sin α = 2R/a, we have cot2 α = 4R2 /a2 − 1; similarly cot2 β = 4R2 /b2 − 1
and cot2 γ = 4R2 /c2 − 1. The inequality can therefore be written as
4 · 9R2
1 1 1
2
4R · + 2 + 2 −3 > 3· −1 ,
a2 b c ( a + b + c )2
or
1 1 1 27
+ 2+ 2 > .
a2 b c ( a + b + c )2
Dividing both sides by 3 and taking the square root gives
s
1 1 1 1 3
· 2
+ 2
+ 2
> .
3 a b c a+b+c
The left side is the quadratic mean of 1a , 1b , 1c and the right side is the harmonic
mean of the same numbers, hence the inequality holds.
The equality holds iff a = b = c.
21
Second day
TS-4. In an acute triangle ABC the angle C is greater than the angle A. Let
AE be a diameter of the circumcircle of the triangle. Let the intersection point
of the ray AC and the tangent of the circumcircle through the vertex B be K.
The perpendicular to AE through K intersects the circumcircle of the triangle
BCK for the second time at point D. Prove that CE bisects the angle BCD.
Solution. Since AE is a diam- D
eter of the circumcircle of the tri-
angle ABC, ∠ ACE = ∠ECK = B
90◦ . So it suffices to show that
∠ ACB = ∠DCK (Fig. 19). Let
L be the point of intersection of E
lines AE and DK. Then ∠ BAC = A L
∠CBK = ∠CDK by the inscribed
angles theorem. Also ∠ ABC = C
∠ AEC = ∠CKD where the latter K
equality follows from the simi- Fig. 19
larity of the right triangles ACE
and ALK. Hence the two triangles ABC and DKC are similar, and therefore
∠ ACB = ∠DCK.
Remark. This problem was proposed to the Baltic Way competition in 2008
(not by Estonia) but was not selected.
22
for all x, y. The case n = 2k implies P( x, y) = ( x2 + y2 )k which satisfies also
the condition P(0, 0) = 0.
Solution 2. Like in Solution 1, express the polynomial as a sum of n + 1
monomials with coefficients a0 , . . . , an and show that n = 2k.
We prove the claim of the problem by induction on k. In case k = 0
(omitting the extra assumption that P is non-constant) the claim holds ob-
viously. Assume now that k > 0 and the claim holds for k − 1. Substituting
t = 0 and t = π2 into P(sin t, cos t) = 1 gives a0 = 1 and an = 1, respec-
tively. Hence the polynomial P( x, y) − ( x2 + y2 )k does not have terms with
x n and yn . Let Q( x, y) be such that P( x, y) − ( x2 + y2 )k ≡ xy · Q( x, y). Then
sin t cos t · Q(sin t, cos t) = 0 for every real number t, hence Q(sin t, cos t) = 0
for every t such that sin 2t 6= 0. By continuity of Q(sin t, cos t) as a function
of t, it follows that Q(sin t, cos t) ≡ 0. Now define R( x, y) = Q( x, y) + ( x2 +
y2 )k−1 . As both Q and R are homogeneous polynomials of degree 2(k − 1),
the assumptions of the problem hold for polynomial R. By the induction hy-
pothesis, R( x, y) ≡ ( x2 + y2 )k−1 . Hence Q( x, y) ≡ 0 and P( x, y) ≡ ( x2 + y2 )k .
TS-6. Every unit square of a n × n board is colored either red or blue so that
among all 2 × 2 squares on this board all possible colorings of 2 × 2 squares
with these two colors are represented (colorings obtained from each other by
rotation and reflection are considered different).
a) Find the least possible value of n.
b) For the least possible value of n find the least possible number of red
unit squares.
Answer: a) 5; b) 10.
Solution. a) Since there are 24 = 16 = 42 possibilities
to color a 2 × 2 square in two colors and a n × n square
contains (n − 1)2 such subsquares, we must have n − 1 >
4, or n > 5. For n = 5 a suitable coloring is given in Fig. 20.
b) Fig. 20 presents a coloring with 10 red squares. We
will show that this is the least possible.
Note that in the 5 × 5 square there are 4 unit squares
in the corners, 12 squares on the sides (not in the corners), Fig. 20
and 9 inner squares. Each corner square is contained in
exactly one, side square in two and inner square in four 2 × 2 squares. All 16
colourings of 2 × 2 squares contain a total of 64 unit squares of which 32 are
red by symmetry. Therefore, if the 5 × 5 square contains k red squares, among
them a corner squares, b side squares and c inner squares, then a + b + c = k
and a + 2b + 4c = 32. The equation a + 2b + 4c = 32 implies c 6 8. If c = 8,
then a = b = 0. If c = 7, then the only possibility to have k < 10 is b = 2 and
a = 0. If c 6 6, then always k = a + b + c > 10.
23
Thus it is enough to show that there are no colorings with required prop-
erties with a = 0 and b 6 2. Indeed, in this case the 5 × 5 square has at least
two sides not containing any red squares. Without loss of generality, let one
of them be the upper side. We saw in part a) that for n = 5 each coloring of
2 × 2 squares must occur exactly once. Since among all 16 colorings of 2 × 2
squares there are 4 such where both upper unit squares are blue, and two
upper rows of the 5 × 5 square contain exactly 4 2 × 2 squares, all four such
colorings must be located in the two upper rows, among these the completely
blue coloring. Since the same is true for the other side which does not contain
any red squares, the two sides must meet and a completely blue 2 × 2 square
must be in the corner where the two sides meet. With-
out loss of generality, let it be the left side. Then the two
squares on Fig. 21 must be red, because otherwise there
would be more than one completely blue 2 × 2 square. But
now there are two 2 × 2 squares with red square in the
lower right corner and the rest of them blue. Therefore
there is no coloring satisfying the conditions with a = 0
Fig. 21 and b 6 2 and the least number of red squares is k = 10.
24
Estonian Math Competitions
2010/2011
University of Tartu
1
Selected Problems from Open Contests
OC-1. (Juniors.) Find all pairs ( a, b) of real numbers with a + b = 1, which
satisfy ( a2 + b2 )( a3 + b3 ) = a4 + b4 .
Answer: (0, 1), (1, 0), and ( 12 , 21 ).
Solution 1. As a3 + b3 = ( a + b)( a2 − ab + b2 ), the given equation can be
expressed as ( a2 + b2 )( a2 − ab + b2 ) = a4 + b4 . Expanding brackets gives
− a3 b + 2a2 b2 − ab3 = 0, which factorizes to − ab( a − b)2 = 0. Hence a = 0
or b = 0 or a − b = 0. Together with the condition a + b = 1, we get the
following solutions: (0, 1), (1, 0), and ( 21 , 12 ).
Solution 2. Denote ab = c. Then a2 + b2 = ( a + b)2 − 2ab = 1 − 2c,
a + b3 = ( a + b)( a2 − ab + b2 ) = 1 · (1 − 2c − c) = 1 − 3c, and a4 + b4 =
3
OC-4. (Juniors.) Find all pairs (n, k) of positive integers that satisfy the
equality n! + (n + 1)! = k! + 120.
Answer: (4, 4), (5, 6).
Solution 1. Note that for every n, n! + (n + 1)! = n! + n! · (n + 1) =
n! · (n + 2) 6 (n + 2)!. Thus if n! + (n + 1)! = k! + 120, then due to 120 = 5!
we have k! + 5! 6 (n + 2)!. This inequality in turn implies k < n + 2 and 5 <
n + 2. Hence 5 6 n + 1, leading to 0 6 (n + 1)! − 5! = k! − n!. Consequently,
k > n, i.e., the cases to be considered are k = n and k = n + 1. If k = n,
then the initial equation leads to (n + 1)! = 120, giving n = 4, k = 4. If
k = n + 1, then analogously n = 5, k = 6.
Solution 2. If k < n, then n! > k!. If, additionally, n > 4, then (n + 1)! >
120, giving n! + (n + 1)! > k! + 120. But if n < 4, then n! + (n + 1)! 6 30 <
k! + 120. The desired equality can hold in neither of the cases.
If k > n + 1, then k! − ( n + 1)! is positive and is also divisible by (n + 1)!,
hence k! − (n + 1)! > (n + 1)!. On the other hand, n! − 120 < (n + 1)!,
giving k! − (n + 1)! > n! − 120. Thus, there is no solution in this case either.
Hence n = k or n + 1 = k, leading to two solutions (n, k) = (4, 4), (5, 6).
3
Solution 3. The equation implies that 120 is divisible by the minimum of
n! and k!. As 120 = 5!, either n 6 5 or k 6 5. Consider both cases.
If n ∈ {1, 2, 3}, then n! + (n + 1)! − k! < 120, whence the equation has
no solution. If n = 4, then 120 = 144 − k!, whence k! = 24 and k = 4. If
n = 5, then 120 = 840 − k!, whence k! = 720 and k = 6.
If k ∈ {1, 2, 3, 4, 5}, then the initial equation implies that n! + (n + 1)!
lies between 121 and 240. This is possible only if n = 4, since if n = 3, then
n! + (n + 1)! = 30, and if n = 5, then n! + (n + 1)! = 840. If n = 4, then
n! + (n + 1)! = 144, which corresponds to k = 4.
OC-6. (Juniors.) A cashier has a stack of n notes lying on top of each other.
He has to turn all notes front side up, however the order of the notes is not
important. Every step consists of taking a block of consecutive notes and
turning them around in the stack. Find the smallest number of steps that
will suffice him to turn all notes in the stack front side up, irrespective of
the initial position of notes.
Answer: n2 for even n, and n+ 1
2 for odd n.
Solution 1. Let the cashier always choose the part of the stack which
starts from the top most note facing the wrong way and ends with the bot-
tom most note facing the wrong way. Then after k steps there are at least
4
k notes facing the right way on top of the stack and so are k notes at the
bottom of the stack. Therefore for even n, after at least n2 steps all the notes
are facing upwards. For odd n it is guaranteed that after n− 1
2 steps there are
n − 1 notes facing the right way, and one more step may be needed to turn
the middle note the right way. So, overall at least n+ 1
2 steps are sufficient.
To show the necessity, assume that the stack itself is located in between
a bigger stack where all the notes are already facing the right way, and call
the number of pairs of neighboring notes where one of the notes is the right
way around and the other is the wrong way around degree of disarrange-
ment. If the degree of disarrangement of the stack is 0, we have achieved
the required situation (we introduce the concept of the bigger stack to en-
sure that if all the notes in our stack are the wrong way around the degree
of disarrangement of the stack is non-zero). At every step, the degree of
disarrangement of the stack cannot decrease by more than 2, since the rel-
ative order of consecutive notes can only change at the boundaries of the
block to be turned around. Assume the initial stack consists of notes facing
alternately the right way and the wrong way, whereby for odd n assume the
top and the bottom notes are the wrong way around. Then for odd n the
degree of disarrangement to start with is n + 1 and so we need at least n+ 2
1
steps. For even n the initial degree of disarrangement is n because one of the
endmost notes is the right way around and thus at least n2 steps are needed.
Remark. In the first part of the solution we cannot claim that at every
step the number of notes facing the wrong way decreases by 2. In addition
to the two endmost notes facing the wrong way, all the notes between them
are also turned around and as a result the number of notes facing the wrong
way may even increase. For that reason it is necessary to monitor the pro-
gression of success from the stack’s ends onwards.
Solution 2. Let us show that every possible stack can be put in order as
required by no more than n2 (for even n) or n+ 1
2 (for odd n) steps as follows.
For n = 2 the statement holds. Indeed, if both notes are facing the wrong
way, we simply turn them around together, and if only one of the notes is
facing the wrong way we only turn this one around. Now partition the
stack into smaller stacks, each containing two consecutive notes and solve
the problem for each stack separately. For even n no more than n2 steps are
required. For odd n the last note has no counterpart, however it still may
require one step, so overall no more than n+ 1
2 steps are needed.
The necessity is shown as in Solution 1.
Solution 3. Let us present one more strategy to achieve the required situ-
ation by n2 (for even n) or n+ 1
2 (for odd n) steps for any stack of notes. If no
more than half of the notes are facing the wrong way we simply turn each
of them around individually. This requires as many steps as is the number
of such notes, i.e. for even n no more than n2 steps and for odd n no more
than n− 1
2 steps. If more than half of the notes are facing the wrong way, first
5
turn the whole stack around and then individually turn around all those
notes facing the wrong way. For even n no more than 1 + ( n2 − 1) = n2 steps
are required. For odd n no more than 1 + n− 1 n +1
2 = 2 steps are required.
The necessity is again shown as in Solution 1.
6
ii) If N = p2 q, where q < p, then N has the divisors 1, q, p, pq, p2 , and
p2 q, in increasing order, and d4 + d5 = pq + p2 = p(q + p) is divisible by
d2 + d3 = q + p and thus, they are all good, too.
iii) If N = p2 q, where p < q < p2 , then N has the divisors 1, p, q, p2 , pq,
and p2 q, in increasing order, and d4 + d5 = p2 + pq = p( p + q) is divisible
by d2 + d3 = p + q. Thus, they are all good, too.
iv) Finally, let N = p2 q, where q > p2 . Then N has the divisors 1, p, p2 ,
q, pq, and p2 q, in increasing order and d4 + d5 = q + pq = q(1 + p) is not
divisible by d2 + d3 = p + p2 = p(1 + p) because the prime number q can-
not be divisible by another prime number p. Thus all these numbers have
exactly 6 different divisors and they all are non-good. To get the smallest
of these numbers, we have to take p and q as small as possible, i.e. p = 2
and q = 5 (to achieve q > p2 = 4). Then N = 22 · 5 = 20. Finally, there
are infinitely many of these numbers N because we have infinitely many
choices for prime numbers p and q such that q > p2 . For example, we can
take p = 2 and q an arbitrary prime number bigger than 5. As there are
infinitely many prime numbers, we have proven the statement.
OC-8. (Seniors.) Prove that none of the integers that contain one 2, one 1,
and all the rest zeros, can be expressed as a sum of two perfect squares or
as a sum of two perfect cubes.
Solution. All the numbers described in the problem are divisible by 3
(as their sum of digits is divisible by 3). Note that all perfect squares leave
the remainder 0 or 1 when divided by 3, and therefore, for the sum of the
two perfect squares to be divisible by 3, they both have to be divisible by
3. Now, as the numbers are both divisible by 3, their squares are divisible
by 9 and thus the sum of the squares is divisible 9. However, the number
described in the problem is not divisible by 9, a contradiction.
Note that when a cube number is divided by 3, it will leave the re-
mainder of either 0, 1 or −1. Indeed, (3k)3 = 9(3k3 ) and (3k ± 1)3 =
27k3 ± 27k2 + 9k ± 1 = 9(3k3 ± 3k2 + k) ± 1. The numbers described in
the problem give the remainder 3 when divided by 9 (as their sum of digits
gives the remainder 3 when divided by 9), therefore we conclude that it is
impossible to express them as sums of two cubes.
7
∠CBB1 (because the sum of opposite C
angles of a cyclic quadrilateral ADBC
is 180◦ ). Thus, the right-angled tri-
angles AA1 C and BB1 C are similar.
From ∠ ACA1 = ∠ BCB1 we see that
∠ ACB = ∠ A1 CB1 . This together with
| AC|
| BC|
= ||AB 1CC|| gives that ACB and B1
1 A B
A1 CB1 are similar. As | AC | > | A1 C |,
we conclude that | AB| > | A1 B1 |. A1
Solution 2. Since the angles CA1 D
and CB1 D are right angles, the points D
C, A1 , D, and B1 form a cyclic quadri- Fig. 3
lateral and thus ∠CAB = ∠CDB =
∠CA1 B1 . Similarly, ∠CBA = ∠CB1 A1 . Therefore the triangles ABC and
A1 B1 C are similar. As |CA| > |CA1 |, we deduce that | AB| > | A1 B1 |.
Solution 3. The radius R of the circumcircle of the quadrilateral CADB
is at least as large as the radius R1 of the circumcircle of the quadrilateral
CA1 DB1 because CD is a chord in the first one and a diameter in the second
one. The sine law in triangles ADB and A1 DB1 gives | AB| = 2R sin ∠ D and
| A1 B1 | = 2R1 sin ∠D. As R > R1 , we deduce | AB| > | A1 B1 |.
Remark. The statement holds for all triangles ABC and all points D on
the circumcircle, given D is not one of the vertices of the triangle.
OC-10. (Seniors.) Find all pairs (m, n) of positive integers for which the
m × n grid contains exactly 225 rectangles whose side lengths are odd and
whose edges lie on the lines of the grid.
Answer: (1, 29), (5, 9), (9, 5), and (29, 1).
Solution. The m × n grid is formed by m + 1 horizontal and n + 1 ver-
tical lines. Number the horizontal lines with numbers from 1 to m + 1 and
the vertical lines with numbers from 1 to n + 1. Rectangles with odd side
lengths arise if and only if two horizontal lines with different parity and
two vertical lines with different parity intersect.
Assume that at least one of the numbers m and n is even. We can assume
without loss of generality that m = 2k. Then there are exactly k + 1 odd-
numbered and k even-numbered horizontal lines and thus there are k(k + 1)
pairs of lines of different parity. But this means that overall the number of
rectangles with odd side lengths is even and cannot be 225. Therefore m
and n are both odd numbers. Let now m = 2k − 1 and n = 2l − 1. Then we
have exactly k even-numbered and k odd-numbered horizontal lines and l
even-numbered and l odd-numbered vertical lines. Overall it is possible to
form k · k · l · l = (kl )2 rectangles with odd side lengths. From (kl )2 = 225
we get kl = 15. The solutions are k = 1, l = 15 or k = 3, l = 5 (or vice
versa). So m = 1, n = 29 or m = 5, n = 9 (or vice versa).
8
OC-11. (Seniors.) Given a triangle ABC where | BC | = a, |CA| = b and
| AB| = c, prove that the equality
1 1 3
+ =
a+b b+c a+b+c
holds if and only if ∠ ABC = 60◦ .
Solution. By finding the common denominator on the left hand side,
transform the equation to ( a + 2b + c)( a + b + c) = 3( a + b)(b + c). Ex-
panding the brackets and simplifying gives b2 = a2 + c2 − ac. Comparing
the latter with the cosine law b2 = a2 + c2 − 2ac cos β, we see that the equal-
ity holds if and only if cos β = 12 , i.e., β = 60◦ .
OC-12. (Seniors.) A square ABCD lies in the coordinate plane with its ver-
tices A and C lying on different coordinate axes. Prove that one of the ver-
tices B or D lies on the line y = x and the other one on y = − x.
Solution 1. Assume without loss of generality that A is located on the
x-axis and C is located on the y-axis, let these points have coordinates of
A ( a, 0) and C (0, c). As the diagonals of a square bisect each other, we know
that the intersection point P of diagonal is also the mid-point of AC, i.e.
−
→
P ( 2a , 2c ) and PC = (− 2a , 2c ).
As the diagonals of a square are perpendicular to each other and of the
−
→ −→
same length, the vectors PB and PD have the same length as the vector
−→
PC and are perpendicular to it. But for a given vector ~u = (s, t), there
are exactly two vectors perpendicular to and having the same length as it:
−→
~v = (−t, s) and −~v = (t, −s). For the vector ~u = PC = (− 2a , 2c ) we get
−
→ −→
~v = ( 2c , 2a ) and w.l.o.g. we can assume that PB = ~v and PD = −~v. Now
from here B( a+ c a+c a−c c− a
2 , 2 ) and D ( 2 , 2 ). Thus, we see that the point B is
located on the line y = x and point D is located on the line y = − x.
Solution 2. W.l.o.g., assume that the vertices of the square are labelled
counter-clockwise with A ( a, 0), C (0, c), where a, c > 0 (other cases are
similar). Let O be the origin, then ∠ AOC = 90◦ , i.e. the circumcircle (with
y
C y B
B C
D
O A x
O A x
D
Fig. 4 Fig. 5
9
diameter AC) of the square ABCD passes through the origin O. Based on
the assumptions made, B definitely lies in the first quadrant and D has to lie
in the second quadrant (Fig. 4) or in the fourth quadrant (Fig. 5), otherwise
the circle with the diameter BD cannot pass the origin. Now note that the
vertices of the square divide its circumcircle into four equal arcs of 90◦ , each
having an inscribed angle of 45◦ subtending on it. Thus, ∠ AOB = ∠ BOC =
45◦ , i.e., B lies on the line with equation y = x (if A = O or C = O, then one
of those angles will lose its meaning, however, the other one is still 45◦ and
that is sufficient). Similarly, ∠COD = 45◦ , if D lies in the second quadrant,
or ∠ AOD = 45◦ , if D lies in the fourth quadrant. In both cases, D lies on
the line y = − x; this condition is also met in the special case D = O.
OC-14. (Seniors.) Let ABC be a triangle with integral side lengths. The
angle bisector drawn from B and the altitude drawn from C meet at point P
inside the triangle. Prove that the ratio of areas of triangles APB and APC
is a rational number.
Solution 1. Let H be the foot of the alti- C
tude drawn from C. First prove that | AH |
and | BH | are rational numbers. For that, L
use the Pythagorean theorem for triangles
ACH and BCH to obtain | AH |2 + |CH |2 = K
2 2 2 2
| AC | and | BH | + |CH | = | BC | . There-
P
fore | AC |2 − | BC |2 = | AH |2 − | BH |2 =
(| AH | − | BH |) · (| AH | + | BH |) = (| AH | − A H B
| BH |) · | AB|. We see that | AH | − | BH | =
| AC|2 −| BC|2 Fig. 6
| AB |
is rational and so are | AH | =
| AB |+(| AH |−| BH |)
2 and | BH | = | AH | − (| AH | − | BH |). Let now K be the
projection of P to BC (see Fig. 6). As P lies on the angle bisector of B,
it is equidistant from both AB and BC, i.e., | PH | = | PK |. Consequently,
10
S APB | AB| · | PH | | AB| S |CP| · | BH | | BH |
= = . As CH ⊥ AB, also BPC = = .
SBPC | BC | · | PK | | BC | S APC |CP| · | AH | | AH |
S | AB| | BH |
Thus, APB = · is rational as a product of two rational numbers.
S APC | BC | | AH |
Solution 2. Let H be the foot of the altitude drawn from C and let L be
the projection of P to AC (see Fig. 6). Now ∠CPL = 90◦ − ∠ ACH = 90◦ −
(90◦ − ∠CAB) = ∠CAB, giving ||PH |
PL |
| PH |
= | PC|·cos ∠CAB
. The angle bisector
| PH | | BH |
theorem gives | PC|
= | BC|
= cos ∠ ABC. Consequently,
S APB | AB| · | PH | | AB| cos ∠ ABC
= = · .
S APC | AC | · | PL| | AC | cos ∠CAB
| AB|
As the side lengths of the triangle ABC are integers, is rational. By the
| AC |
cosine law, the cosines of the angles of triangle ABC are rational, whence
cos ∠ ABC S
is rational. Altogether, APB is rational.
cos ∠CAB S APC
OC-16. (Seniors.) How many positive integers are there that are divisi-
ble by 2010 and that have exactly 2010 divisors (1 and the integer itself
included)?
Answer: 24.
Solution. Let N be a positive integer that is divisible by 2010 and that
has exactly 2010 positive divisors. Since 2010 = 2 · 3 · 5 · 67, also N should
be divisible by these four primes. Thus, N = 2a · 3b · 5c · 67d · s, where a, b, c,
d > 0 and s is not divisible by any of the primes 2, 3, 5, 67. All the factors of
N can be expressed as 2i · 3 j · 5k · 67l · t, where 0 6 i 6 a, 0 6 j 6 b, 0 6 k 6 c,
0 6 l 6 d, and t is a factor of s. There are a + 1 choices for i (from 0 to a) and
similarly, there are b + 1, c + 1 and d + 1 choices for j, k and l, respectively.
Therefore, N has δ( N ) = ( a + 1)(b + 1)(c + 1)(d + 1)δ(s) different factors,
where δ( x ) stands for the number of factors of x. We require δ( N ) = 2010.
As a + 1 > 1, b + 1 > 1, c + 1 > 1, and d + 1 > 1, we see that each of
these numbers is divisible by some prime numbers and the number δ( N ) =
( a + 1)(b + 1)(c + 1)(d + 1)δ(s) can thus be expressed as a product of at
least four prime numbers. But as 2010 itself is a product of exactly four
prime numbers, we conclude that a + 1, b + 1, c + 1, and d + 1 are exactly
those primes 2, 3, 5, and 67, in some order, and δ(s) = 1. From the latter
condition we see that s = 1 because any numbers bigger than 1 has more
than one factor. So for N to satisfy the conditions, N must be expressible as
2a · 3b · 5c · 67d , where a, b, c, d are the numbers 1, 2, 4, and 66 in some order.
Thus there are 4! = 24 numbers satisfying the conditions.
12
Selected Problems from the Final Round
of National Olympiad
FR-1. (Grade 9.) Juku discovered that of the things in his satchel, 60 per-
cent were ugly and 76 percent were useless. He scrapped all things that
were both ugly and useless, and added things that were both beautiful and
useful. After this, of the things in Juku’s satchel, 25 percent are ugly and 45
percent are useless. How many percent of the things in Juku’s satchel were
both beautiful and useful initially?
Answer: 4 percent.
Solution. Observe that the amount of things that are beautiful but use-
less and things that are useful but ugly remained unchanged. The differ-
ence between the percentages of these things was 76% − 60% = 16% before
displacement but is 45% − 25% = 20% after that. Hence the overall num-
ber of things in the satchel decreased 20 : 16 = 1.25 times. Things that are
beautiful but useless form 45% of all things after the displacement, hence
they formed 45% : 1.25 = 36% before it. As there were 100% − 60% = 40%
of beautiful things in total, the things that were both beautiful and useful
constituted 40% − 36% = 4% of the content of the satchel.
Remark. This problem can of course be solved in completely standard
ways via linear equations.
13
b) Suppose it is possible to form a 2 × 2 × 2 cube so that its surface con-
tains only numbers 4, 5, 6. As exactly 3 faces of each unit cube are visible, all
three numbers must occur on those. Place the cube in such a way that the
upper layer has 6 in its southeastern corner (see Fig. 8). Then, as the only
possibility, the upper layer must have 4 in its southwestern corner and 5 in
its northeastern corner. Now it is impossible to place a dice in the north-
western corner since it should touch both of its neighbors with number 1.
15
− + −
+ 0 + 0
− + − −
+ − + 0 +
− 0 0
0 + + − 0 − + 0
+ − 0 − C
0 + − + 0
− + − C
A B
A B D C
D C − + −
0 − + 0 +
+ 0 − + −
A B A B
Fig. 10 Fig. 11
FR-6. (Grade 10.) Every face of a unit cube has one of numbers −1, 0,
1 written on it in such a way that every two faces with a common edge
contain different numbers. Is it possible to form
a) a cube of size 2 × 2 × 2;
b) a cube of size 3 × 3 × 3
so that in the grids that come up on the faces, every two squares with a
common side contain different numbers and the sum of all numbers on
each face equals 0?
Answer: a) yes; b) yes.
Solution. First note that the placement of the numbers on the faces of the
unit cube is unique. Indeed, let a number x be written on some face; then
the neighboring faces contain alternately the other numbers y and z, while
the opposite face again contains x. This means that each of the numbers −1,
0, 1 occurs in one pair of opposite faces. Figures 10 and 11 show suitable
constructions (where − and + denote −1 and 1, respectively).
Remark. These constructions can be easily generalized to arbitrary (even
or odd, respectively) integral size of the cube.
FR-7. (Grade 10.) The size of the angle ABC, expressed in degrees, in a
right triangle ABC is an integer. It is known that for some positive integer
n, one can choose points K0 = A, K2 , . . . , K2n on the hypotenuse AB and
points K1 = C, K3 , . . . , K2n+1 = B on the leg CB in such a way that each
triangle Ki−1 Ki Ki+1 with i = 1, . . . , 2n is isosceles with base Ki−1 Ki+1 . Find
all possible values of the size of angle ABC.
Answer: 2◦ , 6◦ , 10◦ , 18◦ , and 30◦ .
Solution. Let ∠ ABC = α (Fig. 12). Then the base angle of the last
isosceles triangle K2n−1 K2n K2n+1 is α. The base angle of the second last
isosceles triangle K2n−2 K2n−1 K2n has the size 180◦ − (180◦ − 2α) = 2α.
The base angle of the next triangle before it, K2n−3 K2n−2 K2n−1 , has the size
16
180◦ − (180◦ − 4α) − α = 3α. Generally, the size of the base angle of triangle
K2n−i K2n−i+1 K2n−i+2 is 180◦ − (180◦ − 2 · (i − 1)α) − (i − 2)α = iα (i = 3,
. . . , 2n). Thus the base angle of triangle ACK2 = K0 K1 K2 has the size 2nα.
Now in the triangle ABC we get 90◦ = ∠ BAC + ∠ ABC = 2nα + α, whence
◦
α = 2n90+1 . By the conditions of the problem, α must be an integer, hence
2n + 1 is an odd divisor C = K1
of 90 and is greater than
1 (as a triangle cannot 3α K3
have two angles of the 3α
size 90◦ ). Such divisors α
are 3, 5, 9, 15, and 45 4α 4α 2α 2α α
that give the solutions A = K K2 K4 B = K5
0
30◦ , 18◦ , 10◦ , 6◦ , and 2◦ ,
respectively. Fig. 12
FR-8. (Grade 11.) Find all integers that cannot be expressed as a sum of at
least three consecutive terms of some non-constant arithmetic sequence of
integers.
Answer: 1 and −1.
Solution 1. First prove that 1 and −1 are not expressible as the sum of at
least three consecutive terms of an arithmetic sequence of integers. Let a1 ,
a2 , . . . , ak be k consecutive terms of an arithmetic sequence, where k > 3.
They sum up to s = a1 + 2
ak
· k. If k is odd, then s is divisible by k. If k is even,
k
then s is divisible by 2 > 1. In both cases, s differs from 1 and −1.
Now prove that every integer s other than 1 or −1 is expressible as the
sum of at least three consecutive terms of an arithmetic sequence of inte-
gers. If s = 0, then s = −1 + 0 + 1. If s is different from zero and is even,
i.e., s = 2t, where t 6= 0, then −t, 0, t, 2t sum up to 2t = s. If s is odd, i.e.,
s = 2t + 1, then −t + 1, . . . , 0, 1, . . . , t − 1, t, t + 1 are consecutive terms of
an arithmetic sequence; they sum up to t + (t + 1) = 2t + 1 = s, since the
terms −t + 1 through t − 1 mutually cancel.
Solution 2. Let a1 be the first of the consecutive terms and d be the com-
mon difference of consecutive terms. The sum of n consecutive terms is
2a1 + d(n − 1)
s = · n. Thus 2s = (2a1 + d(n − 1))n. If s = 1 or s = −1,
2
then this equality cannot hold because n > 3 divides neither 2 nor −2. If
s = 0, then choose the portion of the arithmetic progression to be −1, 0,
1. If s differs from these numbers, then let n = 2|s|, d be an arbitrary odd
1 − ( n − 1) d −1 − ( n − 1 ) d
number, and a1 = if s > 0, and a1 = if s < 0.
2 2
17
Solution. First assume x > y. Then 3xy + 1 = x3 − y3 = ( x − y)( x2 +
xy + y2 ) > ( x − y) · 3xy. Thus 3xy = x3 − y3 − 1 > 1 − 1 = 0 because
x > y. If 3xy > 0, then 3xy > 3, hence the inequality 3xy + 1 > ( x − y) · 3xy
derived above implies x − y = 1. If 3xy = 0, then either x = 0 or y = 0
and in both cases the only possibility is x − y = 1 again. An elementary
check shows that all pairs ( x, y) = (n + 1, n), where n is an integer, satisfy
the initial equation.
Now assume x = y. Then the equation has no solutions, since the l.h.s.
is 0 while the r.h.s. is positive.
Finally assume x < y. Then the l.h.s. of the equation is negative, show-
ing that xy is negative. Hence x < 0 and y > 0. Denoting − x = z and multi-
plying the equation by (−1) leads to new equation z3 + y3 = 3zy − 1. Then
3zy − 1 = z3 + y3 = (z + y)(z2 − zy + y2 ) > (z + y)zy. Hence z + y < 3,
giving z = y = 1, i.e., x = −1, y = 1, as the only possibility. It is easy to
check that this satisfies the equation.
FR-10. (Grade 11.) Let CM be the median of a triangle ABC. Prove that
the product of the circumradius of ACM and the altitude drawn from M
in ACM equals the product of the circumradius of BCM and the altitude
drawn from M in BCM.
Solution 1. Let ∠CAB = α and ∠CBA = C
β (see Fig. 13), and let r and s be the cir-
cumradii of the triangles ACM and BCM,
respectively. By the sine law in the triangle U
|CM | V
ACM we obtain sin α = 2r, reducing to r =
|CM | |CM | α β
2 sin α . Analogously, s = 2 sin β . Let U and V
A M B
be the feet of altitudes drawn from the point
M in triangles ACM and BCM, respectively. Fig. 13
| AB |
Then | MU | = | AM | sin α = 2 sin α. Anal-
ogously, | MV | = | AB |
2 sin β. Thus r · | MU | =
|CM |·| AB |
4 = s · | MV |.
Solution 2. Let x and y be the altitudes drawn from M in the triangles
ACM and BCM, respectively. Let r and s be the circumradii of these tri-
angles, respectively. The areas of triangles ACM and BCM are equal be-
cause of | AM | = | BM | and the common altitude drawn from C. Therefore
| AC | · x = | BC | · y. Denote ∠ AMC = γ. The sine law gives | AC | = 2r sin γ
and | BC | = 2s sin(180◦ − γ) = 2s sin γ. Hence 2r sin γ · x = 2s sin γ · y. As
γ 6= 0, this implies rx = sy.
Solution 3. The formulas S = abc ah
4R and S = 2 , where a, b, c are the
side lengths, R is the circumradius and h is the altitude corresponding to a,
together give Rh = bc 2 . The product of the circumradius of ACM and the
| AM | · |CM | | BM | · |CM |
altitude drawn from M is thus . Analogously, for
2 2
triangle BCM. These two products are equal, since | AM | = | BM |.
18
FR-11. (Grade 11.) The inhabitants of a city of naturals are natural num-
bers. Every two different inhabitants may either be or not be friends. Call
a city neighborly if every two inhabitants share a common friend if and only
if one of the numbers is divisible by the other. Can a city whose inhabitants
are precisely 1, 2, . . . , 2011 be neighborly?
Answer: no.
Solution 1. Let p be a prime inhabitant of a neighborly city 1
(see Fig. 14). As p is divisible by 1 and p 6= 1, the inhabitants
1 and p share a common friend k. As k is divisible by 1 and i
k 6= 1, the inhabitants 1 and k share a common friend i.
If i 6= p, then k is a common friend of i and p. This means p
that i is divisible by p because the primality of p does not per-
mit the divisibility the other way round. If i = p, then 1 is a k
common friend of k and p. This analogously means that k is
divisible by p. In both cases, the city contains a multiple of p Fig. 14
that is greater than p.
In the city with inhabitants 1, . . . , 2011, the prime inhabitant 2011 has no
larger multiples. Hence this city cannot be neighborly.
Solution 2. Suppose that this city is neighborly. Choose arbitrary 12
prime inhabitants (for instance, the first 12 primes). Each of them shares a
common friend with 1; let these friends be a1 , a2 , . . . , a12 . These numbers are
all different since otherwise one of them would be a common friend to two
prime numbers. W.l.o.g., assume a1 < a2 < . . . < a12 . As 1 is a common
friend of all them, each of a2 , . . . , a12 must be divisible by the previous term.
This implies that a12 > 212 = 2048, a contradiction.
Remark. The use of primality of 2011 in Solution 1 can be replaced with
the application of Chebyshev’s theorem, choosing p arbitrarily in such a
way that 1005 < p < 2010.
19
99 ≡ 41 = 4, 1010 ≡ 02 = 0, 1111 ≡ 13 = 1. Hence the overall sum is
congruent to 3 modulo 5. Consequently, the last digit of this sum is 8.
FR-13. (Grade 12.) Does there exist a positive real number C such that the
inequality
x1 x2 + x1 x3 + x1 x4 + x2 x3 + x2 x4 + x3 x4 6 C ( x1 x2 + x2 x3 + x3 x4 + x4 x1 )
holds for arbitrary positive real numbers x1 , x2 , x3 , x4 ?
Answer: no.
Solution. For simplicity, denote A = x1 x2 + x1 x3 + x1 x4 + x2 x3 + x2 x4 +
x3 x4 and B = x1 x2 + x2 x3 + x3 x4 + x4 x1 . Choose x1 = x3 = u and x2 =
x4 = 1. Then A > x1 x3 = u2 and B = 4u. Hence A B > 4 . As u can be
u
FR-15. (Grade 12.) Ants has three pencils, each of a different color. In how
many ways can he paint the faces of a regular octahedron in such a way that
faces with a common edge always have different colors? Colorings that can
be obtained from each other via rotations of the octahedron are considered
the same.
Answer: 15.
Solution. One color can occur at most 4 times (at most twice among
the faces adjacent to either one of some two opposite vertices). Thus the
possible numbers of colors are 4, 4, 0 or 4, 3, 1 or 4, 2, 2 or 3, 3, 2.
20
i) Case 4, 4, 0. There are 3 possibilities to choose two colors from the
three. After that, there is only one possibility to paint the octahedron. Thus
there are 3 possibilities to paint.
ii) Case 4, 3, 1. Ordering the 3 colors can be done in 6 ways. After that,
there is only one possibility to paint the octahedron, since the color used 4
times must occur twice among the faces adjacent to one vertex and twice
among the faces adjacent to the opposite vertex. The remaining two colors
can be deployed in principle in only one way. Thus there are 6 possibilities
to paint.
iii) Case 4, 2, 2. Choosing the color that is used 4 times can be done in
3 different ways. After that, the octahedron can be painted in only 1 way,
since after 4 faces have been painted with the same color, faces with either of
the other colors must meet at the same vertex. Thus there are 3 possibilities
to paint.
iv) Case 3, 3, 2. Choosing the color that is used only twice can be done
in 3 ways. If the faces painted with this color met at a common vertex V,
the faces adjacent to the opposite vertex would be painted alternately with
the other two colors. But then the remaining two faces adjacent to V would
have to be painted with the same color, that contradicts the case assump-
tion. Hence the color that occurs twice is used on a pair of opposite sides.
The other colors occur alternately on the surface formed by the remaining
six faces. Thus there are 3 possibilities to paint.
Consequently, the number of all colorings is 3 + 6 + 3 + 3 = 15.
21
A6 A5
A2 A1
A7 A4
A
A3
l α
A8 A3
O
A1 A2
A4
Fig. 16 Fig. 17
the center of the 2n-gon, A be the point chosen inside, α be the angle be-
tween line OA and the line perpendicular to a side, and l be the distance
between O and any side of the 2n-gon (see Fig. 17). Then the altitude of
the corresponding triangle is l − |OA| cos α. The altitude of the next tri-
angle of the same color can be expressed similarly but α is replaced with
◦
α + 360
n . Thus the sum of all altitudes of the triangles of the same color is
◦ ( n−1)360◦
nl − |OA| · (cos α + cos(α + 360
n ) + . . . + cos( α + n )). To show that
◦
the sum inside parentheses equals 0, multiply the sum by sin 360
2n . Since
◦ ◦ ( k + 1 )·360◦ ( k − 1 )·360◦
cos(α + k·360 360 1
n ) sin 2n = 2 (sin(α +
2
n ) − sin(α + 2
n )), a tele-
◦
scoping sum emerges and after reduction one obtains − sin(α − 360 2n ) +
1
( n− 2 )·360 ◦
sin α + n = 0. Hence for both colors, the sum of the altitudes
of all triangles of this color is nl.
◦ ( n−1)·360◦
Remark. The sum cos α + cos(α + 360 n ) + . . . + cos(α + n ) in So-
lution 2 can also be computed as follows. Denote z = cos α + i sin α, where i
◦ ◦
is the imaginary unit and let zk = cos k·360 n + i sin k·360
n , k = 0, 1, . . . , n − 1.
Then the sum under consideration is the real part of the complex number
z · z0 + z · z1 + . . . + z · zn−1 . Thus z · z0 + z · z1 + . . . + z · zn−1 = z · (z0 +
z1n −1
z1 + . . . + zn−1 ) = z · (z01 + z11 + z21 + . . . + z1n−1 ) = z · z1 −1 = z · 0 = 0,
whence the sum under consideration is equal to 0.
22
IMO Team Selection Contest
First day
TS-1. Two circles lie completely outside each other. Let A be the point
of intersection of internal common tangents of the circles and let K be the
projection of this point onto their external common tangent. The tangents,
different from the common tangent, to the circles through point K meet the
circles at M1 and M2 . Prove that the line AK bisects the angle M1 KM2 .
Solution 1. Let L1 and L2 be the points of tangency of the external com-
mon tangent of the circles, N1 and N2 be the points of tangency of an in-
ternal common tangent, and O1 and O2 be the centers of the two circles
(see Fig. 18). As all the lines O1 L1 , AK, and O2 L2 are perpendicular to the
line L1 L2 , they are parallel to each other and thus || LL1 K |
= ||O1 A|
. The trian-
2 K| O2 A |
gles O1 AN1 and O2 AN2 are similar because they are both right-angled and
|O A | |O N | |O L |
have the same vertical angles. Thus, |O1 A| = |O1 N1 | = |O1 L1 | . Therefore, the
2 2 2 2 2
right-angled triangles O1 L1 K and O2 L2 K are similar due to proportionality
of their legs. Hence, ∠ L1 KO1 = ∠ L2 KO2 . As ∠ L1 KM1 = 2∠ L1 KO1 and
∠L2 KM2 = 2∠L2 KO2 , we also get that ∠L1 KM1 = ∠L2 KM2 . Together with
the equality ∠ L1 KA = ∠ L2 KA = 90◦ this implies ∠ M1 KA = ∠ M2 KA.
Solution 2. Both of the circles appear at the same angle, when viewed
from the point A. To solve the problem, it is enough to show that both of the
circles also appear at the same angle, when viewed from the point K. Let the
centers of the circles have the coordinates O1 ( a1 , b1 ) and O2 ( a2 , b2 ) and let
r1 and r2 be the radii of the circles. The two circles appear at the same angle
from the point P( x, y) if and only if |Or1P| = |Or2P| , i.e., √ r1
2 2
=
1 2 ( x − a1 ) +( y −b1 )
√ r2
2 2
. Simple algebra shows that this equation is equivalent to
( x − a2 ) +( y −b2 )
(r12 − r22 ) x2 + (r12 − r22 )y2 + c1 x + c2 y + c3 = 0, where c1 , c2 , and c3 are some
L2
L1 K
N1
O1 O2
A
M1
M2
N2
Fig. 18
23
constants. If r1 = r2 , then the statement clearly holds. If r1 6= r2 , then the
last equation is that of a circle. Point A as well as the point D of intersection
of the external common tangents both lie on that circle, and from symmetry,
the diameter of that circle is AD. As AK is perpendicular to the external
common tangent of the circles, the point K also lies on that circle.
Remark. The statement would hold even if we swapped the internal and
external tangents of the circles and considered angle M1 KM2 as the angle
between the lines KM1 and KM2 instead.
TS-2. Let n be a positive integer. Prove that for each factor m of the num-
ber 1 + 2 + . . . + n such that m > n, the set {1, 2, . . . , n} can be partitioned
into disjoint subsets, the sum of the elements of each being equal to m.
Solution. For every positive integer k, denote Sk = {1, 2, . . . , k} and
k ·( k +1)
sk = 1 + 2 + . . . + k = 2 . Prove the claim by induction: assume there
exists the required partitions of S1 , . . . , Sn−1 and prove the same for Sn . Fix
an arbitrary m such that m | sn , m > n.
First assume m > 2n. Let d = smn . To construct d disjoint subsets of Sn
with equal sum, partition the set {n, n − 1, . . . , n − 2d + 1} into subsets
Mi = {n + 1 − i, n − 2d + i }, where i = 1, . . . , d. As m > 2n implies
( n−2d )·( n+1−2d)
> (n−2d
2d )·2d
s
n + 1 > 4d, one gets n−d 2d = 2d = n − 2d. Note
( n−2d )·( n+1−2d)
also that d | 2 , since (n − 2d) · (n + 1 − 2d) ≡ n(n + 1) ≡ 0
(mod 2d). Hence by the induction hypothesis, there exist disjoint subsets
L1 , . . . , Ld of Sn−2d with equal sum. Taking Mi ∪ Li for each i = 1, . . . , d
forms the desired partition of Sn .
Now assume n 6 m < 2n. If m = n, then the task is trivial (n must be
odd to be a divisor of sn , so take sets {i, n − i }, i = 1, 2, . . . , n− 1
2 , and {n}).
If m > n, then form the subsets Mi = {m − 1 − n + i, n + 1 − i } of the set
{n, n − 1, . . . , m − n} for i = 1, . . . , n − ⌈ m2−1 ⌉, the sum of the elements of
each being m. The solution is complete if the remaining numbers in Sn can
be divided into sets, the sum of the elements of each being also m.
If m is odd, then the set of remaining numbers is {1, 2, . . . , m − n − 1}.
For m = n + 1, this set is empty and the partition is trivial, so assume
( m−n−1)·( m−n)
m > n + 1. As m > m − n − 1 and m | 2 (the latter following
from (m − n − 1) · (m − n) ≡ (n + 1)n ≡ 0 (mod m) and the parity of m),
the desired partition exists by the induction hypothesis.
If m is even, then the set of remaining numbers of Sn also includes m2 .
But m2 | sm−n−1 again by m | (m − n − 1) · (m − n). The inequality m < 2n
implies m2 < n, so also m2 > m − n > m − n − 1. Here, m = n + 1 is
impossible, since n + 1 | sn implies 2 | n and 2 6 | n + 1. Hence the induction
hypothesis gives the existence of a partition of Sm−n−1 into subsets, the sum
s − n −1
of the elements of each being m2 . The number of these subsets is mm/2 =
( m−n−1)·( m−n)
m = m − 2n − 1 + 2 · (n2m
+1 ) n
, which is odd. Together with the
number m2 , the subsets can be grouped by two to form the desired partition.
24
TS-3. Does there exist an operation ∗ on the set of all integers such that
the following conditions hold simultaneously:
(1) for any integers x, y, z, ( x ∗ y) ∗ z = x ∗ (y ∗ z);
(2) for any integers x and y, x ∗ x ∗ y = y ∗ x ∗ x = y?
Answer: yes.
Solution. Define an operation ⊕ on the set of all non-negative integers,
which maps two non-negative integers a and b to a non-negative integer
a ⊕ b, such that for all i = 0, 1, . . . , ( a ⊕ b)i = ( ai + bi ) mod 2, where ni
stands for the binary digit corresponding to 2i in the binary representa-
tion of n. This operation satisfies condition (1) for all non-negative integers
because addition modulo 2 satisfies it. The operation also satisfies condi-
tion (2) because if x, y ∈ {0, 1}, then ( x + x + y) mod 2 = y mod 2 =
(y + x + x ) mod 2. As the set of non-negative integers as well as the set
of all integers are countable, there exists one-to-one correspondence f be-
tween these sets (e.g. mapping a non-negative integer x to the integer
(−1) x ⌊ x + 1
2 ⌋). Every integer can therefore be uniquely expressed in the form
f (n), where n is a non-negative integer. Therefore we can define the opera-
tion ∗ by the formula f ( x ) ∗ f (y) = f ( x ⊕ y). Following from the construc-
tion, both conditions (1) and (2) still hold.
Remark. This problem was inspired by problem 5 on Baltic Way 2006.
Second day
TS-4. Let a, b, c be positive real numbers such that 2a2 + b2 = 9c2 . Prove
that
2c c √
+ > 3.
a b
Solution 1. Using the AM-GM inequality for three terms twice, one gets
√ √
2c c (2b + a)c (2b + a) 2a2 + b2 ( b + b + a ) a 2 + a 2 + b2
+ = = = >
a b ab p √3ab 3ab
√ √ √ √
3 3
3 b2 a 3 a 4 b2
3
3 3 b2 a · a 2 b 3 3ab √
> = = = 3.
3ab 3ab 3ab
Solution 2. Using HM-QM inequality for a, a, b gives
√
r r r
3 3 a 2 + a 2 + b2 2a2 + b2 9c2
= 6 = = = 3c.
2 1 1 1 1 3 3 3
+ + +
a b a a b
Thus
2 1
3 √
+ · c > √ = 3,
a b 3
which implies the desired inequality.
25
Solution 3. By AM-GM for two terms,
2c c 2 (2b + a)2 c2 (2b + a)2 (2a2 + b2 )
+ = 2 2
= =
a b a b 9a2 b2
(( a2 + b2 ) + 4ab + 3b2 )(a2 + ( a2 + b2 )) (6ab + 3b2 )( a2 + 2ab)
= 2 2
> =
9a b 9a2 b2
(2a + b)( a + 2b) 2a2 + 5ab + 2b2 4ab + 5ab
= = > = 3.
3ab 3ab 3ab
Solution 4. The square of the l.h.s. of the desired inequality is
2c c 2 (2b + a)2 (2a2 + b2 ) 1 a 2 b2
+ = = 2 + 2 + .
a b 9a2 b2 9 b a2
Denoting b = x, the desired inequality reduces to 9 (2 + x )2 (2 + x12 ) > 3,
a 1
TS-5. Prove that if n and k are positive integers such that 1 < k < n − 1,
then the binomial coefficient (nk) is divisible by at least two different primes.
Solution 1. Assume w.l.o.g. that n > 2k (if n < 2k, then interchange the
roles of k and n − k). Let p be an arbitrary prime number. Consider the
numbers that remain into the numerator of the expression
n · ( n − 1) · . . . · ( n − k + 1)
n
=
k k · ( k − 1) · . . . · 1
after reducing all factors by the highest power of p by which they are di-
visible. Suppose that some two of the k factors resulting after this step are
equal. Then the corresponding initial factors are of the form s · pi and s · p j ,
where i > j. But then n > s · pi > p · s · p j > p · (n − k) > 2 · (n − k),
which contradicts the assumption n > 2k. Hence the k new factors are pair-
wise different. As 1 < k < n − 1, the numerator initially contains at least
two consecutive numbers, at least one of which is not divisible by p. This
number does not change in the process described above. By the assumption
n > 2k, this number is greater than k. Consequently, the product remaining
in the numerator after elimination of powers of p is greater than the de-
nominator k · (k − 1) · . . . · 1. This means that the powers of p in the original
numerator cannot be completely cancelled out with the denominator. So
the canonical representation of (nk) cannot consist of a power of p only.
Solution 2. Suppose that for some n and k,
n · ( n − 1) · . . . · ( n − k + 1)
n
= = pt,
k k · ( k − 1) · . . . · 1
where p is a prime number and t is some positive integer. Let m be a num-
ber in {n, n − 1, . . . , n − k + 1}, in the canonical representation of which
26
the exponent of p is the largest. Then the exponent of p in the canonical
representation of n, n − 1, . . . , m + 1 coincides with that in the canonical
representation of n − m, n − m − 1, . . . , 1, respectively. Similarly, the expo-
nent of p in the canonical representation of m − 1, . . . , n − k + 1 coincides
with that in the canonical representation of 1, . . . , m − 1 − n + k, respec-
tively. Consequently, the exponent of p in the canonical representation of
the product n(n − 1) . . . (m + 1)(m − 1) . . . (n − k + 1) equals to that in the
canonical representation of the product (n − m)! (m − 1 − n + k)!. Since
( k − 1) ! k−1
k!
= k· =k·
(n − m)! (m − 1 − n + k)! (n − m)! (k − 1 − n + m)! n−m
is clearly an integer, the exponent of p in the canonical representation of
(n − m)! (m − 1 − n + k)! does not exceed that in the canonical representa-
tion of k!. Hence, the exponent of p in the canonical representation of (nk)
does not exceed that in the canonical representation of m. As the assump-
tions of the problem imply (nk) > (n2 ) > n > m, this leads to a contradiction.
Remark. More straightforward solutions can be presented using either
Legendre’s formula or Kummer’s theorem.
28
Estonian Math Competitions
2011/2012
University of Tartu
Problem authors Juhan Aru, Maksim Ivanov, Urve Kangro, Oleg Koshik,
Toomas Krips, Tuan Le (USA), Jim Leahy (Ireland), Härmel Nestra,
Heiki Niglas, Erik Paemurru, Reimo Palm
Translators Urve Kangro, Erik Paemurru
Editor Reimo Palm
3
Selected Problems from Open Contests
O1. (Juniors.) Find all four-digit numbers, which after deleting any one
digit turn into a three-digit number that is a divisor of the original number.
Answer: 1100, 1200, 1500, 2200, 2400, 3300, 3600, 4400, 4800, 5500, 6600,
7700, 8800, 9900.
Solution. Let abcd be a such number. Since abcd is divisible by abc, we
have d = 0. Since abcd = abc0 is divisible by abd = ab0, we have c = 0.
Since abcd = ab00 is divisible by acd = a00 and by bcd = b00, the number
ab is divisible by a and b. So b = ax and 10a = by with integer x and y.
Therefore 10a = axy, whence xy = 10. If x = 1, y = 10, then a = b, which
gives 9 possible numbers 1100, 2200, 3300, 4400, 5500, 6600, 7700, 8800, 9900.
If x = 2, y = 5, then 2a = b, which gives 4 possibilities 1200, 2400, 3600,
4800. If x = 5, y = 2, then 5a = b, which gives 1 number 1500. The case
x = 10, y = 1 is impossible, since a and b must be one-digit numbers.
O2. (Juniors.) Find the minimum number of colours required to paint all
points with integer coordinates in the plane in such a way that no two
points which are exactly five units apart have the same color.
Answer: 2.
Solution. Obviously at least 2 colors are necessary. Color all points ( x, y)
with even sum of coordinates with one color and all other points with an-
other color. All points that are at distance 5 from ( x, y) are ( x ± 4, y ± 3),
( x ± 3; y ± 4), ( x ± 5, y), ( x, y ± 5). In each case, the sum of coordinates has
the parity different from that of ( x, y). Therefore they are colored differently
from ( x, y).
O3. (Juniors.) A hiking club wants to hike around a lake along an exactly
circular route. On the shoreline they determine two points, which are the
most distant from each other, and start to walk along the circle, which has
these two points as the endpoints of its diameter. Can they be sure that,
independent of the shape of the lake, they do not have to swim across the
lake on any part of their route?
Answer: No.
Solution. Suppose the shape of the lake is an equilateral triangle. Then
the two points which are the most distant from each other are two vertices
of the triangle. The circle, which has these two points as the endpoints of
its diameter, does not cover the whole triangle,
√
because the distance of the
3
third vertex from the center of the circle is 2 of the length of the side of the
1
triangle, but the radius of the circle is only 2 of this length.
O4. (Juniors.) Two circles c and c′ with centers O and O′ lie completely
outside each other. Points A, B, and C lie on the circle c and points A′ ,
4
C′
C
P O O′
A
A′
c B
c′ B′
Fig. 1
O5.h (Juniors.)
i Let n be a positive integer and a1 , . . . , a2n be real numbers
1 1
in − 2 , 2 . Leaving out any one of the numbers, the sum of the remaining
2n − 1 numbers is always an integer. Prove that a1 = . . . = a2n .
Solution. Assume that there exist ai and a j which are not equal. Let
S = a1 + . . . + a2n . Since S − ai and S − a j are integers, their difference
(S − ai ) − (S − a j ) = a j − ai is also an integer. Since a j − ai 6= 0, and they
h i
belong to − 21 , 21 , their difference can be only ±1, this happens when ai
and a j are 12 and − 21 in any order. Let ak be any of the given numbers. Since
(S − ai ) − (S − ak ) = ak − ai is an integer, ak must also be either 12 or − 21 .
Hence all numbers ai are either 12 or − 12 . It follows that the sum of any two
numbers ai and a j is an integer. As we have an even number of them, the
sum of all the numbers S is also an integer. But then S − ai = S ± 21 cannot
be an integer, a contradiction. Therefore all numbers ai must be equal.
5
O6. (Juniors.) Is it possible that the perimeter of a triangle whose side
lengths are integers, is divisible by the double of the longest side length?
Answer: no.
Solution. Let the side lenghts of the triangle be integers a, b, c. Without
loss of generality we may assume that c ≥ a and c ≥ b. Suppose that the
perimeter of the triangle a + b + c is divisible by double of the longest side
length 2c. Since 0 < a + b + c ≤ 3c < 2 · 2c, the perimeter a + b + c can be
divisible by 2c only in the case when a + b + c = 2c. But then a + b = c,
which violates the triangle inequality a + b > c.
O7. (Seniors.) For any positive integer n let an be the largest power of 2 that
divides n (e.g. a2011 = 1, a2012 = 4). Prove that for any positive integers i
and j with i < j, the sum a1 + a 1 + . . . + a1 is a fractional number.
i i +1 j
Solution. First prove that the largest power of 2 among the numbers ai ,
ai+1 , . . . , a j is unique. Let 2s be the largest of the numbers ai , ai+1 , . . . , a j .
If there were k and l with i ≤ k < l ≤ j such that ak = al = 2s , then they
must be of the form k = 2s u and l = 2s v, where u and v are odd numbers.
Since k < l, we have u < v and u + 1 < v. Since u + 1 is even, the num-
ber m = 2s (u + 1) has a divisor 2s+1, and k < m < l, which contradicts
the choice of s. Thus the largest power of 2 appears only once among the
numbers ai , ai+1 , . . . , a j . Converting the fractions to the common denomi-
nator the fraction with the largest denominator gives 1 in the numerator, all
others give a positive power of 2, i.e. an even number. Consequently the
numerator is odd and cannot cancel with the denominator.
O9. (Seniors.) Let ABC be a triangle with median AK. Let O be the circum-
center of the triangle ABK.
a) Prove that if O lies on a midline of the triangle ABC, but does not
coincide with its endpoints, then ABC is a right triangle.
b) Is the statement still true if O can coincide with an endpoint of the
midsegment?
Solution. a) Let L and M be the midpoints of the sides CA and AB,
respectively. If O lies on the segment KM (Fig. 2), then the segment KM
and the perpendicular bisector of AB have two different common points O
and M, hence KM is the perpendicular bisector of AB. Since KM is parallel
to AC and is perpendicular to AB, the angle at vertex A must be right. If
O lies on the segment LM (Fig. 3), then we get similarly that the angle at
vertex B must be right. If O lies on the segment KL (Fig. 4), then on one
hand ∠ ABC is acute, because OK and MB are perpendicular to MO, the
perpendicular bisector of AB, and |OK | < | LK | = | MB|. On the other
hand, ∠ ABK must be obtuse, since the circumcenter O of the triangle ABK
lies outside of the triangle, a contradiction. Thus this case is not possible.
b) If the triangle ABC is equilateral, then the median AK is also the alti-
tude and ABK is a right triangle with the hypotenuse AB. The circumcenter
O of the last triangle is the midpoint of AB, i.e. an endpoint of a midseg-
ment of the triangle, but ABC is not a right triangle.
7
exactly n dark squares visible by covering the paper as follows. First put
one figure on the lower left corner of the paper. Then put two figures so
that they cover the dark squares of the first figure and their dark squares
form a diagonal of length 3. Then add 3 figures so that they cover the dark
squares of the previous figures and their dark squares form a diagonal of
length 4. Repeat this until we have a diagonal of length n from one corner
of the paper to the opposite corner. To cover all the squares repeat the same
steps starting from the upper right corner of the paper.
O11. (Seniors.) The teacher drew a 3 × 3 table in Juku’s exercise book and
wrote a number in every position of the table. Then he gave Juku the fol-
lowing task.
1) Turn the next page and draw a similar table. Write in the first row
the numbers obtained by subtracting the numbers in the third row of
the corresponding column from the numbers in the second row of the
corresponding column in the previous table. Similarly, the numbers
in the second and third row are obtained as differences of the third
and the first, and the first and the second row.
2) Turn the next page and draw a new table. Write in the first column
the numbers obtained by subtracting the numbers in the third column
from the numbers in the second column in the corresponding row in
the previous table. Similarly, the numbers in the second and third
column are obtained as differences of the third and the first, and the
first and the second column.
Repeat in turns steps 1 and 2 until you reach a table where all the numbers
are zeroes. Juku has reached the end of the third page and has not yet
reached the table with all zeroes in it. Prove that his task never ends.
Solution. First note that after step 1 we get a table where the column
sums of the table are 0, and after step 2 we get a table where the row sums
of the table are 0. Suppose that after some steps we reach the table with all
zeroes in it. By symmetry we can consider the case where we get this table
after step 2. Then the table on the previous step was
a a a
b b b
c c c
where at least one of the numbers a, b, c is not zero. This table was obtained
after step 1, hence a + b + c = 0. The table on the previous step was
d e f
d−c e−c f −c
d+b g+b f +b
This was also written by Juku, because he computed at least 2 tables. Since
this table was obtained after step 2, we must have d + e + f = d − c + e −
8
c + f − c = d + b + g + b + f + b = 0. From the first equality it follows that
c = 0, from the equality of the first and third expression b = 0, and since
a + b + c = 0, we have a = 0, which contradicts the assumption that at least
one of the numbers a, b, c is not zero.
O12. (Seniors.) Prove that for any positive integer n the sum of the first n
primes is greater than n2 .
Solution. First notice that the n-th prime p n satisfies the inequality pn ≥
2n − 1. Indeed, the claim holds for the first prime p1 = 2. Since all other
primes are odd and there is exactly n − 1 odd numbers between 2 and 2n,
there are at most n prime numbers less or equal to 2n − 1, hence pn ≥ 2n −
1. Now consider the sum of the n first primes P = p1 + p2 + . . . + pn . Since
pk ≥ 2k − 1 for any k, and additionally p1 = 2 > 1, the sum P is strictly
greater than the sum of n first odd numbers S = 1 + 3 + . . . + 2n − 1 =
= (12 − 02 ) + (22 − 12 ) + . . . + (n2 − (n − 1)2 ) = n2 . So P > S = n2 .
O14. (Seniors.) Let ABC be an acute triangle and D an interior point of its
side AC. We call a side of the triangle ABD friendly, if the excircle of ABD
tangent to that side has its center on the circumcircle of ABC. Prove that
there are exactly two friendly sides of ABD if and only if | BD | = | DC |.
Solution. Let E, F and G be the centers of excircles touching BD, AD and
AB respectively, and let ω be the circumcircle of ABC (see Fig. 5). To prove
the assertion of the problem, we will show that F and G cannot both lie on
ω and that E ∈ ω ⇐⇒ | BD | = | DC | ⇐⇒ F ∈ ω.
9
As AF and AG are bisectors of the two E
complementary angles of BAD, the point G B
A lies on the segment FG. Thus, only one
of the rays AF and AG can cut the circle ω
again, and therefore only one of F and G
can lie on ω.
To show that E ∈ ω iff | BD | = | DC |, A D C
we first note that E lies on ω iff ∠CAE = F ω
∠CBE. Since ∠CAE = 12 ∠BAD = 21 (π −
∠ ADB − ∠ ABD ) = 12 (π − (π − 2∠BDE) − Fig. 5
π π
(π − 2∠DBE)) = ∠BDE + ∠DBE − 2 = 2 − ∠BED, we see that E ∈ ω
is equivalent to ∠CBE = π2 − ∠ BED, i.e BC and DE being perpendicular.
Since DE is the bisector of BDC, this occurs iff | BD | = | DC |.
It remains to show that | BD | = | DC | iff F ∈ ω. Point F lies on ω iff
∠ AFB = ∠BCD. Using the fact that ∠BAF = ∠BAD + 21 (π − ∠BAD ) =
1 π −∠ ABD −∠BAD
2 ( π + ∠ BAD ), we get ∠ AFB = π − ∠ ABF − ∠ BAF = 2 =
∠ ADB ∠BCD +∠CBD
2 = 2 . Thus ∠ AFB = ∠ BCD is equivalent to ∠ BCD =
∠CBD, i.e., | BD | = | DC |.
Remark. The claim holds in the case of right or obtuse triangle, too. The
problem with the above proof is that if ∠ ABC > 90◦ then point E may fall
inside triangle ABC, whence equality ∠CAE = ∠CBE is no more equiva-
lent to E ∈ ω. Nevertheless, one can show that if | BD | = | DC |, then E must
lie outside triangle ABC, extending the validity of the claim to the obtuse
case.
10
Selected Problems from the Final Round
of National Olympiad
F1. (Grade 9.) Integers a, b, c are such that a + b + c is divisible by 6, and
a2 + b2 + c2 is divisible by 36. Does it imply that a3 + b3 + c3 is divisible by
a) 8; b) 27?
Answer: a) yes; b) no.
Solution. a) As the sum of a, b, and c is divisible 6, and is therefore even,
there must be either 0 or 2 odd numbers among the three. If we had 2 odd
numbers, the sum of the squares a2 + b2 + c2 would give a remainder of
0 + 1 + 1 = 2 when dividing by 4. But this is not possible, since the sum is
divisible by 36, and therefore also by 4. So, all the numbers a, b, c are even.
Hence, all the numbers a3 , b3 , c3 are divisible by 8, and so is their sum.
b) If a = 8, b = c = 2, then all of the premises are fulfilled: 8 + 2 + 2 = 12
is divisible by 6 and 82 + 22 + 22 = 72 is divisible by 36. But 83 + 23 + 23 =
528 is not divisible by 9, and therefore, it is not divisible by 27.
F2. (Grade 9.) Let ABC be an isosceles triangle with | AB| = | AC |. The
bisector of angle ABC meets the side AC at the point D.
a) Is the triangle ABD isosceles whenever the triangle BCD is isosceles?
b) Is the triangle BCD isosceles whenever the triangle ABD is isosceles?
Answer: a) yes; b) no.
Solution. a) Denote ∠ BAC = α and ∠ ABC = C
∠ ACB = β (Fig. 6). Assume that the triangle BCD β
is isosceles. If |CB| = |CD |, then the angles ∠CBD, D
∠CDB and ∠BCD would be 2β , β2 and β respectively, β
β β 2
which implies 2 · + β =
2 180◦ giving β = 90◦ .This α 2
is impossible, since the triangle ABC has two angles A B
β
equal to β. If | DB| = | DC |, then we would get 2 = Fig. 6
β, which is also impossible. This leaves the only
option | BC | = | BD |. Then, the triangles ABC and BCD are similar, since all
corresponding angles are the same. Therefore ∠ DBA = ∠ DBC = ∠ BAC =
∠BAD, showing that the triangle ABD is isosceles with
C | DA| = | DB|.
b) If the angles of the triangle ABC are 73 · 180◦ , 27 ·
180◦ , 27 · 180◦ (Fig. 7), then ∠ ADB = 180◦ − ∠ BAD −
D
∠ ABD = 180◦ − 37 · 180◦ − 71 · 180◦ = 37 · 180◦ = ∠BAD,
which shows that the triangle ABD is isosceles with
| BA| = | BD |. At the same time, the angles in the trian-
A B
gle BCD are 17 · 180◦ , 72 · 180◦ , 74 · 180◦ , which are pair-
Fig. 7 wise different, so the triangle BCD is not isosceles.
11
F3. (Grade 9.) An equilateral triangle with side length 3 is
divided into 9 equilateral triangles with side length 1. An in-
teger from 1 to 10 is written into every point that is a vertex
of a small triangle (colored vertices on the figure), such that
all numbers are written exactly once. For every small trian-
gle, the sum of the numbers in its three vertices is written inside it. Prove
that at least three of those sums are greater than 11.
Solution. In a triangle, which has 10 at one vertex, the sum is at least
13. If 10 is not at one of the vertices of the large triangle, the number of
triangles with sum greater than 12 is at least 3 and the problem is solved. If
10 is at the vertex of the large triangle, then look, where is the number 9. If
9 does not lie at a vertex of the large triangle, then there are at least 3 small
triangles, with a sum of at least 12, and the problem is solved. If 9 is at a
vertex a the large triangle, then look, where is the number 8. If 8 does not
lie at the vertex of a large triangle, then it is at a vertex of at least 3 small
triangles. In at least two of them the sum is at least 12, and as at most one
of these can overlap with one of the triangles found earlier, the problem is
solved. If 8 is at a vertex of the large triangle, then either the sum in that
triangle is at least 12, meaning the problem is solved, or the other vertices
in the triangle have numbers 1 and 2. In the last case, the numbers 3, 4
and 5 are the smallest numbers whose positions are not set, but these give
a sum of 12. So, we can simply choose any triangle who vertices we have
not looked at yet as our third triangle (there are three of these triangles).
F4. (Grade 9.) Jüri wishes to draw n circles and any number of lines on
the plane such that all the lines meet at one point, and for every two circles
there exist two lines that touch both of these circles.
a) Is it possible for Jüri to solve this problem for any n ≥ 2?
b) For which natural numbers n is it possible to solve this problem if in
addition all the circles must have the same radius?
Answer: a) yes; b) n ≤ 4.
Solution. a) Jüri can draw two lines and draw any number of circles such
that they touch both of the lines (Fig. 8).
b) Assume that Jüri has solved the problem for some n, where n > 1.
Fig. 8 Fig. 9
12
Let O be the intersection point of all the lines. Look at any circle c. From
the premises of the problem we see that the circle c touches two of the lines
drawn by Jüri, which we call k and l. But any one circle can only touch up
to two lines drawn from one point. So, the circle c does not have any more
lines touching it. If c′ is any other circle drawn by Jüri, then the common
tangents of c and c′ can only be k and l. Therefore, k and l are the common
tangents of all the circles. Two lines divide the plane into four sectors, inside
each can be only one circle with the previously set radius (Fig. 9). So, for
n > 4 the problem has no solution but for n ≤ 4, it obviously has.
F5. (Grade 10.) Find all pairs (n, m) of positive integers such that the arith-
metic and geometric means of m and n are different two-digit numbers con-
sisting of the same digits.
Answer: (32, 98), (98, 32).
Solution. Let 10a + b be the arithmetic mean of the given numbers, where
a and b are decimal digits. Let 10a + b + x and 10a + b − x be the numbers
we are searching for. Then, by the premises
q
(10a + b + x )(10a + b − x ) = 10b + a,
which after squaring and simplifying gives x2 = 99( a2 − b2 ). So, x2 is
divisible by 99, implying x2 is divisible by 3 and 11. Since 3 and 11 are
primes, x itself is divisible by 3 and 11, and therefore by 33. Denoting x =
33z, we get
99 · 11z2 x2
11z2 = = = a2 − b2 = ( a + b)( a − b),
99 99
from which we see, that the product ( a + b)( a − b) is divisible by 11. Since
11 is a prime, either a + b or a − b is divisible by 11. Since a − b 6= 0 and
a and b are single-digit numbers, we must have a + b = 11. Therefore
a − b = z2 . Since a − b and a + b are either both odd or both even, z must be
odd. So, z = 1, since z ≥ 3 implies x ≥ 99, but 10a + b − x must be positive.
So, a = 6, b = 5, x = 33, and the corresponding pair is (98, 32).
F6. (Grade 10.) We say that two real numbers r and s are close if |r − s| =
10u for some integer u. Let y = ax + b be a linear function, for which there
exist close numbers x1 and x2 so that the corresponding y1 and y2 are also
close. Prove that for any close numbers x1′ and x2′ , the corresponding y1′ and
y2′ are also close.
Solution. From the premises we get | x1 − x2 | = 10u and |y1 − y2 | =
|( ax1 + b) − ( ax2 + b)| = 10v for some integers u, v. So,
10v = |( ax1 + b) − ( ax2 + b)| = | a( x1 − x2 )| = | a| · | x1 − x2 | = | a| · 10u ,
v
10
which gives | a| = 10 v − u . Let x ′ , x ′ be any close real numbers, | x ′ −
u = 10 1 2 1
x2 | = 10 . Then |y1 − y2 | = |( ax1′ + b) − ( ax2′ + b)| = | a( x1′ − x2′ )| =
′ w ′ ′
13
| a| · | x1′ − x2′ | = 10v−u · 10w = 10w+v−u. Since u, v, w are integers, w + v − u
is also an integer, which shows that y1′ , y2′ are close.
F7. (Grade 10.) Let ABC be a triangle on the plane. The angle bisector from
the vertex A meets the side BC at P, and the median from the vertex B meets
the side AC at M. The lines AB and MP meet at the point K. Prove that if
| PC|
| BP |
= 2, then AP and CK are perpendicular.
Solution 1. Let K ′ be a point on the ray AB, such
that B is the midpoint of the line segment AK ′ (Fig. C
10). Then CB is the median of the triangle ACK . ′
As P divides this line segment in the ratio 2 : 1, P
must be the centroid of the triangle ACK ′ . So, K ′ M,
M
which is also a median of the triangle ACK ′ , must
pass through the point P. Therefore, K = K ′ . So, B is P
the midpoint of the line segment AK. As AP passes
through the point P, AP is also a median of the trian- A B K′
gle ACK. By the premises it is also an angle bisector.
So, the triangle ACK is isosceles with | AC | = | AK | Fig. 10
and AP is its height. Therefore, AP ⊥ CK.
Solution 2. As in solution 1, we show that B is the midpoint of AK. So
BM joins the midpoints of sides in the triangle ACK and therefore BM k
CK. An the angle bisector divides the opposite side in the same ratio as
| AC| | PC|
the corresponding sides, so | AB| = | BP| = 2. As M is the midpoint of AC,
we have | AB| = | AM |. An angle bisector, drawn from the vertex opposite
the base in an isosceles triangle, is also the height in that triangle, giving
AP ⊥ BM. It follows that AP ⊥ CK.
14
Fig. 11 Fig. 12 Fig. 13 Fig. 14
Indeed, the three triangles can be chosen so that they leave uncovered the
central number (Fig. 11) or one of the corner numbers (Fig. 12, 13, 14).
Solution 2. Let m be the number in the center of the large triangle. Then,
when adding the sums of the three corner triangles we get the sum of all
the numbers from 1 to 10, except m, so the sum of the corner triangles is
55 − m. If m ≤ 7, then the sum is at least 48.
In the rest of the cases, consider any three triangles around the center
point, such that no two of them share a side. Adding the numbers in them,
we get the sum of all the numbers from 1 to 10, except the three numbers
in the corners, while we add the center number three times. So the sum of
those triangles is at least 21 + 3m. If m ≥ 9, then the sum is at least 48.
This leaves the case m = 8. The sum of the numbers in the triangles in
the corners is at least 55 − 8 = 47, so at least one of them contains a sum
that is at least 16. If none of the triangles contains a sum 17 or greater, the
number 16 must occur in two different triangles. The sum of the numbers
in any three triangles around the center point, chosen like above, is at least
21 + 3 · 8 = 45. So, in both triples at least one of the triangles contains a
sum of at least 15, and since triangles sharing an edge cannot contain the
same sum as the corresponding sums differ by exactly one term, at least
one of the six triangles around the center point contains a sum of at least 16.
Thus we can pick the three desired triangles from among either one corner
triangle and two central triangles or two corner triangle and one central
triangle that contain the largest numbers.
F9. (Grade 10.) Find all triples of positive integers ( x, y, z), for which
x · y! + 2y · x! = z!.
Answer: (2, 1, 3) and (n, n + 1, n + 2) for every positive integer n.
Solution. Since the left-hand side is greater than both x! and y!, obviously
z > x and z > y. So, both sides of the equation are divisible by both x! and
y!. Therefore, x · y! is divisible by x!, which means that y! is divisible by
( x − 1)!, giving y ≥ x − 1. Analogously, 2y · x! is divisible by y!, meaning
2 · x! is divisible by (y − 1)!. The case x = 1, y = 3 is not a solution, the case
x > 1 gives 2 · x! < ( x + 1)!, which implies x ≥ y − 1. This leaves us to look
through the cases −1 ≤ y − x ≤ 1.
• If y = x − 1, then the equation simplifies to (2x − 1) · x! = z!. As
( x + 1)( x + 2) > 2x − 1, we have 2x − 1 = x + 1 and z = x + 1. This
gives the solution x = 2, y = 1, z = 3.
15
• If y = x, the equation simplifies to 3x · x! = z!. As ( x + 1)( x + 2) > 3x,
we have 3x = x + 1, but this does not give integer solutions.
• If y = x + 1, the equation simplifies to ( x 2 + 3x + 2) · x! = z! or ( x +
2)! = z!. From here we get a family of solutions x = n, y = n + 1,
z = n + 2.
F10. (Grade 11.) In his last research, professor P was concentrating on nat-
ural numbers with a certain property. It is known that whenever a natural
number x has this property, all multiples of x also have this property. Let
a1 , . . . , an be positive integers such that all their divisors that are greater
than one have the property professor P studied. Is it true that all divisors
greater than one of the product a1 . . . an definitely have this property?
Answer: yes.
Solution 1. Let k > 1 be any divisor of the product a1 . . . an . Then k has
a prime divisor p, which is also a divisor of the product a1 . . . an . As p is a
prime, there exists i, such that p is a divisor of ai . As all the divisors of ai
greater than 1 have the property, p also has this property. By the premise,
all the multiples of p have the property, so k has the property.
Solution 2. Let k > 1 be any divisor of the product a1 . . . an . If k were
relatively prime to all ai , it would be relatively prime to the product a1 . . . an ,
but gcd(k, a1 . . . an ) = k > 1. Hence gcd(k, ai ) > 1 for some ai . As a divisor
of ai , the number gcd(k, ai ) has the property studied by professor P. As a
multiple of gcd(k, ai ), also k has the same property.
F11. (Grade 11.) a) Find all positive integers n, such that the sum of all
integers from 1 to n + 1 can be represented as the sum of n consecutive
integers.
b) Find all positive integers n, for which there exists an integer a, such
that the sum of the integers from a to a + n is equal to the sum of the integers
from a + n + 1 to a + 2n.
Answer: a) 1; b) all positive integers.
Solution 1. a) Clearly the sum of the first two positive integers can be
represented as the sum of one positive integer. Now, let n ≥ 2 and let us
show that the sum of the n + 1 first positive integers cannot be represented
as a sum of n consecutive integers. Indeed, on one hand 1 + 2 + . . . + n +
(n + 1) > 2 + 3 + . . . + n + (n + 1), on the other hand 1 + 2 + . . . + n + (n +
1) < 1 + 2 + 3 + . . . + n + ( n + 1) + 1 = 3 + . . . + n + ( n + 1) + 2 + 2 ≤
3 + . . . + n + (n + 1) + (n + 2). So the sum of n + 1 first positive integers
1 + . . . + (n + 1) lies between 2 + . . . + (n + 1) and 3 + . . . + (n + 2), which
both are consecutive sums of n consecutive integers. So, the number 1 +
. . . + (n + 1) is not a sum of n consecutive integers.
b) Let n be any positive integer. To solve the problem, it suffices to see
that n2 + (n2 + 1) + . . . + (n2 + n) = n2 · (n + 1) + (1 + . . . + n) = n · (n2 +
n ) + ( 1 + . . . + n ) = ( n2 + n + 1 ) + . . . + ( n2 + n + n ) .
16
Solution 2. We use the formula for the sum of arithmetic progression.
a) If n is odd, then the sum of n consecutive integers is divisible by n.
So, if the number 1 + 2 + . . . + (n + 1) = n+ 1
2 · ( n + 2) was the sum of n
consecutive integers, it would be divisible by n+ 1
2 and by n. As n + 1 and
n are relatively prime, the same clearly holds for n+ 1
2 and n. Therefore,
n +1 n +1
2 · ( n + 2) should be divisible by 2 · n, meaning that n + 2 should be
divisible by n.
If n is even, the sum of n consecutive integers is divisible by n2 . So, if
1 + 2 + . . . + ( n + 1) = ( n + 1) · n+ 2
2 was the sum of n consecutive integers,
it would be divisible by both n + 1 and n2 . As n + 1 and n are relatively
prime, also n + 1 and n2 are relatively prime. Thus, (n + 1) · n+ 2
2 should be
n
divisible by (n + 1) · 2 , implying that n + 2 is divisible by n.
So, in all cases n + 2 is divisible by n, which is equivalent to saying 2 is
divisible by n. So, n = 1 or n = 2. Clearly 1 + 2 is the sum of one integer,
but 1 + 2 + 3 = 6, being an even number, cannot be represented as the sum
of two consecutive integers.
b) If a is the first of the two consecutive integers, then the problem can be
represented as the equation ( a + a + n)(n + 1)/2 = ( a + n + 1 + a + 2n)n/2.
By simplifying we see that it is equivalent to a = n2 . This means that the
sum of n + 1 consecutive integers, first of which is n2 , is the sum of the next
n consecutive integers. So, the desired numbers exist for every n.
F12. (Grade 11.) The sides AB and AC of the triangle ABC touch the circle
c respectively at points B′ and C ′ . The center L of the circle c lies on the
side BC. The circumcenter O of triangle ABC lies on the shorter arc B′ C ′ of
the circle c. Prove that the circumcircle of ABC and the circle c meet at two
points.
Solution. Let r be the circumradius of ABC,
let s be the radius of c and α = ∠ BAC (Fig. 15). C
By tangency, | AB′ | = | AC ′ |. Thus ∠C ′ B′ A = C ′ c
∠B′ C ′ A = π2 − α2 whence, by property of in-
L
scribed angle, ∠ B′ OC ′ = π − π2 − α2 = π2 + α2 .
′ ′
Clearly ∠ B OC > ∠ BOC = 2α, leading to 2 + π
α O
α π A
2 > 2α. Hence α < 3 . Now let K be the mid- B′
B
point of side BC. From the right triangle KOC,
one gets |KO| = |OC | cos ∠KOC = r cos α. By
the inequality obtained above, cos α > cos π3 = Fig. 15
1
2 . On the other hand, | KO | ≤ | LO | = s, leading
to 12 r < r cos α = |KO| ≤ s or r < 2s. As c passes through the circumcenter
of ABC, this inequality shows that these circles must intersect.
Remark. This problem, proposed by Estonia, appeared in the IMO 2011
shortlist as G1.
17
F13. (Grade 11.) A finite grid is covered with 1 × 2 cards in such a way that
the edges of the cards match with the lines of the grid, no card lies over the
edge of the grid, and every square is covered by exactly two cards. Prove
that one can remove some of the cards in such a way that every square will
be covered by exactly one card.
Solution. Choose any square covered by two cards, and choose one of
these cards. Move that card to a neighbouring square, and choose the other
card that is covering that square. From there we move to the next square,
etc., until we return to the first square. We cannot return to any other square
visited previously, since in all squares except the first one, both cards have
been chosen already. If we color the rectangular grid like a chessboard, then
after an odd number of moves, we reach a square with the opposite color,
and after an even number of moves, we reach a square with the same color.
Therefore, the number of chosen cards is even. So, we can remove every
second chosen card. All of the remaining squares we passed through will
be covered by exactly one card. If after this, there are still squares that are
covered by two cards, we repeat the process with a new randomly chosen
square which is covered by two cards. We can never move from a square
covered by two cards to a square covered by exactly one card, since all the
squares covered by exactly one card were previously connected to squares
now covered by exactly one card. So, after a finite number of steps we can
find a new cycle, from which we can remove every second card. We repeat,
until all squares are covered by exactly one card.
F14. (Grade 11.) There are 2012 points marked in a square with side length
11. Prove that one can choose an equilateral triangle with side length 12
which covers at least 671 points.
Solution. Place two equilateral triangles with side
lengths 12 on the square in such a way that both have
one vertex lie on the side of the square and the oppo-
site sides of these vertices partially coincide with the
other side of the square and with each other (Fig. 16).
The area common to both triangles forms an equi- Fig. 16
lateral triangle of side length 1. Position the third
equilateral triangle with side length 12 between the two triangles, turned
180◦ , such that the lowermost vertex of that triangle coincides with the up-
permost vertex of the small triangle. To show that the square is fully cov-
ered by these triangles, we must show that the sum of the heights of the
large
√
and the small triangle is at least 11, which is equaivalent to showing
3
√
2 · (12 + 1) > 11. As we can simplify this equation to 13 3 > 22 and
3 · 169 > 484, we see that it holds. Therefore at least a third of the 2012
points or at least 671 points lie in one of the three chosen triangles.
18
F15. (Grade 12.) Find all pairs ( x, y) of positive integers such that
1 249 1 1
+ + 2 = .
x2 xy y 2012
Answer: (503, 1006), (1006, 503).
Solution 1. Let gcd ( x, y) = d and x = ad, y = bd. Then the equation can
2 + b2
be written as a +a249ab
2 b2 d 2
1
= 2012 or
a2 b2 d2 = 2012( a2 + 249ab + b2 ).
As a and b are relatively prime, a2 and b2 are both relatively prime to a2 +
249ab + b2 and therefore both they must be divisors of 2012. As 2012 = 22 ·
503 and 503 is a prime, the possible cases are ( a, b) = (1, 1), ( a, b) = (1, 2),
( a, b) = (2, 1). If we substitute ( a, b) = (1, 1) into the last equation, we
get d2 = 2012 · 251, which is not solvable in integers. The other two cases
give 4d2 = 2012 · 503, from which d = 503. This leads to the solutions
( x, y) = (503, 1006) and ( x, y) = (1006, 503).
Solution 2. Multiplying both of the sides by 2012x 2 y2 , we get
2012x2 + 249 · 2012xy + 2012y2 = x2 y2 .
From the left-hand side we see that both sides of the equation must be di-
visible by 503. As 503 is a prime, one of the numbers x and y must be
divisible by 503. So x2 or y2 is divisible by 5032, giving that both sides
of the equation are divisible by 5032. If x is divisible by 503, the sum-
mands 2012x2 and 249 · 2012xy on the left-hand side are divisible by 5032,
meaning that 2012y2 is divisible by 5032. Therefore y is divisible by 503.
Analogously, we get that if y is divisible by 503, then x is also divisible by
503. Consequently, both x and y are divisible by 503. Denote x = 503a,
y = 503b. Then the equation, after dividing both sides by 5033 , simplyfies
to 4a2 + 996ab + 4b2 = 503a2 b2 . Assume a ≥ b. If b ≥ 2, then 503a2 b2 ≥
503a2 · 2b = 1006a2 b = 4a2 b + 4a2 b + 998a2 b > 4a2 + 4b2 + 996ab, so the
last equation cannot hold. Therefore b = 1. Now we get a quadratic equa-
tion 499a2 − 996a − 4 = 0 with respect to a, whose only positive solution is
a = 2. From here we obtain the solution (1006, 503) to our original equa-
tion. The case b ≥ a is symmetrical and gives the solution (503, 1006).
F16. (Grade 12.) a) Prove that for every real number x the arithmetic mean
√ √
of 1 + sin x and 1 − sin x is equal to one of the following: sin 2x , cos 2x ,
− sin x2 , − cos 2x .
b) Can one leave out one of the four numbers listed in part a) in such a
way that the claim still holds?
Answer: b) no.
Solution 1. a) Denote the arithmetic mean given in the problem by A( x ).
As
x x x x x x 2
1 + sin x = sin2 + cos2 + 2 sin cos = sin + cos ,
2 2 2 2 2 2
19
x x x x x x 2
1 − sin x = sin2 + cos2 − 2 sin cos = sin − cos ,
2 2 2 2 2 2
we get
√ √
1 + sin x + 1 − sin x | sin x2 + cos x2 | + | sin 2x − cos x2 |
A( x ) = = .
2 2
Depending on the signs of the numbers sin x2 + cos x2 and sin x2 − cos x2 , one
of the trigonometric functions in the numerator cancels out and the other
one is doubled, with either a positive or a negative sign. Therefore, A( x ) is
equal to one of the numbers sin x2 , cos 2x , − sin x2 , − cos 2x .
b) Clearly A( x ) = 1, whenever x is one of the numbers 0, π, 2π, 3π.
Nevertheless, each of these four values makes a unique expression among
sin 2x , cos x2 , − sin 2x , − cos x2 evaluate to 1. Therefore, none of these four can
be left out.
Solution 2. Part a) can also be proven as follows. Let A( x ) be the same
as in the first solution. Then
√ √ !2 p
1 + sin x + 1 − sin x 2 + 2 1 − sin2 x 1 + | cos x |
= = ,
2 4 2
q q
1+| cos x | 1+cos x
so that A( x ) = 2 . Therefore, if cos x ≥ 0, then A ( x ) = 2 =
q
± cos x2 ; if cos x < 0, then A( x ) = 1−cos 2
x
= ± sin x2 .
F17. (Grade 12.) In an acute triangle ABC, a point P is chosen such that all
points symmetrical to P with respect to the sides of ABC lie on the circum-
circle of ABC. Prove that P is the orthocenter of ABC.
Solution. Let A′ , B′ , C ′ be points symmetric to
the point P with respect to the sides BC, CA, AB
C A′
(Fig. 17). Then |C ′ A| = | PA| = | B′ A|, giving that
′ ′ B ′
the arcs AC and AB of the circumcircle of the tri-
angle ABC are equal. Since A and C ′ are on the
same half-plane from the line BB′ , and C on the P
′ ′
other one, we have ∠C CA = ∠ B CA = ∠ PCA.
A B
Since P and C ′ are on the same side from the line
AC, the points P, C, C ′ are collinear. Since PC ′ ⊥
AB, we must also have PC ⊥ AB, that is, the point C′
P lies on the height drawn from the vertex C in the
triangle ABC. Analogously we see that P is on the Fig. 17
other two heights.
Remark 1. The converse—the points symmetric to the orthocenter with
respect to the sides of the triangle lie on the circumcircle of the triangle—is
a known result in elementary geometry that can also be used to solve this
problem. Namely, the point P lies on the reflections of arcs AB, BC, CA from
the corresponding lines AB, BC, CA of the circumcircle. By the theorem
20
mentioned, the intersection point of the height lies on the same arcs. But
the circles, whose arcs are the reflections, already meet twice pairwise at
the points A, B, C. Therefore, they cannot have two common intersection
points.
Remark 2. The claim of the problem, as well as the theorem given in
Remark 1, hold for all triangles, not just acute ones.
F18. (Grade 12.) There are 2n soldiers standing in a line, where n is a posi-
tive integer. The soldiers can rearrange themselves into a new line only in
the following way: the soldiers standing at odd numbered positions move
to the front of the row, keeping their positions with respect to each other,
and the soldiers previously standing at even numbered positions move to
the end of the row, keeping their positions with respect to each other. Prove
that after n rearrangements the soldiers stand in the same ordering as in the
beginning.
Solution 1. The last soldier does not change its position. The rest of the
soldiers regroup just as in the case, when the last soldier was not there, and
the number of the soldiers was 2n − 1. So, it suffices to prove the claim
for 2n − 1 soldiers. We show that after n rearrangements the soldiers are
in positions, which can be found in the original line by counting cyclically
every 2i -th soldier (after the last soldier we go to the first one). Indeed,
after 0 rearrangements, the claim clearly holds, and every rearrangement
makes us cyclically count every second soldier in the previous line (after the
last soldier we go to the second one), the first soldier will still be counted
first. After n rearrangements the soldiers in the new line can be found by
counting every 2n -th soldier in the old line with 2n − 1 soldiers. Since the
remainder of 2n when divided by 2n − 1 is 1, this is equivalent to simply
counting the soldiers. This means that we get back the original line.
Solution 2. Enumerate the soldiers starting from 0, and write the num-
bers in binary form (adding leading zeros to make the lengths of the binary
codes equal; for example for n = 3 we have the numbers 000, 001, 010, 011,
100, 101, 110, 111). After a rearrangement the soldiers stand in such a way
that when reinterpreting the last digit as the first one (but leaving the order
of the rest of the digits the same), the soldiers are again enumerated by con-
secutive numbers. After n rearrangements the binary code of the soldiers
has returned to the original, so every soldier’s position corresponds to their
original position in the line.
F19. (Grade 12.) a) Does there exist a function from real numbers to real
numbers, which is not constantly zero and whose derivative’s graph can be
obtained by reflecting the graph of the original function with respect to the
y-axis?
b) Does there exist a function from real numbers to real numbers, which
is not constantly zero and whose derivative’s graph can be obtained by
21
shifting the graph of the original function towards the positive side of the
x-axis by one unit?
Answer: a) yes; b) yes.
Solution. a) What the problem asks is equivalent to finding a function f
such that f ′ ( x ) = f (− x ) for all real numbers x. This is so, for example, for
the function f ( x ) = sin x + cos x, since f ′ ( x ) = cos x − sin x = cos(− x ) +
sin(− x ) = f (− x ).
b) The premise about the graphs is equivalent to saying that for every
real number x, we have f ′ ( x ) = f ( x − 1). Assume that we have a > 1
such that ln a = a−1 . Then, defining f ( x ) = a x , we get f ′ ( x ) = a x ln a =
a x · a−1 = a x −1 = f ( x − 1). It remains to make sure that such a number
a > 1 exists. Since ln 1 = 0 < 1 = 1−1 and ln e = 1 > e−1 , the graphs of the
continuous functions g( x ) = ln x and h( x ) = x −1 intersect at some point
a > 1. This is the number we were looking for.
Remark. One can prove that in part a) precisely all functions of the
form f ( x ) = c · (sin x + cos x ), where c 6= 0, satisfy the√
premises. The an-
2
swers can be written in a different form, for example 2 · (sin x + cos x ) =
sin x + π4 .
22
x3 , or 2y2 = x 2 ( x − 1). Let d = gcd( x, y) and x = dn, y = dm. Then
2m2 = n2 (dn − 1). If n had a nontrivial prime divisor, then it must also
divide m, a contradiction. Hence n = 1 and the equation is 2m2 = d − 1, or
d = 2m2 + 1. By choosing m freely we get the triples above.
Remark 2. One can also solve the problem by first showing that there
exist infinitely many quadruples (−m, m, −n, n + 1) with 2m2 = n2 + n that
satisfy the conditions of the problem.
S2. For a given positive integer n one has to choose positive integers a0 ,
a1 , . . . so that the following conditions hold:
(1) ai = ai+n for any i;
(2) ai is not divisible by n for any i;
(3) ai+ ai is divisible by ai for any i.
For which positive integers n > 1 is this possible only if the numbers a0 , a1 ,
. . . are all equal?
Answer: for all primes.
Solution. Let n be a prime. By condition (1) the sequence a0 , a1 , . . . con-
tains only finitely many different numbers. If am is maximal of them, then
by condition (3) am+ am must also be maximal. Let us prove that if am is
maximal of the numbers, then am+k· am is also maximal for any k ≥ 0. This
holds for k = 0. If the claim holds for k, then am+( k+1)· am = am+k· am + am =
am+k· am + am+k · am = am+k· am = am . This proves the claim. By condition (2) am
is not divisible by n. Since n is prime, the numbers am and n are relatively
prime. Hence among the numbers m + k · am , where 0 ≤ k < n, there is one
in each congruence class modulo n. Hence all members of the sequence are
maximal, i.e. they are equal.
Suppose n is a composite number; let m be its divisor with 1 < m < n.
For any k < m choose ak = m + k · n and continue the sequence with period
m. Condition (1) holds, since n is a multiple of m. Condition (2) holds,
since all members of the sequence are congruent to m modulo n. For the
condition (3) notice that all members of the sequence are divisible by m.
Hence i and i + ai are always congruent modulo m, therefore ai = ai+ ai . At
the same time not all the numbers are equal.
23
A
A
D X
B′
H D
K K′
L K′
P K P L
A′
C C
B
B Y
Fig. 18 Fig. 19
B′ lie on a line parallel to CD. Denote this line by A′ B′ (even in the case
A′ = B′ = K).
Let d( X, l ) be the distance of point X from line l, and let S∆ be the area of
triangle ∆. By two angles, △ ACK ∼ △ BDK and △ PAD ∼ △ PBC, whence
| AK |
| BK |
= || BD
AC |
|
| AP | | AD |
and | BP| = | BC| . At the same time
d( A, CD ) S△ ACD | AC | · | AD | · sin ∠CAD | AC | | AD |
= = = · ,
d( B, CD ) S△ BCD | BD | · | BC | · sin ∠CBD | BD | | BC |
d( A, KP) S△ AKP | AK | · | AP| · sin ∠KAP | AK | | AP|
= = = · .
d( B, KP) S△ BKP | BK | · | BP| · sin ∠KBP | BK | | BP|
Therefore
| AL| d( A, KP) d( A, CD )
= = .
| LB| d( B, KP) d( B, CD )
Considering instead of the cyclic quadrilateral ABCD the quadrilateral de-
termined by points A, B, A′ , B′ , and instead of P and L the points H and
K ′ correspondingly, we get similarly that
| AK ′ | d( A, KH ) d( A, A′ B′ )
= = .
|K ′ B| d( B, KH ) d( B, A′ B′ )
This equality holds also in the special case A′ = B′ = K. Indeed, let the
projections of points A and B to the line A′ B′ be X and Y correspondingly
(Fig. 19), then ∠ AKX = ∠KDC = ∠KBA = ∠ AKK ′ , ∠ BKY = ∠KCD =
∠KAB = ∠BKK ′ , whence △ AKX ∼ = △ AKK ′ and △ BKY ∼ = △ BKK ′ . It
′ ′ | AK ′ | d ( A,A ′ B ′ )
follows that | AK | = | AX |, | BK | = | BY | and |K ′ B| = d( B,A′ B′ ) .
24
Since C and D lie on the shorter arc AB, we A
have ∠ BCA = ∠ BDA > π2 . Thus the line A′ B′ α
is farther from the points A and B than the line δ
CD. Since | AD | > | BC |, we have ∠ ABD > D
∠CAB and also ∠KBA > ∠KAB, which implies β
|KA| > |KB|. Hence d( A, CD ) + d(K, CD ) >
d( B, CD ) + d(K, CD ), or d( A, CD ) > d( B, CD ). K
P ξ L
All together
| AL| d( A, CD ) d( A, A′ B′ ) | AK ′ | α
= > ′ ′
= ′ . δ
| LB| d( B, CD ) d( B, A B ) |K B| C β
Hence L lies farther from A than K′
on the seg- B
ment AB, therefore ∠ ALK < ∠ AK ′ K = π2 , i.e.
∠ ALK is acute. Fig. 20
Solution 2. Denote ∠KAB = ∠KCD = α, ∠KBA = ∠KDC = β, ∠KAC =
∠KBD = δ and ∠ ALK = ξ (Fig. 20). Then ∠KDB = α + β − δ = ∠KCA.
The condition | AD | > | BC | is equivalent to β > α, and points C and D
being located in the shorter arc AB is equivalent to the inequality α + β −
|KD | sin α
δ < π2 . In triangle KCD we get |KC| = sin β . From triangles KDP and
|KP |
KCP we obtain sin( α+ β− δ )
= sin(ξ|−( KD |
,
|KP |
β− δ )) sin( α+ β− δ )
= sin(ξ|+(
KC |
α− δ ))
, respec-
|KD | sin( ξ −( β− δ )) |KD |
tively. Consequently, |KC| = sin( ξ +( α−δ)) . Expressions of |KC| together yield
sin α sin( ξ −( β− δ )) sin ξ cos( β− δ )−cos ξ sin( β− δ )
sin β = sin( ξ +( α− δ )) = sin ξ cos ( α− δ )+cos ξ sin( α− δ ) , that in turn implies
Second day
S4. Let ABC be a triangle where | AB| = | AC |. Points P and Q are differ-
ent from the vertices of the triangle and lie on the sides AB and AC, respec-
tively. Prove that the circumcircle of the triangle APQ passes through the
circumcenter of ABC if and only if | AP| = |CQ|.
25
Solution. Without loss of generality, we A
can assume that | AP| ≤ | AQ|. Let O be
the circumcenter of ABC. Let R be the in- P
tersection point of the bisector of ∠ BAC
with the circumcircle of the triangle PAQ
— we then have | RB| = | RC | (Fig. 21). R Q
Also, ∠ APR = 180◦ − ∠ AQR = ∠CQR O
and | RP| = | RQ| (since ∠ RAP = ∠ RAQ). B C
So, | AP| = |CQ| ⇐⇒ △ APR ∼ = △CQR
⇐⇒ | RA| = | RC | ⇐⇒ R = O (where Fig. 21
∼
| RA| = | RC | ⇒ △ APR = △CQR by two sides and obtuse angle).
Remark. This problem has been taken from the booklet “The Coins of
Harpland and 20+10 other maths problems from Ireland” (edited by Bernd
Kreussler), its author is Jim Leahy. The solution here is new.
S5. Let x, y, z be positive real numbers whose sum is 2012. Find the max-
imum value of
( x2 + y2 + z2 )( x3 + y3 + z3 )
.
( x 4 + y 4 + z4 )
Answer: 2012.
( x 2 +y2 +z2 )( x 3 +y3 +z3 )
Solution. If x = y = z = 2012 3 , then ( x 4 + y 4 + z4 )
= 2012. Now we
prove that for all x, y, z satisfying the premises we have
( x2 + y2 + z2 )( x3 + y3 + z3 )
≤ 2012.
( x 4 + y 4 + z4 )
It suffices to show that ( x2 + y2 + z2 )( x 3 + y3 + z3 ) ≤ 2012( x4 + y4 + z4 ), or
( x2 + y2 + z2 )( x3 + y3 + z3 ) ≤ ( x + y + z)( x4 + y4 + z4 ). Multiplying out,
simplifying and rearranging the terms gives xy( x − y)( x2 − y2 ) + xz( x −
z)( x2 − z2 ) + yz(y − z)(y2 − z2 ) ≥ 0. Since the differences in the brackets
in every product have equal signs, the products are non-negative, showing
that the necessary inequality holds.
Remark. The inequality that we get after multiplying out and canceling
x5 , y5 , z5 , is a special case of Muirhead inequality (with exponent vectors
(4, 1, 0) and (3, 2, 0)).
S6. On an m × m board, at the midpoints of the unit squares there are some
ants. At the time 0 each ant starts moving with speed 1 parallel to some
edge of the board until it meets an ant moving in the opposite direction
or until it reaches the edge of the board. When two ants moving in the
opposite direction meet each other, both turn 90◦ clockwise and continue
moving parallel to another edge of the board. Upon reaching the edge of
the board the ant falls off the board.
a) Prove that eventually all the ants will have fallen off the board.
b) Find the latest possible moment for the last ant to fall off the board.
26
Answer: b) 32 m − 1.
Solution 1. Let the lower left corner of the board be the origin. Divide
the units of time and space by 2; then the squares are of dimensions 2 × 2,
the coordinates of the midpoints of the squares are odd positive integers,
and the speed of the ants is still 1.
We prove by induction that at integer time moments the coordinates of
the ants are integers and the sum of the coordinates for any fixed ant has
the same parity as the time moment. In addition, the ants can meet only at
integer time moments. At time t = 0 all coordinates of the ants are odd,
so their sum is even. Suppose that at an integer time moment t = k the
coordinates of the ants are integers and the sum of the coordinates for any
fixed ant has the same parity as the time moment. If two of the ants were
to meet each other within the next time unit, they have to move toward
each other from time t = k, hence one of their coordinates must be the
same. Since the parity of the sum of their coordinates was the same at time
t = k, another of their coordinates had to differ by at least 2. Hence they
cannot meet before time t = k + 1. Between time moments t = k and
t = k + 1 every ant has changed only one of its coordinates by 1, hence at
time t = k + 1 the parity of the sum of the coordinates is again the same as
the parity of the time moment.
Next we will prove by induction that for any point with integer coor-
dinates ( x, y) there are no collisions at this point after the time moment
t = x + y − 2. For x = y = 1 this is obviously true, since there are no colli-
sions in the middle of the lower left square (otherwise one of the ants has to
arrive to this point from the edge of the board). Let ( x, y) be arbitrary and
suppose that the claim holds for all points with the sum of the coordinates
less than x + y. Suppose that a collision takes place at point ( x, y) at time t.
One of the participants had to arrive from a point, where one of the coordi-
nates was smaller; w.l.o.g. we can assume that this was the x-coordinate. If
this ant has not collided with anyone before, then t ≤ x − 1 ≤ x + y − 2. If
the last collision of this ant occurred at time t′ < t, then the coordinates
of the last collision were ( x − (t − t′ ), y). By the induction assumption
t′ ≤ x − (t − t′ ) + y − 2, hence t ≤ x + y − 2.
By symmetry the claim holds when another corner is chosen as the ori-
gin. Let the last collision of a particular ant occur at the point ( x, y), where
the coordinates are taken with respect to the nearest corner. W.l.o.g., we
can assume x ≤ y. The time from the last collision to the falling off the
edge of the ants participating in the collision is at most 2m − x, hence the
time elapsed from the start is at most x + y − 2 + 2m − x ≤ 3m − 2. By this
time all ants have fallen off the edge. With respect to the original units the
maximal time is 32 m − 1.
For any m the maximal time can be achieved, if in the beginning there
are 2 ants at the adjoining corners of the board moving toward each other.
At the moment t = m− 1
2 the pair collides and one of the ants starts moving
27
toward the center, falling off the board at time t = 32 m − 1.
Solution 2. Part a) can also be solved as follows. For each ant consider
the distance to the edge in the direction of its motion. After an ant falls
this distance will remain 0. Observe that as long as an ant moves without
collision, this distance decreases with speed 1.
Consider now the sum of all such distances. When a collision happens,
the sum of the distances of the two corresponding ants is m, both right
before and right after the collision. Thus as long as there are ants left on
the board, the total sum decreases with the speed of at least 1. Since in the
beginning this sum is a finite number, after some time this sum will become
0 and thus all ants will have fallen off the board.
Remark. This problem, proposed by Estonia, appeared in the IMO 2011
shortlist as C5.
28
Estonian Math Competitions
2012/2013
Tartu 2013
WE THANK:
University of Tartu
Problem authors: Juhan Aru, Maksim Ivanov, Kairi Kangro, Urve Kangro,
Oleg Koshik, Toomas Krips, Härmel Nestra, Uve Nummert, Erik Paemurru
Translators: Urve Kangro, Härmel Nestra
Editor: Härmel Nestra
*
This booklet contains problems that occurred in the open contests, the final round of
national olympiad and the team selection contest. For the open contests and the final
round, selection has been made to include only problems that have not been taken from
other competitions or problem sources and seem to be interesting enough. The team
selection contest is presented entirely.
1
Selected Problems from Open Contests
1 1 1
O-1. (Juniors.) Nonzero integers a, b and c satisfy + + = 0. Prove that among a,
a b c
b, c there are two integers which have a common divisor larger than 1.
Solution: Multiplying the given equation by abc we get bc + ca + ab = 0. If a, b, c were
all odd, then bc, ca and ab were also odd and their sum could not be 0. If one of the
numbers a, b, c was even and the others were odd, then two of the numbers bc, ca and
ab were even and one odd, which also would not add up to 0. Hence at least two of the
numbers a, b, c are even, which satisfy the conditions.
O-2. (Juniors.) Teacher tells Jüri two nonzero integers a and b such that b is divisible by
a. Jüri has to find a nonzero integer c such that c is divisible by b and all solutions of the
quadratic equation ax2 + bx + c = 0 are integers. Can Jüri always solve the problem?
Answer: Yes.
Solution: By the conditions of the problem there is an integer q such that b = aq. Let
c = −2aq2 ; then c 6= 0 and c is divisible by b. The quadratic equation ax2 + bx + c = 0
or ax2 + aqx − 2aq2 = 0 has solutions q and −2q.
O-3. (Juniors.) Inside a circle c with the center O there are two circles c1 and c2 which
go through O and are tangent to the circle c at points A and B crespectively. Prove that
the circles c1 and c2 have a common point which lies in the segment AB.
Solution: The radius AO of the circle c is perpendicular to the common tangent to cir-
cles c and c1 at the point A, hence AO is a diameter of the circle c1 . Similarly BO is a
diameter of the circle c2 . If the circles c1 and c2 are tangent at the point O (Fig. 1), then
the diameters AO and BO are both perpendicular to the common tangent to c1 and c2 at
the point O, whence the lines AO and BO coincide, i.e. O lies in the segment AB. If the
circles c1 and c2 intersect at O (Fig. 2), then let M be the other intersection point of the
circles. Since ∠ AMO = 90◦ and ∠ BMO = 90◦ (angles at the circumference supported
by a diameter), the lines AM and BM coincide and M lies in the segment AB.
O-4. (Juniors.) Numbers 1, . . . , 200 are written on a blackboard in one line. Juku has to
write in front of each number plus or minus sign so that for any positive integer n ≤ 100
the number itself and one of its multiples have different signs. Which numbers must he
assign a minus sign in order to get the maximal possible value of the expression?
Answer: The numbers 51, . . . , 100.
Solution: If Juku writes a minus in front of the number 51, . . . , 100 and a plus in front
of the others, then the conditions of the problem are satisfied: for 51 ≤ n ≤ 100, the
numbers n and 2n have different signs; for n ≤ 50 there is at least one multiple of n
among the numbers 51, . . . , 100.
To show that this arrangement of the signs gives the maximal value of the expression,
2
B
B
M
N
O
B A A
O P
M
A K L C
1 1
O-5. (Juniors.) Kärt writes the fractions and on the blackboard and Märt writes
2 3
10 positive integers on the paper, which he does not show to Kärt. Then Kärt starts to
write fractions on the blackboard by the following rule: on each step she chooses two
a c
fractions and which are already on the blackboard and writes on the blackboard
b d
a+c
the fraction after reducing. Can Kärt always choose the fractions so that after a
b+d
number of steps she writes on the blackboard a fraction whose denominator is coprime
with all the numbers Märt has written on the paper?
Answer: Yes.
2
Solution: The first fraction that Kärt adds to the blackboard has to be . On every fol-
5
1
lowing move, let Kärt pick as one fraction and the latest written fraction as the other
2
fraction. Ignoring the reducing step, this means that the denominator of every added
fraction is larger than the previous fraction by 2, or that the denominators of the frac-
tions are consecutive odd numbers. This is indeed the case, because all fractions added
k
in this way are irreducible (these fractions have the form , and k is always coprime
2k + 1
with 2k + 1 because any common divisor would also divide (2k + 1) − 2k = 1).
Therefore the denominators of the fractions that Kärt writes include all prime numbers
except 2. Since there are infinitely many prime numbers, Kärt will eventually write a
3
fraction with a prime denominator that is larger than all of the numbers written by Märt,
and hence coprime with them.
O-6. (Juniors.) Publisher Soothsayer published a reference book claiming that for each
real number x and positive even number n the equality (1 + x )n ≥ 2n x holds. Is this
claim true?
Answer: No.
1
Solution: The inequality does not hold for example when x = and n = 4.
2
Remark: This is a plausible mistake, because a similar inequality (1 + x )n ≥ 2nx holds
for any real number x and positive even integer n.
O-7. (Juniors.) In an isosceles right triangle ABC the right angle is at vertex C. On the
side AC points K, L and on the side BC points M, N are chosen so that they divide the
corresponding side into three equal segments. Prove that there is exactly one point P
inside the triangle ABC such that ∠KPL = ∠ MPN = 45◦ .
Solution: Without loss of generality let the points on the side AC be in the order A, K,
L, C and on the side BC in the order C, M, N, B (see Fig. 3). Choose the point P so that the
quadrilateral LCMP is a square. Then |KL| = | LC | = | LP| and | MN | = |CM | = | MP|,
i.e. KLP and PMN are isosceles right triangles, so ∠KPL = ∠ MPN = 45◦ . Since
∠KPN = 45◦ + 90◦ + 45◦ = 180◦ , the point P lies inside the segment KN, whose all
points except the endpoints are inside the triangle ABC.
To show that P is the only point with the required properties, let P0 be an arbitrary point
inside the triangle ABC which satisfies ∠KP0 L = ∠ MP0 N = 45◦ . Since P and P0 are on
the same side of the line KL and ∠KPL = ∠KP0 L, the point P0 lies on the circumcircle
of the triangle KPL; similarly it also lies on the circumcircle of the triangle MPN. Since
∠KLP = ∠ PMN = 90◦ , the segments KP and PN are the diameters of the circles.
Since the diameters KP and PN lie on the same straight line KN, they have a common
perpendicular at the point P which is tangent to both circles at this point. Hence the
point P is the only common point of these circles, i.e. P0 = P.
O-8. (Juniors.) The numbers 1, 2, . . . , 2012 are written on the blackboard in some
order, each of them exactly once. Between each two neighboring numbers the absolute
value of their difference is written and the original numbers are erased. This is repeated
until only one number is left on the blackboard. What is the largest possible number
that can be left on the blackboard?
Answer: 2010.
Solution: The largest number on the blackboard cannot increase on any step, because the
absolute value of the difference of two nonnegative numbers cannot be greater than the
maximum of these two numbers. Since in the beginning all the numbers are different
and positive, after the first step the largest possible number is 2011 and the smallest
possible number is 1. After the second step the largest possible number is 2010 and
hence the number left on the blackboard in the end cannot be larger than 2010.
The number 2010 can be left on the blackboard, for example when in the begin-
ning the numbers are written in the order 2012, 1, 2, 3, . . . , 2011. Then after the first
4
step there are the numbers 2011, 1, 1, . . . , 1, and after the second step the numbers
2010, 0, 0, . . . , 0. On each following step the number of zeroes decreases by one and
in the end only the number 2010 remains.
O-10. (Seniors.) Find all positive integers which are exactly 2013 times bigger than the
sum of their digits.
Answer: 36234.
Solution: Note that the minimal value of a k-digit number is 10k−1 and the maximal value
of the cross-sum multiplied by 2013 is 9k · 2013. Since 9 · 7 · 2013 = 126819 < 1000000
we can consider only numbers with up to 6 digits. Since then the cross-sum is at most
54, it is enough to consider numbers in the form n · 2013 with 1 ≤ n ≤ 54.
Since 2013 is divisible by 3, n · 2013 and its cross-sum are divisible by 3. Since the cross-
sum must be equal to n, n · 2013 is divisible by 9. But then its cross-sum and hence also n
is divisible by 9. It remains to consider the cases n = 9, 18, . . . , 54 which can be checked
by hand and see that only n = 18 satisfies the conditions.
O-11. (Seniors.) Find all remainders which one can get when dividing by 6 an integer
n which satisfies n3 = m2 + m + 1 for some integer m.
Answer: 1.
Solution: Numbers n and n3 give the same remainder when dividing by 6. Also, m2 +
m + 1 is odd and gives the remainder 0 or 1 when dividing by 3. The only possibility to
get 0 as the remainder is when m = 3k + 1, but then
O-12. (Seniors.) Prove that for any integer n ≥ 3 we have (2n)! < n2n .
5
Solution 1: For n = 3 the claim holds: (2n)! = 6! = 720 and n2n = 36 = 729.
Suppose n ≥ 4. Divide the numbers 2, 3, . . . , 2n − 2 into pairs (k, 2n − k ) with 2 ≤ k ≤
n − 1, leaving n alone. For each pair we have
(2n)! < 1 · n2n−3 · (2n − 1) · (2n) < n2n−3 · (2n)2 = 4n2n−1 ≤ n2n .
Solution 2: For n = 3 the claim holds. Suppose the claim holds for n; to show that it also
(n + 1)2n
holds for n + 1 it is enough to show the inequality (2n + 1)(2n + 2) < 2n
( n + 1)2 .
n
2 2 (n + 1)2n
Since (2n + 1)(2n + 2) < (2n + 2) = 4(n + 1) , it is enough to show that > 4.
n n2n
1
This is equivalent with 1 + > 2 which holds for all n ≥ 2.
n
O-13. (Seniors.) Inside a circle c there are circles c1 , c2 and c3 which are tangent to
c at points A, B and C correspondingly, which are all different. Circles c2 and c3 have
a common point K in the segment BC, circles c3 and c1 have a common point L in the
segment CA, and circles c1 and c2 have a common point M in the segment AB. Prove
that the circles c1 , c2 and c3 intersect in the center of the circle c.
Solution: Take a point X on the common tangent to the circles c1 and c which lies on the
other side of the line AB from the point C. Then ∠ ALM = ∠XAM = ∠XAB = ∠ ACB
| AM|
(Fig. 4). Consequently ML k BC. Similarly KM k CA and LK k AB. If = λ, then
| AB|
| BK | | BM| |CL| |CK | | AM|
= = 1 − λ and = = 1 − (1 − λ) = λ, whence λ = =
| BC | | BA| |CA| |CB| | AB|
| AL| 1
= 1 − λ. Hence λ = , therefore the triangles AML, MBK and LKC are all similar
| AC | 2
1
to ABC with the factor . Thus the radii of their circumcircles c1 , c2 and c3 are equal to
2
half of the radius of the circumcircle c of the triangle ABC. Since the circles c and c1 are
tangent, the diameter of c1 and the radius of c, both drawn from the tangent point A,
coincide. Hence the circle c1 goes through the center of the circle c; similarly the circles
c2 and c3 go through the center of the circle c.
O-14. (Seniors.) For which positive integers m and n is it possible to write the numbers
1, 2, . . . , 2mn into the white squares of a 2m × 2n checkerboard in such a way that the
sum of the numbers in every row is the same, and the sum of the numbers in every
column is the same?
Answer: For all even m and n, except when m = n = 2.
Solution: All the numbers sum up to mn(2mn + 1). For odd m this is not divisible by 2n,
breaking the equality of all column sums. Thus m and likewise also n cannot be odd.
In the case m = n = 2 we cannot write the numbers as required, because the numbers
6
B
X
M
K
A
a ∗
L
∗
C
Figure 4 Figure 5
O-15. (Seniors.) Let a and b be positive integers such that b is divisible by a and writing
b
a and b one after another in this order gives ( a + b)2 . Prove that = 6.
a
Solution: Let n be the number of digits of b and let b = ka. Then by the conditions of the
problem, 10n · a + ka = ( a + ka)2 , or
10n + k
a= . (1)
( k + 1)2
If k were odd, then the numerator on the r.h.s. of (1) would be odd and the denominator
even, so a could not be an integer. Hence k is even.
7
If k = 2 then the cross-sum of 10n + 2 is 3, which is not divisible by (2 + 1)2 = 9. The
case k = 4 also leads to a contradiction, since 10n + 4 ends with 4, hence cannot be
divisible by (4 + 1)2 = 25. Thus k ≥ 6.
In the following we show first that k ≤ 8 and finally that k 6= 8. The assumptions
ka = b ≥ 10n−1 give 10ka ≥ 10n . Equality (1) implies
(8 − k) · ka ≥ a − k . (2)
As a is positive, (8 − k ) · ka > −k. As both sides of this inequality are divisible by k, this
implies (8 − k ) · ka ≥ 0. Consequently 8 − k ≥ 0, i.e., k ≤ 8.
10n + 8
If k = 8, the inequality (2) implies a ≤ 8 whereas the equality (1) reduces to a = .
81
Hence a ends with digit 8, leaving a = 8 and b = 8 · 8 = 64 as the only possibility. But
864 6= (8 + 64)2 , contradicting the conditions of the problem.
Remark: It is not hard to show that the smallest numbers satisfying the conditions of
1036 + 6
the problem are a = = 20408163265306122448979591836734694 and b = 6a =
49
122448979591836734693877551020408164.
x2 + 4xy + y2
O-16. (Seniors.) Let x and y be different positive integers. Prove that is
x 3 − y3
never an integer.
Solution 1: By symmetry we can assume that x > y. If x − y = 1, then
x2 + 4xy + y2 3xy
3 3
· ( x − y) − 1 = 2
x −y x + xy + y2
8
F2
D c2
F1 F2
F1 Q
A1
A2
Q
O1 c1
O2 D
A2 A1
c1 O1 O2
P
E1 c
c2
B
E2
E1 P
c
B E2
Figure 6 Figure 7
9
As line O1O2 is perpendicular to BD, line O1O2 is also perpendicular to E1 F1 . Thus the
line segment O1O2 entirely lies on the perpendicular bisector of E1 F1 . This means that
the midpoint of line segment O1O2 is equidistant from E1 and F1 .
Altogether, we have shown that these four points lie on a circle with its center at the
midpoint of the line segment O1O2 .
Remark: The chains of equations (3) and (4) hold as given in the situation depicted in
Fig. 6, where F1 and O1 lie at the same side from line A1 E1 . There are other situations
where F1 and O1 lie at different sides from A1 E1 or O1 lies on the line A1 E1 or circle c lies
inside circles c1 and c2 (see Fig. 7). Despite the equations having a little different form,
the final result ∠E2 E1O1 = 90◦ still holds.
O-18. (Seniors.) Eha and Koit play the following game. In the beginning of the game
at each vertex of a square there is an empty box. At each step each player has two
possibilities: either add one stone to an arbitrary box, or to move each box clockwise to
the next vertex of the square.
Koit begins and they make in turns 2012 steps (each player 1006). Then Koit marks one
of the vertices of the square and lets Eha make one more step. Koit wins if after this step
the number of stones in some box is larger than the number of stones in the box at the
vertex Koit marked; otherwise Eha wins. Which player has a winning strategy?
Answer: Eha.
Solution: First show that Eha can guarantee that before Koit’s move the number of stones
in the boxes lying at opposite corners of the square are equal. In the beginning it is true,
since all the boxes are empty. Let before Koit’s move the numbers of stones in the boxes
be ( a, b, a, b). If Koit adds one stone to a box, then Eha can add a stone to the box at the
opposite corner; if Koit moves the boxes cyclically, Eha also moves the boxes cyclically,
so the condition still holds. Hence Eha can make the moves so that after step 2012 the
numbers of the stones in the boxes are ( a, b, a, b). Without loss of generality we can
assume that a ≥ b. If Koit marks a vertex with a box with a stones then Eha adds one
stone to the box and wins. If Koit marks a vertex with a box with b stones, then Eha
moves the boxes cyclically and still wins.
O-19. (Seniors.) Find all functions f from the set of all positive integers to the same set
such that, for all positive integers a1 , . . . , ak with k > 0, the sum a1 + . . . + ak divides the
sum f ( a1 ) + . . . + f ( ak ).
Answer: All functions given by f (n) = an, a ∈ N.
Solution: Suppose that f is a function that satisfies the conditions of the problem. We
claim that f (n) = f (n − 1) + f (1) for all integers n > 1. Indeed, for any integer m > n,
we have m | f (n) + f (m − n) and m | f (n − 1) + f (1) + f (m − n) by conditions of the
problem. Hence the difference f (n) − ( f (n − 1) + f (1)) is also divisible by m. As m was
arbitrary, this implies that f (n) − ( f (n − 1) + f (1)) is divisible by an infinite number of
different integers, i.e., is equal to 0. This completes the proof of the claim.
Easy induction now gives that necessarily f (n) = n f (1). It is straightforward to verify
that all functions of the form f (n) = an satisfy the conditions of the problem.
10
Selected Problems from the Final Round of National
Olympiad
F-1. (Grade 9.) Consider hexagons whose internal angles are all equal.
(i) Prove that for any such hexagon the sum of the lengths of any two neighbouring
sides is equal to the sum of the lengths of their opposite sides.
(ii) Does there exist such a hexagon with side lengths 1, 2, 3, 4, 5 and 6 in some order?
(i) Let the hexagon be ABCDEF. It suffices to show that | AB| + | BC | = | DE| + | EF |.
Let K be the intersection point of rays FA and CB and let L be the intersection
point of rays FE and CD (Fig. 8). The size of every internal angle of the hexagon
is 120◦ , whence triangles KAB and LDE are equilateral. The quadrilateral FKCL
is a parallelogram since its opposite sides are parallel. This implies |KC | = | LF |
or |KB| + | BC | = | LE| + | EF |, which together with |KB| = | AB| and | LE| = | DE|
implies the desired equality | AB| + | BC | = | DE| + | EF |.
(ii) Take a parallelogram with side lengths 7 and 5 and internal angles 60◦ and 120◦ ,
and cut off equilateral triangles with side lengths 1 and 2 at its acute angles. This
gives rise to a hexagon with all internal angles having size 120◦ and side lengths 1,
4, 5, 2, 3, 6 (Fig. 9).
Solution 2:
(i) Note that the external angles of the hexagon have size 60◦ . Any two opposite sides
of the hexagon are parallel, as they are separated by exactly three external angles.
Consider a line s perpendicular to opposite sides CD and FA of the hexagon
ABCDEF (Fig. 10). As all other sides form the same angle 30◦ with line s, the
lengths of these sides are proportional to the lengths of the projections of the sides
to line s. The sum of the lengths of the projections of sides AB and BC is equal
to the distance between the parallel lines CD and FA and the same holds also for
the opposite sides DE and EF. Therefore the sum of the lengths of the projections
of sides AB and BC is equal to that of sides DE and EF, whence the sums of the
lengths of the sides are equal as well.
C D L
5 2
B
4 2 2
E
K 1 1 3
A F 1 6
Figure 8 Figure 9
11
C D
B
s
E
A F
Figure 10 Figure 11
x d d
c c
e e
x
b
b
f b−x f
x a x a
(ii) Figure 11 shows a hexagon in a triangular grid with distance between neighbouring
nodes being 1. The side lengths of the hexagon are 1, 4, 5, 2, 3 and 6.
Solution 3: Consider two types of transformations on hexagons that maintain the prop-
erty that all internal angles are of the same size.
(1) Prolonging two opposite sides by the same quantity x (Fig. 12); this causes the side
lengths to change according to the template
( a, b, c, d, e, f ) ←→ ( a+ x, b, c, d+ x, e, f ) .
(2) Prolonging the two neighbouring sides of one particular side by the same quantity
x (Fig. 13); this causes the side lengths to change according to the template
( a, b, c, d, e, f ) ←→ ( a+ x, b− x, c+ x, d, e, f )
A straightforward check shows that both transformations maintain the desired prop-
erty, no matter of in which direction the transformations are applied.
(i) As the internal angles of all regular hexagons are equal, it suffices to show that
an arbitrary hexagon with all internal angles equal can be turned into a regular
hexagon by a finite sequence of the transformations above.
Indeed, let the side lengths of a given hexagon with all internal angles equal be
( a, b, c, d, e, f ). Assume w.l.o.g. that d ≥ a and f ≥ c. Choose a quantity s such
that s ≥ max( a, b, c); by applying the transformation (1) thrice, we can obtain a
hexagon with three consecutive sides having the same length:
12
By the assumption made above we have d0 ≥ s and f 0 ≥ s. W.l.o.g., assume also
d0 ≤ f 0 . The transformation
(2)
(s, s, s, d0 , e0 , f 0 ) −→ (s, s, s, s, e00 , f 00 )
leads to a hexagon with four consecutive sides of equal length. But this must be
regular since all of its internal angles are equal (Fig. 14).
(ii) Such a hexagon can be obtained from a regular hexagon with side length 1 by the
following transformations:
F-2. (Grade 9.) Two children are playing noughts and crosses with changed rules.
In each move, either of the players may draw into an empty square of a 3 × 3 board
either a nought or a cross according to one’s wish. Moves are made alternately and the
winner is the one after whose move a row, a column or a long diagonal becomes filled
with three similar signs. Is there a player with a winning strategy, and if yes then who?
Answer: Yes, the first player.
Solution: The first player may play the first move into the middle square and, later
on, make the immediately winning move if there is any and play symmetrically to the
opponent’s last move w.r.t. the center of the board otherwise.
Suppose that the opponent wins. As the central square is occupied, the winning move
must be played either into a corner or in the middle of an edge of the board. According
to the first player’s strategy, the position before the winning move was symmetric w.r.t.
the center of the board. Consequently, the square symmetric to the winning move is
empty in the final position, which in turn implies that the three signs of the same type
appear along an edge of the board. Before the winning move, there must already have
been two of these signs present and, by symmetry, similarly also at the opposite edge.
Three of these four signs must already have been there before the last move of the first
player. As two of these three must have been in one line, the first player could win in
her last move, which contradicts the chosen strategy.
Remark: It is also easy to argue by case study.
F-3. (Grade 10.) Can 2013 be represented as the difference of two cubes of integers?
Answer: No.
Solution 1: Suppose that 2013 = x3 − y3 where x and y are integers. Note that
As 2013 is divisible by 3 and so is 3xy( x − y), the difference ( x − y)3 must be divisible
by 3. Thus also x − y is divisible by 3 as 3 is prime. Consequently, 3xy( x − y) is divisible
by 32 and ( x − y)3 is divisible by 33 , whence the sum x3 − y3 is divisible by 32 . But 2013
is not divisible by higher powers of 3. The contradiction shows that 2013 cannot be
represented as the difference of two cubes of integers.
13
A B
A B A B
P P O
D C
D C D C
F-4. (Grade 10.) The bases of trapezoid ABCD are AB and CD, and the intersection
| PA| | PB|
point of its diagonals is P. Prove that if = then the trapezoid is isosceles.
| PD | | PC |
| PA| | PD |
Solution 1: By assumptions, = . As the bases AB and CD are parallel, we
| PB| | PC |
| PA| | PC |
have also = (Fig. 15). Hence | PC | = | PD |. Similarity of triangles APD and
| PB| | PD |
| AD | | PD |
BPC implies = = 1, thus | AD | = | BC | as needed.
| BC | | PC |
Solution 2: By assumptions, triangles APD and BPC are similar. Thus ∠ ADB = ∠ ACB
(Fig. 16), showing that quadrilateral ABCD is cyclic. But if a quadrilateral with parallel
opposite sides has a circumcircle, the bisectors of these sides coincide as they have the
same direction and both pass through the circumcenter of the quadrilateral (Fig. 17). By
symmetry w.r.t. this line, the other pair of opposite sides have equal lengths.
F-5. (Grade 10.) Each unit square in a 5 × 5 table is coloured either blue or yellow.
Prove that there exists a rectangle with sides parallel to the edges of the table, such that
the four unit squares in its corners have the same colour.
Solution 1: Each row contains at least 3 squares with the same colour. Similarly, the
dominating colour must be the same in at least 3 rows. W.l.o.g., suppose that the first 3
rows contain at least 3 blue squares each. If the first two rows contain two blue squares
in the same columns then the desired rectangle exists. Otherwise, each column contains
at least one blue square in these two rows (in Fig. 18, the blue squares are consecutive
w.l.o.g.). Thus in one of the first two rows there are two blue squares that are in the
same columns with the blue squares in the third row. These form the desired figure.
Solution 2: The first row containes three squares of the same colour, say, blue. If in
some of the remaining four rows there are two blue squares aligned to the blue squares
in the first row then the desired rectangle exists. Otherwise, each of these four rows
14
contains two yellow squares aligned with the blue squares in the first row. But two
yellow squares can be placed into three columns in 3 different ways only. Thus there
exist two rows where these two yellow squares are in the same columns. These squares
form the desired figure.
F-6. (Grade 10.) Jüri draws a circle c with radius 3 and n circles with radius 1 on a
paper. Find the minimal n for which he can draw the circles in such a way that it would
not be possible to draw inside the circle with radius 3 any new circles with radius 1
having at most one common point with each of the previously drawn circles.
Answer: 3.
Solution: Let O be the center of circle c. Suppose that circles c1 , c2 , c3 with radius 1
and centers O1 , O2 , O3 , respectively, are placed in such a way that they touch circle c
internally and O1 , O2 , O3 are vertices of an equilateral triangle (coloured dark in Fig. 19).
No new circle can be placed to the same line with two existing circles with radius 1
because the three circles would require free area with length 6 which is possible only
along the diameter of circle c. The other ways of placement are in the middle or to the
other side of the narrower area between two circles (uncoloured in Fig. 19).
The circle with radius 1 and center O touches all three circles, hence the location is
fixed. The circle with radius 1 and center O10 being symmetric to point O w.r.t. line O2O3
touches circles c2 and c3 and has at least one common point also with circle c, as it is
in one line with the circle in the middle and circle c1 . Hence this location is fixed, too.
Note that after moving the circle c2 towards point O the circle will intersect the circle
with center O10 (since ∠OO2O10 < 90◦ ) and also the circle in the middle. Thus if all circles
c1 , c2 , c3 are moved a bit towards point O, chances to add new circles disappear.
Consequently, if n ≥ 3 then Jüri can draw the circles in such a way that new circles
cannot be added in the required fashion. Show now that in the case n ≤ 2 a new
circle can always be added; it suffices to consider the case n = 2. Let c1 and c2 be
the given circles with radius 1 and centers O1 and O2 . W.l.o.g., O1 6= O. Choose AB
as the diameter of c which is perpendicular to O1O (Fig. 20). Consider two circles with
radius 1, touching the circle c at points A and B, respectively. Circle c1 does not preclude
drawing either of them as it is located inside the strip with width 2 surrounding the line
O1O (dark in Fig. 20), where neither of the two circles outreach. Circle c2 can preclude
at most one of the two circles since it cannot outreach to both sides of the strip.
O2 c
O1 O
O1
O O1′
O3
15
F-7. (Grade 11.) Let a finite decimal fraction be given. Juku starts appending digits to
this fraction in such a way that each new digit equals the remainder of the sum of all
digits existing so far in division by 10. (For instance, if the initial fraction is 27.35 then
the digits added to the end are 7, 4, 8 etc.)
Prove that the infinite decimal fraction obtained this way represents a rational number.
Solution: It suffices to show that the infinite decimal fraction is periodic. For that, note
that each new digit except for the first digit is congruent to twice the previous digit
modulo 10. Indeed, let a1 , . . . , ak−1 , ak be the existing at some time moment digits where
ak is already added by Juku. Then the next digit ak+1 satisfies
Hence each new digit is uniquely determined by the last existing digit. As there are only
a finite number of different digits, some digit must be added repeatedly. According to
the fact just proven, all following digits are repeated as well.
Remark: This solution can be reformulated without mentioning the fact that each new
digit except for the first is congruent to twice the previous one. After noting that Juku
must add some digit repeatedly, denote their position numbers in the decimal fraction
by m and n. Hence the digits before the mth digit and the digits before the nth digit sum
up to congruent numbers modulo 10. Adding the mth and the nth digit, respectively,
to the sums maintains the congruence. This means that the next digits are also equal.
Thus the digits start repeating periodically.
F-8. (Grade 11.) Let n > 1 be an integer and a1 , a2 , . . . , an some real numbers, the sum
of which is 0 and the sum of the absolute values of which is 1. Prove that
n−1
| a1 + 2a2 + . . . + nan | ≤ .
2
| a 1 + . . . + a k | = | a k +1 + . . . + a n | ,
| a 1 + . . . + a k | + | a k +1 + . . . + a n | ≤ | a 1 | + . . . + | a k | + | a k +1 | + . . . + | a n | = 1 .
1
Consequently, | ak+1 + . . . + an | ≤ for each k = 1, . . . , n − 1. Now
2
| a1 + 2a2 + . . . + nan |
= |( a1 + . . . + an ) + ( a2 + . . . + an ) + . . . + ( an−1 + an ) + an |
≤ | a 1 + . . . + a n | + | a 2 + . . . + a n | + . . . + | a n −1 + a n | + | a n |
n −1
≤ 0 + 21 + 12 + . . . + 21 + 21 = 2 .
Solution 2. Let A+ and A− be the sum of positive numbers and the sum of negative
numbers, respectively. By the assumptions, A+ + A− = 0 and A+ − A− = 1, implying
1 1
A+ = and A− = − .
2 2
16
B B
β β
C
F α F C
G G′
E E
β D
α D α
A A
Figure 21 Figure 22
By increasing the coefficients of positive terms and decreasing the coefficients of nega-
tive terms, the whole sum can only increase, and by decreasing the coefficients of posi-
tive terms and increasing that of negative terms, the sum can only decrease. Thus
n −1
a1 + 2a2 + . . . + nan ≤ n · A+ + 1 · A− = 2 ,
a1 + 2a2 + . . . + nan ≥ 1 · A+ + n · A− = − n−2
1
.
n−1
Consequently, | a1 + 2a2 + . . . + nan | ≤ .
2
n−1 1
Remark: The bound can be achieved for every n by taking a1 = − , a2 = . . . =
2 2
1
an−1 = 0 and an = .
2
F-9. (Grade 11.) A convex quadrilateral ABCD where ∠ DAB + ∠ ABC < 180◦ is
given on a plane. Let E be a point different from the vertices of the quadrilateral on
the line interval AB such that the circumcircles of triangles AED and BEC intersect
inside the quadrilateral ABCD at point F. Point G is defined so that ∠ DCG = ∠ DAB,
∠CDG = ∠ ABC and triangle CDG is located outside quadrilateral ABCD. Prove that
the points E, F, G are collinear.
Solution: Denote ∠ DAB = α and ∠ ABC = β (Fig. 21). From cyclic quadrilaterals AEFD
and BEFC one obtains
∠ DFE = 180◦ − ∠ DAE = 180◦ − α ,
∠CFE = 180◦ − ∠CBE = 180◦ − β ,
17
These equalities imply that G 0 = G. Thus G belongs to line EF.
Remark 2: The claim of the problem holds also if point F does not have to be inside the
quadrilateral ABCD. Then G may also be located between E and F.
F-10. (Grade 11.) A (2k + 1) × (2k + 1) table, where k is a positive integer, contains
one real number in each entry, where these numbers are pairwise different. After each
row, one writes the median of the row, i.e., the number occurring in this row such that
the row contains the same amount of numbers less than it and greater than it. Let m be
the median of the column of medians. Prove that more than a quarter of the numbers
initially in the table are less than m.
Solution: Each row contains k numbers less than the median and k numbers greater than
the median. Thus k + 1 numbers in each row do not exceed the median of that row. In
rows whose median does not exceed m, these k + 1 numbers do not exceed m either.
There are k + 1 such rows. Consequently, there are at least (k + 1)2 numbers in the table
that do not exceed m. Only one of them is equal to m, whence (k + 1)2 − 1 = k2 + 2k
numbers are less than m. As k is positive by assumption, we have 1 < 4k and 4k + 1 <
k2 + 2k k2 + 2k k2 + 2k 1
4k + 4k = 8k. Now 2
= 2
> 2
= and we are done.
(2k + 1) 4k + 4k + 1 4k + 8k 4
F-11. (Grade 11.) For which natural numbers n ≥ 3 is it possible to cut a regular n-
gon into smaller pieces with regular polygonal shape? (The pieces may have different
number of sides.)
Answer: 3, 4, 6, 12.
Solution: A regular triangle can be partitioned into four regular triangles of equal size
(Fig. 23), a regular quadrilateral can be partitioned into four regular quadrilaterals with
equal size (Fig. 24) and a regular hexagon can be partitioned into six regular triangles
of equal size (Fig. 25). By building alternately equilateral triangles and squares onto the
sides of a regular 12-gon, just a regular hexagon remains (Fig. 26), whence also a regular
12-gon can be partitioned in the required way.
Show now that other regular polygons cannot be partitioned into smaller regular poly-
gons. For that, consider an arbitrary polygon that is partitioned into regular polygons.
As the size of an internal angle of a regular polygon is less than 180◦ and not less than
60◦ , at most two regular polygons can meet at each vertex.
If a vertex of the big n-gon is filled by just one smaller polygon then this piece is an n-
gon itself. Beside it, there must be space for at least one regular polygon. No more than
two regular polygons can be placed there since the sum of the internal angles of these
polygons and the n-gon itself would exceed 180◦ . Two new pieces can be placed only
if all these three pieces are triangular, which gives n = 3. It remains to study the case
where there is exactly one polygon beside the n-gonal piece. The size of the internal
angle of the n-gon being at most 120◦ implies n ≤ 6. The case n = 5 is impossible as its
external angles are of size 72◦ but no regular polygon has internal angles of size strictly
between 60◦ and 90◦ .
If each vertex of the big n-gon is the meetpoint of two smaller regular polygons then one
of them must be a triangle since other regular polygons have internal angles of size 90◦
or more. Beside a triangle, there is space for a triangle, a quadrilateral or a pentagon.
18
Figure 23 Figure 24 Figure 25 Figure 26
60◦
108◦ 108◦
84◦
Figure 27
In the first two cases, the size of the internal angles of the n-gon will be 120◦ and 150◦ ,
respectively, covering the cases n = 6 and n = 12. It remains to show that the third
case with a triangle and a pentagon meeting at each vertex is impossible. Indeed, the
side length of the pentagon must coincide with the side length of the initial big n-gon,
because it is impossible to place a regular polygon beside the pentagon along one side.
For the same reason, another pentagon must be built to the second next side along the
boundary of the initial polygon. These two pentagons meet at the third vertex of the
triangle built to the side between (Fig. 27). But the ulterior angle between the sides of
the pentagons at the meeting point has size 360◦ − 2 · 108◦ − 60◦ = 84◦ , which cannot
be filled with interior angles of regular polygons.
F-12. (Grade 12.) Find the smallest natural number n for which there exist integers
a1 , . . . , an (that do not have to be different) such that a41 + . . . + a4n = 2013.
Answer: 14.
Solution: Note that the fourth powers of even numbers are divisible by 16 and the fourth
powers of odd numbers are congruent to 1 modulo 16. As 2013 ≡ 13 (mod 16), the
desired representation must contain at least 13 odd summands.
Suppose that no more summands are needed. As 74 = 2401 > 2013, each summand
must be 14 = 1, 34 = 81 or 54 = 625. There can be at most 3 summands 625 since
4 · 625 > 2013. Therefore the number of summands not divisible by 5 is at least 10.
The fourth power of an integer not divisible by 5 is congruent to 1 modulo 5, whereas
2013 ≡ 3 (mod 5). Hence the number of summands not divisible by 5 must be at least
13. This shows that the representation contains only summands 1 and 81, but 13 such
numbers sum up to at most 13 · 81 which is less than 2013. Thus representations with
13 summands are impossible.
On the other hand, 14 fourth powers is enough as 64 + 54 + 34 + 11 · 14 = 2013.
Remark: The fact that 2013 cannot be represented as the sum of 13 odd fourth powers
can also be proved without calculations modulo 5. Suppose that
19
where ai = 2bi + 1 for every i = 1, . . . , 13. We may assume that each bi is 0, 1 or 2. As
4 4 3 2 4 3 3 2 1
(2x + 1) = 16x + 32x + 24x + 8x + 1 = 16 x + 2x + x + x + 1
2 2
and
3 1 1 1 1
x4 + 2x3 + x2 + x = x4 + 2x3 + x2 + x2 + x = x2 ( x + 1)2 + x ( x + 1) ,
2 2 2 2 2
2013 − 1 − 1 − . . . − 1
the equality (5) reduces to f (b1 ) + f (b2 ) + . . . + f (b13 ) = = 125
16
1
where f ( x ) = x2 ( x + 1)2 + x ( x + 1). Thus 125 should be representable as the sum of
2
13 integers, each of which is f (0) = 0, f (1) = 5 or f (2) = 39. Obviously the number of
summands 39 is at most 3 and, as 125 − 13 · 5 = 60, at least 2. The cases with two and
three summands 39 give no solution.
F-13. (Grade 12.) Real numbers x1 , x2 , x3 , x4 in [0; 1] are such that the product
K = | x1 − x2 | · | x1 − x3 | · | x1 − x4 | · | x2 − x3 | · | x2 − x4 | · | x3 − x4 |
1 4
is as large as possible. Prove that>K> .
27 243
Solution 1: If some two numbers among x1 , x2 , x3 , x4 are equal then K = 0 which is not
maximal. Thus assume w.l.o.g. that x1 > x2 > x3 > x4 . Applying AM-GM for x1 − x2 ,
x2 − x3 and x3 − x4 gives
( x1 − x2 ) + ( x2 − x3 ) + ( x3 − x4 ) x − x4 1
q
3
( x1 − x2 )( x2 − x3 )( x3 − x4 ) ≤ = 1 ≤ ,
3 3 3
1
i.e., | x1 − x2 | · | x2 − x3 | · | x3 − x4 | ≤
. Among the remaining factors | x1 − x3 |, | x1 − x4 |,
27
| x2 − x4 |, at least one is less than 1. Hence we conclude the left-hand inequality needed.
3 1
For the second inequality, note that if x1 = 1, x2 = , x3 = , x4 = 0 then
4 4
1 3 1 3 1 9 4
K= · ·1· · · = > ,
4 4 2 4 4 512 243
since 9 · 243 = 2187 > 2048 = 4 · 512.
Solution 2: W.l.o.g., assume the inequalities x1 > x2 > x3 > x4 . In addition, assume that
x1 = 1 and x4 = 0 as otherwise K can be made larger. Substituting x2 = y and x3 = z
for simplicity, one obtains
20
case y + z = 1 and the product (1 − z)y is the largest in the same case y + z = 1.
Consequently, also K obtains its largest value in the case y + z = 1. Substituting 1 − z at
place of y, one gets K = z2 (1 − z)2 (1 − 2z) = (z(1 − z))2 (1 − 2z).
Let f (z) = (z(1 − z))2 (1 − 2z), then f 0 (z) = 2z(1 − z)(1 − 5z + 5z2 ). The roots of√f 0
5− 5
within (0, 1) are the roots of the quadratic polynomial 5z2 − 5z + 1, namely
√ √ 10
5− 5
5+ 5 1
and = 1− . As f (0) = f = 0 and f (z) > 0 whenever 0 < z <
10 10 2 √
1 5− 5
, the maximum of f is achieved at z = . Thus the maximum value of K is
2 √ ! √ √ 10
5− 5 5 5
f = . This number satisfies both inequalities of the problem.
10 125 125
(i) Prove that the sum of the radii of circles c1 , c2 and c3 does not exceed one quarter
of the perimeter of the triangle C1 C2 C3 .
(ii) Prove that if the sum of the radii of circles c1 , c2 and c3 equals one quarter of the
perimeter of the triangle C1 C2 C3 then the triangle C1 C2 C3 is equilateral.
Solution: Let the radii of the circles c1 , c2 , c3 be r1 , r2 , r3 , and the radii of the circles k1 , k2 ,
k3 be R1 , R2 , R3 , respectively (Fig. 28). By assumptions,
1
R 1 + R 2 = | K1 K2 | = 2 |C1 C2 | ,
1
R 2 + R 3 = | K2 K3 | = 2 |C2 C3 | ,
1
R 3 + R 1 = | K3 K1 | = 2 |C3 C1 | ,
1
which sum up to 2R1 + 2R2 + 2R3 = (|C1 C2 | + |C2 C3 | + |C3 C1 |). The assumptions
2
also imply inequalities
1 1
r1 + R3 ≤ 2 |C1 C2 | , R3 + r2 ≤ 2 |C1 C2 | ,
1 1
r2 + R1 ≤ 2 |C2 C3 | , R1 + r3 ≤ 2 |C2 C3 | ,
1 1
r3 + R2 ≤ 2 |C3 C1 | , R2 + r1 ≤ 2 |C3 C1 | ,
which sum up to 2r1 + 2r2 + 2r3 + 2R1 + 2R2 + 2R3 ≤ |C1 C2 | + |C2 C3 | + |C3 C1 |.
1
(i) Altogether, we obtain the inequality 2r1 + 2r2 + 2r3 ≤ (|C1 C2 | + |C2 C3 | + |C3 C1 |),
2
1
which implies r1 + r2 + r3 ≤ (|C1 C2 | + |C2 C3 | + |C3 C1 |) as desired.
4
21
C3
c3
2 4 2 4 2 6
6 8 8 6 8 4
K2 K1
k2 k1
Figure 29
c1
c2 2 9 4
C1 K3 C2
7 5 3
k3 6 1 8
Figure 28 Figure 30
1
(ii) Suppose that r1 + r2 + r3 = (|C1 C2 | + |C2 C3 | + |C3 C1 |). Then all inequalities
4
1
above must hold as equalities. The equalities r1 + R3 = |C1 C2 | = r2 + R3 im-
2
ply r1 = r2 , analogously r1 = r3 . Denoting r = r1 = r2 = r3 , we get
1
r + R3 = 2 |C1 C2 | = R1 + R2 ,
1
r + R2 = 2 |C1 C3 | = R1 + R3 ,
F-15. (Grade 12.) Define magic square as a 3 × 3 table where each cell contains one
number from 1 to 9 so that all these numbers are used and all row sums and column
sums are equal. Prove that any two magic squares can be obtained from each other via
the following transformations: interchanging two rows, interchanging two columns,
rotating the square, reflecting the square w.r.t. its diagonal.
Solution: As all the transformations are invertible, it suffices to show that every magic
square can be turned to one particular magic square by these transformations.
The sum of all numbers in a magic square is 45, whence the numbers in each row and
each column must sum up to 15. As this is odd, exactly 0 or 2 of the three summands
must be even. There are 4 even numbers in use, hence 2 even numbers must be in some
two rows and 0 even number in the remaining one. The same holds for columns.
Hence the even numbers 2, 4, 6, 8 occur in the corners of some rectangle with sides
parallel to the edges of the table. By interchanging rows or columns one can move the
even numbers to the corners of the whole table. There are 3 possibilities to locate these
four numbers into the corners, that can not be obtained from each other by rotations and
reflections of the table (Fig. 29). The last two of them cannot occur in the magic square
because the missing numbers in the first and third column would coincide. Hence only
the first possibility remains. Its completion to a magic square is unique (Fig. 30).
22
IMO Team Selection Contest
First day
S-1. Find all prime numbers p for which one can find a positive integer m and non-
negative integers a0 , a1 , . . . , am less than p such that
a0 + a1 + . . . + am−1 + am = 11.
Answer: 2003.
Solution: Subtracting the second equation from the first one gives
a1 ( p − 1) + . . . + am ( pm − 1) = 2002.
1. If p = 2 then m = 10 as 210 < 2013 < 211 . The second equation implies that all
ai s must be ones, but 1 + 2 + 22 + . . . + 210 = 211 − 1 = 2047. Hence there is no
solution in this case.
2. Let p = 3. As 2013 = 2 · 3 + 32 + 2 · 33 + 2 · 35 + 2 · 36 whereas 2 + 1 + 2 + 2 + 2 =
9 6= 11, this case gives no solution either.
3. Let p = 23. As 2013 = 12 + 18 · 23 + 3 · 232 while 12 + 18 + 3 > 11, this case gives
no solution either.
4. For p = 2003, we get 2013 = 10 + 2003 and 10 + 1 = 11, so the conditions are
satisfied.
Consequently, 2003 is the only prime number with the desired property.
23
2π
Ai and Ai+1 for i = 1, . . . , n − 2 is equal to αi = − (n − 2 − i )ε (see Fig. 31 for
n−2
n = 6). The angle between the radii drawn to the points An−1 and A1 is then equal to
(n − 2)(n − 3)
α n −1 = ε. Place the point An outside of the circle on the extension of the
2
radius containing An−1 at the same distance d from An−1 as the distance between points
A1 and A2 . It is straightforward to verify that the points A1 , . . . , An satisfy the condition
of the problem.
On the other hand, suppose that there exists a construction for n = 3. Let the cyclic
walk be A1 → A2 → A3 → A1 . Then d( A1 , A2 ) > d( A1 , A3 ), d( A2 , A3 ) > d( A2 , A1 )
and d( A3 , A1 ) > d( A3 , A2 ), where d( X, Y ) denotes the distance between X and Y. But
these three inequalities cannot hold simultaneously.
S-3. Let x1 , . . . , xn be non-negative real numbers, not all of which are zeros.
(ii) Show that, for each n ≥ 1, both inequalities can hold as equalities.
n
(The last inequality is proved by + k ≤ n + 1, as it is equivalent to (n − k)(k − 1) ≥ 0.)
k
This gives us the necessary upper bound; this bound is achieved for instance if x1 =
xn = 1 and x2 = . . . = xn−1 = 0.
For the lower bound, estimate the numerator by Cauchy-Schwarz inequality:
! ! !2 !2
n n n n
r
x xk p
∑ kk ∑ kxk ≥ ∑ k
· kxk = ∑ xk ;
k =1 k =1 k =1 k =1
24
O C
A3 A2
τ
α2 D β B
α3 α1 d A
α4
α5
A4 A5 A1
A6 Q
Figure 31 Figure 32
Second day
S-4. Let D be the point different from B on the hypotenuse AB of a right triangle ABC
such that |CB| = |CD |. Let O be the circumcenter of triangle ACD. Rays OD and CB
intersect at point P, and the line through point O perpendicular to side AB and ray CD
intersect at point Q. Points A, C, P, Q are concyclic. Does this imply that ACPQ is a
square?
which implies that isosceles triangles BDC and DCO are similar. Thus ∠ BDC = ∠ DCO,
i.e., OC k AB, whence ∠QOC = 90◦ = ∠QAC. So O lies on the circle determined by A,
C, P, Q. Therefore
1 1 1
∠ ACQ = ∠ AOQ = ∠ AOD = ∠ AOP = ∠ ACP.
2 2 2
Consequently, the diagonal of the rectangle ACPQ bisects the angle of the rectangle,
whence ACPQ is a square.
Remark: It turns out from the solution that the conditions of the problem determine the
3
shape of the triangle ABC, namely ∠ ABC = π.
8
25
S-5. Call a tuple (bm , bm+1 , . . . , bn ) of integers perfect if both following conditions are
fulfilled:
Prove that if n − m is large enough then there is no perfect tuples, and find all perfect
tuples with the maximal number of components.
Answer: 20 + 1, 21 + 1, 22 + 1, 23 + 1, 24 + 1 .
S-6. A class consists of 7 boys and 13 girls. During the first three months of the school
year, each boy has communicated with each girl at least once. Prove that there exist two
boys and two girls such that both boys communicated with both girls first time in the
same month.
Solution 1: Call the first communication between a boy and a girl their acquaintance.
During the 3 months, there are 7 · 13 = 91 acquaintances in total. Thus there exists a
month when there was at least 31 aquaintances. Let the boys be denoted by p1 through
p7 and let Ti , i = 1, . . . , 7, be the set of girls to whom pi acquainted in this month. We
have to show that there exist distinct i and j such that Ti ∩ Tj contains at least 2 girls.
W.l.o.g., assume the inequalities | T1 | ≥ | T2 | ≥ . . . ≥ | T7 |. Consider two cases.
| T1 ∪ T2 ∪ T3 ∪ T4 | ≥ | T1 | + | T2 | + | T3 | + | T4 | − k ≥ 20 − 6 = 14
26
2)
2)
2)
∪T
∪T
∪T
T1
T1
T1
1
1
\T
\T
\T
\(
\(
\(
T1
T2
T3
T1
T2
T3
T1
T2
T3
p1 p2 p3 p1 p2 p3 p1 p2 p3
Figure 33
girls in the month under consideration. (For proving the first inequality, note that
girls that belong to one or two subsets count once in the r.h.s., girls belonging to
three subsets not count and girls in all four subsets count −2 times.)
This is a contradiction since there are only 13 girls.
7
Solution 2: Let A be the set of all combinations of two boys, | A| = = 21. Say
2
that girl t determines an element { p1 , p2 } of A if t acquainted with p1 and p2 in the same
month. If in the ithmonth
a girlt acquainted
with exactly ni boys, where i = 1, 2, 3, then
n1 n2 n3
t determines + + elements of A. Applying Jensen’s inequality for
2 2 2
x ( x − 1)
n1 n2 n3 n1 + n2 + n3 7 2
f (x) = gives + + ≥ 3· f = 3· f =4 .
2 2 2 2 3 3 3
n1 n2 n3 n1 n2 n3
As + + is an integer, + + ≥ 5. Hence all girls
2 2 2 2 2 2
determine at least 13 · 5 = 65 elements of A in total.
As 65 ≥ 3 · 21 + 1, an element { p1∗ , p2∗ } of A is determined by at least 4 girls by the
pigeonhole principle. Consequently there exists a month in which this couple of boys is
determined by at least two girls.
Solution 3: Suppose that there is no required pairs of boys and girls. Like in Solution
1, consider a month with 31 or more acquaintances. Let gi be the number of girls who
acquainted with exactly i boys in this month, i = 0, 1, . . . , 7. We get a system of inequal-
27
ities
g2 + 3g3 + 6g4 + 10g5 + 15g6 + 21g7 ≤ 21,
g1 + g2 + g3 + g4 + g5 + g6 + g7 ≤ 13, (6)
g1 + 2g2 + 3g3 + 4g4 + 5g5 + 6g6 + 7g7 ≥ 31.
If at least one girl aquainted with 5 or more boys then g5 + g6 + g7 ≥ 1. The system (6)
then reduces to
g2 + 3g3 + 6g4 + 10( g5 + g6 + g7 − 1) ≤ 11,
g1 + g2 + g3 + g4 + ( g5 + g6 + g7 − 1) ≤ 12,
g1 + 2g2 + 3g3 + 4g4 + 7( g5 + g6 + g7 − 1) ≥ 24.
The first two inequalities sum up to g1 + 2g2 + 4g3 + 7( g4 − 1) ≤ 27. In the light of the
third inequality, this is possible only if g3 = 0 and g4 = 1. Then the second and third
inequalities give g1 + g2 ≤ 12 and g1 + 2g2 ≥ 27, which contradict each other since
g1 + 2g2 ≤ 2( g1 + g2 ).
Hence also g4 = 0 and our system of inequalities reduces to
g2 + 3g3 ≤ 21,
g1 + g2 + g3 ≤ 13,
g1 + 2g2 + 3g3 ≥ 31.
Subtracting the first inequality from the third one, we obtain g1 + g2 ≥ 10. Subtracting
twice the second inequality from the third one, we get g3 − g1 ≥ 5. The two inequalities
obtained sum up to g2 + g3 ≥ 15, contradicting the second inequality.
Remark 1: After decreasing either the number of boys or the number of girls, the claim
of the problem would not hold anymore.
Remark 2: This problem is a variant of the problem F-5 from the Final Round for the 10th
grade. In terms of that problem, here we take 7 × 13 table instead of 5 × 5 and use three
colours instead of two.
28
Problems Listed by Topic
Number theory: O-1, O-5, O-10, O-11, O-15, F-3, F-7, F-12, S-1, S-5
Algebra: O-2, O-6, O-9, O-12, O-16, O-19, F-8, F-13, S-3
Geometry: O-3, O-7, O-13, O-17, F-1, F-4, F-6, F-9, F-14, S-4
Combinatorics: O-4, O-8, O-14, O-18, F-2, F-5, F-10, F-11, F-15, S-2, S-6
Singapore Mathematical
Olympiad Team
Selection Test
And Trainings
1.1.* Let N = {1, 2, 3, ...} be the set of all natural numbers and f : N → N be a function.
Suppose f (1) = 1, f (2n) = f (n) and f (2n + 1) = f (2n) + 1 for all natural numbers
n.
(i) Calculate the maximum value M of f (n) for n ∈ N with 1 ≤ n ≤ 1994.
(ii) Find all n ∈ N, with 1 ≤ n ≤ 1994, such that f (n) = M .
1.2. ABC is a triangle with ∠A > 90o . On the side BC, two distinct points P and Q are
chosen such that ∠BAP = ∠P AQ and BP · CQ = BC · P Q. Calculate the size of
∠P AC.
1.3. In a dance, a group S of 1994 students stand in a big circle. Each student claps the
hands of each of his two neighbours a number of times. For each student x, let f (x) be
the total number of times x claps the hands of his neighbours. As an example, suppose
there are 3 students A, B and C. A claps hand with B two times, B claps hand with
C three times and C claps hand with A five times. Then f (A) = 7, f (B) = 5 and
f (C) = 8.
(i) Prove that {f (x) | x ∈ S} =
6 {n | n is an integer, 2 ≤ n ≤ 1995}.
(ii) Find an example in which
{f (x) | x ∈ S} = {n | n is an integer, n 6= 3, 2 ≤ n ≤ 1996}.
1
2.1. Let f (x) = 1+x where x is a positive real number, and for any positive integer n, let
gn (x) = x + f (x) + f (f (x)) + · · · + f (f (. . . f (x))),
the last term being f composed with itself n times. Prove that
(i) gn (x) > gn (y) if x > y > 0.
(ii) gn (1) = F1
F2 + FF23 + · · · + FFn+1
n+2
, where F1 = F2 = 1 and Fn+2 = Fn+1 + Fn for n ≥ 1.
2.2. Let ABC be an acute-angled triangle. Suppose that the altitude of 4ABC at B
intersects the circle with diameter AC at P and Q, and the altitude at C intersects
the circle with diameter AB at M and N . Prove that P, Q, M and N lie on a circle.
2.3. Show that a path on a rectangular grid which starts at the northwest corner, goes
through each point on the grid exactly once, and ends at the southeast corner divides
the grid into two equal halves: (a) those regions opening north or east; and (b) those
regions opening south or west.
•.................................................•.......................•.................•..................•.................•..................................................•........................•........
......................... ......................... ..
•...........................................................................•...............................................................•...........................................................•...............................................................•.......... •..........................................................................•.............. •.........
......................................................................................... ........................ ...
............................ .......
•...............................•.................................•.................................•.............................................................................•.............. •................................•..... •........
.................. ...
.......
•................................................................•....................................................................•.....................................................................•..................................................................................•............. •...............................................................•.....................................................................•........
............................................................. ..............................
............................. ..............
•...............................•.................................•.................................•...................................•..... •................................•................................•...
(The figure above shows a path meeting the conditions of the problem on a 5 × 8 grid.
The shaded regions are those opening north or east while the rest open south or west.)
1995/96
1.1. Let P be a point on the side AB of a square ABCD and Q a point on the side BC.
Let H be the foot of the perpendicular from B to P C. Suppose that BP = BQ.
Prove that QH is perpendicular to HD.
1.2. For each positive integer k, prove that there is a perfect square of the form n2k − 7,
where n is a positive integer.
1.3. Let S = {0, 1, 2, . . . , 1994}. Let a and b be two positive numbers in S which are
relatively prime. Prove that the elements of S can be arranged into a sequence
s1 , s2 , s3 , . . . , s1995 such that si+1 − si ≡ ±a or ± b (mod 1995) for i = 1, 2, . . . , 1994.
2.1. Let C, B, E be three points on a straight line l in that order. Suppose that A and D
are two points on the same side of l such that
(i) ∠ACE = ∠CDE = 90o and
(ii) CA = CB = CD.
Let F be the point of intersection of the segment AB and the circumcircle of 4ADC.
Prove that F is the incentre of 4CDE.
2.2. Prove that there is a function f from the set of all natural numbers to itself such that
for any natural number n, f (f (n)) = n2 .
2.3. Let S be a sequence n1 , n2 , . . . , n1995 of positive integers such that n1 + · · · + n1995 =
m < 3990. Prove that for each integer q with 1 ≤ q ≤ m, there is a sequence
ni1 , ni2 , . . . , nik , where 1 ≤ i1 < i2 < · · · < ik ≤ 1995, ni1 + · · · + nik = q and k
depends on q.
1996/97
1.1. Let ABC be a triangle and let D, E and F be the midpoints of the sides AB, BC
and CA respectively. Suppose that the angle bisector of ∠BDC meets BC at the
point M and the angle bisector of ∠ADC meets AC at the point N . Let M N and
CD intersect at O and let the line EO meet AC at P and the line F O meet BC at
Q. Prove that CD = P Q.
1.2. Let an be the number of n-digit integers formed by 1, 2 √ and 3 which do not contain
any consecutive 1’s. Prove that an is equal to ( 21 + √13 )( 3 + 1)n rounded off to the
nearest integer.
1.3. Let f : R −→ R be a function from the set R of real numbers to itself. Find all such
functions f satisfying the two properties:
(a) f (x + f (y)) = y + f (x) for all x, y ∈ R,
f (x)
(b) the set : x is a nonzero real number is finite.
x
2.1. Four integers a0 , b0 , c0 , d0 are written on a circle in the clockwise direction. In the first
step, we replace a0 , b0 , c0 , d0 by a1 , b1 , c1 , d1 , where a1 = a0 − b0 , b1 = b0 − c0 , c1 =
c0 − d0 , d1 = d0 − a0 . In the second step, we replace a1 , b1 , c1 , d1 by a2 , b2 , c2 , d2 ,
where a2 = a1 − b1 , b2 = b1 − c1 , c2 = c1 − d1 , d2 = d1 − a1 . In general, at the
kth step, we have numbers ak , bk , ck , dk on the circle where ak = ak−1 − bk−1 , bk =
bk−1 − ck−1 , ck = ck−1 − dk−1 , dk = dk−1 − ak−1 . After 1997 such replacements, we
set a = a1997 , b = b1997 , c = c1997 , d = d1997 . Is it possible that all the numbers
|bc − ad|, |ac − bd|, |ab − cd| are primes ? Justify your answer.
2.2. For any positive integer n, evaluate
b n+1
2
c
X n−i+1
,
i
i=0
m!
m
where = and b n+1
2 c is the greatest integer less than or equal to
k k!(m − k)!
n+1
2 .
2.3. Suppose the numbers a0 , a1 , a2 , . . . , an satisfy the following conditions:
1 1
a0 = , ak+1 = ak + a2k for k = 0, 1, . . . , n − 1.
2 n
1
Prove that 1 − < an < 1.
n
1997/98
Ai ∩ Aj 6= ∅, Ai ∩ Ak 6= ∅, Aj ∩ Ak 6= ∅ imply Ai ∩ Aj ∩ Ak 6= ∅.
2.1. Let I be the centre of the inscribed circle of the non-isosceles triangle ABC, and let
the circle touch the sides BC, CA, AB at the points A1 , B1 , C1 respectively. Prove
that the centres of the circumcircles of 4AIA1 , 4BIB1 and 4CIC1 are collinear.
2.2. Let a1 ≥ · · · ≥ an ≥ an+1 = 0 be a sequence of real numbers. Prove that
v
u n n √
uX X √ √
t ak ≤ k ak − ak+1 .
k=1 k=1
2.3. Let p and q be distinct positive integers. Suppose p2 and q 3 are terms of an infinite
arithmetic progression whose terms are positive integers. Show that the arithmetic
progression contains the sixth power of some integer.
1998/99
x3 − mx2 + mx − (m2 + 1) = 0
2.1. Let M and N be two points on the side BC of a triangle ABC such that BM =
M N = N C. A line parallel to AC meets the segments AB, AM and AN at the
points D, E and F respectively. Prove that EF = 3DE.
2.2. Find all possible values of
x−p −x − 1
+ ,
p p
where x is a real number and p is a nonzero integer.
Here bzc denotes the greatest integer less than or equal to z.
2.3. Let f (x) = x1998 − x199 + x19 + 1. Prove that there is an infinite set of prime numbers,
each dividing at least one of the integers f (1), f (2), f (3), f (4), · · · .
1999/2000
1.1. In a triangle ABC, AB > AC, the external bisector of angle A meets the circumcircle
of triangle ABC at E, and F is the foot of the perpendicular from E onto AB. Prove
that 2AF = AB − AC.
1.2. Find all prime numbers p such that 5p + 12p is a perfect square.
1.3. There are n blue points and n red points on a straight line. Prove that the sum of
all distances between pairs of points of the same colour is less than or equal to the
sum of all distances between pairs of points of different colours.
2.1. Find all functions f : R −→ R such for any x, y ∈ R,
2.2. In a triangle ABC, ∠C = 60◦ , D, E, F are points on the sides BC, AB, AC respec-
tively, and M is the intersection point of AD and BF . Suppose that CDEF is a
rhombus. Prove that DF 2 = DM · DA.
Let n be any integer ≥ 2. Prove that 1/pq = 1/2, where the summation is over all
P
2.3.
integers p, q which satisfy 0 < p < q ≤ n, p + q > n, (p, q) = 1.
2000/2001
1.1. Let a, b, c, d be four positive integers such that each of them is a difference of two
squares of positive integers. Prove that abcd is also a difference of two squares of
positive integers.
1.2. Let P, Q be points taken on the side BC of a triangle ABC, in the order B, P, Q, C.
Let the circumcircles of 4P AB, 4QAC intersect at M (6= A) and those of 4P AC,
4QAB at N . Prove that A, M, N are collinear if and only if P and Q are symmetric
in the midpoint A0 of BC.
1.3. A game of Jai Alai has eight players and starts with players P1 and P2 on court and
the other players P3 , P4 , P5 , P6 , P7 , P8 waiting in a queue. After each point is played,
the loser goes to the end of the queue; the winner adds 1 point to his score and stays
on the court; and the player at the head of the queue comes on to contest the next
point. Play continues until someone has scored 7 points. At that moment, we observe
that a total of 37 points have been scored by all eight players. Determine who has
won and justify your answer.
2.1. In the acute triangle ABC, let D be the foot of the perpendicular from A to BC,
let E be the foot of the perpendicular from D to AC, and let F be a point on the
line segment DE. Prove that AF is perpendicular to BE if and only if F E/F D =
BD/DC.
2.2. Determine all the integers n > 1 such that
n
X n−1
X
x2i ≥ xn xi
i=1 i=1
1.1. Let A, B, C, D, E be five distinct points on a circle Γ in the clockwise order and let
the extensions of CD and AE meet at a point Y outside Γ. Suppose X is a point on
the extension of AC such that XB is tangent to Γ at B. Prove that XY = XB if
and only if XY is parallel DE.
1.2. Let n be a positive integer and (x1 , x2 , . . . , x2n ), xi = 0 or 1, i = 1, 2, . . . , 2n be a
sequence of 2n integers. Let Sn be the sum
Sn = x1 x2 + x3 x4 + · · · + x2n−1 x2n .
1.3. For every positive integer n, show that there is a positive integer k such that
2k 2 + 2001k + 3 ≡ 0 (mod 2n ).
2.2. For each real number x, bxc is the greatest integer less than or equal to x. For
example b2.8c = 2. Let r ≥ 0 be a real number such that for all integers m, n, m|n
implies bmrc|bnrc. Prove that r is an integer.
2.3. Find all functions f : [0, ∞) −→ [0, ∞) such that f (f (x)) + f (x) = 12x, for all x ≥ 0.
2002/2003
1.1. Determine whether there exists a positive integer n such that the sum of the digits
of n2 is 2002.
1.2. Three chords AB, CD and EF of a circle intersect at the midpoint M of AB. Show
that if CE produced and DF produced meet the line AB at the points P and Q
respectively, then M is also the midpoint of P Q.
1.3. In how many ways can n2 distinct real numbers be arranged into an n × n array (aij )
such that maxj mini aij = mini maxj aij ?
f (m + f (f (n))) = −f (f (m + 1)) − n
2003/2004
1.1. Let N be the fourth root of a product of 8 consecutive positive integers. Prove that
the greatest integer less than or equal to N is even.
1.2. Let Γ be a circle with center I, the incenter of triangle ABC. Let D, E, F be points of
intersection of Γ with the lines from I that are perpendicular to the sides BC, CA, AB
respectively. Prove that AD, BE, CF are concurrent.
1.3. Find all pairs of integers (x, y) satisfying x5 + y 5 = (x + y)3 .
2.1. Let A, B, C, D be four distinct points arranged in order on a circle. The tangent to
the circle at A meets the ray CB at K and the tangent to the circle at B meets the
ray DA at H. Suppose BK = BC and AH = AD. Prove that the quadrilateral
ABCD is a trapezium.
2.2. Determine the smallest constant k > 0 such that
ab bc ca
+ + ≤ k(a + b + c),
a + b + 2c b + c + 2a c + a + 2b
for all a, b, c > 0.
2.3. Consider an n × n square lattice with points colored either black or white. A square
path is a closed path in the shape of a square with edges parallel to the edges of the
lattice. Let M (n) be the minimum number of black points needed for an n × n square
lattice so that every square path has at least one black point on it. Prove that
2 2
(n − 1)2 ≤ M (n) ≤ n2 .
7 7
(*The numbering 1.1 refers to the first question of the selection test in the first day, while 2.1 refers
to the first question of the selection test in the second day.)
Solutions to National Team Selection Tests
Prepared by Tay Tiong Seng and Wong Yan Loi
1994/95
1.1 It can be proved by induction that f (n) is the number of ones in the binary represen-
tation of n.
(i) There can be at most 10 ones in the binary representation of a natural number if it is
less than or equal to 1994 = 11111001010(2) . Hence M = 10.
(ii) For any natural number n less than or equal to 1994, f (n) = 10 if and only if n is
1023 = 1111111111(2) ,
1535 = 10111111111(2) ,
1791 = 11011111111(2) ,
1919 = 11101111111(2) ,
1983 = 11110111111(2) .
1.2. Stewart’s theorem. In 4ABC, D is a point on BC such that AD bisects ∠A. Then
AB : BD = AC : CD.
1st solution
A
...............
....... .... .....
....... ... .. ....
....... .... ..... .........
.......
.......
.
......
....
.
...
.
... ..
....
.....
R
.....
.....
....... ... .... . .... .. .........
....
........ .
... . .
.. ..
.....................................................................................................................
.
B P Q C
AB BP
Applying Stewart’s theorem to 4ABQ, we have = .
AQ PQ
BC AB
Given BP · CQ = BC · P Q, it follows that = .
CQ AQ
Now let R be the point on AC such that QR is parallel to BA.
AB BC AB
Then = = .
RQ CQ AQ
Hence RQ = AQ and ∠QAR = ∠QRA.
Therefore ∠P AC = ∠P AQ + ∠QAR = 12 (∠BAQ + ∠QAR + ∠QRA) = π2 .
2nd solution
CB PB AB
Since = = , by Stewart’s theorem, AC is the external angle bisector of ∠BAQ.
CQ PQ AQ
π
Hence ∠P AC = .
2
P
1.3. (i) Note that twice the total number of clappings is equal to x∈S f (x) which cannot
be the odd number 2 + 3 + 4 + · · · + 1995.
Each circle in the diagram represents a student x and the number in the circle represents
f (x). The number on each edge represents the number of times the two adjacent students
clap hands with each other. Taking n = 499 gives an example of the problem.
2.1. (i) Denote the function f (x) composed with itself n times by f (n) (x). Also let g0 (x)
be the identity function. Note that f (2) (x) is strictly increasing for x > 0. We shall prove
by induction on n that gn (x) is strictly increasing for x > 0. It can easily be checked that
g1 (x) is strictly increasing for x > 0.
Suppose for n ≥ 2, g1 (x), ..., gn−1 (x) are strictly increasing. Let x > y > 0. We have
gn (x) − gn (y) = (x − y) + (f (x) − f (y)) + (f (2) (x) − f (2) (y)) + · · · + (f (n) (x) − f (n) (y))
= (g1 (x) − g1 (y)) + (gn−2 (f (2) (x)) − gn−2 (f (2) (y))) > 0.
By induction, gn (x) is strictly increasing.
F1 Fi Fi+1 F1 Fn+1
(ii) Note that = 1 and f ( )= . Hence + ··· + = gn (1).
F2 Fi+1 Fi+2 F2 Fn+2
B
2.2. Since 4ADP is similar to 4AP C, we have ............
... .... .......
.
... .. ........
AP /AD = AC/AP . Hence AP 2 = AD · AC = .... .......P
............................
...................................
.... .......
...... .... .. ......... ..... ......
.......... ..... ........... .......... ........ ........
(BD cot A) · AC = 2(ABC) cot A, where (ABC) is ...
... ... ..... ...
.
...... .....
...... ..... .....
.
... ....... .... ...... ..... ..
the area of 4ABC. Similarly, AM 2 = ... ........ ... ...... .... ...
p2(ABC) cot A.
........... ..
............. ............. ........... ...
......... ..
...... .. ..
...........................................................................................................
Hence AP = AQ = AM = AN = 2(ABC) cot A. A ... ... .. C
...
... D .
.
....
...
..
... ..
This shows that P,
pQ, M, N lie on the circle centered
...
...
.....
.
.
....
.. .....
...
. ..
.....
...... .... .....
at A with radius 2(ABC) cot A. ........ .. .....
.......
..................................
2.3. Let such a path be given. First the following facts are observed.
(i) The number of edges of the path is nm − 1.
(ii) By induction, each region with s squares is adjacent to 2s + 1 edges of the path.
(iii) Each edge on the north or east side of the grid which is not included in the path
corresponds to exactly one shaded region.
Let the number of shaded regions be k and let s1 , s2 , ..., sk be the number of squares in each
of these regions. From (iii), it follows that the number of edges of the path on the north
and east side of the grid is (m − 1) + (n − 1) − k. Hence by (ii), the total number of edges
k
X
of the path is (2si + 1) + [(m − 1) + (n − 1) − k]. By (i), we have
i=1
k
X
(2si + 1) + [(m − 1) + (n − 1) − k] = nm − 1.
i=1
k
X 1
From this the total number of shaded squares is si = (m − 1)(n − 1).
i=1
2
This problem appears in the American Mathematical Monthly. (See The American Mathe-
matical Monthly, Vol.104, No.6, June-July 1997, p572-573.)
1995/96
B .........................................Q
............................................................ C
1.1. Let BH intersect AD at F . Then 4AF B is congruent to .........
.
...
...
...
.........
.........
.. ...
...
... ... ... ......... ...
... .... ..... ................
4BP C. Hence AF = BP = BQ. Therefore F D = QC and ... .... .... .............
... ..................
...
...
...
............. .......... ...
QCDF is a rectangle. Since ∠CHF = 90o , the circumcircle P H ...
...
... .......
......
... ......
...
...
...
... ...
...... ...
... ......
of the rectangle QCDF passes through H. As QD is also a ...
...
... ......
......
.
.......
...
...
...
...
diameter of this circle, we have ∠QHD = 90o . ... ...... ...
... ......
... ...... ....
... . ... ..
.
........................................................................................................
A F D
1.2. Suppose there is a perfect square a2 of the form n2k − 7 for some positive integer n.
Then a is necessarily odd. We shall show how to produce a perfect square of the form
n0 2k+1 − 7 for some positive integer n0 . If n is even, then a2 = (n/2)2k+1 − 7 is of the
required form. Suppose that n is odd. We wish to choose a positive integer m such that
(a + m)2 is of the desired form.
Consider (a + m)2 = a2 + 2am + m2 = −7 + n2k + m(m + 2a). If we choose m = 2k−1 , then
m(m + 2a) is an odd multiple of 2k . Consequently, (a + m)2 is of the form n0 2k+1 − 7 for
some positive integer n0 . Now the solution of this problem can be completed by induction
on k.
1.3. Let p be the smallest positive integer such that pa ≡ 0 (mod 1995), i.e. pa = 1995k for
some positive integer k. Let q = 1995/p. Then q is an integer and it divides a. We claim
that
In this sequence, there are q blocks of a, a, . . . , a, b or −a, −a, . . . , −a, b making a total of
pq = 1995 terms. For each i = 1, 2 . . . , 1995, let si be the sum of the first i terms of this
sequence. Then by the result above, {s1 , s2 . . . , s1995 } = S and si+1 − si = ±a or ± b
(mod 1995).
2.2. Let N be the set of all natural numbers. Let A = {n2 | n ∈ N}. Let N \ A =
{n1 , n2 , n3 , . . . }. Define f as follows:
1 if n = 1,
n2i if n = n2i−1 , i = 1, 2, . . .
2
f (n) = n2i−1 if n = n2i , i = 1, 2, . . .
k 2k
n22i
if n = n2i−1 , k = 1, 2, . . .
2k+1 k
n2i−1 if n = n22i ,
k = 1, 2, . . .
(Note: The function above comes from the following consideration. First, f (1) must be 1.
Let f (2) = 3. Then f (3) = 22 , f (22 ) = 32 , f (32 ) = 24 etc.. Next, let f (5) = 6. Continuing
as before, we have f (6) = 52 , f (52 ) = 62 , f (62 ) = 54 etc..)
2.3. Let N = {1, 2, ..., 1995}. Let q be an integer with 1 ≤ q ≤ m. We shall prove the
following statement S(q) by induction (on q):
P
S(q): There exists a subset Iq of N such that i∈Iq ni = q.
S(1) is true because one of the ni ’s must be 1. Now assume that for some q with 1 ≤ q < m,
S(i) is true for i ≤ q. Then |Iq | ≤ q and 1994.
If ni > q + 1 for all i ∈ N \ Iq , then i∈N ni ≥ q + (q + 2)(1995 − |Iq |) = (1996 − |Iq |)q +
P
This problem appears in the American Mathematical Monthly with 1995 replaced by k and
3990 replaced by 2k. The proof above works for the general case too. See (The American
Mathematical Monthly, Vol.105, No.3, March 1998, pg 273-274.)
1996/97
BM DB AN AD C .......
...........
= and = . .
.
..... .. ...
..... .... ....
MC DC NC DC .....
.
.... .. ....
.... .... ...
...
..... ... ...
.... ..
BM AN .
....
.
...
.
.
..
..
.
.
...
...
As AD = DB, we have = . F .....
.
..
..
.
.. ...
E
..
..................................................................................
. .
MC NC . .
..
.
.
..
.. .
.... .............. O .
.
.
........ .. ........... . ....... .
...... ....
..
.
N ..... .... .... .... ....................................... .... .... ..... .......
.. .. ...
.
M
..... ...
Hence N M//AB and 4ABC ∼ 4N M C. .
..
..... . ..........
.... ....................
.. ..... ..
... .. ..........
...
. ...........
. ........ ....
..
........ .... . .....
................ ... .... ...... ..
P .
....................................................................................................................................
....
Q
AB AC BC .
....
.
...
. ..
. ..
... .
.
.
.
. .... .... ...
...
Therefore = = . . ...
..................................................................................................................................................
.
NM NC MC A D B
BM DB DB + DC BM + M C BC AB
Since = , we have = = = .
MC DC DC MC MC NM
F E + DC 2F E
On the other hand, F E = 12 AB = DB. Therefore, = .
DC NM
1 1 2
Consequently, + = .
F E DC NM
Applying Menelaus’s Theorem to 4CM N for the lines EP and F Q and using the fact that
OM = ON , we have
CP OM CE CE CQ ON F C FC
= · = and = · = .
PN ON M E ME QM OM F N FN
CE FC CQ CP
Since F E//AB//N M , we have = . Therefore = so that F E//P Q.
ME FN QM PN
Hence P QEF is a trapezoid and O is the intersection point of its two diagonals.
1 1 2
From this, it follows that + = . Consequently, P Q = DC.
FE PQ NM
1.2. It can be shown that an satisfies the recurrence relation: an = 2an−1 + 2an−2 with
a1 = 3 and a2 = 8. Solving this difference equation gives
1 1 √ 1 1 √
an = ( + √ )(1 + 3)n + (−1)n+1 ( √ − )( 3 − 1)n .
2 3 3 2
√
Next we shall show that ( √13 − 12 )( 3 − 1)n < 0.5 for n ≥ 1. This is because
1 1 √ 1 1 √ 1
for n ≥ 1, 0 < ( √ − )( 3 − 1)n ≤ ( √ − )( 3 − 1) < (1 − )(2 − 1) = 0.5.
3 2 3 2 2
1 1 √
Thus an = ( + √ )(1 + 3)n rounded off to the nearest integer.
2 3
1.3. 1st solution
f (t)
Since the set { | t 6= 0} is finite, there exist distinct positive integers m1 , m2 with
t
6 −f (x) such that
m1 , m2 =
By letting x = 1, we obtain [f (1)]2 = 1 so that f (1) = ±1. Consequently, f (x) = ±x. Also
the functions f (x) = x and f (x) = −x clearly satisfy the two given conditions.
2nd solution
(i) First we prove that f (0) = 0. Putting x = 0 = y, we have f (f (0)) = f (0). If f (0) = a,
then f (0) = f (f (0)) = f (a). Thus a + f (0) = f (0 + f (a)) = f (f (0)) = f (0), whence a = 0.
(ii) Putting x = 0, we have f (f (y) = y for all y.
(iii) We will prove that f (x) = ±x for all x.
Suppose for some p, f (p) = cp for some constant c 6= ±1. Then f (p + f (p)) = p + f (p).
Let q = p + f (p). Then q 6= 0 and f (q) = q. Thus f (q + f (q)) = q + f (q) and f (2q) = 2q.
Inductively we have f (nq) = nq for any positive integer n. Now f (nq + f (p)) = p + f (nq).
So f (nq + cp) = p + nq. Thus f (nq + cp)/(nq + cp) = 1 − (c − 1)p/(nq + cp). Since c − 1 6= 0
and there are infinitely many choices for n so that nq + cp 6= 0, this gives an infinite number
of members in the set {f (x)/x} contradicting the second condition. Thus c = ±1.
(iv) For f (p) = p, we will prove that f (x) = x for all x.
If f(−p) = p, then −p = f (f (−p)) = f (p) = p which is impossible. Thus f (−p) = −p.
Suppose there exists r such that f (r) = −r. Then f (r+f (p)) = p+f (r), i.e., f (r+p) = p−r.
Therefore f (r + p)/(r + p) = (p − r)/(r + p) 6= ±1. (Note that the denominator is not zero.)
(v) From the above we conclude that either f (x) = x for all x or f (x) = −x for all x.
Clearly these functions satisfy the two given conditions. Thus these are the only two
functions required.
2.1. Let a, b, c, d represent the numbers at any stage subsequent to the initial one. Then
a + b + c + d = 0 so that d = −(a + b + c). It follows that
n−i+1
2.2. is equal to the number of i-subsets of the set S = {1, 2, . . . , n} containing
i
no consecutive integers. Hence the required sum is just the number an of subsets of S
containing no consecutive integers. It can be shown easily that an satisfies the recurrence
relation: an = an−1 + an−2 with a0 = 1 and a1 = 2. This can also be derived from the
identity:
n−i+1 (n − 1) − i + 1 (n − 2) − (i − 1) + 1
= + .
i i i−1
From this, we obtain
b n+1
2
c √ √ !n √ √ !n
5+3 5 1+ 5 5−3 5 1− 5
X n−i+1
= + .
i 10 2 10 2
i=0
since (2n − r)2 > (2n − r + 1)(2n − (r + 1)). Hence the induction hypothesis is true for
k = r + 1. This completes the induction step.
When k = n, we get
1 1 n+1 n
1− <1− = < an < = 1,
n n+2 n+2 2n − n
the required inequality.
1997/98
1.2. We will prove the statement by induction on n. It obviously holds for n = 2. Assume
that n > 2 and that the statement is true for any integer less than n. We distinguish two
cases.
Case 1. There are no i and j such that Ai ∪ Aj = S and |Ai ∩ Aj | = 1.
Let x be an arbitrary element in S. The number of sets Ai not containing x is at most
2n−2 − 1 by the induction hypothesis. The number of subsets of S containing x is 2n−1 .
At most half of these appear as a set Ai , since if x ∈ Ai , then there is no j such that
Aj = (S − Ai ) ∪ {x} for otherwise |Ai ∩ Aj | = 1. Thus the number of sets Ai is at most
2n−2 − 1 + 2n−2 = 2n−1 − 1.
Case 2. There is an element x ∈ S such that A1 ∪ A2 = S and A1 ∩ A2 = {x}.
Let |A1 | = r + 1 and |A2 | = s + 1. Then r + s = n − 1. The number of sets Ai such that
Ai ⊆ A1 is at most 2r − 1 by the induction hypothesis. Similarly the number of sets Ai such
that Ai ⊆ A2 is at most 2s − 1.
If Ai is not a subset of A1 and A2 , then A1 ∩ Ai 6= ∅, A2 ∩ Ai 6= ∅. Since A1 ∩ A2 6= ∅, we
have A1 ∩ A2 ∩ Ai 6= ∅. Thus A1 ∩ A2 ∩ Ai = {x}. Thus Ai = {x} ∪ (Ai − A1 ) ∪ (Ai − A2 ),
and since the nonempty sets Ai − A1 and Ai − A2 can be chosen in 2s − 1 and 2r − 1 ways,
respectively, the number of these sets is at most (2s − 1)(2r − 1). Adding up these partial
results we obtain the result that the number of Ai ’s is at most 2n−1 − 1.
2nd solution
We shall prove that the statement holds for any integer k ≥ 4, not just k = 1998. Consider
any polynomial F (x) with integer coefficients satisfying the given inequality 0 ≤ F (c) ≤ k
for every c ∈ {0, 1, . . . , k + 1}. Note that F (k + 1) = F (0) because F (k + 1) − F (0) is a
multiple of k + 1 not exceeding k in absolute value. Hence
F (x) − F (0) = x(x − k − 1)G(x)
where G(x) is a polynomial with integer coefficients. Thus
k ≥ |F (c) − F (0)| = c(k + 1 − c)|G(c)| for each c ∈ {1, 2 . . . , k}. (2)
The inequality c(k + 1 − c) > k holds for each c ∈ {1, 2, . . . , k − 1} which is not an empty
set if k ≥ 3. Thus for any c in this set, |G(c)| < 1. Since G(c) is an integer, G(c) = 0. Thus
2, 3, . . . , k − 1 are roots of G(x), which yields
F (x) − F (0) = x(x − 2)(x − 3) · · · (x − k + 1)(x − k − 1)H(x). (3)
We still need to prove that H(1) = H(k) = 0. For both c = 1 and c = k, (3) implies that
k ≥ |F (c) − F (0)| = (k − 2)! · k · |H(c)|.
Now (k − 2)! > 1 since k ≥ 4. Therefore |H(c)| < 1 and hence H(c) = 0.
For k = 1, 2, 3 we have the following counterexamples:
F (x) = x(2 − x) for k = 1
F (x) = x(3 − x) for k = 2
F (x) = x(4 − x)(x − 2)2 for k = 3
C2 A C1 B
Let the line perpendicular to CI and passing through C meet AB at C2 . By analogy,
we denote the points A2 and B2 . It’s obvious that the centres of the circumcircles of
AIA1 , BIB1 and CIC1 are the middle points of A2 I, B2 I and C2 I, respectively. So it’s
sufficient to prove that A2 , B2 and C2 are collinear. Let’s note that CC2 is the exterior
C2 A B2 A
bisector of ∠ACB, and so C 2B
CA
= CB . By analogy B 2C
BA
= BC and A 2B AB
A2 C = AC . Thus
C2 A B2 C A2 B CA BC AB 3
C2 B B2 A A2 C = CB BA AC = 1 and by Menelaus’ Theorem , the points A2 , B2 and C2 are
collinear.
2nd solution
Let A0 , B 0 , C 0 be the midpoints of AI, BI, CI, respectively. Let the perpendicular bisectors
of AI and BI meet at C 00 . A00 and B 00 are similarly defined.
A
........
....... ....... ..............
.................... ...
B 00
.. .... ......... ........ ..... ....
... .... ....... ............. ............
... ......... ....... ............. . ...
.. ..... ....... .. .............. ........ ...
A0
.
. . ................. . ... ..
. .. .. ...
... ...... .. ....... ..
...
. ......
. . ....... ................ ............. .. ... ........ .
...
. .. ....... ....... .
... ............................ .... .................. ...
..................
. .. ......
...................... .... C 00
.............
..
... ... ....
... .... ....
.
. ..
....
.... ............
.
............
....... .............
....... ...
.
...
..
.... . ... ................
... .. ............... .... .... ....
. . ...
........ ....
....... ..
.... ....
.... ....
.... .... ... ...
......
............. .. .. ............
. . . .
... .......................................
I ....
....
.......
....... ....
....... ..
........
.
... ... ....
. .. ...... .. .
..
. . . .
.
. .... .... . .
....
.. .. .......
.... .... . . . ....... .... ............. .. .
...
.......
.... .... ... ..... .. ....... .... ........ .............
............. . ....
.. .......
. .... .... .... ... ... ............. ... ... ... ............. .... ..
..
.......
.......
... .
.. .. .. . .. .
.. . . . .. . ... ..
............... . .. ..
.
... ... .
. ...
. ...........
.
.. .
... ... ...
. .....
. .
.......
.......
.... .... .... .... .... .... .... .... .... .... .... .... .... .... .... .... .... .... .... .... ......... .... .... ............... .... .... .... .... ......... ..... .... .... .... .... .... .... .... .... .... .... . ............. .......
. .
.. . . . . . .. .. .. .
. .............
A000 ....
... . .... ... ..... ... .
. ....
.......
..
B0
.
. .......... . ... . . . .
.............
..... .. . ............. ............
C0
.
. . .
..... .. . ..
. . .. .......... ...
. . . .
. .
.. ............. .......
......... .....
. ... .. ..... .
..............................................................................................................................................................................................................................................................................................................
..
B
...
...
...
...
A1...
...
..
.
...
C
...
... ...
... ...
... ..
... ... .
...
... ...
... ... ...
... ... ...
... ... ..
... .
. .
...
... ...
... ... ...
... ... ...
... ... ..
... .
. ...
.
... ...
... ... ...
...
... ... ...
... .. ...
... ..... ....
... ..
... ... ..
... ... ...
... .. ...
... ... ...
... ... ...
... .. ...
... ... ..
...... ..
..... ..
......
A00
Then the circumcentre A000 of AIA1 is the intersection of B 00 C 00 with B 0 C 0 . Likewise the
circumcentre B 000 of BIB1 is the intersection of A00 C 00 with A0 C 0 and the circumcentre C 000
of CIC1 is the intersection of A00 B 00 with A0 B 0 .
First we note that the circumcentre of AIB lies on the line CI. To prove this, let the
circumcircle of AIB meet CI at another point X. Then ∠XAB = ∠XIB = 12 (∠B + ∠C).
Thus ∠XAI = ∠XAB + ∠BAI = 90◦ . Thus XI is a diameter and the circumcentre which
is C 00 is on the line CI. Similarly, A00 is on AI and B 00 is on BI.
Now we consider the triangles A0 B 0 C 0 and A00 B 00 C 00 . The lines A0 A00 , B 0 B 00 , and C 0 C 00 are
concurrent (at I), thus by Desargues’ Theorem4 , the three points, namely, the intersections
of B 00 C 00 with B 0 C 0 , A00 C 00 with A0 C 0 and A00 B 00 with A0 B 0 are collinear.
We prove this by induction on n. For n = 1 the void sum has value zero and the result is
clear. Assume that the result holds for a certain n ≥ 1. Consider a1 ≥ · · · ≥ an+1 ≥ an+2 =
0. Write S = nk=1 ak and b = an+1 . It suffices to prove that
P
√ √ √ q
S+b− S ≤ − nb + (n + 1)b.
√
This holds trivially when b = 0. And if b > 0, division by b takes it into the form
√ √ √ √
U + 1 − U ≤ n + 1 − n,
2nd solution
√ √
Set xk = ak − ak+1 , for k = 1, . . . , n. Then
Note that the coefficient of xk x` (where k < `) in the last sum is equal to k. The square of
the right-hand side of the asserted inequality is equal to
n √
X 2 n
X X √
kxk = kx2k + 2 k`xk x` . (4)
k=1 k=1 1≤k<l≤n
And since the value of (3) is obviously not greater than the value of (4), the result follows.
√ √
3rd solution Let ck = k − k − 1, then the inequality can be transformed to
v
u n n
uX X √
t ak ≤ ak ck .
k=1 k=1
√ √
Also 2 a1 a2 c1 c2 = a1 a2 (1 − c22 ). Hence
n
X X √
ak (c2k − 1) + 2 ai aj ci cj
k=2 0≤i<j≤n
n n
X X √
ak c2k − 1 + ak−1 ak 1 − c2k
≥
k=2 k=2
n
2 √
X
= 1 − ck ak−1 ak − ak ≥ 0.
k=2
√
since ak−1 ak − ak ≥ 0 and 1 − c2k ≥ 0. This completes the proof.
From solutions 2 and 3, we can conclude that equality holds if and only if there exists an
index m such that a1 = · · · = am and ak = 0 for k > m.
And since d and e are coprime and h = de, the latter two equations imply
Case 2. gcd(d, e) > 1. Let p be a prime divisor of d and e. Assume that pα is the greatest
power of p dividing a and pβ is the greatest power of p dividing h. Recalling that h = de
with e being coprime to a, we see that β > α ≥ 1. If follows that for each term of the
progression (a + ih : i = 0, 1, . . .), the greatest power of p which divides it is pα . Since x2
and y 3 are in the progression, α must be divisible by 2 and 3. So α = 6γ for some integer
γ; hence α ≥ 6.
The progression (p−6 (a + ih) : i = 1, 2, . . .) with common difference h/p6 < h has integer
terms and contains the numbers (x/p3 )2 and (y/p2 )3 . By the induction hypothesis it con-
tains a term z 6 for some integer z. Thus (pz)6 is a term in the original progression. This
completes the induction.
2nd solution
We use the same notation as in the first solution.
The assertion is proved by induction on h. The case d = 1 is trivially true.
(a) gcd(a, h) = 1. (a−1 exists mod h.) In this case, we have (y/x)6 ≡ a (mod h).
(b) gcd(a, h) = r > 1. Pick a prime p dividing r and let α be the largest positive integer
such that pα divides r. If α ≥ 6, then
x 3 a y 2 a d
( ) ≡ 6, ( ) ≡ 6 (mod ).
p2 p p3 p p6
a d
By induction hypothesis, there exists z such that z 6 ≡ (mod 6 ). Then (zp)6 ≡ a
p6 p
(mod h). So we suppose 0 < α < 6. From x3 ≡ a, y 2 ≡ a (mod h), we have
x3 a y2 a d
α
≡ α, ≡ α (mod α ). (∗)
p p pα p p
h d
(i) gcd(p, ) = 1. (p−1 exists mod α .) Multiply both sides of (*) by pα−6 . We have
pα p
x 3 a y 2 a d
( ) ≡ 6, ( ) ≡ 6 (mod ).
p2 p p3 p pα
a d
By induction hypothesis, there exists z such that z 6 ≡ (mod ). Write a = pα a0 ,
p6 pα
then there is an integer m such that
h
(pz)6 − pα a0 = m .
pα
Since α < 6, pα divides the left hand side of the equation. Thus it also divides m, whence
(pz)6 ≡ pα a0 = a (mod h).
h
(ii) gcd(p, α ) = p. Then pα is the largest power of p dividing a. Furthermore, α is a
p
multiple of 3. To see this write x = pβ x0 , where p does not divide x0 and let x = a + kh
for some positive integer k. Then p3β x0 3 = x3 = a + kh = pα (a0 + pkh0 ) for some integer
a0 , h0 with gcd(a0 , p) = 1. Consequently, α = 3β. Similarly, α is a multiple of 2. Therefore,
α ≥ 6, and this case does not arise.
Footnotes
1. Ptolemy’s Theorem. For any quadrilateral ABCD, we have
AB · CD + BC · DA ≥ AC · BD
BX CY AZ
· · = 1.
XC Y A ZB
4. Desargues’ Theorem. Given any pair of triangles ABC and A0 B 0 C 0 , the following are
equivalent: (i) The lines AA0 , BB 0 and CC 0 are concurrent. (ii) The points of intersection
of AB with A0 B 0 , AC with A0 C 0 , BC with B 0 C 0 are collinear.
1988/89
1.1. Suppose p is an integer such that p3 − mp2 + mp − (m2 + 1) = 0. It follows that
(p2 + m)(p − m) = 1. Since p and m are integers, we have either
(1) p2 + m = p − m = −1, or
(2) p2 + m = p − m = 1.
In case (1), we have m = p + 1, and so p2 + p + 1 = −1 or p2 + p + 2 = 0 which has no real
solution.
In case (2), we have m = p − 1, and so p2 + p − 1 = 1 or p2 + p − 2 = 0 which has the
solutions p = −2 and 1.
Hence, m = −3 and 0 are the integer values of m for which the given equation has an
integer solution.
..........................................................................................................................................................................................................................................................................
..................................... .........
................................................
........
...................................................
.........
.............................................. ....
..................................... ...................................... ........................................ ..................................... ...
..................................... . ........ .......... .......... .
....................................................................................................................................................................................................................................................................................................................
........ ......... ......... ........
.... ................................................... .............................................. ................................................ ...................................................
... ........................................ ..................................... ...................................... ........................................
1.2. It is only necessary to partition the checker- ...
.
........................................
..........
.....................................
..........
......................................
. ........
........................................
..........
................................................................................................................................................................................................................................ ...................................................................................
board into a closed path one square wide. One way .....................................
.....................................
............................
.
.
....................................
.
. ........
.............................
.
.
......................................
.
. .........
...........................
.
.
...................................
.
. ........
....
...
..................................... .................... ..................... ................... .
..............................................................................................................................................................................................................................................................................................
to do this is shown in the diagram. The squares ..
...
..............................
........................................
...........................
.....................................
.............................
......................................
..............................
........................................
... ........................................ ..................................... ...................................... ........................................
... .............................. ............................. ............................ ..............................
lie with alternating colours along the closed path. ................................................................................................................................................................................................................................................................................................................
.................................... ..................................... ........................................ ..................................... ...
.................................... ..................................... ........................................ ..................................... ...
The removal of two squares of opposite colours .................................... ........................... ..............................
........................................................................................................................................................................................................................................................................................................
............................. ...
... ........................................ ..................................... ..................................... ........................................
... ........................................ ..................................... ..................................... ........................................
from any two positions along the path will cut the ... ............................. ............................ ...........................
......................................................................................................................................................................................................................................................................................................................
.............................
.......................................... ..................................... ........................................ ..................................... ...
path into two open-ended segments (or one seg- ....................................
....................................
.....................................
.....................................
........................................
........................................
.....................................
.....................................
...
...
................................ .................................... ........................................ .................................... ..
ment if the removed squares are adjacent on the ...........................................................................................................................................................................................................................................................................................
... ........................................ ..................................... ..................................... ........................................
... ........................................ ..................................... ..................................... ........................................
.. ............................. ............................ ........................... .............................
path). ................................................................................................................................................................................................................................................
Each segment must consist of an even number of squares, so each segment must be com-
pletely covered by dominoes.
(i) xi = ±1 for i = 1, . . . , n,
(ii) 0 ≤ x1 + x2 + · · · + xr < 4, for r = 1, 2, . . . , n − 1,
(iii) x1 + x2 + · · · + xn = 4.
Also let Sn0 be the set of all n-tuples (x1 , x2 , . . . , xn ) such that
(i) xi = ±1 for i = 1, . . . , n,
(ii) −2 ≤ x1 + x2 + · · · + xr < 2, for r = 1, 2, . . . , n − 1,
(iii) x1 + x2 + · · · + xn = 2.
Let |Sn | = an and |Sn0 | = bn . First note that an = 0 if n is odd. Hence we consider only
even values of n.
Let (x1 , x2 . . . , x2k ) be an element in S2k . Then (x1 , x2 ) = (1, 1) or (1, −1). If (x1 , x2 ) =
0
(1, 1), then (x3 , x4 . . . , x2k ) is an element of S2k−2 . If (x1 , x2 ) = (1, −1), then (x3 , x4 . . . , x2k )
is an element of S2k−2 . Conversely, if each element of S2k−2 0 is augmented at the beginning
by two ones, it gives rise to an element of S2k . Similarly, if each element of S2k−2 is
augmented at the beginning by 1 and -1, it gives rise to an element of S2k . This shows that
a2k = b2k−2 + a2k−2 .
Next consider an element (x1 , x2 . . . , x2k ) in S2k 0 . Then (x , x ) = (1, −1), (−1, 1) or
1 2
(−1, −1). Hence, (x3 , x4 . . . , x2k ) is an element of S2k−2 0 if (x1 , x2 ) = (1, −1) or (−1, 1),
and it is an element of S2k−2 if (x1 , x2 ) = (−1, −1). By augmenting each element of S2k−2 0
0
at the beginning by either 1,-1 or -1,1, it gives rise to an element of S2k . Similarly, by
augmenting each element of S2k−2 at the beginning by −1, −1, we get an element of S2k 0 .
(i) xi = ±1 for i = 1, . . . , n,
(ii) 0 ≤ x1 + x2 + · · · + xr < 4, for r = 1, 2, . . . , n − 1,
(iii) x1 + x2 + · · · + xn = 2.
Let |Sn00 | = cn . Again, we only have to consider even values of n. Note that each sequence
in S2k must end in two “1”s. By dropping these two ones, we obtain a sequence in S2k−2 00 .
00
Conversely, each sequence in S2k−2 can be augmented at the end by two “1”s to get a
sequence in S2k . Hence a2k = c2k−2 .
Let’s examine the end terms of each sequence in S2k 00 . The last three terms of any sequence
00
in S2k are as follow:
−1 1 1, 1 − 1 1, 1 1 − 1, −1 − 1 1, −1 1 − 1. (∗)
For the first three cases, one can replace them by a single “1” to get a member of S2k−2 00 .
For the last two cases, one can drop the last two terms to get a member of S2k−2 . 00
Conversely, for any sequence in S2k−200 , which ends in a “1”, one can replace the “1” by any
one of the first three endings in (*) to get a sequence in S2k00 .
Let the number of sequences in S2k−200 which end in a “1” be x. Let the number of sequences
00
in S2k−2 which end in a “-1” be y. In the latter case, observe that if this last “-1” is replaced
by a “1”, then a sequence in S2k−2 is obtained. Hence, y = a2k−2 .
00
Similarly, for any sequence in S2k−2 which ends in a “-1”, it can be replaced by any one of
the last two endings in (*) to get a sequence in S2k 00 .
.
A
............
..............
... ... ... .....
.. ... .... ........
2.1. Let H and J be the points on AB such D .
.......... ... .... .......
...... ................ .... .........
E
that HE and JF are parallel to BC. H ...
.
... ... ....... ....
... ....... ....
.....
.....
.....
... ... ..... ...
..... ..
.....
... .
. ........
.....
.....
... ..
.
.... ... ... ... ... .... ... ... ... ... ......... ... F .....
GC FA JF JF J .
.
...
.
..
.
........
. ..
........
.....
.....
.....
..
. ..
. . ..
... ...... .....
Then = = = . .
.
..
.. ..
.
..
.
... .....
.
.
... ..... ..
.....
.....
.....
NC NA BN 2N C .
.
...
.. .
.
.
..
.
.
...
.
...
.
.......
.
..
. ...
.......
.....
.......
..........
... ..
. ... ..
..... .....
.....
..
. ..
. .... ....... .....
... ..
. . ....... .....
Hence JF = 2GC. ... .
. ...
.......................................................................................................................................................................
. .
. ...
B M N G C
EH AE GC GC
Also = = = . This shows that GC = 2EH. Therefore,
MB AM MC 2M B
DF FJ
JF = 4EH. As 4DEH is similar to 4DF J, we have = = 4.
DE EH
Consequently, EF = DF − DE = 4DE − DE = 3DE.
2.2. Let r = x−bxc. Write bxc = lp+q, where l, q ∈ Z and 0 ≤ q < |p|. Hence x = lp+q +r.
Now
x−p −x − 1 q+r −q − r − 1
b c+b c=b c+b c − 1.
p p p p
q+r −q − r − 1
So it suffices to find the value of the expression A ≡ b c+b c.
p p
(i) (p > 0.)
(
q+r −q − r − 1 −2 if q = p − 1 and r > 0
Then b c = 0 and b c= .
p p −1 otherwise
(
−2 if q = p − 1 and r > 0
Hence, A = .
−1 otherwise
(ii) (p = −1.) (
q+r −q if r = 0 −q − r − 1
Then b c= and b c = q + 1.
p −q − 1 if r > 0 p
(
1 if r = 0
Hence, A = .
0 if r > 0
(iii) (p < −1.)
( In this case, we have (
q+r 0 if q + r = 0 −q − r − 1 1 if q + r + 1 ≥ −p
b c= and b c= .
p −1 if q + r > 0 p 0 if q + r + 1 < −p
(
0 if q + r = 0 or q + r ≥ −p − 1
Hence, A = .
−1 if 0 < q + r < −p − 1
Therefore the possible values of the expression A − 1 are -3,-2,-1, and 0.
(Remark: Let P A be the tangent at A with P inside the sector of ∠QAE. As AB > AC,
we have ∠C > ∠B. Hence, ∠P AB = ∠C > ∠B = ∠QAP . This implies that E is on the
arc AB not containing C. Also, ∠EBF = ∠P AE < ∠EAB so that BF > AF . Hence, A0
is between F and B.)
1.2. The problem can be changed to find all integers m such that 5m + 12m is a perfect
square. Again the only answer is m = 2. We shall give the solution in this more general
case. (The solution of the original problem is easy by considering mod 5 or mod 10.)
One solution is p = 2 and we assert that it is the only solution. If p = 2k + 1 is odd, then
52k+1 + 122k+1 ≡ 22k+1 ≡ 2 · 4k ≡ 2(−1)k ≡ 2 or 3 (mod 5). However the square of an
integer can only be 0, 1 or 4 (mod 5). So 5p + 12p is not a square when p is odd.
Now suppose that 52n + 122n = t2 with n ≥ 2. Then
If 5 divides both factors on the right, it must also divide their difference which means it
divides 12. But this is impossible. Thus t − 12n = 1 and
f (a) f (b)
− a2 = − b2 .
a b
2.3. Let f (n) be the given sum. The summands that appear in f (n) but not in f (n − 1)
are those of the form ap = 1/pn where 1 ≤ p < n, (p, n) = 1; the summands in f (n − 1)
but not in f (n) are those of the form bp = 1/p(n − p) where 1 ≤ p < n − p, (p, n − p) = 1,
equivalently (p, n) = 1. (For example, if n = 10, those summands in f (10) but not in f (9)
1 1 1 1 1 1
are 1×10 , 3×10 , 7×10 , 9×10 , while those which are in f (9) but not in f (10) are 1×9 , 3×7 .)
Hence summing only over values of p such that (p, n) = 1, we have
X X X
f (n) − f (n − 1) = ap − bp = (ap + an−p − bp ).
p<n 2p<n 2p<n
But ap + an−p − bp = 0; hence f (n) = f (n − 1) for all n ≥ 3, and the result follows.
2000/2001
1.1 It suffices to prove that the product of two differences of two squares is also a difference
of two squares. Let a = x2 − y 2 and b = r2 − s2 . Then, ab = (x − y)(r − s)(x + y)(r + s) =
(xr + ys − yr − xs)(xr + ys + yr + xs) = (xr + ys)2 − (yr + xs)2 .
There is another characterization of a difference of two squares. Namely, a positive integer
n is a difference of two squares of positive integers if and only if n 6= 1, 2, 4, and n 6≡ 2
(mod 4). The result also follows from this characterization.
....................................................
......... .......
....... ......
.... ...... ......
..... .....
..
..
....... .....
..... ..
..... ..
. ..
...
. . . ....... .. ....... ....... ..... ....
.
....
...
...
......
. A
...... ....
. .. ..
....... ..........................................
... .. ....... ............. ......
1.2. Let K and L be points of intersection of ...
....
..
.....
...
....... ......................................... . .......
... .
.. . ..... ..
.......
......
.....
.....
... ... .. .
.. .......... ... . ... ......... .
. . . .....
.. ...
the line BC with the lines AM and AN re- ....
... ...
..
..
.
....
.
...
..
........ .... ... .... .... ........
... ... . ... ... ....
.
...... ... .. ... ... .......
..
...
...
...
...
... .
. .... .
. ...
... ... ... ... ... ....... .... ...
spectively. Suppose that the line BC is the ...
...
...
.
.
..
.. ........ ..
. . . .. .
... ... ...
.. .... .... ....
.....
.
.
.
.
...
....... ...
...
...
...
..
. . ...... . ..... ... .
. ..
.. .
... ..
x-axis of a coordinate system with origin B, ...
...
...
...
. .
..
.
.
...
. .
.......
..
.
...
.
.
. .... .. .... ... .. ..
.
. .... .. ....
.
.
. ...
. ..
..
. ..
.
...
N
The point K is on the radical axis of the circumcircles of 4P AB and 4QAC, hence its
powers k(k − p) and (k − q)(k − c) with respect to these two circles are equal. It follows
that k = cq/(c + q − p). Similarly, we have l = cp/(c + p − q), interchanging the roles of p
and q. We easily find that l = k if and only if p + q = c and the result follows.
1.3 Each time a player loses a match, he has to wait six games before his turn comes again.
If x is the number of games before his first turn, then the player will win if x + 7r + 7 = 37,
where r ≥ 0 is an integer and 0 ≤ x ≤ 6. Here r counts the number of times he lost. From
this, we obtain x = 2 and r = 4. Thus the second player in the queue wins. That is P4
wins.
2.1. Let G be the point on CE such that DG is parallel to
BE. Then ∠EBD = ∠GDC. Also EG/GC = BD/DC. ......
A
.........
.. .........
Note that 4ADE is similar to 4DCE. Then, .... .... .........
.
. .. .
... .... .... .....
... ... ... ...
... ... ... ....
.... ... ... ...
... ... ... ....
F E/F D = BD/DC ..
.
...
.
. ... ...
.....
...
...
...
...
...
.... ... ... ...
⇐⇒ EG/GC = F E/F D ..
.
..
.. .
.
..
.
.
.
...
...
...
...
...
.... .... ... ...
. ..
⇐⇒ 4ADF is similar to 4DCG ..
.
...
. .
...
.
.
...
...
...
...
...
...
.
. .
.
... .
... E
.....
................ ................ .....
.
...
⇐⇒ ∠DAF = ∠GDC .. . . ...
.
....... .
..
. . . .
. . ..
................... ... ........ .
. .
.
.
.
.
.
.
G
...
..
... ............. .... ....... ... ... ......
.. ............. ....... ... ... ... ... ...
F
⇐⇒ ∠DAF = ∠EBD B ..
. ................... ...... .
. .. ... . ...
. ... . . .
..................................................................................................................................................................... C
...
.
⇐⇒ AF ⊥BE. D
2nd Solution Let A = (0, a), B = (−b, 0), C = (c, 0), D = (0, 0), E = (x, y), F = (tx, ty),
where a, , b, c, t > 0. DE⊥AC implies that (x, y) = (sa, sc) for some s. E lies on AC implies
that s = ac/(a2 + c2 ). Hence, x = a2 c/(a2 + c2 ) and y = ac2 /(a2 + c2 ).
Then,
AF ⊥BE
⇐⇒ (tx, ty − a) · (x + b, y) = 0
⇐⇒ tx2 + txb + ty 2 − ay = 0
ta4 c2 2c ta2 c4 a 2 c2
⇐⇒ (a2 +c2 )2
+ atba
2 +c2 + (a2 +c2 )2 − a2 +c2 = 0
2.2 For n ≥ 6, take xn−5 = xn−4 = xn−3 = xn−2 = xn−1 = 1/2 and xn = 1 and zero
for other xi . Then the left hand side of the inequality is 9/4, while the right hand side is
5/2. So the inequality is not valid for n ≥ 6. We shall prove that the inequality holds for
n = 2, 3, 4, 5. The cases n = 2 and 3 can be verified easily. Let’s consider the case n = 5.
(The case n = 4 can be proved in a similar way.) The inequality to be proved is equivalent
to
x25 − (x1 + x2 + x3 + x4 )x5 + (x21 + x22 + x23 + x24 ) ≥ 0.
Regard this as a quadratic equation in x5 . It suffices to prove that its discriminant is less
than or equal to zero. The discriminant is equal to (x1 + x2 + x3 + x4 )2 − 4(x21 + x22 + x23 + x24 )
which can simplified to −[(x1 −x2 )2 +(x1 −x3 )2 +(x1 −x4 )2 +(x2 −x3 )2 +(x2 −x4 )2 +(x3 −x4 )2 ].
It is obviously less than or equal to zero.
2x + 1 = 3 y (1)
has two solutions in integers, (x, y) = (3, 2), (1, 1) and the equation
2x − 1 = 3 y (2)
has the solution (x, y) = (2, 1). By examining L(1), . . . , L(11), we see that the only solutions
are:
m = 1, 2, 6.
[(1) has solutions (x, y) = (3, 2), (1, 1). Assume that x > 3. As 3y − 1 = 2x = 16(2x−4 ) is
divisible by 16, it implies that y ≡ 0 (mod 4). Write y = 4k. Thus 34k − 1 = 81k − 1 =
80(81k−1 + 81k−2 + · · · + 1) which is not a power of 2.
(2) has a solution (x, y) = (2, 1). Now assume that y > 1. Then 2x − 1 is divisible by 9.
This implies that x must be even. Write x = 2x1 . Then 2x = 4x1 . For 4x1 ≡ 1 (mod 9), we
have x1 = 3k. Thus 2x − 1 = 64k − 1 = 63(64k−1 + 64k−2 + · · · + 1). Thus it cannot be a
power of 3.]
2001/2002
1.1 Suppose XY = XB. Then XY 2 = B
........
............
......................................................................................................................................
........
....... .. .
...... ....
X
....... ......
XB 2 = XC · XA so that XY : XC = ...............
.
......
.....
.....
C
..... ..... ...
...
......
...... ...
...
..... .......... ..
... ..
XA : XY . This shows that 4XCY is sim- ...
.. .
......
. .........
...... ..........
.
.
...
... ...... ...... ...
... ...... ......
D ..
ilar to 4XY A. Hence ∠EDY = ∠XAY = ....
.. ......
. . . .. . ...... ...
.
..........
..
.
.
...
.. ...... ........ ...
.. ...... .. ... ..
∠XY C. Therefore, XY is parallel to DE. ..... ...... ..
....
....... .. ... .....
.
. ..
.
.
... . .
...... ... ... .... ...
... ...... ... ... .... ....
The converse is similar. ... ......
... ...... .. .. ... ..
... .. .
........ .
....... ... ...
... .......... ... .. ..
A ....................................................................................................................................................................
...
... ...
... Y
.....
..... ....... E
..... .....
...... .....
....... ......
........
............ . . . ............
..................................
1.2 Use Induction. We can prove by induction that On = 22n−1 − 2n−1 and En = 22n−1 +
2n−1 . We merely have to note that
From here we have En + On = (1 + 3)n and En − On = (3 − 1)n . The result then follows.
1.3 We show more generally that ak 2 + bk + c ≡ 0 (mod 2n ) has a solution for all n
whenever b is odd and a or c is even. For n = 1, take k = 0 if c is even and k = 1
if c is odd. Now suppose the claim is true for all n. If c is even, then, by assumption,
the congruence 2at2 + bt + c/2 ≡ 0 (mod 2n ) has some solution t. Letting k = 2t we get
ak 2 + bk + c = 2(2at2 + bt + c/2) ≡ 0 (mod 2n+1 ). If c is odd, then a is even, so a + b + c is
even; hence, by assumption, the congruence 2at2 + (2a + b)t + (a + b + c)/2 ≡ 0 (mod 2n )
has some solution t. Letting k = 2t + 1 yields
Thus, whether c is even or odd, the claim is true for n + 1, and so by induction for all n.
2nd Solution by Tan Kiat Chuan, Tay Wei En Joel, Leung Ngai-Hang Zachary,
Kenneth Tay
It suffices to show that 2k 2 + 2001k + 3, k = 0, 1, · · · , 2n − 1 forms a complete residue
class modulo 2n . Suppose there are distinct integers k1 , k2 , 0 ≤ k1 , k2 ≤ 2n − 1 such that
2k12 + 2001k1 + 3 ≡ 2k22 + 2001k2 + 3 (mod 2n ). That is (k1 − k2 )(2k1 + 2k2 − 2001) ≡
(mod 2n ). Since 2k1 + 2k2 + 2001 is odd, we must have 2n divides k1 − k2 . Thus k1 ≡ k2
(mod 2n ). Since 0 ≤ k1 , k2 ≤ 2n − 1, we have k1 = k2 . Therefore, 2k 2 + 2001k + 3
k = 0, 1, · · · , 2n − 1 forms a complete residue class modulo 2n .
3
Alternatively, Consider the function f (x) = x 2 for x > 0. f 00 (x) = √3x > 0 for x > 0. Hence,
f is concave upward. By Jensen’s Inequality, for any three positive numbers z1 , z2 , z3 ,
z1 + z2 + z3 f (z1 ) + f (z2 ) + f (z3 )
f ≤ .
3 3
That is
(x21 + x22 + x23 )3
≤ 3.
(x31 + x32 + x33 )2
[x61 + 2x31 x32 − 3x41 x22 ] + [x61 + 2x31 x33 − 3x41 x23 ] + [x62 + 2x32 x31 − 3x42 x21 ]
+[x62 + 2x32 x33 − 3x42 x23 ] + [x63 + 2x33 x31 − 3x43 x21 ] + [x63 + 2x33 x32 − 3x43 x22 ]
+[2x31 x32 + 2x31 x33 + 2x32 x33 − 6x21 x22 x23 ] ≥ 0.
Each term in the square brackets is non-negative by the AM-GM inequality. So the result
follows.
[2x31 x32 + 2x31 x33 + 2x32 x33 − 6x21 x22 x23 ] + [x61 + x62 + 4x31 x32 − 3x41 x22 − 3x21 x42 ]
+[x61 + x63 + 4x31 x33 − 3x41 x23 − 3x21 x43 ] + [x62 + x63 + 4x32 x33 − 3x42 x23 − 3x22 x43 ] ≥ 0.
The first term is nonnegative by rearrangement inequality. The next three can be shown to
be nonnegative by using rearrangement inequality as follows:
x61 + x62 + 4x31 x32 = x31 (x31 + x32 ) + x32 (x32 + x31 ) + 2x31 x32
≥ x31 (x21 x2 + x1 x22 ) + x32 (x1 x22 + x21 x2 ) + 2x31 x32
= x51 x2 + x41 x22 + x1 x52 + x21 x42 + 2x31 x32
= (x51 x2 + x31 x32 ) + (x1 x52 + x31 x32 ) + x41 x22 + x21 x42
≥ 3x41 x22 + 3x21 x42 .
Alternatively,
x61 + x62 + 4x31 x32 = (x61 + x31 x32 + x31 x32 ) + (x62 + x31 x32 + x31 x32 ) ≥ 3x41 x22 + 3x21 x42 .
2.2 Suppose r is not an integer, choose an integer a such that ar 6= barc > 1. (Note that
r > 0. If r is irrational, choose any large positive integer a. If r = p/q is rational, choose a
large positive integer a such that (a, q) = 1.) Let k be the unique integer such that
1 1
≤ ar − barc < .
k+1 k
Then
k+1
1 ≤ (k + 1)(ar − barc) < ≤ 2.
k
Since
b(k + 1)arc = (k + 1)barc + b(k + 1)(ar − barc)c = (k + 1)barc + 1
we see that barc does not divide b(k + 1)arc. Thus m = a, n = (k + 1)a form a counter
example.
2.3 Fix any x ≥ 0. Let f [0] (x) = x and f [1] (x) = f (x). For n ≥ 1, let f [n] (x) = f (f [n−1] (x)).
Then the above functional equation gives
Using the initial conditions f [0] (x) = x and f [1] (x) = f (x), we have C1 = (f (x) + 4x)/7 and
C2 = (3x − f (x))/7. Therefore,
1 1
f [n] (x) = (f (x) + 4x)3n + (3x − f (x))(−4)n .
7 7
Since f (x) ≥ 0, f [n] (x) ≥ 0 for all n ≥ 0. By taking n even, we have 17 (f (x) + 4x)3n +
1 n 1 n
7 (3x − f (x))4 ≥ 0. From this, 3x − f (x) ≥ 0. By taking n odd, we have 7 (f (x) + 4x)3 −
1 n
7 (3x − f (x))4 ≥ 0. From this, 3x − f (x) ≤ 0. Consequently, f (x) = 3x. One can easily
verify that f (x) = 3x satisfies the given functional equation.
Alternatively, suppose for some a, f (a) = 3a + c where c ∈ R. Then f [n] (a) = 3n a + kn c.
One can obtain a recurrence relation in kn and use it to prove that c = 0. Finally check
that f (x) = 3x satisfies the given condition.
2002/2003
1.2 This is in fact the Butterfly Theorem. There are many proofs of this result. See the
discussion on page 45 in Geometry Revisited by Coxeter and Greitzer. Here we give two
proofs. ........................................
............. ........ .........
C .......
........................
.......
......
First, apply sine rule to .
..
...
.... ..
......... .......... ........................
. α .....
.
.............
. ..
.. .............
......
.....
.....
.....
... ...
. ..... . ....
. . ...
4CM P , .
..
... ..
.
.. .. ......
......
......
................
. .. .. ...................
.
E .....................
.. ...
. ..... γ β ............
. .. ........ ........................
.. ...... .
... .. .....
................. ............ .. ..
. .............
.............
.. ... . .. ..
MP CP Q A .
. M ... .. ........ .
θ .... . . ... .........
........................................................................................................................................................................................................................................................................................................................................................
.......... . . .... .. . .
. . P
= − (1) ..........
..........
..........
.
.. ..
.. ..
.
.......... ... .. .................... ..... .... . .
θ
....... .. .... .... . ...... ...................
γ
.....
..
B.
.
.
.
. ...
...
...
sin α sin(γ + θ) ........................
............... α
.....
.....
.....
.
..
..
.
...
..... ..
F ... .............
... ..........
..........
.....
.
.....
..
. ....
... .......... ..... .. ...
... .......... .. ...
Apply sine rule to 4EM P , ...
...
...
..........
..........
..........
.....
.....
.....
..... .
.. ....
. ..
... .......... ..... ... ....
..........
...
... β
.......... .
.......... ......... .. .....
........... .........
.
....
MP EP .....
.....
...........
...
.
= − (2) .....
...... D
......
.....
..
...... ......
sin β sin θ .......
.........
.............
............................... .. .. ... .
. ..........
.......
MP2 CP · EP
(1) × (2) gives = . – (3)
sin α sin β sin(γ + θ) sin θ
QM QD
For 4QDM , = . – (4)
sin β sin(γ + θ)
QM QF
For 4QF M , = . – (5)
sin α sin θ
QM 2 QD · QF
(4) × (5) gives = . – (6)
sin α sin β sin(γ + θ) sin θ
CP · EP QD · QF
By (3) and (6), 2
=
MP QM 2
M P 2 − AM 2 QM 2 − AM 2
Or equivalently, = .
MP2 QM 2
That is M P = QM .
Second solution Set up a rectangular coordinate system with M as the origin and QP as
the x-axis. Let the equation of the circle be x2 + (y + c)2 = r2 . Let the coordinates of C, D,
F and E be (p1 , ap1 ) and (p2 , ap2 ) (q1 , bq1 ), (q2 , bq2 ), respectively. Direct calculation shows
that the x-intercept of CE at the point P is p1 q2 (b−a)/(bq2 −ap1 ) and the x-intercept of DF
at Q is p2 q1 (a − b)/(ap2 − bq1 ). It suffices to verify that p1 q2 /(bq2 − ap1 ) = p2 q1 /(ap2 − bq1 ).
This equation can be rearranged to
The line CD has equation y = ax. Thus the x-coordinates of C and D are the roots of the
equation x2 + (ax + c)2 = r2 . That is p1 and p2 are the roots of the quadratic equation
(1 + a2 )x2 + 2acx + (c2 − r2 ) = 0. Similarly, q1 and q2 are the roots of the quadratic equation
(1 + b2 )x2 + 2bcx + (c2 − r2 ) = 0. Using the relations between roots and coefficients, we have
p1 + p2 = −2ac/(1 + a2 ) and p1 p2 = (c2 − r2 )/(1 + a2 ). Similarly, q1 + q2 = −2bc/(1 + b2 )
and q1 q2 = (c2 − r2 )/(1 + b2 ). With these, (∗) can be easily verified.
1.3 If maxj mini aij = mini maxj aij = aαβ , then clearly aαβ is at once the largest number
in the αth row and the smallest numbers in the βth column, and hence
aαj < aαβ < aiβ for all i 6= α and for all j 6= β. (∗)
Conversely, if (∗) holds for some aαβ , then mini aij ≤ aαj < aαβ for all j 6= β and maxj aij ≥
aiβ > aαβ for all i 6= α would imply that maxj mini aij = aαβ = mini maxj aij . To obtain a
required configuration, it is therefore necessary and sufficient to choose any 2n − 1 of the
given n2 numbers, say x1 < x2 < · · · < x2n−1 . Put xn anywhere in the array. Then put
x1 , x2 , . . . , xn−1 in the same row as xn and put xn+1 , xn+2 , . . . , x2n−1 in the same column
as xn . The remaining n2 − 2n + 1 numbers can be used to fill up the remaining n2 − 2n + 1
positions. Therefore, the total numbers of such configurations is
!
n2 (n2 )!(n!)2
· n2 · [(n − 1)!]2 · (n2 − 2n + 1)! = .
2n − 1 (2n − 1)!
2.1 The answer is 290. First observe that the arithmetic sequences {p + mk, k ∈ N} and
{q + nk, k ∈ N} are disjoint if and only if p − q 6= ln − km for all integers k, l, which holds if
and only if gcd (m, n) does not divide p − q. Therefore, the required r cannot be relatively
prime to 10 = (2)(5), 26 = (2)(13) and 29. We start by choosing r to be the smallest
lcm of d1 , d2 , d3 where d1 , d2 , d3 are factors (greater than 1) of 10, 26 and 29 respectively.
The smallest such r is 58 = (2)(29). In this case, gcd (10, 58) = 2, gcd (26, 58) = 2 and
gcd (29, 58) = 29. We also require b − 3 6≡ 0 (mod 2), b − 6 6≡ 0 (mod 2) and b − 5 6≡ 0
(mod 29). But there is no solution for b from the first two equations. Therefore we cannot
take r = 58. The next smallest lcm would be 290 = (5)(2)(29). In this case, a simple
checking using the above criterion shows that {1 + 290k, k ∈ N} is disjoint from A
...................................................
2.2 Let the radius of Γ be r1 and the radius ........... ........................ S
........ ...........
....... .......
.......
....... ..... .. ....
... ..............
.
..... ..... P
of the inscribed circle be r2 and its center be .
....
.
.
.
....
.
..
..
.
..
.
..
...
.....
.. .........
..........
...........
... .. .. ... ..... ....
F . First we have P B 2 = AB · BM because Γ .
.
..
.
...
.
...
.
...
..
F ....
·
.
...
.
..
.
..
.
..
·Q
.. ..... .....
...
...
...
. .
... ... ... ... .... ...
4AP B is similar to 4P M B. ...
....
...
...
... .
..
.
..
...
.. . . ...
...
..
.. ...
.....
......
...
...
...
... ... .. .. ..
Using Pythagoras’ Theorem, RO2 = F O2 − ...
...
..... ...
...... ..
........ .
........
.. ..
......
.
.
..
... ...
...
....
...............................................................................................................................................................................................
...
...
.
A B
F R2 = (r1 − r2 )2 − r22 = r12 − 2r1 r2 = AO2 − O R M
2AO · RM .
Thus, AO2 − RO2 = 2AO · RM = AB · RM . Therefore, AB · RB − RB 2 = AR · RB =
AO2 − RO2 = AB · RM . From this, we have AB · (RB − RM ) = RB 2 . Therefore,
P B 2 = AB · BM = AB · (RB − RM ) = RB 2 and so P B = RB.
Note that since ∠P AM = ∠BP M , we have P B = RB if and only if ∠BP R = ∠BRP
if and only if ∠AP R = ∠M P R if and only if P R bisects ∠AP M . With this observation,
if we inscribe another circle in the curvilinear triangle P BM touching M B at a point
R0 , then ∠RP R0 = 45◦ . Note also that S, Q, B are in fact collinear. Using this, we can
obtain another solution as follow. Let the extension of P M meet the circle at M 0 . Then
BR2 = BQ · BS = BQ2 + BQ · QS = BQ2 + P Q · QM 0 = BQ2 + (P M − M Q)(M 0 M +
M Q) = BQ2 + (P M − M Q)(P M + M Q) = BQ2 + P M 2 − M Q2 = M B 2 + P M 2 = P B 2 .
There is an even shorter proof due to Colin Tan. The fact that S, Q, B are collinear gives
∠ASQ = 90◦ . Thus 4ABS is similar to 4QBM so that AB/BS = BQ/BM . Therefore,
P B 2 = AB · BM = BS · BQ = BR2 .
Put m = 1 in the original functional equation and denote for simplicity f (f (2)) by k. We
obtain f (f (f (n)) + 1) = −k − n. Using this and equating (1) and (2), we get f (−m − k) −
f (−n − k) = m − n. Letting m = −p + k and n = −p + k + 1, we have f (p) − f (p − 1) = −1.
Inductively, we obtain f (p) = f (0)−p. Thus, f (f (p)) = f (f (0)−p) = f (0)−(f (0)−p) = p.
Substituting this into the original functional equation, we have f (m + n) = −m − 1 − n. In
other words, f (n) = −n − 1 for all n. Indeed, f (n) = −n − 1 satisfies the given functional
equation.
(2nd solution by Teo Wei Hao) Setting m = 0, we obtain f 3 (n) = −f 2(1) − n. This
functional relation immediately shows that f is bijective, because f f 2 (n) = −f 2 (1) − n
and f (p) = f (q) =⇒ f 3 (p) = f 3 (q) =⇒ p = q. Now we may let f 2 (n0 ) = 1 for some n0 .
The original functional equation becomes f (m + 1) = −f 2 (m + 1) − n0 . Using the fact
that f is bijective, we may let f (m + 1) = x, so that f (x) = −x − n0 . Applying this on
f 2 (n0 ) = 1 gives n0 = 1. Therefore, f (x) = −x − 1.
(3rd solution by Colin Tan) Let f 2 (1) = c. Put n = 1, m = 0 and replace n by f (n), we
get, respectively,
f (m + c) = −f 2 (m + 1) − 1 (1)
3
f (n) = −c − n (2)
3 2
f (m + f (n)) = −f (m + 1) − f (n) (3)
f (m + c) + 1 = f (m − c − n) + f (n) (4)
2003/2004
1.1 Let M = x(x + 1)(x + 2)(x + 3)(x + 4)(x + 5)(x + 6)(x + 7), where x is a positive integer.
Then M = (x2 + 7x)(x2 + 7x + 6)(x2 + 7x + 10)(x2 + 7x + 12). Let a = x2 + 7x + 6 =
(x + 1)(x + 6) ≥ 2 × 7 = 14. Thus M = (a − 6)a(a + 4)(a + 6) = a4 + 4a(a + 3)(a − 12) > a4 .
Also (a + 1)4 − M = 42a2 + 148a + 1 > 0. Therefore, a4 < M < (a + 1)4 . Consequently,
1
bN c = bM 4 c = a = (x + 1)(x + 6) which is an even integer.
C ...
..................
........... ... ...........
.............................. .....γ
1.2 Let the intersection of AD, BE, CF .
z
.. ..... . . .... ...................γ
. . .. .
.
.........................
.. .
.. .....................
............... .........
.................. ..
with BC, CA, AB be D0 , E 0 , F 0 respec- .
...........
.
.. .....
..............
.....
.. .
.....
. . .
.
.
.
.
..
... ..............
...
...
..........
..........
..........
E ............
. ..................................................... .... ..........
tively. It is easy to establish that ∠F AF 0 = ..
. . ... . ........ ... ........
........ ............... ................ .
.
. .. .
..... ... .......
........
.
........... .....
.. . ..
............. .
.
.
. .
. . .
........ D
........
....
..... . . .. . . .... ............... .........
.
∠EAE 0 = α, F BF 0 = ∠DBD0 = β,
.
... . .
.... .................. .. .
.
. .. . .
. . . . .
. ......
.. ...... 0 ...... .......
. . .........
..
.
.....
.
.
. .. ........ .
. E ...... .......
.. .. .
.
.
.
.
.... .
..
..
.
... .
...
........... ....
. . ......
......
....... .. ... ..... ...... .. ............ 0 ......
∠DCD0 = ∠ECE 0 = γ. Also AE = AF =
.
. ..... ......
..
.
. .. ....... ...
.
. ..
. ..
..
..... ...... .. ..............
..... .....................
.
.
. . . . . .. ..
... . .. D .....
.
.
.
... ...
......
.... ... ....... ..
. .... ..
. . .
. . ......
.... ............ ... . .. . . ................. ......... .. ...
. . .....
.... . . .. ...... . .... ..
x, BF = BD = y, CD = CE = z. The .
...
.. ...... ....
. .
.
..
.. ...
...
... ........ ............. ... .. . I .
... .
. .
.
. .. ......
........ ...
.
. ... .......
... .....
.... ........ .... . . . .. . ..
.
. . . ........ .. .
. .
... .. ....
. . .
ratio AF 0 /F 0 B equals to the ratio of the al- .
... .....
...α.....
... ........ ... .
...
.
... ............... .....
.........
........
...
.
.... ....
... ..
. ...
. . . .
......
. ..
... .......... .........β....
.
.... ..
.
. . . ..... . ... .......... ... ...
............ ............... ..... ..... ... ..
. .. .. .
........ ........
.
titudes from A and B on CF of the trian- . . ...... ........
... ...
.................
. ....
..
....
.........
..... ..........
...
.
.
. .
........
... . .......
...
. ... . ..............
... ..
A ............................................................................................................................................................................................................................................................................ B
................ α . ...... . . .
gles AF C and BF C and hence as the ratio ................
................ ...........
. ...
...................... F 0....
. .
.... .
.. . .
.. .
. .
. . . . β
................... . . . . .
.. . . .
x .................. ..............
..................... .... ......................................... y
of their areas. .....................................
F
AF 0 BD0 CE 0
It follows that = 1, so by Ceva’s Theorem, AD, BE and CF are concurrent.
F 0 B D0 C E 0 A
1.3 The integer pair (x, y) is a solution of the given equation if and only if x + y = 0 or
(x, y) = (0 ± 1), (±1, 0), or ±(2, 2). Clearly, if x + y = 0, then (x, y) is a solution. Assume
now that (x, y) is a solution with x + y 6= 0.
We first show that xy ≥ 0. Dividing both sides of x5 + y 5 = (x + y)3 by x + y yields
x4 − x3 y + x2 y 2 − xy 3 + y 4 = (x + y)2 .
This is equivalent to
(x2 + y 2 )2 + x2 y 2 = (x + y)2 (xy + 1),
and it follows that xy ≥ 0.
Next we show that |x + y| ≤ 4. The convexity of the function f (t) = t5 on [0, ∞) implies
that for nonnegative x and y,
5
x5 + y 5 x+y 1
≥ , or equivalently, x5 + y 5 ≥ (x + y)5 .
2 2 16
If x + y > 4, then x5 + y 5 > (x + y)3 . Similarly, if x and y are both nonpositive with
x + y < −4, then x5 + y 5 < (x + y)3 .
Finally, examining the cases where xy ≥ 0 and |x + y| = 1, 2, 3 or 4, we find the solutions
(x, y) = (0 ± 1), (±1, 0), or ±(2, 2).
...............................
............... .........
......... .......
2.1 We have to show that ei- ....... ......
...... ......
.. .. ..... .....
. ....
. .....
.....
ther AB is parallel to CD or ...
..... .
...
...
A..
.
.
...............................................................................................................................................................................................................................................................................................
...
D
AD is parallel to BC. Us- H .......
.......
....... .
... ....
.... ...
.. ...
.. ....
.
. .
....... .... .. .. ...
ing the powers of H nd K re- .......
.......
.......
.......
........ .....
... ...
...
...
.
... ....
...
.
...
....... ... ... ... ... ...
spective to the circle, we have .......
.......
.......
....... ..
... .....
.. .
...
...
.... ..
..
..
. .
.
..
..
.
.
. . . .
. .
.
HB 2 = HA · HD = 2HA2 and
....... ... ...
....... .... ... ... ... ...
....... ..
. ... ... ... ...
.......... ... ... ... ..
..
.. ............ ..
KA2 = KB · KC = 2KB 2 . ...
...
. ....... ...
...
....... ...
....... ..... ....
...
... .
...
... ....
...
..
.
.
Note that ∠HBA = ∠KAB. Applying sine rule to triangles ABK and BAH, we obtain
KA/KB = sin(∠ABK)/ sin(∠KAB) and HB/HA = sin(∠HAB)/ sin(∠HBA). Thus
sin(∠ABK) = sin(∠HAB). Therefore, either ∠ABK = ∠HAB or ∠ABK + ∠HAB =
180◦ . Consequently, AB is parallel to CD or AD is parallel to BC.
2.2 The smallest such value of k is 1/4. First note that for x, y > 0,
1 4xy 1 (x + y)2 1 1 1 1
= · ≤ · = + ,
x+y x + y 4xy (x + y) 4xy 4 x y
ab bc ca
with equality if and only if x = y. We then have + +
a + b + 2c b + c + 2a c + a + 2b
ab 1 1 bc 1 1 ca 1 1 1
≤ + + + + + = (a + b + c),
4 a+c b+c 4 c+a b+a 4 c+b a+b 4
with equality if and only if a + b = b + c = c + a, that is a = b = c.
No calculator is allowed
1. In 4ABC, ∠C = 90◦ . D is a point on the line AB. E and F are, respectively, the
feet of the perpendiculars from A and B to the line DC. ` and m are, respectively,
lines through E and F parallel to CB and CA. Prove that the lines AB, ` and m
are concurrent.
......
B
..............
m `
... ......
....
........ ..... ..
........ ......... ....
.. ........ .... ....
....
... ......... .....
... .... ..
.
..........
..
.. . .
.
I
... ....................
. .
..
. ..
...... ..
...
... .
. .
....... .. .... .... ..
...
.
... ... . .... .
.... ..
. ...... .... ...
. .
...
..... .... .. .
.. ...
..
..
..
.... ... .. . . ... ..... ..
. .
... . .. .
...
..
.. ....A
... ........... .
.. .
..
. .. . ... ..
.
. .
..
...
...
. ..
..
. .... .
...
.
...
..
..
..
........ .......... . ..
. . .
. .....
..
. ..
. ...
.
......... .
.
.....
..... .. .. ... ...
. ..
.......... .
. . ..
.. .
... .
. ...
. ...
........ .
. ..... .. ... .. ...
....... . . .
.....
. .. ..
. .. . . ..
.......
. .
. .
.
. ........ ...
. .. .. ... .
...
.... .
. .. .
... ... ....
..
........ . ....
.
.
.....
.....
. .
. .
.... .. ..... .... ... ..
...
..... .
. ...
α ... .. . ...
...................................................................................................................................................................................................................................................
D E C P F
Let ∠ACE = α. Then
EP1 + P2 F = EF
⇔ EI1 sin α + F I2 cos α = AC cos α + BC sin α
⇔ (F I2 − AC) cos α = (BC − EI1 ) sin α
CF CE
⇔ AC cos α = BC sin α
CD CD
CF CE
⇔ cos α = sin α
BC AC
⇔ sin α cos α = cos α sin α
Since the last line is true, we have EP1 + P2 F = EF , i.e., I1 = I2 and we are done.
DF DF
¡ −CD ¢ AC·CF
Note IF = AC · CD , so that IF − AC = AC · CD − AC = AC DFCD = CD .
1
Soln: By taking the geometric mean of the terms on each side of the inequalities
1 a+b 1 1 a+b+c 1
a= (a + a + a), ≥ (a + (ab) 2 + b) and = (a + b + c),
3 2 3 3 3
we have
· µ ¶ µ ¶¸ 13
a+b a+b+c 1 1 1 1 1
a· · ≥ (a + a + a) 3 (a + (ab) 2 + b) 3 (a + b + c) 3 .
2 3 3
1 1 3 1
(a + (ab) 2 + b)(a + b + c) ≥ (a + a 4 b 4 + (bc) 2 )2 .
Thus, we have
· µ ¶ µ ¶¸ 13
a+b a+b+c 1 1 1 3 1 2
a· · ≥ (a + a + a) 3 (a + a 4 b 4 + (bc) 2 ) 3 .
2 3 3
2 2
1 1 3 1 2 1 2 1 1 3 1 1 1 1
(a+a+a) 3 (a+a 4 b 4 +(bc) 2 ) 3 ≥ a 3 a 3 +a 3 (a 4 b 4 ) 3 +a 3 ((bc) 2 ) 3 = a + (ab) 2 + (abc) 3 .
Consequently,
· µ ¶ µ ¶¸ 13 1 1
a+b a+b+c a + (ab) 2 + (abc) 3
a· · ≥ .
2 3 3
Level. Medium
2
3. Let n be a positive integer such that the sum of all its positive divisors (inclusive
of n) equals to 2n + 1. Prove that n is an odd perfect square.
3
Singapore International Mathematical Olympiad
No calculator is allowed
3
x + 2x2 + · · · + 7x7 + 45x10 = = y + 2y 2 + · · · + 7y 7 + 8y 8 + 9y 9 .
2
3
≥ y + 2y 2 + 3y 3 + 4y 4
2
1 1 1 1
≥ + 2( )2 + 3( )3 + 4( )4 ,
2 2 2 2
3
= x + 2x2 + · · · + 7x7 + 45x10
2
≤ y + 2y 2 + · · · + 7y 7 + 45y 10
3
< y + 2y 2 + · · · + 7y 7 + 8y 8 + 9y 9 = ,
2
5. Two circles Γ1 and Γ2 lying outside each other are tangent to a straight line ` at
points E and F respectively such that both are on the same side of `. The two
internal common tangents `1 and `2 to the two circles intersect at the point A such
that `1 meets ` at B and is tangent to Γ2 at H, and `2 meets ` at C and is tangent
to Γ1 at G. Suppose the extensions of EG and F H intersect at P . Prove that P A
is perpendicular to `.
4
Soln: Let the extension of P A meet ` at D. Let ∠GP A = x1 and ∠HP A = x2 .
ED P E sin x1
Applying sine rule to 4P ED and 4P F D, we have = · .
DF P F sin x2
P
.......
.........
.
.... ........
..
... ... ...
............. ...
... ..............
.
.
...
. ... .. x
...
...
....1 .....
... ...2 x
.
.. ... ...
.
. ... ...
... ...
...
. ..... ...
..
. .
... ...
.
.. ... ...
.
.. . ...
.
. .
... ...
....
. ..
. ...
..
. ... ...
.
.. .
. ...
. .. .... ...
...
.
.. .
..
.. . ...
..
. ... ...
... ... ...
... ... ...
2 ` . ..
.... .
.
.
.
...
...
..
. ....
.... .
...
.. . ..................................... ...
....... ..
. .
......
G ..
.
.
.
.
...
...
...
...
.
.
..
.....
.
... ........
. .........
. .
.........
.
.
...
...
...
... .......
` 1
.
...
.
...
..
... ...... ..
.
. H ... ..............
.. .. ... ...... . .
.
. ...............................
.. .
. . . .. ...
.. .
...
... ..... ... ....... ... ......
.. ... .....
..... ... ........... .. .....
.... O ... A ....... ... .. ...
... 1...... ... ........... ... ... ... ... ........... ... ... ... .......................................... ...... ...... ...
...
... . ... .. ...
. . . ....
. .
.. . . . .
. .
. ...
... ......... ... ....... ... ... ..... ...
...
...
.. ...
. ...
. .
.
..
..
... .
... .. ....... .. . ...... ..
. ..... .. . .
. O
... ..
.
. 2 ...
.
Γ 1 .......
... . ...
.
.. ....
.. ... ...
. ..
. ....
.................
..
...
. .
..
.
.
.. ..... ...
.......
. . .
... .
... .. .
.
...
.
.
Γ2
..... .
....... . .
...... ..... .
.
......
...... ....... .. ..... .... .....
...... ....... . .... ......
...............................................................................................................................................................................................................................................
`
EB D CF
sin x1 AG sin x2 AH
By sine rule on 4P AG and 4P AH, we have = and = .
sin ∠AGP PA sin ∠AHP PA
Thus
sin x1 AG sin ∠AGP AG sin ∠P EF O1 E P F
= · = · = · ,
sin x2 AH sin ∠AHP AH sin ∠P F E O2 F P E
where O1 , O2 are the centres of Γ1 and Γ2 respectively. Therefore,
ED P E sin x1 O1 E
= · = .
DF P F sin x2 O2 F
Since O1 A : O2 A = O1 E : O2 F , AD is parallel to O1 E. Thus, P A is perpendicular to
`.
6. Let k ∈ N and the numbers {1, 2, 3, . . . , 4k} are placed arbitrarily at 4k points
on the circumference of a circle. Prove that there exist 2k nonintersecting chords
joining these 4k points in pairs so that the difference between the numbers at the
endpoints of each chord is ≤ 3k − 1.
Soln: f (k) = 3k − 1
Lemma. Given any 2n-gon with n red vertices and n blue vertices, we can always
find n non-intersecting line segment with different coloured endpoints.
Induction. Clearly, There are 2 consecutive points of different colours. Draw a line
segment linking these 2 points. Now induct.
S
Now, Set A = {1, 2, . . . , k} {3k + 1, 3k + 2, . . . , 4k}. Set B = {k + 1, k + 2, . . . , 3k}.
5
By the above lemma, we can find a set of 2k lines satisfying given condition.
To prove 3k − 2 not possible, consider the clockwise arrangement of points {1, 3k +
1, 2, 3k + 2, 3, 3k + 2, . . . k, 4k} at the top and {k + 1, k + 2, k + 3, . . . , 3k} at the bottom.
Since no two consecutive vertices can be connected with a line at the top, all 2k line
must joint a top point and a bottom point. Hence, points 1 and 3k must be joined,
contradicting the assumption that the largest difference is 3k − 1.
6
Singapore International Mathematical Olympiad
National Team Selection Test I 2006
1. Let AN C, CLB and BKA be triangles erected on the outside of the triangle
ABC such that ∠N AC = ∠KBA = ∠LCB and ∠N CA = ∠KAB = ∠LBC.
Let D, E, G and H be the midpoints of AB, LK, CA and N A respectively.
Prove that DEGH is a parallelogram.
3. Let n be a positive integer such that the sum of all its positive divisors (in-
clusive of n) equals to 2n + 1. Prove that n is an odd perfect square.
1. Let AN C, CLB and BKA be triangles erected on the outside of the trian-
gle ABC such that ∠N AC = ∠KBA = ∠LCB and ∠N CA = ∠KAB =
∠LBC. Let D, E, G and H be the midpoints of AB, LK, CA and N A re-
spectively. Prove that DEGH is a parallelogram.
Solution. The given condition means that 4AN C, 4CLB and 4BKA are
all similar.
C L
......................................
........................... ... ......
......... ..............
........... ....... ... .......
.
.................. .......
.......
... ......
.....
.
........ ....... ...
........
...
.. .....
.... ....... .....
.. .... .
.... .......
. . . .. ....... ..... .....
.....
... .... .
....... .....
......
G.......
. ..
.....
.... ... ... ... ...
R .. .. .
................ .....
.....
. . . ...
.. . .. .....
N .. .
..... ................
.. . ... ... ... ... .
.. ... ... ... ..
. ... ... ... ... ...... ...... ... . .
.
...... .. ........
.
........ .....
... ............. . ........ ..........
.
........ ....... ... E
........... ......
. ....
... .......... ............ ....... .....
....... .....
H ... .............. ... ... ... . . ...
.... ...... ... ..... ....... .....
....... ..... . P
.. ... .. ... ... ... ...
............ ... ....... ............ ......
......... ... ... ... ... ...
. ........... . ... ......... .........
A ..........................................................................................................................................................................................................................................................................................
............... ............... ..... ..
...............
............... D ....................
............... ......... . .
......
.... B
...............
............... ... .......................... ............
............... ... ........
..
............... ... ........
...............
............... .
.
..............................
....F
...
...
K
[Remark by Lim Wei Quan] Let M be the 4th vertice of the parallelogram
AKBM . Let F be the midpoint of BC. Then since triangles BM D, BLF ,
CN G are similar, M L = DF ×BL/BF = AC/2×2CN/CA = CN Similarly,
M N = CL Thus, M N CL is a parallelogram This gives, DE = M L/2 =
CN/2 = GH Also, DE||M L||CN ||GH Therefore, DEGH is a parallelogram.
n
xk 1 n−1 1 1
X X
= Sn · + Sk − .
k=1 k n k=1 k k+1
Thus
n n−1 n−1
X x X
1 1
1 1
k X
= Sk − ≤ |Sk | −
k=1 k k k + 1 k=1 k k+1
k=1
n−1
X 1 1 1 1 1
≤ − = 1− .
k=1 2 k k+1 2 n
(b) At any stage, it is not possible for there to be i < j for which pi = pi+1 ,
pj = pj+1 , and pi+1 − pj ≤ j − i − 1 (that is, the average decrease per
column from column i + 1 to column j is 1 or less).
In the proofs of (a), (b) and (c), we use the following terminology. Let a
k-switch be the movement of one pebble from column k to column k + 1, and
for any column i let a drop be the quantity pi − pi+1 .
To prove (a), suppose a sequence of valid moves resulted in pi < pi+1 for the
first time at some stage. Then the move leading to this stage must have been
an i-switch, but it would be contrary to the condition that column i have at
least 2 more pebbles than column i + 1, to allow switches.
To prove (b), if such a configuration were obtainable, there would be a mini-
mum value of j − i overall such obtainable configurations, and we now show
that there is no minimum. Suppose p1 , p2 , . . . was such a minimal configura-
tion. It cannot be that j = i + 1, for what would columns i, i + 1, i + 2 look
like just before the move that made the height equal? The move must have
been a k-switch for i − 1 ≤ k ≤ i + 2, but if so the configuration before the
switch was impossible (not decreasing).
Now suppose j > i + 1. Consider the first configuration C in the sequence for
which columns i, i + 1, j, j + 1 are at their final heights. Note that from pi+1
to pj the column decrease by exactly one each time in C, because if there
was a drop of 2 or more at some point, there would have to be another drop
of 0 in this interval to obtain an average of 1 or less, and thus j − i is not
minimal. The move leading to C was either an i-switch or a j-switch. If it
was the former, at the previous stage columns i + 1 and i + 2 had the same
height, violating the minimality of j − i. A similar contradiction arises if the
move was a j-switch.
Finally, to prove (c), if any drop is 2 or more, the configuration isn’t final.
However, if all drops are 0 or 1, and there were two drops of 0 between
nonempty columns (say between i and i + 1 and between j and j + 1), then
(b) would be violated. Thus a final configuration that satisfied (b) also
satisfies (c).
Now B, A0 and C are collinear points, one on each side (extended if necessary)
of triangle AGB 0 . By Menelaus’ theorem, we have
AA0 GB B 0 C
· · = −1. (2)
A0 G BB 0 CA
Similarly, from triangle CGB 0 and collinear points C 0 , B and A, we have
CC 0 GB B 0 A
· · = −1. (3)
C 0 G BB 0 AC
C0
....
... ...
... .....
.. .. ...
A A .
...
...
...
...
........... ..... ...
... ... ...... .... ............... ...
... .. ...... ... ..........
...........
...
.... ..... ........ .... . ..
..... .....
...
..
.
... ... .....
..... ...
. ...... ..... .....
. ...
... ... ..... ... . .
..... ..... ....
..
... ... ..... .... . .
..... ....... ....
.
.... ... ..... ..
. ...... ....... ...
... ... ..... .
. ... ..... ... ..... ......... G
0 ..... ... ..... 0 ... ..... ........
C B .... .. .. .....
....
.
. .........
.
.
... ...........
.
...
.
B 0................... ..............
...
.
. ........
....
...
. .
.. ..
. ....... ........ ....
..... ....... ......... ..............
........ .. ....... . ..... ..... . ..... ... ......
.
... ........ .. ....... ..... .... ...
. . . ..... ... ......
... ...... ..... . ......
... ............. ... ....... ..... ... ......
....... ..... ................ ..... ... ....... ..... ... .....
.. ......... .. .....
... ......... ..... ... .....
.
.
...
.
..
............
. G .
...
.
.
.. ..........
..
..........
.....
..... .
...
.
...........
.. . ..... ...
..... ...
.....
.....
.....
.
.. ........... ... .... ........ ......... ...
. .
.......... ..
. ..
........ .....
.... ........... ...
. . ........... ...... ... .
.......... .. . .....
.....
.. .............. ... ....... ... . . .. ........... ........
. .
.... .....
.
.......................................................................................................................................................................... . .... .
...........................................................................................................................................................................................................
B A0 C B C A0
By adding 1 to each member of AG/GA0 = CG/GC 0 from (1), we have
AA0 CC 0
= . (4)
GA0 GC 0
An easy combination of (2),(3),(4) gives
CB 0 = B 0 A. (5)
Since these are directed segments, (5) means that B 0 is the midpoint of AC.
Similarly, by selecting other triangles, we obtain C 0 as midpoint of AB, and
A0 as midpoint of BC. Thus G is the centroid of 4ABC.
So (1) implies that A0 , B 0 , C 0 lie in the open segments BC, CA, AB, respec-
tively, and the “open segments” restriction in the hypothesis is unnecessary.
In order for any other conclusion to hold, we must interpret the symbols in
(1) to mean undirected segments. Then, upon replacing them by directed
segments we see that either all ratios have the same sign - the case already
discussed - or two have one sign and the third the opposite sign. Suppose,
say,
AG CG BG
0
= 0
=− . (6)
GA GC GB 0
By the original argument, the first equation implies (5) so that B 0 is the
midpoint of AC. From AG/GA0 = −BG/GB 0 , we have
AG GA0 BG GB 0
+ − 1 = − − + 1,
GA0 GA0 GB 0 GB 0
whence
AA0 BB 0
= 2 − . (7)
GA0 GB 0
Now (5) implies that B 0 C/CA = −1/2, whence (2) becomes
AA0 GB
· = 2. (8)
A0 G BB 0
Putting GB/BB 0 = GB 0 /BB 0 − 1 in (8) and eliminating AA0 /A0 G from (7)
and (8), we obtain
2r2 − r + 1 = 0, (9)
where r = GB 0 /BB 0 . But the roots of (9) are not real, so that this case
cannot hold. Thus the case of medians is the only conclusion.
To verify that this S is indeed valid, simply check that if any one of the first
eleven values of a are changed, then at least two of a12 , a13 , a14 , a15 must also
be changed, and if any two of the first eleven values of a are changed, then
at least one of a12 , a13 , a14 , a15 must also be changed.
6. Let n be a positive integer such that the sum of all its positive divisors
(inclusive of n) equals to 2n + 1. Prove that n is an odd perfect square.
1. Find all pairs of nonnegative integers (x, y) satisfying (14y)x + y x+y = 2007.
4. Two circles Γ1 and Γ2 touch externally at the point C and internally at points
A and B respectively with another circle Γ centred at O. Suppose the common
tangent of Γ1 and Γ2 at C meets Γ at P with P A = P B. Prove that P O is
perpendicular to AB.
1. Find all pairs of nonnegative integers (x, y) satisfying (14y)x + y x+y = 2007.
Solution. Suppose x and y are nonnegative integers satisfying (14y)x + y x+y =
2007. Note that y ≥ 1 because y = 0 does not satisfy the equation. For x ≥ 3,
we have (14y)x + y x+y ≥ 14x ≥ 143 = 2744 > 2007. Thus x = 0, 1 or 2. If
x = 0, then the equation becomes 1 + y y = 2007. That is y y = 2006 = (2)(17)(59)
which obviously has no solution in y. If x = 1, then 14y + y y+1 = 2007. That is
y(14 + y y ) = 2007 = 32 (223). Thus y = 1, 3, 9 or 223. But none of these satisfy
14y + y y+1 = 2007. If x = 2, then y 2 (142 + y y ) = 32 (223). Since y = 1 does not
satisfy this equation, we must have y = 3. Consequently x = 2 and y = 3 and they
do satisfy the given equation. Therefore (x, y) = (2, 3) is the only solution to the
equation.
E .....................
..................................................
...........
..........
......... .. .... .........
. .. . . ........ .. ..
........
.......
.. . .. .... .. . ......
......
..... .. .. ......
. . . . .
. .. ..... . . .. .....
.....
....... .. . ..
.. .....
..
.... .. . .....
.. ..... . . ..
..
.....
....
.. .. .. . ...
.... .
.. ..
..
...
...
... . ...
... .. .. ...
. . . .
..
..
. . . .. ...
... . . ...
.. .. .. ...
. . .. ...
... .. .. ...
.... .. ... ...
... .. .. ...
... .. . ...
... . .. .. ...
. .. ...
... .. .. ...
...
·O
.. ... ...
....
...
... ..
..
.. ..
..
.. .
.....
. .
.. ..
........................
. .
...
D
... .. .. ................................. .........
... .. ............. ....... . .
.........
... .. ............ .... .............. ... ... ..
.. ............ .
... ............. ........
....... ...
.. .. ..
...
.
..
. ........ .
.............. ......
. .
... .... ....
... .. ...
..
. ............. .......
.. . ..
... .. ..
... ............ ....... ..
.. .... ........
... .. ............. .......
............. .......
...
...
... .. ..
..
........ . . . . . . .. . . . ............
.......
............. N .
................................................... ......... .......
.. ... ... ..
....
... .....
... .. ........................ ......................................................
....................................................................................................................................................................................................................................................................................................................................................
..................... . . .
A ...
...
................
................
................ M ... ..
....
. .. . . .. ....... .. .
.......
....... R
.
... .......
.. .......
.
...
.
C
...
..... ....................... .......
....... .. ........
..... . . . ... .......................... ....... ..... ................
..... .. . ................... .. ....... . .
.. .. ..
..... .... ................ .
................ .............. ................
..... ... .. ...............
..... ........ ...... ..
.......... ... ... ... ... ... ... ... ... ... ... ... ... ... ... ... ... ... ... ... ... ... ... ... ... .................................................
...... ....
F .......
.......
........
.......... ........
.......
......
..
B
............ ..........
...................................................................
(1234),(2345),(3456),(4567),(5678),(6781),(7812),
(8123),(1256),(2367),(3478),(4581),(1357),(2468)
4. Two circles Γ1 and Γ2 touch externally at the point C and internally at points A and
B respectively with another circle Γ centred at O. Suppose the common tangent
of Γ1 and Γ2 at C meets Γ at P with P A = P B. Prove that P O is perpendicular
to AB.
Solution. Let r1 , r2 and r be the radii of Γ1 , Γ2 and Γ respectively. It suffices to
prove that r1 = r2 . Then P, O, C are collinear and P O is perpendicular to AB.
P
...........................................................
......... ..... ... .......
....... ......
...... ... ... ... ......
........ .. .. ... .....
......
.
.
.... .... .... .....
....
. ... .. ... .....
. ...
.
.. .. .
.
. ... ...
.
.... ..
. ..
. ...
...
...
...
... ..
. ..
. . ...
.. ...
.... ..
. . ... ...
... ... ..
. .
... ...
... ... ..
. ... ...
...
... X ..
..... ... .........
...
...........
. ....... .. .......
..
.
.
.
. .. ...
. ..
.
.........
. .. .
.
Y
... ........
...
..
.
... .... .
. . .
. . . . ..
... .
..... ... ..... .. ....
. . .. ... .....
. ....
... .. . ... . .. ... ... .
... .. .
. .. .
....... .
... .... .... . .
.
... ... ... ...... ... ... ...
... ... ... ... ... ... ...
... ..
...... ....
..... ..
.. C .
.
.
.....
. ....
...
... ........
... .....
. ..
..... .. ..
. .... ... ..... .
....... .
.. ..... . ..
..... .... .....
............. .............. .......
...................................
A ................
........
........... ........
.....
B
.........................................
P
Solution. Let S = i ai . Denote by L and R the expressions on the left and right
hand side of the proposed inequality. We transform L and R using the identity
X X
(ai + aj ) = (n − 1) ai . (1)
i<j i
And thus:
!
X ai aj X1 (ai − aj )2 n−1 1 X (ai − aj )2
L= = ai + aj − = ·S − . (2)
i<j
ai + aj i<j
4 ai + aj 4 4 i<j ai + aj
X
To express R we express the sum ai aj in two ways; in the second transformation
i<j
identity (1) will be applied to the squares of the numbers ai :
!
X 1 X
ai aj = S2 − a2i ;
i<j
2 i
X 1 X 2 n−1X 2 1X
ai aj = ai + a2j − (ai − aj )2 = ai − (ai − aj )2 .
i<j
2 i<j 2 i
2 i<j
Multiplying the first of these equalities by n − 1 and adding the second one we
obtain
X n−1 1X
n ai aj = · S2 − (ai − aj )2 .
i<j
2 2 i<j
Hence
n X n−1 1 X (ai − aj )2
R= ai aj = ·S− . (3)
2S i<j 4 4 i<j S
Now compare (2) and (3). Since S ≥ ai + aj for any i < j, the claim L ≥ R results.
6. Let A, B, C be 3 points on the plane with integral coordinates. Prove that there
exists a point P with integral coordinates distinct from A, B and C such that
the interiors of the segments P A, P B and P C do not contain points with integral
coordinates.
Solution. Let A = (a1 , a2 ), B = (b1 , b2 ), C = (c1 , c2 ) where a1 , a2 , b1 , b2 , c1 , c2 are
integers. If a lattice point X = (x1 , x2 ) is between A = (a1 , a2 ) and B = (b1 , b2 ),
then there exist relatively prime positive integers u and v such that (u + v)xi =
uai + vbi , i = 1, 2. That is (u + v)(xi − bi ) = u(ai − bi ), i = 1, 2. This implies
that (u + v) divides ai − bi , for i = 1, 2. That is a1 − b1 and a2 − b2 have a
common factor. Therefore, it suffices to find a lattice point P = (x1 , x2 ) such that
gcd(x1 − a1 , x2 − a2 ) = 1, gcd(x1 − b1 , x2 − b2 ) = 1 and gcd(x1 − c1 , x2 − c2 ) = 1.
Let p = 2 or 3. There are at least 4 distinct values (m1 , m2 ) modulo p. Thus
there exists (r1 , r2 ) such that (r1 , r2 ) 6≡ (a1 , a2 ), (b1 , b2 ), (c1 , c2 ) (mod 2). Similarly,
there exists (s1 , s2 ) such that (s1 , s2 ) 6≡ (a1 , a2 ), (b1 , b2 ), (c1 , c2 ) (mod 3). By the
Chinese Remainder Theorem, there exists (p1 , p2 ) such that p1 ≡ r1 (mod 2),
p1 ≡ s1 (mod 3); and p2 ≡ r2 (mod 2), p2 ≡ s2 (mod 3). We can also require
p1 6= a1 , b1 , c1 and p2 6= a2 , b2 , c2 . This ensures that (p1 , p2 ) 6≡ (a1 , a2 ), (b1 , b2 ),
(c1 , c2 ) (mod 2) and (mod 3).
Suppose p is a prime larger than 3 such that p divides p2 − a2 , p2 − b2 , or p2 − c2 .
There are only a finite numbers of such primes p. Let the set of all such primes be Y .
For each p ∈ Y , pick an integer zp 6≡ a1 , b1 , c1 mod p. Since p > 3, this is possible.
Apply the Chinese Remainder Theorem to find an x1 such that x1 ≡ p1 (mod 2),
x1 ≡ p1 (mod 3) and x1 ≡ zp (mod p), p ∈ Y . Take x2 = p2 . Then P = (x1 , x2 )
is the desired point.
Singapore International Mathematical Olympiad
National Team Selection Test I 2008
P103 p−1 .
3. Find all odd primes p, if any, so that p divides n=1 n
Singapore International Mathematical Olympiad
National Team Selection Test II 2008
4. Let C be a circle centred at O, and let ABP be a line segment such that A, B lie
on C and P is a point outside C. Let C be a point on C such that P C is tangent
to C and let D be the point on C such that CD is a diameter of C and intersects
AB inside C. Suppose that the lines DB and OP intersect at E. Prove that AC is
perpendicular to CE.
for all x, y ∈ R.
6. Fifty teams participate in a round robin competition over 50 days. Moreover, all
the teams (at least two) that show up in any day must play against each other.
Prove that on every pair of consecutive days, there is a team that has to play on
those two days.
Solution to National Team Selection Test 2007
1. Find all pairs of nonnegative integers (x, y) satisfying (14y)x + y x+y = 2007.
Solution. Suppose x and y are nonnegative integers satisfying (14y)x + y x+y = 2007.
Note that y ≥ 1 because y = 0 does not satisfy the equation. For x ≥ 3, we have
(14y)x + y x+y ≥ 14x ≥ 143 = 2744 > 2007. Thus x = 0, 1 or 2. If x = 0, then the equation
becomes 1 + y y = 2007. That is y y = 2006 = (2)(17)(59) which obviously has no solution in
y. If x = 1, then 14y + y y+1 = 2007. That is y(14 + y y ) = 2007 = 32 (223). Thus y = 1, 3, 9
or 223. But none of these satisfy 14y + y y+1 = 2007. If x = 2, then y 2 (142 + y y ) = 32 (223).
Since y = 1 does not satisfy this equation, we must have y = 3. Consequently x = 2 and
y = 3 and they do satisfy the given equation. Therefore (x, y) = (2, 3) is the only solution
to the equation.
2. Let ABCD be a convex quadrilateral inscribed in a circle with M and N the midpoints
of the diagonals AC and BD respectively. Suppose AC bisects ∠BM D. Prove that BD
bisects ∠AN C.
Solution. Let O be the center of the circle. Let the extensions of CN and DM meet the
circle at E and F respectively. Join AE and BF .
E ......................
................................................
...........
........... .........
......... .. .... ........
. . .. . . ........ .. .. .......
. .. . ..... . .. ..
......
......
... . .... . . .
. ......
. .. ..... .
. .. .....
.....
....... .. . ..
.. .....
.....
...
. .. . . .....
.... . ..
. .. . . . ....
. . .. .. .. ...
... . . .. ...
... .. ...
. . . .. .. ...
...
. . . . .
..
...
...
..
. . . ...
... ... .. ...
... .. .. ...
.. . .. ...
. . .. ...
.... .. ... ...
... .. . ...
... .. .. ...
... . .. .. ...
. ..
... .. .. ...
... .. .. ...
·O
... .. .. ...
...
... ..
.. ..
.. . .
... . . . . .
.........................
. .. D
.. . .
... .
.................. .........
... .. ............ .. ... .. ... .
... .. ..... .... . . ..
. ......
. . ........
.. .. ........
... .. ............. ..
.
... ... ...
... .. ............. ......... ... ... ...
... .. ............ ....... ...
.. ............. ....... ... .... ....
...
... .. . .
..................... ............. ..
.. ..
. .. ..
........ ...... .
.. .... ........
... .. ............. .......
... .. ............. .......
...
... .. ..
....... . . . . . . . .. . . ............
................... N ..
............................................. ........ ...... .. ..
.. .. .. ..
.
. ..
.
...
.. .
.....
... .. ........................ ......................................................
....................................................................................................................................................................................................................................................................................................................................................
..................... . .. . .. .....
A ...
...
................
. . . .................
................ ......... M . .. . ..... ................
........
....... R ... ........
.. .......
C
...
..... .................
. . .
.......
. .
. .. ...............
..... ... ................ ....... . .......
..... ..... ................
................
.......
. ... ..........
..... .... ................ .............. ................
..... ..... ............. ..... .............
..... ........
...... ... ... ... ... ... ... ... ... ... ... ... ... ... ... ... ... ... ... ... ... ... ... ... ... ... ..................................................
...... ...
F ....... ...... B
....... .......
........ ........
.........
............ .. .
. ..............
....................... ......
...................................
First note that F is the reflection of B about the line OM so that M F = M B. Thus
∠BF M = ∠BM C and F B is parallel to M C. Therefore ∠BAD = ∠BF M = ∠BM C. As
∠ADB = ∠M CB, we have 4ADB is similar to 4M CB. Therefore, BD DA DA
BC = CM = AC/2
AC DA DA
so that BC = BD/2 = N B . As ∠DAC = ∠N BC, we see that 4DAC is similar to
4N BC so that ∠BN C = ∠ADC = ∠AEC. This means AE is parallel to BD. As N O
is perpendicular to BD, we have the extension of N O is perpendicular to AE and meets
AE at its midpoint. Thus 4AN E is isosceles with ∠N EA = ∠N AE. Consequently,
∠CN R = ∠N EA = ∠N AE = ∠AN R.
2nd Solution. Set up a coordinate system in which M is the origin, AC is the x-axis and
the center of the circle lies on the y-axis with coordinate (0, d). Let the radius of the circle
be r and let the gradients of the √ lines M D and M √B be α and −α respectively. Then the
coordinates of A and C are (− r2 − d2 , 0) and ( r2 − d2 , 0) respectively. By solving the
equation of the circle x2 + (y − d)2 = r2 and the equation y = αx of the line M D, we get
the coordinate of D equal to
√ √ !!
αd + r2 − d2 + α2 r2 αd + r2 − d2 + α2 r2
,α .
1 + α2 1 + α2
Replacing α by −α, we obtain the coordinate of B as
√ √ !!
−αd + r2 − d2 + α2 r2 −αd + r2 − d2 + α2 r2
, −α .
1 + α2 1 + α2
√ !
r 2 − d2 + α 2 r 2 α 2 d
Therefore, the midpoint N of BD has coordinate , . Let AC
1 + α2 1 + α2
2 2
r −d
intersect BD at R. Then the x-coordinate of R is found to be √r2 −d 2 +α2 r 2
. Thus a direct
calculation gives
2 p 2
2r2 − 2d2 + α2 r2 + 2 (r2 − d2 )(r2 − d2 + α2 r2 )
RA NA
= p = .
RC 2r2 − 2d2 + α2 r2 − 2 (r2 − d2 )(r2 − d2 + α2 r2 ) NC
RA NA
That is RC = NC . Using the Angle Bisector Theorem, we have ∠AN B = ∠CN B.
(1234),(2345),(3456),(4567),(5678),(6781),(7812),
(8123),(1256),(2367),(3478),(4581),(1357),(2468)
4. Two circles Γ1 and Γ2 touch externally at the point C and internally at points A and B
respectively with another circle Γ centred at O. Suppose the common tangent of Γ1 and
Γ2 at C meets Γ at P with P A = P B. Prove that P O is perpendicular to AB.
Solution. Let r1 , r2 and r be the radii of Γ1 , Γ2 and Γ respectively. It suffices to prove
that r1 = r2 . Then P, O, C are collinear and P O is perpendicular to AB.
P
.......................................................
.......... .. .......
....... ... .. .... ......
.......... ... ... .. ......
.....
.....
. .. .. ... .....
.
...... .... .... .... .....
.... ... ... .... ...
... ...
.... ..
. ..
. .... ...
... ..
. ..
. ... ...
..
. ... ..
. .
...
...
...
.... ..
. ..
. ... ...
... ... ..
. .
... ...
... ... ..
. . ...
...
...
X . . . . . ........
.
..
.
............
.
..
.
.
.
. .... .
. .
. .
.
...
...........
. .
Y
... . .
..
..
... ... ..... ...
. ...... .. .......
.. . .
.
... .......
. . ..
... .. ... ..
.
..... . ...
. .
. .. ... .....
. ....
... .... .. ... .. .. ... ... ..
... .. . ........ ... .... .... .. .
... ... ... ..... ... ... ..
... ... ... ... . ..
......
..... ....
.. C ..
...
......
...
... ........
.. ...
...... ..
. . . ....
. ... ......
....... . . .......
..... .... ........ ..
...... ..... ....
A ..................................
.......
.........
.......
.................................
.........
B
................. .................... . .
.
..........
First AX/AP = r1 /r so that P X/P A = (r − r1 )/r. Also qP C 2 = P X · P A. Thus
r
P A2 /P C 2 = P A/P X = r/(r − r1 ). Therefore, P A = P C r−r 1
. Similarly, P B =
q
r
P C r−r 1
. As P A = P B, we have r1 = r2 .
And thus:
X1 (ai − aj )2 1 X (ai − aj )2
X ai aj n−1
L= = ai + aj − = ·S− . (2)
a + aj
i<j i i<j
4 ai + aj 4 4 i<j ai + aj
X
To express R we express the sum ai aj in two ways; in the second transformation identity
i<j
(1) will be applied to the squares of the numbers ai :
!
X 1 X
ai aj = S2 − a2i ;
i<j
2 i
X 1X 2 n−1X 2 1X
ai aj = ai + a2j − (ai − aj )2 = ai − (ai − aj )2 .
i<j
2 i<j 2 i
2 i<j
Multiplying the first of these equalities by n − 1 and adding the second one we obtain
X n−1 1X
n ai aj = · S2 − (ai − aj )2 .
i<j
2 2 i<j
Hence
n X n−1 1 X (ai − aj )2
R= ai aj = ·S− . (3)
2S i<j 4 4 i<j S
Now compare (2) and (3). Since S ≥ ai + aj for any i < j, the claim L ≥ R results.
6. Let A, B, C be 3 points on the plane with integral coordinates. Prove that there exists a
point P with integral coordinates distinct from A, B and C such that the interiors of the
segments P A, P B and P C do not contain points with integral coordinates.
Solution. Let A = (a1 , a2 ), B = (b1 , b2 ), C = (c1 , c2 ) where a1 , a2 , b1 , b2 , c1 , c2 are integers.
If a lattice point X = (x1 , x2 ) is between A = (a1 , a2 ) and B = (b1 , b2 ), then there exist
relatively prime positive integers u and v such that (u + v)xi = uai + vbi , i = 1, 2. That
is (u + v)(xi − bi ) = u(ai − bi ), i = 1, 2. This implies that (u + v) divides ai − bi , for
i = 1, 2. That is a1 − b1 and a2 − b2 have a common factor. Therefore, it suffices to find
a lattice point P = (x1 , x2 ) such that gcd(x1 − a1 , x2 − a2 ) = 1, gcd(x1 − b1 , x2 − b2 ) = 1
and gcd(x1 − c1 , x2 − c2 ) = 1.
Let p = 2 or 3. There are at least 4 distinct values (m1 , m2 ) modulo p. Thus there exists
(r1 , r2 ) such that (r1 , r2 ) 6≡ (a1 , a2 ), (b1 , b2 ), (c1 , c2 ) (mod 2). Similarly, there exists (s1 , s2 )
such that (s1 , s2 ) 6≡ (a1 , a2 ), (b1 , b2 ), (c1 , c2 ) (mod 3). By the Chinese Remainder Theorem,
there exists (p1 , p2 ) such that p1 ≡ r1 (mod 2), p1 ≡ s1 (mod 3); and p2 ≡ r2 (mod 2),
p2 ≡ s2 (mod 3). We can also require p1 6= a1 , b1 , c1 and p2 6= a2 , b2 , c2 . This ensures that
(p1 , p2 ) 6≡ (a1 , a2 ), (b1 , b2 ), (c1 , c2 ) (mod 2) and (mod 3).
Suppose p is a prime larger than 3 such that p divides p2 − a2 , p2 − b2 , or p2 − c2 . There
are only a finite numbers of such primes p. Let the set of all such primes be Y . For each
p ∈ Y , pick an integer zp 6≡ a1 , b1 , c1 mod p. Since p > 3, this is possible. Apply the
Chinese Remainder Theorem to find an x1 such that x1 ≡ p1 (mod 2), x1 ≡ p1 (mod 3)
and x1 ≡ zp (mod p), p ∈ Y . Take x2 = p2 . Then P = (x1 , x2 ) is the desired point.
Singapore International Mathematical Olympiad
Training Problems
18 January 2003
1. Let M be a point on the segment AB. Squares AM CD and M BEF are erected on
the same side of AB with F lying on M C. The circumcircles of AM CD and M BEF
meet at a second point N . Prove that N is the intersection of the lines AF and BC.
2. Let DM be the diameter of the incircle of a triangle ABC where D is the point at
which the incircle touches the side AC. The extension of BM meets AC at K. Prove
that AK = CD.
3. Tangents P A and P B are drawn from a point P outside a circle Γ. A line through P
intersects AB at S and Γ at Q and R. Prove that P S is the harmonic mean of P R
and P Q.
4. (IMO 1981)Three circles of equal radius have a common point O and lie inside a given
triangle. Each circle touches a pair of sides of the triangle. Prove that the incenter
and the circumcenter of the triangle are collinear with the point O.
1. Let M be a point on the segment AB. Squares AM CD and M BEF are erected on
the same side of AB with F lying on M C. The circumcircles of AM CD and M BEF
meet at a second point N . Prove that N is the intersection of the lines AF and BC.
Solution
...................................
............ ........
....... .......
......
D.....................................................................................................................C
...... ........
... .. .........
... .. ... .. ...
... .... ... .. ....
... .
. ... .. ...
.... .... ... .. ...
. ...
... ... ...
... .. ..
... .. .....
...
...
....
..
... N
... ................................
... .
. F .
................. .... ........ E
. ................................................
... .... ... ......... . . .. ...
...
.... ... ... ... .... ... .. .. .
...
. ..
. ... ... .
. ... .. .. ... .....
...
... .... . ... ... .
.
. .
. ..
. .. .. .
... .. ... ... . . ... ... ..
. . ..
. .. .... ....
... ... . ... ... ... ... .. ... ...
... .. .. ... ..... ... .. ...
. .... ...
....... .... . . .
........................................................................................................................................................
...... . .
A ...... ...... ............................... B
....... ...... M
.......... .......
..........................................
2. Let DM be the diameter of the incircle of a triangle ABC where D is the point at
which the incircle touches the side AC. The extension of BM meets AC at K. Prove
that AK = CD.
Solution
B
..
.......
...... ..
... .. ...
.
.... .... .....
.. . ...
... .. ...
... ... ...
... .... ...
. ... . ...
....
...
. ...
... . ...
. .
.. ............................
.
. M .
.
..
............. ....
. ........ .....
. . . ........ ...
..........
.
. .....
. ..... ...
..... ... ... .....
.....
.... ... ... ....
.... .. .. ...
.. ....... ... ....
. ...
.. .. ... .
.. ... .....
.
... ... ... ... ......
... ... . ..
. ....
.... ... .
.
. .... ... ....
..... ...
. .
... .
.
. .. ....
.
.
.. ...
...
.
... . ....
... ...
.. ... ...
... ...
..... ....
... ... ...
... ... ....
....
. ..... .
..
. .. .
. . . ........ ...
...
.. ....... .
. ....... .
. . . ... . ..
A............................................................................................................................................................................................
. . . . . ...
C
.. ........ .......... ...
..
.
.
.. ........... ..... .
.
K D . .... ........
. ..
...
....... ...
... ....... ...... ...
............... ...... ...
.......... .....
..
.........
. ..... .....
.
.. ....... ...
.....
.
. .
... .... ....
... ... ..
... ... ..
....
. ......
......
..
...
. .....
......
. .....
....... ...
. . ..... ...
.. ...
. ...
.
.. ... ...
.. ....
... ...
. ......
... ...
. ......
.... ... ......
.. ... ...
.... ... ...
... . ..
... 0 ... ...
... O ... ....
... ... ....
...
... ... ...
...
... ... .
... ...
... ...
... ...
... .. .
...
... ...
... ..
... ...
... .....
.... ... .
..... .....
..... .....
..... .....
.....
...... .........
.
...... ......
....... ......
....... .......
......... ........
...........
..................... . . . . ..... . . . ............
.......................
2
Consider a homothety centered at B carrying the incircle to the excircle. The diameter
M D of the incircle is mapped to the diameter M 0 D0 of the excircle. Since M D is
perpendicular to AC, M 0 D0 is also perpendicular to AC. Therefore M 0 must be the
point K. That is the excircle touches AC at K. Therefore, AK = (a + b − c)/2 = CD.
3. Tangents P A and P B are drawn from a point P outside a circle Γ. A line through P
intersects AB at S and Γ at Q and R. Prove that P S is the harmonic mean of P R
and P Q.
A
.............................................
.......... ... .......................
Q .......
...... ... ...... ..........
................
. ...
..... ...........
..... ..........
.... .............................
..
.
.
.. ................ .... ....
...
..........
..........
. ......................... ..........
.... . ................ .....
R .................
..........
..........
..
. S ..
. .
.
... .................. ..........
.... .... ... ................
................
..........
.
... ... ... ................ ...................
... ... ... ................ .........
... ................ ..........
.... ... ... ........................
.............
... ... ..... P
... ... ... ..........
... ... .. ..........
... .
. ...
.
.
.............. .
..
.
... ..
. ... ........
.......
.
...
.... ...
. ..........
... ..........
... ... ... ..........
... ... .. ..........
... .
. .....
......
. ............
.....
.... .....
.. ......
..... ..........
...... ... ..... ..........
...... ...............
.......
.......... ... .......................
.................................. ... .
..
B
Thus
PR PQ
SR = QR · , SQ = QR · .
PR + PQ PR + PQ
Also
QR · P R (P Q − P R) · P R 2P R · P Q
P S = P R + RS = P R + = PR + = .
PR + PQ PR + PQ PR + PQ
That is P S is the harmonic mean of P R and P Q.
(Second Solution by Colin Tan) Let M be the midpoint of QR. Then to prove P S =
2P R·P Q/(P R+P Q) is equivalent to prove that P S ·P M = P R·P Q. Or equivalently,
P S·P M = P B 2 , since P R·P Q = P B 2 . Therefore, we have to show that P B is tangent
to the circumcircle of 4SM B. Let O be the centre of Γ. Then O, M, P, B are concyclic
and OP is perpendicular to AB. Hence, ∠P BA = ∠P OB = ∠SM B. Therefore, P B
is tangent to the circumcircle of 4SM B.
3
...........
....................................
.
A
........ ... ........................
Q ...........
...... ... ................
..... ...........
.....
.... .............................. M ... ..... ..........
.
.. .
. .... ..........
....
...
. ................
.
.
....• .
.
.... ................................ ....
..
...
.................. R
..........
..........
..........
..........
..
... .. ..
. .. ...
.
S . .. .. . . .
... ...................... ..........
..........
. ... .. ................. ......
.... .. .. .... .
... ................
................ ..................
... .. .. ... ... ................ ..........
... .. .. .. . .............. ........
.... ..... ... ....... ..... ... ... ... ... ... ... ... ......... ... ... ... ... ... ... ... ... ... ... ... ... ... ... ... ... ... ... ... ...............................................
... .. .. ...
.. . .. ..
..
.......
..........
P
... O .. . .. . . .. ..........
... .. ..........
... .. .. ..
. . .
. .... ...
. ..............
.
..
... . .. ..
...
. ..........
... ..
.. .. ....
.
... ..........
... .........
... .. .. .. .. ..........
... .. .. ... ..... ....
...............
..... .. ..... .. ......
..... ..... ..........
...... ....... ..... ...........
......
....... ....... ..............................
.......... ................
....................................
B
Since AP = BP , we may cancel the common factor (AS + SB), thus obtaining
P S 2 + AS · SB − AP 2 = 0.
Since AS · SB = QS · SR = (P Q − P S)(P S − P R) = (P Q + P R) · P S − P S 2 − P Q · P R
and P A2 = P Q · P S, we have (P Q + P R) · P S = 2P Q · P R. Thus, P S is the harmonic
mean of P R and P Q.
4. (IMO 1981)Three circles of equal radius have a common point O and lie inside a given
triangle. Each circle touches a pair of sides of the triangle. Prove that the incenter
and the circumcenter of the triangle are collinear with the point O.
Solution
.................
........................................................... A
..............
............ .. ...
.......... ........ .................
... .......... ... . ... .......
.. ... . .... .
..... ... .... .......
......
......... .... .. ... ......
......... .. .. .. .... ......
......
........ .. .. .. ...
... .....
....... ...
. .
. . .....
.....
....... ..
.. .
.
... .....
.... .. . ... .....
... .. . .
... .....
...
... ... . ....
... ... .. ... ...
.. . . . ....
...
. .. .. ..
...
...
...
...
. .... .................................. . ...
... . .. ............ . ...........
.
... ...
.
.......... ...... .. .
...
. .. .
. ........ .. . ...
...
... .........
. .
.
. .
. .
.... ... ...
... .... . . . . ...
.. ... . ...... ...
. . .
. ..
... ...
..
. . ...
.
. . ... ...
... . . ..... .
. . ...
... ......
. .
. ...
. ...
.... ... ... . 0 ...... ...
... ..
.
.. ... .
.
. A .....
. . ...
... .... .... . .........
. .
.. ... ...
... ..
. ... .... .. ... . ....
.
.
...
... ... .. . ... . ... ...
... .... ... ..
.. .. .... . . ..
. .... ...
... .. . ... .. .
. ...
... .... . .... .. ... . ... ... ...
.. ... . . ... ............. ... ...
C
·O
... .. .. .......... . . . .. .. ... .
........... .. .. .
. . .. . .. .. .
........... . ...
.... .. .. ............... .
. . ... . . . .. . .
................ .
.. .. . . .
... ..
. .......... ..... ........ . ... .... ... . .
. .. ...... ...
. ..
... . .. . . .
...... ..... .... .. . ... ...... . . ..
..... .. . ..
.......
...
...
...
...
... ....
......
..
...
........
...
...
...
...
.. .
··I
.. . . .
.
. . ..
... .....
.. .........
...
...... ...........
.
.
.. ............................... ......................... .
...
...
...
... ..
.
.
........
.
.....
......
...
... ..
...
.
...
....
4
Let A0 , B 0 , C 0 be the centres of the circles inside 4ABC. As AA0 , BB 0 , CC 0 are angle
bisectors, they meet at the incenter I of triangle ABC. I is also the incenter of the
triangle A0 B 0 C 0 . The circles are of the same radii. Thus A0 and B 0 are of equal distance
from AB so that AB is parallel to A0 B 0 . Similarly, BC is parallel to B 0 C 0 and A0 C 0 is
parallel to AC. That is 4ABC is similar to 4A0 B 0 C 0 . Consider a homothety centred
at I sending A0 to A, B 0 to B and C 0 to C. Thus the circumcentre O of 4A0 B 0 C 0
is mapped to the circumcentre C of 4ABC under this homothety. Therefore, I, C, O
are collinear.
5
Singapore International Mathematical Olympiad
Training Problems
25 January 2003
4. (Crux 2333) Points D and E are on the sides AC and AB of 4ABC. Suppose F and G
are points of BC and ED, respectively, such that BF : F C = EG : GD = BE : CD.
Prove that GF is parallel to the angle bisector of ∠BAC.
5. (Balkan Math Olympiad 2002) Let O be the center of the circle through the points
A, B, C and let D be the midpoint of AB. Let E be the centroid of triangle ACD.
Prove that the line CD is perpendicular to the line OE if and only if AB = AC.
1. In a quadrilateral ACGE, H is the intersection of AG and CE, the lines AE and CG
meet at I and the lines AC and EG meet at D. Let B be the intersection of the line
IH and AC. Prove that AB/BC = AD/DC, or equivalently, DB is the harmonic
mean of DA and DC.
Solution Apply Menelaus’ Theorem to 4ACI, 4AEC and 4CEI with transversals
EGD, IHB and AHG respectively. We have
CD AE IG AB CH EI CG IA EH
= 1, = 1, = 1.
DA EI GC BC HE IA GI AE HC
The result is obtained by multiplying these three equations together.
I .
.......
... .......
... .....
.. .........
.
.. ... ...
.. ... ...
... ... ...
... ... ...
... ... ...
... ... ....
... ... ...
. ... ... ...
... ... ...
.. ... ...
. ... ...
... ... ....
...
E .
.. .
... ....................
...
...
...
...
... .............. ... ...
.. .............. . ...
. .
. ..
.
...... ........ .
... ...
... .. ...
...... ....... ... F ...
..
. ... . . .. . ...
...... ..........
... ...... ........ .
...
. ...... ............. .....
.. ...... .. ....... ..
. . . ...... .. . G ........
..
H . . . .. ......................
.. .
. .....
.
... ................. ... ........
.. .. .. .
... .. . .............. ................ . .......
.
..............
... .. ... ....... .... .......
... .............. ... ...... ... .......
.......
..
. ............. ... ........... .......
...................... . . .
.........................................................................................................................................................................................
.
. . ...
.
A B C D
2
Note that all these ratios are equivalent to each other and they are just different ways
to express the location of S. Similarly,
P S0 2P Q0 P S0 2P R0
= and = .
S 0 R0 Q0 R0 S 0 Q0 Q0 R0
............
................................... A
........ ...................
Q..................... ...
...
................
..... ..........
..... .........
.................................. ... ..... ..........
. ... ........ ................ . ... ..........
. ..... ... ... ................. . ... ..........
.... .. . . ......... . ..........
..
.....
..... . .............................
. R ..........
..
.
. .....
......
S .
.
.
. .......................
.. ...
.... ...
................
..........
..........
.... ..... .. ..... ... ................ .......
..... .
.. ...... . ... ................ ...................
... .....
..... .
. .... ... .
... ................
...................................
... ..... .. ..... .
. .. . ... .........................
... ..... .. .....
..... ....... ... ................
... ........ ..
. ................
.......................
P
...
... X ....
. ..
......... ......... ..
. .........
....................................
. .
...............................
... ..... ... .......... ... ..............
... ..... ..... .............. ...................
.
... ..... .... ..... ....
..... .. ...........................
..... ... ...
...
... ......
. . ................... ...............
Γ ...
.... .
. .
.......
S 0 .. ........................ 0
.
. ......
.. .. .
......
. ................
..... .....
.
..... ......... ...........................
.................
....
R
..... ...................
... ..
...... .................. ..... ..........
.........
. ... ............................
...
0 ................. .
.
.. ....
...
Q ................................
A0
XR P R0 QR P R QR
0
= 0 0
= ,
XQ PQ Q R P Q0 Q0 R0
3
By taking c × (1) − f × (2), we have
1 1 1
(c − f )x + cd + − (c + f ) y = 0 (3)
k xD xB
This is the equation of a line passing through A (as there is no constant term) and
the intersection of DF and BC. Hence, it is the equation of the line AG. Similarly,
the equation of the line AE is given by
1 1 1
(c − f )x − cd + − (c + f ) y = 0 (4)
k xD xB
From (3) and (4), we see that the slopes of the lines AG and AE are negative of each
other. Therefore, ∠GAC = ∠EAC.
4. (Crux 2333) Points D and E are on the sides AC and AB of 4ABC. Suppose F and G
are points of BC and ED, respectively, such that BF : F C = EG : GD = BE : CD.
Prove that GF is parallel to the angle bisector of ∠BAC.
C
.....
... ..
... ...
Solution Let A be the origin of a rectangular ...... ....
. ...
... ...
... ...
coordinates system. For each of the points in the .....
... ...
...
.
... ...
question, we use the small case letter in bold face . ... ....
....... ....
F
D ...
. ..
....
.
.... ....
...... ...
...
.. .... ......
.
to denote the position vector of that point. First .....
..
... .................
....
.... ...
...
...
.... G ... ...
we have e = pb and d = qc for some p, q in (0, 1). .
......................................................................................
.
A E B
tc + b td + e tqc + pb
Let t = BF/F C. Then f = and g = = . Since BE = tCD, so
t+1 t+1 t+1
(1 − p)|b| = t(1 − q)|c|. Thus,
c b
This is parallel to + , which is in the direction of the angle bisector of ∠BAC.
|c| |b|
4
5. (Balkan Math Olympiad 2002) Let O be the center of the circle through the points
A, B, C and let D be the midpoint of AB. Let E be the centroid of triangle ACD.
Prove that the line CD is perpendicular to the line OE if and only if AB = AC.
A
............
............... ........ ......................
........ ... .. .......
...... .. ... ......
......
Solution Set the origin at O. As in the last .
............
..
.
.
... ....
...
...
.....
.....
....
...
.... ... . ...
question, for each of the points in the question, ..
.
...
.
..
.
..
...
.
...
...
...
...
...
..
. ..
. . ...
.. ...
·O·
we use the small case letter in bold face to denote .... . E ... ...
... D ...... . . ...
.... ... ...... ........ ... .
.. ........ ... ...
the position vector of that point. Then d = 12 (a+ ...
...
... ..
.
.
..
. .....
.....
..
.....
....
...
.
... ..
.
...
.
... ... ...
b) and e = 31 (a + c + d) = 16 (3a + b + 2c), ... ..... ..
... ... .....
..... ..... ...
... ... ..... .. ..
...
... ... ..... ... ..
d − c = 12 (a + b − 2c). ..... ... ..... .. .
..... .. ........ ........
....... ... ..
...........................................................................................
....... ..
.. ...
B .........
...........................................
. C
5
Singapore International Mathematical Olympiad
Training Problems
8 February 2003
1. Determine whether there exist an integer polynomial f (x) together with integers a, b
and c satisfying the following conditions.
(i) ac 6= bc.
(ii) f (a) = a, f (b) = b, c2 + f (c)2 + f (0)2 = 2cf (0).
Solution The second condition in (ii) implies that (c − f (0))2 + f (c)2 = 0. That
is f (c) = 0 and f (0) = c. As f is an integer polynomial and f (a) = a, we have
(a − c)|(f (a) − f (c)). That is (a − c)|a. Also (a − 0)|(f (a) − f (0)) so that a|(a − c).
By (i), c 6= 0. Thus, a − c = −a giving c = 2a. Similarly, c = 2b. But this gives a = b,
contradicting (i). Therefore, no such integer polynomial exists.
Solution Let q(x) = p(x) − 827x. Then q(x) is a polynomial of degree 4. As q(1) =
q(2) = q(3) = 0, we have q(x) = (x − 1)(x − 2)(x − 3)(x − r), for some r. Therefore,
1 1 1
4 (p(9) + p(−5)) = 4 (q(9) + q(−5)) + 827 = 4 ((8)(7)(6)(9 − r) + (6)(7)(8)(5 + r)) =
1176 + 827 = 2003.
Solution From the relation between roots and coefficients of a polynomial equation,
we have
a+b+c = −a (1)
ab + bc + ac =b (2)
abc = −c (3)
2
Singapore International Mathematical Olympiad
Training Problems
15 February 2003
1. Let n be an odd integer which is not a multiple of 5. Prove that there exists a strictly
positive integer k such that n divides a string of k 1’s, i.e.
n| 11...11
| {z } .
k1’s
n! = mk .
3. Show that for all integers A, B, there exists an integer C such that the following sets
M1 = {x2 + Ax + B : x ∈ Z} and M2 = {2x2 + 2x + C : x ∈ Z} are disjoint.
4. Let m be a strictly positive integer. Show that there exists infinitely many pairs of integers
(x, y) such that
(a) x, y are relatively prime
(b) y divides x2 + m
(c) x divides y 2 + m
(d) x + y ≥ m + 1
6. Let n be a non negative integer. Suppose that there exists rational numbers p, q, r such
that
n = p2 + q 2 + r 2 .
Prove that there exists integers a, b, c such that
n = a2 + b2 + c2 .
Solutions
1. From the given conditions gcd(n, 10) = 1. But gcd(9, 10) = 1 and hence gcd(9n, 10) = 1.
Thus by Euler’s Theorem,
10φ(9n) ≡ 1 (mod 9n),
which implies the desired result.
2. Using Bertrand’s Postulate, there exists a prime p satisfying n2 < p < n for all n ≥ 3. Now
note that 2p > n, hence p only has a single power in n!, i.e. k = 1. Hence (m, n, k) =
(n!, n, 1) is a solution triplet. If n = 2, we have 2 = mk , hence we must have m = 2, k = 1.
If n = 1, we must have 1 = mk , or m = 1, k ∈ ℵ thus (m, n, k) = (1, 1, k) is another solution
triplet. Thus the only solutions to the equation are
but y|(x2 + m) implies that y|x2 (x21 + m), but gcd(x, y) = 1 implies that y|(x21 + m). Now
x1 > y ≤ x. Repeat the same argument to generate y1 , but instead consider
x21 + m = yy1 .
Then (x1 , y1 ) is also a solution, with x1 + y1 > x + y. Continue this process to generate
(x2 , y2 ),... and since m is fixed, xn + yn ≥ m + 1 for some n, thus (xn , yn ), (xn+1 , yn+1 ), ...
is a set of infinitely many solution pairs which satisfies all given conditions.
5. (a) Consider ai = ki + 1, bj = mj + 1. Suppose that two of the residues are the same.
Then mk divides ai bj − as bt = (ki + 1)(mj + 1) − (ks + 1)(mt + 1) = km(ij − st) +
m(j − t) + k(i − s), and thus m|k(i − s) but gcd(m, k) = 1, hence m|(i − s), and since
|i − s| < m, we must have i = s and similarly j = t and we are done.
(b) Suppose all the residues are distinct. Then 0 is one the residues. WLOG, suppose
mk|a1 b1 . Hence there exists a0 , b0 such that a0 |a1 , b0 |b1 and mk = a0 b0 . Suppose now
that for i 6= s, a0 |(ai − as ). Then we have mk = a0 b0 |(ai b1 − as b1 ), which is a contra-
diction. Hence all the ai ’s cannot have the same residue modulo a0 , similarly, all the
bj ’s cannot have the same residue modulo b0 . Thus we must have a0 ≥ m, b0 ≥ k thus
a0 = m, b0 = k.
Now let p be a prime divisor of m and k. p > 1 since gcd(m, k) > 1. Since all the ai ’s
form a distinct set of residues modulo m, there are m − m p between them which are
k
not divisible by p. Similarly, there are k − p bj ’s which are not divisible by p. On the
other hand all the ai bj ’s form a set of reduced residues modulo mk by our assumption,
and hence between them, there are mk − mk p which are not divisible by p. But
m k mk
m− k− = mk −
p p p
if and only if m = 0, k = 0 or p = 1, which is a contradiction.
6. If n = 0, the result is clear. So suppose n > 0. Suppose the set of points (x1 , x2 , x3 ) which
lies on the sphere
n = x2 + y 2 + z 2
are all rational points. We will obtain a contradiction. Now there exists an integer point
u = (u1 , u2 , ..., un ) such that ad = u, where d ≥ 2. Suppose that a and u are chosen such
that d is minimal. Then let x0 , y 0 z 0 be the integers closest to x, y, z, where a = (x, y, z).
Then |x − x0 | ≤ 21 , |y − y 0 | ≤ 12 and |z − z 0 | ≤ 12 , hence ||a − a0 || < 1, where a0 = (x0 , y 0 , z 0 ).
Now consider the line connecting a and a0 . This will intersect the sphere x2 + y 2 + z 2 = n
at two points, one at a and the other which we call b. The equation of the line is given by
a0 + λ(a − a0 ). Now b lies on the sphere so
One of the solutions to this equation is given by λ = 1, which correspond to the point a.
0 ||2 −n
The other thus is given by λ = ||a
||a−a0 ||2
. Now
2 d1
||a − a0 ||2 = ||a0 ||2 + ||a||2 − 2 < a0 , a >= ||a0 ||2 + n − < a0 , u >= ,
d d
d(||a0 ||2 −n)
where d1 ∈ ℵ and since ||a − a0 ||2 < 1 we have d1 < d. Hence λ = d1 and we have
b = a0 + λ(a − a0 )
||a0 ||2 − n
= a0 + (u − da0 )
d1
v
=
d1
where v is an integer point. Now b = vd1 with d1 < d contradicts our assumption that d is
minimal.
Note that a generalisation is not possible using this method since ||a − a0 ||2 < 1 will NOT
be satisfied for higher dimension spaces. For a one dimensional space, i.e. the real line,
this result is obvious. For a two dimensional space, i.e. the plane, this argument works.
Singapore International Mathematical Olympiad
Training Problems
08 March 2003
Now consider
S1 = {(a, b, c) ∈ S, a > b + c}.
Assume that |S1 | is odd. Show that p can be expressed as a sum of two squares.
2. Prove that the circle with equation x2 + y 2 = 1 contains an infinite number of points with
rational coordinates such that the distance between each pair of the points is irrational.
2. Let p be a prime and p ≡ 1 (mod 4). Then there exists integers a and b such that p = a2 +b2 ,
and by Dirichlet’s Theorem, there exists infinitely primes of the form 4k + 1, thus consider
the set of points given by !
a2 − b2 2ab
xp = ,
p p
All these points lie on the unit circle, and are irreducible fractions. If p 6= q, we have
2|ad − bc|
||xp xq || = √
pq
and since p 6= q are primes, the above fraction is irrational and we are done.
3. Note that if f (x0 ) is a solution, −f (x0 ) is also a solution. It is clear that f (x0 ) 6= 0. So
2
assume WLOG f (x0 ) = p < 0. Then we have f (2n−1 x0 ) = 2 f (2n−2 x0 ) − 1 ≥ −1,
and thus −1 ≤ f (2n−1 x0 ) < 0 for all natural numbers n. Now we obtain a better upper
bound. We claim that we have −1 ≤ f (2n−1 x0 ) ≤ − 41 . Suppose that there exists a natural
number m such that 0 > f (2m−1 x0 ) > − 41 . Then we have −1 ≤ f (2m x0 ) < − 78 , and hence
17
1 > f (2m+1 x0 ) > 32 > 0, a contradiction. Now from the given functional, we have
1 1 1
f (2x) + = 2 f (x) − f (x) + ,
2 2 2
and hence for all natural numbers n, we have
f (2n x0 ) + 1 = 2 f (2n−1 x0 ) − 1 f (2n−1 x0 ) + 1 ≥ 2 1 + 1 f (2n−1 x0 ) + 1 ,
2 2 2 2 4 2
or equivalently
f (2n−1 x0 ) + 1 ≤ 2 f (2n x0 ) + 1 .
2 3 2
Hence we have
n+1 n+1 n
f (x0 ) + 1 ≤ 2 f (2x0 ) + 1 ≤ ... ≤ 2 f (2n+1 x0 ) + 1 ≤ 2 1 2
1+ ≤ ,
2 3 2 3 2 3 2 3
and letting n → ∞, we have
f (x0 ) + 1 → 0,
2
and thus f (x0 ) = − 12 . Hence we have f (x0 ) = ± 12 .
Singapore International Mathematical Olympiad 2003
National Team Training
Geometry
(8) Let P and Q be the feet of the perpendiculars from the orthocenter on
triangle ABC onto the internal and external bisectors of ∠A. Assume that
∠A is not a right angle, show that the line through P and Q bisects BC.
Solution. Let H be the orthocenter and consider the circle C with diameter
AH. Observe that P, S, Q, D all lie on this circle. Since ∠P AQ is a right
angle, P Q is another diameter of C. Also ∠DAQ = ∠SAQ, so the arcs DQ
and SQ on C have the same length. Thus the diameter P Q must be the
perpendicular bisector of the segment DS. Now consider the circle C with
diameter BC. S and D both lie on C . Hence P Q, which is the perpendicu-
lar bisector of the chord DS of C , must pass through the center of C , which
is the midpoint of BC.
1
2
Solution. Let F be the foot of the perpendicular from A onto BC. If Q0 is the midpoint of
the arc BC and A0 is the midpoint of BC, it is clear that AM/M Q = AF/M 0 Q ≥ AF/A0 Q0 .
AM BN CP
Therefore, the minimum value of M Q + N R + P S will be obtained uniquely where Q, R, S
are the midpoints of the arcs BC, CA and AB respectively. Thus, we will henceforth
assume that Q, R, S are positioned so that AQ, BR, QS are the angle bisectors of angles
A, B, C respectively.
A
.............
..........
......
.................... ..............................
........
........................... .......
.... ......
...... ... .. ... ...... .....
.
..... .
.. ... ... ...... .....
.....
..
... .
.. .. ... ...... .....
.
.... .
.. .. ... ......
... ..... ...
... ... ... . ...
.
. .. . ... ..... ...
.. .
. .
.. ... ......
... ... . ... ...... ...
... ... .
. ... ..
..... ...
.. .
. .
. .... ...... ...
... ..
.. ..
. ... ...... ...
.. .. .
. ... .
..... ...
.... ...
.. .
.
.. ... .
..... ...
... .
. . ... ...... ...
.. .
.. .
. ... ....... ...
... .
. . .
. ... ...... ...
... .. ..
. . ...... .
. ...
...
..
.
.
. .
.
.
. ... ......
. .
....
... .. . ... ..... .
... ... . . . .... .
.... 0 ...... 0 ............ .....
.
... ...
...... .. F A . MM
..................................................................................................................................................................................
.
... ... .... ...
B ...
...
...
...
..
...
...
...
...
..
... ....
... C
... . ... .. . .
.
... .
..... ... .. ...
..... ... ... .. .....
.....
...... .. ... ... .........
...... . . .
..... ....... ..
....... ... ..
........ ... ........
........... ........
....................................................
Q
Q0
We know that the angle-bisector AQ cuts the side BC of length a in the ratio c : b so
BM = ca/(b + c) and M C = ba/(b + c). Therefore,
AM AM 2 AM 2 AM 2 (b + c)2
= = = . − − − (∗)
MQ AM · M Q BM · M C a2 bc
1
From Stewart’s Theorem,
c2 M C + b2 BM − aBM · M C − aAM 2 = 0.
It follows that
ca2 b
AM 2 = bc − .
(b + c)2
Substituting this into (*), we have
µ ¶2
AM b+c
= − 1.
MQ a
Similarly,
µ ¶2 µ ¶2
BN a+c CP a+b
= − 1, = − 1.
NR b PS c
Thus we have
µ ¶2 µ ¶2 µ ¶2
AM BN CP b+c a+c a+b
+ + ≥ + + − 3.
MQ NR PS a b c
Equality holds if and only if a = b = c; that is to say, if and only if triangle ABC is
equilateral and M.N.P are the midpoints of the sides.
3. Does there exist a convex pentagon, all of whose vertices are lattice points in the xy-
plane, with no lattice point in the interior? (A point in the xy-plane is called a lattice
point if it has integer coordinates.)
Solution. The answer is No. A convex lattice pentagon must have an interior lattice
point. To see this, note that every lattice point (x, y) belongs to one of the four classes
K00 , K01 , K10 and K11 , where the index pair ij is determined by taking i ≡ x and j ≡ y
modulo 2. A convex lattice pentagon has five vertices, so two of them, say P and Q belong
to the same class, which implies that their midpoint R is also a lattice point. If P and Q
are endpoints of a diagonal of the pentagon, then R is an interior lattice point. If P and
Q are the endpoints of an edge, say edge AB of the pentagon ABCDE, we continue by
considering the convex lattice pentagon ARCDE and so on. This case cannot continue
indefinitely, because if so, there would be an infinite sequence of distinct lattice points
within a finite region of the coordinate plane, which is not true.
2
4. A piece of cardboard in the shape of a square is to be cut into n acute- angled triangles.
Find the smallest n for which this can be done. Show at least one way to do it this
minimum n.
Solution. Suppose the square has been cut into n acute-angled triangles. Form a graph
whose vertices are the vertices of the square and the vertices of the triangles. Any two
consecutive vertices on a side of a triangle or on a side of a square are joined by an edge.
There are no other edges. There are three types of vertices:
(a) Vertices of the square: These are of degree at least 3.
(b) Vertices which are in the interior of a side of a triangle or the square. These are
of degree at least 4. Denote the number of vertices of these type by b
(c) Vertices of the triangles which do not lie in the interior of a side. These are of
degree at least 5. Denote the number of vertices of these type by c
Let m be the number of edges. Then
2m ≥ 4 × 3 + 4b + 5c.
(4 + b + c) + (n + 1) − m = 3, or m = b + c + n + 3.
Substitute this into the inequality, we get c ≥ 2. Each vertex of type (c) is associated with
at least 5 triangles and the triangles associated to 2 vertices can have a overlap of at most
2. Thus we get n ≥ 8. See the picture below for a construction which gives n = 8.
............................................................................................................................................................................
.. ............... .... ...
........ ..... ......... .....
.. ......... ... . . ......... ......
...... ......... ...
............ . .... .... ........................ .
.. .. .......... .... .. . .. . ... .. ..
... .. ..................... .. .................... ... ....
.. ..... . .. .......................... .. ....
.. ....... ..
.. .. .
.
.. . . . . .. ... .
... .
. .
....
... ....... ...... ....... .... .... ...... .... ....... ...... ...
.. . ..... .... .... ...
.. .
.
. ..... ... ... ...
... ... ... .... ... ...
.. ... ...... .... ... ...
.. .
. ... ...
... ... ..... ... ... ...
.. .
.. ... ... .. ...
.. .. . . .
.
... .. ...
.... .. .... .....
...
.. ..... ... ... .. ... ..
...
.. ...... .. ... ...
.. ... ......... ...
... .. . .... ... ...
.. ... . .. .
... ...
.. .
. . . . .
... ..
. ..... ..... ... ....
.. .
. . . ... ....
.. .. .. .. . . . ..
...
.. ........... ....... ...
. ... .... ..... .. ..... .
. ....... ........ ...
..
.
. .... . .. . ...... ......
.. .... .
.......... ....... ... ...
. . . ...... ....
... . ....... . . . .
...... ..
.. ... ... . . . . ... ... . ...
.. ... ..... ......... .. ...... ... ... ...
.........
..... ...
....... ......
. .. .
.
.
.
.
. .
.....
... ...
... ...
..... ...... ....... .... ... ...
......................................................................................................................................................
5. (Prize Problem) Let x be a positive rational number. Prove that there exist a unique
set of integers a1 , a2 . . . , ak , with 0 ≤ an ≤ n − 1 for n > 1 such that
a2 a3 ak
x = a1 + + + ··· + .
2! 3! k!
Show also that there exist a set of integers, 106 < b1 < b2 < · · · bm such that
1 1 1
x= + + ··· + .
b1 b2 bm
3
Solution. First note that x can be written in the form m/n!, where m, n are positive
integers. We shall prove by induction on n that
m a2 an
= a1 + + ··· +
n! 2! n!
where 0 ≤ ai ≤ i − 1 for i ≥ 2. For n = 1, a1 = m, and ai = 0 for i > 1 works. Now
assume that it holds for some n. Let
Then
m q r
= + .
(n + 1)! n! (n + 1)!
The result then follows by applying the induction hypothesis on q/n! and putting r = ak+1 .
Next we prove uniqueness. Suppose
a2 a3 ak b2 b3 bk
x = a1 + + + ··· + = b1 + + + ··· + .
2! 3! k! 2! 3! k!
Then
ak ≡ bk ≡ x(k!) (mod k)
and it follows that ak = bk . By considering x − (ak /k!), we can show that ak−1 = bk−1 .
Thus uniqueness follows.
To prove the second part we first note that if ai > 0, then ai | i! and thus ai /i! is
the reciprocal of a positive integer. Moreover, (i − 1)! < i!/ai ≤ i!. Thus all the positive
integers are different.
Now if x < 1, then a1 = 0 and so x can be expressed as the sum of reciprocals of
different positive integers.
Now we consider the general case. Let x be any positive rational number and m be
an integer > 106 such that m > 1/x and n be the largest integer ≥ m such that
1 1 1
x≥ + + ··· + .
m m+1 n
Let
1 1 1
y =x− − − ··· − .
m m+1 n
Then 0 ≤ y < 1/(n + 1). Thus y can be written as a sum of reciprocals of different positive
integers:
1 1
y= + ··· + .
q1 qj
Since 1/qi < y < 1/(n + 1), qi > (n + 1). Thus
1 1 1 1 1
x= + + ··· + + + ··· +
m m+1 n q1 qj
4
as required.
6. Given a segment AB of length 1, define the set M of points as follows: A, B ∈ M and if
X, Y ∈ M , then M contains the point Z in the segment XY for which Y Z = 3XZ. Prove
that M does not contain the midpoint of AB.
Solution. Represent A by 0 and B by 1 on the number line. Denote by Mn the set of points
of the segment AB obtained from A, B by not more than n iterations. It can be proved
by induction that Mn consists of all points in [0, 1] represented by 3k/4n and (3k − 2)/4n ,
where k is an integer. Thus M consists of numbers of the form 3k/4n and (3k − 2)/4n . To
prove our assertion, we need to show that 1/2 cannot be expressed in this form. Suppose
1/2 = 3k/4n , then 4n = 6k which has no solution. Suppose 1/2 = (3k − 2)/4n , then
6n = 4n + 4 which has no solution. Thus 1/2 6∈ M .
2nd soln (Joel): Represent each point by a coordinate in the set [0, 1] with A = 0,
B = 1. For any two points x, y ∈ M , the point (3x + y)/4 ∈ M . Now take decimal
representations. Note that each point is a finite decimal. It’s easy to see that the digit
sum of the decimal representation ≡ 0or1 (mod 3) for any point in M since it initially
holds for A, B and if it holds x, y then it also holds for (3x + y)/4. Since midpoint of AB
has decimal representation of 0.5, it is not in M .
7. (Prize Problem) Let a1 , a2 , . . . , an , n ≥ 1, be real numbers ≥ 1 and A = 1 + a1 +
· · · + an . Define xk , 0 ≤ k ≤ n by
1
x0 = 1, xk = , 1 ≤ k ≤ n.
1 + ak xk−1
Prove that
n2 A
x1 + x2 + · · · + xn > .
n2 + A2
(Hint: Let yk = 1/xk )
ak
Solution. Let yk = 1/xk . We then have yk = 1 + yk−1 . From yk−1 ≥ 1, ak ≥ 1 we obtain
µ ¶
1 ak 1
− 1 (ak − 1) ≤ 0 ⇔ 1 + ≤ ak + .
yk−1 yk−1 yk−1
ak 1
So yk = 1 + yk−1 ≤ ak + yk−1 . We have
n
X n
X n
X n
X n−1
X 1 X 1 n−1 X 1 n
1 1
yk ≤ ak + = ak + + =A+ <A+ .
yk−1 y0 yk yk yk
k=1 k=1 k=1 k=1 k=1 k=1 k=1
Pn Pn
Let t = k=1 1/yk . Then k=1 yk ≥ n2 /t. So for t > 0,
n2 t < A + t ⇔ t2 + At − n2 ≥ 0
√
−A + A2 + 4n2 2n2
⇔ t> = √
2 A + A2 + 4n2
2n2 n2 A
≥ = .
A + A + (2n2 /A) n 2 + A2
5
Training problems 1 April 2003
8. In a group of interpreters each one speaks one or several languages, 24 of them speak
Japanese, 24 Chinese and 24 English. Prove that it is possible to select a subgroup in
which exactly 12 interpreters speak Japanese, exactly 12 speak Chinese and exactly 12
speak English.
Solution: Suppose that in a group of interpreters n speak Japanese, n speak Chinese and
n speak English. Denote these groups by A, B, C. Put p = |A ∩ B c ∩ C c |, q = |Ac ∩ B ∩ C c |,
r = |Ac ∩ B c ∩ C|, a = |Ac ∩ B ∩ C|, b = |A ∩ B c ∩ C|, c = |A ∩ B ∩ C c |, d = |A ∩ B ∩ C|.
A group of interpreters is called a k-group if exactly k interpreters speak Japanese,
exactly k speak Chinese and exactly k speak English.
We shall prove by induction on n, that for n ≥ 2, it’s possible to find a 2-group inside
an n-group.
When n = 2, it’s trivially true. Now suppose n > 2 is an integer and that for each k,
2 ≤ k < n, the result is true.
1. a, b, c > 0: It’s enough to select one interpreter from each of the sets:
Ac ∩ B ∩ C, A ∩ B c ∩ C, A ∩ B ∩ C c.
1
Without loss of generality, we can assume that a1 ≤ a2 ≤ a3 . The solutions are of the
form
(a1 , . . . , a7 ) = (a, a + b, a + c, d, d + b, d + c, n − (a + b + c + 2d))
for (independent) nonnegative integers a, b, c, d.
The set {S(1, 0, 0), S(0, 1, 0), S(0, 0, 1)} gives a 1-groups. The set {S(0, 1, 0), S(1, 0, 1)}
gives b 1-groups. The set {S(0, 0, 1), S(1, 1, 0)} gives c 1-groups. The set S(1, 1, 1) gives
24 − (a + b + c + 2d) 1-groups. The set {S(0, 1, 1), S(1, 0, 1), S(1, 1, 0)} gives d 2-groups.
If (a) + (b) + (c) + (24 − (a + b + c + 2d)) = n − 2d ≥ 2, then there are 2 1-groups
which will combine to give a 2-group. Otherwise, n − 2d ≤ 1, or 2d ≥ n − 1 or d ≥ 1. We
still have a 2-group.
Apply this result to the case n = 24, we have a 2-group. Remove this 2-group, we are
left with the case with n = 22. Continuing this way, we can find 6 distinct 2-groups and
they combine to give a 12-group as desired.
9. Points P1 , . . . , Pn are placed inside or on the boundary of a disk of radius 1 in such a
way that the minimum distance dn between any two of these points has its largest possible
value Dn . Calculate Dn for n = 2, . . . , 7. Justify your answers.
Solution: Suppose n ≤ 6. Decompose the disk by its radii into n congruent regions so that
one of the points is on one of the radii. Then there is one region (including its boundary)
which contains 2 of the points. Since the distance between any two points in a region is at
most 2 sin π/n, then dn ≤ 2 sin π/n. If points Pj are placed in the vertices of regular n-gon
inscribed in the boundary of the disk, then dn = 2 sin π/n. Therefore Dn = 2 sin π/n.
For n = 7, we have D7 ≤ D6 = 1. If one of the given points is placed in the center of
the disk and if the other 6 points are placed at the vertices of the regular hexagon inscribed
in the boundary of the disk, then d7 = 1. Thus D7 = 1.
10. Prove that in any triangle, a line passing through the incentre cuts the perimeter of
the triangle in half if and only if it halves the area of the triangle.
Solution: Let ABC be the triangle and O, r denote t he incentre and inradius. Let l be a
line passing through O. It intersects one side, say BC, at an interior point. Without loss
of generality, let it intersect the side AC at P . (Note that P may coincide with A.) Let
x = P C, y = QC. For a triangle XY Z, denote its area by [XY Z]. Then
r(a + b + c) r(x + y)
[ABC] = , [CP Q] =
2 2
the latter because the altitudes of 4OCP, 4OCQ from O are both r. The line l halves
the area iff
a + b + c = 2(x + y)
iff l halves the perimeter.
11. Nine positive integers a1 < a2 < · · · < a9 are such that all the sums (of at least one
and at most nine different terms) that can be made up of them are different. Prove that
a9 > 100.
Solution: Assume that a9 ≤ 100. Let S be the set of those sums ≥ a4 of 8at
most 5 terms
8
out of a1 , . . . , a8 . The number of sums of at most 5 terms is 1 + · · · + 5 = 218. Those
2
that can be less than a4 are made up of a1 , a2 , a3 and there are at most 7 of them. Thus
|S| ≥ 211. The greatest sum in S is a4 + · · · + a8 < a4 + 4a9 and therefore all the sums
are in [a4 , a4 + 4a9 ]. The inequality |S| ≥ 2a9 implies that there are 3 numbers which are
congruent mod a9 . Thus 2 of them must have a difference of a9 , a contradiction.
12. The quadrilateral ABCD inscribes in a circle with centre O. Let BA meet CD at P ,
AD meet BC at Q and AC meet BD at M . Show that O is the orthocentre of triangle
P QM .
Solution: Let R be the radius of the circle. As ∠QM D > ∠CBD = ∠DAM , one can
extend QM to QF such that ∠F AD = ∠QM D. Then A, D, M, F are concyclic. Also
∠QBD = ∠DAM = ∠DF M so that B, F, D, Q are concyclic.
P
.......
...... ..
....
. ..... ....
.
...... ...
..... ..
............................ A ...... ...
... ................... .
... .............
........ .
.
.
....... ........ ...
...... ...... .......... ...
..... ..... ..... ........ ...
.......
. .......... .. .... .............. .
.
. .. .. ... ...
.
... ...... ..
... ...... .. .... .......... ...
..... .. .... ... .. ...
..... ...
....... .
. . . ... .
. .
.
... ......
. ...... ........ ... ........
... ... ... ... .......
...
..
......
......
..... F .
..... ............. ......
. ...
..........................
D
........
.... ......
..
....
........ ......... ... .... .... .... .....
.. ... ......
.
... ...........
.
... ....................................
.... . ............
·
......
M ... ....... ... ... ....
... ...... . ...
(Alternate Solution) Let OM meet P Q at W . From W draw tangents to the circle touching
it at H and G Then H, M, G are collinear as M is the pole of P Q. Let HG meet P Q at
Z. Then the cross ratio (H, G; M, Z) = −1. Since W M bisects ∠HW G, we have OW or
M W is perpendicular to P Q.
13. Suppose a1 , a2 , · · · , an are n ≥ 3 positive numbers such that (a21 + a22 + · · · + a2n )2 >
(n − 1)(a41 + a42 + · · · + a4n ). Prove that any three such a0i s form the lengths of the sides of
a triangle.
Solution: First we prove the assertion when n = 3. Let’s write the numbers as a, b, c.
They satisfy the inequality : (a2 + b2 + c2 )2 > 2(a4 + b4 + c4 ). We may assume without
3
loss of generality that a ≥ b ≥ c. Then
Thus, |b − c| < a < |b + c|. Therefore, a, b, c are the lengths of the sides of a triangle.
In the general case, we can simply show that a1 , a2 , a3 are the lengths of the sides
of a triangle. Using Cauchy-Schwarz inequality, we have (n− 1)(a41 + a42 + · · · + a4n ) ≤
2 2 2 2 2
a21 +a22 +a23 a21 +a22 +a23
a +a +a
1· 2 +1· 2 + 1 · a4 + · · · + 1 · an ≤ (n−1) 2 1 22 3 a44 + · · · + a4n .
2 2
From this, we get (a21 + a22 + a23 )2 > 2(a41 + a42 + a43 ). Using the case for n = 3, a1 , a2 , a3
are the lengths of the sides of a triangle.
4
Training problems 10 April 2003
14. Let A be a set with 8 elements. Find the maximum number of distinct 3-element
subsets of A such that the intersection of any two of them is not a 2-element set.
Solution. Let G be a graph with vertex set A. Two vertices are adjacent if and only if they
belong to the same 3-element subset. Thus 3 vertices that belong to the same 3-element
subset will form a 3-cycle (C3 ). The question asks for the maximum number of C3 s that
G can contain subject to the condition that every 2 of these C3 s do not have a common
edge. (The graph may, however, contain other C3 . But these will share an edge with the
C3 s we want.) We may further assume that every edge belongs to some C3 .
Since at most 3 C3 can share a common vertex, every vertex if od degree at most 6.
Thus G has at most (6 × 8)/2 = 24 edges and so at most 8 C3 s.
An example with 8 sets is:
2nd solution. This is not easier than the first but it introduces a very useful idea of
an incidence matrix.
Let B1 , . . . , Bn be 3-element subsets of A such that |Bi ∩ Bj | 6= 2. Form an inci-
dence matrix with rows indexed by B1 , . . . , Bn and with columns indexed by the elements
a1 , . . . , a8 of A. An entry (Bi , aj ) is 1 if aj ∈ Bi and is 0 otherwise. Then there are 3 ones
in every row. Call a pair of ones in the same column a 1-pair. The given condition states
that there is at most 1 1-pair in every pair of rows. Suppose there is a column, say column
1, that has 4 ones, say in the first 4 rows. Then in the submatrix formed by the first 4
rows and the last 7 columns, there are 8 ones. Thus there is at least one 1-pair. Hence
some pair of rows has two 1-pairs, a contradiction.
Thus every column has at most 3 ones. Counting the total number of ones in the
incidence matrix in 2 different ways, we conclude that n ≤ 8.
It’s not hard to get an example with 8 sets. Thus the answer is 8.
15. Find all primes p for which p(2p−1 − 1) is the kth power of a positive integer for some
k > 1.
Solution. Let p(2p−1 − 1) = xk for some positive integers x, k. It’s clear that p 6= 2.
Thus p = 2q + 1. Write x = py. Then (2q + 1)(2q − 1) = pk−1 y k . Since at least one of
2q + 1 and 2q − 1 is the kth power of an integer since they are coprime.
Case (1): 2q − 1 = z k . Then 2q = z k + 1. If k is even, then z k + 1 is not divisible by
4. Hence q = 1, p = 3 and p(2p−1 − 1) = 32 .
If k = 2` + 1, i.e., odd, then
1
The last equality contradicts with α < q.
Case (2): 2q + 1 = z k . Then 2q = z k − 1. If k is odd we get a contradiction as before.
If k = 2`, then (z ` −1)(z ` +1) = 2q and since gcd(z ` −1, z ` +1) = 2, we have z ` −1 = 2,
i.e., q = 3, p = 7, p(2p−1 − 1) = 212 .
Thus the answers are p = 3, 7.
16. Let k be a given real number. Find all functions f : (0, ∞) → (0, ∞) such that the
following equality holds for all positive real number x:
x
kx2 f (1/x) + f (x) = .
x+1
1 1
kxf (1/x) + f (x) =
x x+1
(1 − k 2 )f (x) 1 − kx
= .
x x+1
x 1 − kx
f (x) =
x + 1 1 − k2
It’s easy to see that this expression satisfies the given functional equation.
If k = ±1, there is no solution. For f (x) > 0 for all positive x, we also need −1 < k <
0. Thus such a function exists only when −1 < k < 0. In this case the unique solution is
x 1−kx
f (x) = x+1 1−k2 .
Solution. Write
2
Since (ai − 2)(ai+1 − 2) ≥ 0, −2ai ai+1 ≤ −4(ai + ai+1 − 2) and
µ ¶
ai+2 − 2
Ai ≤ ai + ai+1 + ai+2 − 4 1 + .
ai + ai+1 − ai+2
.
. .
........
......
.....
..
O . .. .
...
...
..
. ...
.
.
.....
. ..... ..... ..
. ..... .. .. ...
.... .... ..... .. ....
.. . .... ...
.
. .
. ....... ... .
... .. ..... ..... .. ...
...
.
.
.. ...
... ..... P
.....
..... ........
..
. .
..... .. ...
.. .....
. . . .. ..
.......
....... .....
•S
. ... .. ... . .
... ..... ....... ... . ...
....... .....
... ..... ......... ... ... ... ..... ...
..
...
...
... ....
..... ..... Q
.. ... ... .. .... .............
....... ......
.......
.... ...
..... ........ .... .
........ . . . .
. . ..
.... ..... . ... .
. ..... . ......... ... ..
... ......... ..... .. ............. ..... .... ..
.. ..... ........... ..... ...
..... .......... ... ..... ....
....
. .....
....
.
.. ......
.......
.
..
... . ....
.
....... ... ........
N
..... .... ......
..........
....
..
... ..... .................... ..... .. ...
...
.
. ... .
..
. ..... • .........
.. .....
.... ... .. .
.. .
..
........ ................. •.
...
...
... .
..........................
.
. ... ..... .....
... .... R .. . ...
.............. ................. ...
..
..
... .........
. .
.....
.. .. . ..... .
...............
..............
.
V
.. ............
..
.. ..
...
... ....
.
...
. ......... .
.. ..... .............. .. ......... .... ....
....... ................. . .. .
.........................................................................................................................................................................................
A M B
3
When does equality hold?
Solution. Let E and F be the midpoints of the diagonals AC and BD. In every
quadrilateral the following relation due to Euler holds:
AB · CD + AD · BC = AC · BD.
Hence,
(AC − BD)2 + 4EF 2 = (AB − CD)2 + (AD − BC)2 .
Let us prove that 4EF 2 ≥ (AD − BC)2 . This will implies the stated inequality. Let M
be the midpoint of AB. In the triangle M EF , we have AD = 2M F , BC = 2M E, and
from triangle’s inequality, EF ≥ |M E − M F |, hence 2EF ≥ |BC − AD| and 4EF 2 ≥
(AD − BC)2 .
The equality holds if and only if the points M, E, F are collinear, which happens if
and only if AB is parallel to CD, that is ABCD is either an isosceles trapezium or a
rectangle.
20. Let Γ be a convex polygon with 2000 sides and P an interior point which does not lie
on any diagonal of Γ. Prove that P is in the interior of an even number of triangles formed
using the vertices of Γ.
Solution. First observe that if P lies in a quadrilateral, then it is contained in the interiors
of two triangles. Next, if a triangle 4 contains P , then any quadrilateral containing 4
also contains P . As each triangle in Γ is contained in 1997 quadrilaterals, the point P ∈ 4
is contained in 1997 quadrilaterals. Let m be the number of quadrilaterals containing P
and n the number of triangles containing P . Then 2m = 1997n. Hence, n must be even.
Here, we are counting the number of pairs (4, Q), where P lies in the triangle 4 which is
inside the quadrilateral Q.
4
Singapore International Mathematical Olympiad Committee
22-9-2001
1. Consider the set of integers A = {2a 3b 5c : 0 ≤ a, b, c ≤ 5}. Find the smallest number n
such that whenever S is a subset of A with n elements, you can find two numbers p, q in
A with p | q.
2. “Words” are formed with the letters A and B. Using the words x1 , x2 , . . ., xn we can
form a new word if we write these words consecutively one next to another: x1 x2 . . . xn .
A word is called a palindrome, if it is not changed after rewriting its letters in the reverse
order. Prove that any word with 1995 letters A and B can be formed with less than 800
palindromes.
3. Let n ≥ 2 be an integer and M = {1, 2, . . . , n}. For every k ∈ {1, 2, . . . , n − 1}, let
1 X
xk = (min A + max A).
n + 1 A⊂M
|A|=k
4. The lattice frame construction of 2 × 2 × 2 cube is formed with 54 metal shafts of length
1 (points of shafts’ connection are called junctions). An ant starts from some junction A
and creeps along the shafts in accordance with the following rule: when the ant reaches
the next junction it turns to a perpendicular shaft. t some moment the ant reaches the
initial junction A; there is no junction (except for A) where the ant has been twice. What
is the maximum length of the ant’s path?
5. Let n black and n white objects be placed on the circumference of a circle, and define
any set of m consecutive objects from this cyclic sequence to be an m-chain.
(a) Prove that for each natural number k ≤ n, there exists a chain of 2k consecutive
pieces on the circle of which exactly k are black.
(b) Prove that there are at least two such chains that are disjoint if
√
k≤ 2n + 2 − 2
1
Singapore International Mathematical Olympiad Committee
29-9-2001
6. We are given 1999 rectangles with sides of integer not exceeding 1998. Prove that
among these 1999 rectangles there are rectangles, say A, B and C such that A will fit
inside B and B will fit inside C.
7. We are given N lines (N > 1) in a plane, no two of which are parallel and no three
of which have a point in common. Prove that it is possible to assign, to each region of
the plane determined by these lines, a non-zero integer of absolute value not exceeding N ,
such that the sum of the integers on either side of any of the given lines is equal to 0.
8. Let S be a set of 2n + 1 points in the plane such that no three are collinear and no four
concyclic. A circle will be called good if it has 3 points of S on its circumference, n − 1
points in its interior and n − 1 in its exterior. Prove that the number of good circles has
the same parity as n.
2
Singapore International Mathematical Olympiad Committee
6-10-2001
10. Γ1 and Γ2 are two circles on the plane such that Γ1 and Γ2 lie outside each other.
An external common tangent to the two circles touches Γ1 at A and Γ2 at C and an
internal common tangent to the two circles touches Γ1 at B and Γ2 at D. Prove that the
intersection of AB and CD lie on the line joining the centres of Γ1 and Γ2
12. Let AD, BE and CF be the altitudes of an acute-angled triangle ABC. Let P be a
point on DF and K the point of intersection between AP and EF . Suppose Q is a point
on EK such that ∠KAQ = ∠DAC. Prove that AP bisects ∠F P Q.
13. In a square ABCD, C is a circular arc centred at A with radius AB, P and M are
points on CD and BC respectively such that P M is tangent to C. Let AP and AM
intersect BD at Q and N respectively. Prove that the vertices of the pentagon P QN M C
lie on a circle.
3
Singapore International Mathematical Olympiad Committee
13-10-2001
14. In triangle ABC, the angle bisectors of angle B and C meet the median AD at points
E and F respectively. If BE = CF , prove that 4ABC is isosceles.
15. Let ABCD be a convex quadrilateral. Prove that there exists a point E in the plane
of ABCD such that 4ABE is similar to 4CDE.
16. Let P, Q be points taken on the side BC of a triangle ABC, in the order B, P, Q, C.
Let the circumcircles of P AB, QAC intersect at M (6= A) and those of P AC, QAB at N
(6= A). Prove that A, M, N are collinear if and only if P, Q are symmetric in the midpoint
A0 of BC.
17. About a set of four concurrent circles of the same radius r, four of the common tangents
are drawn to determine the circumscribing quadrilateral ABCD. Prove that ABCD is a
cyclic quadrilateral.
18. Three circles of the same radius r meet at common point. Prove that the triangle
having the other three points of intersections as vertices has circumradius equal to r
4
Singapore International Mathematical Olympiad Committee
27-10-2001
25. (IMO 95) Let a, b and c be positive real numbers such that abc = 1. Prove that
1 1 1 3
+ 3 + 3 ≥ .
a3 (b + c) b (a + b) c (a + b) 2
5
Singapore International Mathematical Olympiad Committee
3-11-2001
(a) What other condition does x1 need to satisfy so that all the numbers of the
sequence are positive?
28. Suppose that a function f defined on the positive integers satisfies f (1) = 1, f (2) = 2,
and
f (n + 2) = f (n + 2 − f (n + 1)) + f (n + 1 − f (n)), (n ≥ 1).
(c) Determine, with justification, all values of n for which f (n) = 210 + 1.
29. Determine the number of all sequences {x1 , x2 , . . . , xn }, with xi ∈ {a, b, c} for i =
1, 2, . . . , n that satisfy x1 = xn = a and xi 6= xi+1 for i = 1, 2, . . . , n − 1.
30. Given is a prime p > 3. Set q = p3 . Define the sequence {an } by:
½
n for n = 0, 1, 2, . . . , p − 1,
an =
an−1 + an−p for n > p − 1.
31. A and B are two candidates taking part in an election. Assume that A receives m
votes and B receives n votes, where m, n ∈ N and m > n. Find the number of ways in
which the ballots can be arranged in order that when they are counted, one at a time, the
number of votes for A will always be more than that for B at any time during the counting
process.
6
Singapore International Mathematical Olympiad Committee
10-11-2001
32. Find all prime numbers p for which the number p2 + 11 has exactly 6 different divisors
(including 1 and the number itself.)
33. Determine all pairs (a, b) of positive integers such that ab2 + b + 7 divides a2 b + a + b.
35. (10th grade) Let d(n) denote the greatest odd divisor of the natural number n. Define
the function f : N → N by f (2n − 1) = 2n , f (2n) = n + 2n/d(n) for all n ∈ N. Find all k
such that f (f (. . . (1) . . .)) = 1997 where f is iterated k times.
36. Given three real numbers such that the sum of any two of them is not equal to 1,
prove that there are two numbers x and y such that xy/(x + y − 1) does not belong to the
interval (0, 1).
7
Singapore International Mathematical Olympiad Committee
17-11-2001
37. In the parliament of country A, each MP has at most 3 enemies. Prove that it is
always possible to separate the parliament into two houses so that every MP in each house
has at most one enemy in his own house.
39. Start with n pairwise different integers x1 , x2 , x3 , xn , n > 2 and repeat the following
step:
x1 + x2 x2 + x3 xn + x1
T : (x1 , x2 , · · · , xn ) → ( , ,···, ).
2 2 2
Show that T, T 2 , · · ·, finally leads to nonintegral component.
41. Is it possible to arrange the integers 1, 1, 2, 2, · · ·, 1998, 1998, such that there are
exactly i − 1 other numbers between any two i’s?
42. A rectangular floor can be covered by n 2 × 2 and m 1 × 4 tiles, one tile got smashed.
Show that one can not substitute that tile by the other type (2 × 2 or 1 × 4).
44. In how many ways can you tile a 2 × n rectangle by 1 × 1 squares and L trominoes?
45. In how many ways can you tile a 2 × n rectangle by 2 × 2 squares and L trominoes?
46. Let a1 = 0, |a2 | = |a1 + 1|, · · · , |an | = |an−1 + 1|. Prove that
a1 + a2 + · · · + an 1
≥− .
n 2
47. Find the number an of all permutations σ of {1, 2, . . . , , n} with |σ(i) − i| ≤ 1 for all i.
48. Can you select from 1, 12 , 14 , 18 , . . . an infinite geometric sequence with sum (a) 15 ? (b) 17 ?
8
50. Let 0 < a < b, a0 = a and b0 = b. For n ≥ 0, define
p an + bn
an+1 = an bn , bn+1 = .
2
Show that lim an = lim bn .
n→∞ n→∞
51. Let a0 , a1 = 1, an = 2an−1 + an−2 , n > 1. Show that 2k |an if and only if 2k |n.
9
Singapore International Mathematical Olympiad Committee
24-11-2001
52. Determine all α ∈ R such that there exists a nonconstant function f : R → R such
that
f (α(x + y)) = f (x) + f (y).
56. Find all functions f : N → N with the property that for all n ∈ N,
1 1 1 f (f (n))
+ + ··· + = .
f (1)f (2) f (2)f (3) f (n)f (n + 1) f (n + 1)
(a) f (x, x) = x,
10
Singapore International Mathematical Olympiad Committee
1-12-2001
a2n
58. Let a0 = 2001 and an+1 = an +1 . Find the largest integer smaller than or equal to
a1001 .
59. Given a polynomial f (x) = x100 − 600x99 + · · · with 100 real roots and that f (7) > 1,
show that at least one of the roots is greater than 7.
60. We define S(n) as the number of ones in the binary representation of n. Does there
2
exist a positive integer n such that S(n ) 501
S(n) < 2001 ?
61. A semicircle S is drawn on one side of a straight line l. C and D are points on S. The
tangents to S at C and D meet l at B and A respectively, with the center of the semicircle
between them. Let E be the point of intersection of AC and BD, and F be the point on
l such that EF is perpendicular to l. Prove that EF bisects ∠CF D.
62. At a round table are 2002 girls, playing a game with n cards. Initially, one girl holds
all the cards. In each turn, if at least one girl holds at least two cards, one of these girls
must pass a card to each of her two neighbours. The game ends when each girl is holding
at most one card.
(b) Prove that if n < 2002, then the game must end.
11
Singapore International Mathematical Olympiad Committee
8-12-2001
m
63. Suppose {ai }i=n is a finite sequence of integers, and there is a Pprime p and some
m
k, n ≤ k ≤ m such that p|ak but p 6 |aj , n ≤ j ≤ m, j 6= k. Prove that i=n a1i cannot be
an integer.
Pm 1
64. Prove that for any choice of m and n, m, n > 1, i=n i cannot be a positive integer.
65. There is a tournament where 10 teams take part, and each pair of teams plays against
each other once and only once. Define a cycle {A, B, C} to be such that team A beats
team B, B beats C and C beats A. Two cycles {A, B, C} and {C, A, B} are considered the
same. Find the largest possible number of cycles after all the teams have played against
each other.
66. Consider the recursions xn+1 = 2xn + 3yn , yn+1 = xn + 2yn with x1 = 2, y1 = 1.
Show that for each integer n ≥ 1, there is a positive integer Kn such that
³ ´
2
x2n+1 = 2 Kn2 + (Kn + 1) .
67. Suppose that ABCD is a tetrahedron and its four altitudes AA1 , BB1 , CC1 , DD1
intersect at the point H. Let A2 , B2 , C2 be points on AA1 , BB1 , CC1 respectively such that
AA2 : A2 A1 = BB2 : B2 B1 = CC2 : C2 C1 = 2 : 1. Show that the points H, A2 , B2 , C2 , D1
are on a sphere.
12
Singapore International Mathematical Olympiad Committee
15-12-2001
DA · DB · AB + DB · DC · BC + DC · DA · CA = AB · BC · CA.
71. Consider the polynomial p(x) = x2001 + a1 x2000 + a2 x1999 + ... + a2000 x + 1. where
all the ai ’s are nonnegative. If the equation p(x) = 0 has 2001 real roots, prove that
p(2001) ≥ 20022001 .
1
(a) a0 = 2001 ,
1
Prove that 1 − 2000n < 2000an < 1.
13
Singapore International Mathematical Olympiad Committee
22-12-2001
74. Let ABC be an equilateral triangle. Draw the semicircle Γ which has BC as its
diameter, where Γ lies on the opposite side of BC as A. Show that the straight lines
drawn from A that trisect the line BC also trisect Γ when they are extended.
75. ABC is an equilateral triangle. P is the midpoint of arc AC of its circumcircle, and
M is an arbitrary point of the arc. N is the midpoint of BM and K is the foot of the
perpendicular from P to M C. Prove that AN K is an equilateral triangle.
76. Two concentric circles are given with a common centre O. From a point A on the
outer circle, two tangents to the inner circle are drawn, meeting the latter at D and E
respectively. AD and ED are extended to meet the outer circle at C and B respectively.
AB 2 BE
Show that ( BC ) = BD .
77. Ali Baba the carpet merchant has a rectangular piece of carpet whose dimensions
are unknown. Unfortunately, his tape measure is broken and he has no other measuring
instruments. However, he finds that if he lays it flat on the floor of either of his storerooms,
then each corner of the carpet touches a different wall of that room. He knows that the
sides of the carpet are integral numbers of feet, and that his two storerooms have the
same (unknown) length, but widths of 38 and 50 feet respectively. What are the carpet’s
dimensions?
78. Let I be the incentre of the non-isosceles triangle ABC. Let the incircle of ABC
touch the sides BC, CA and AB at the points A1 , B1 and C1 respectively. Prove that the
circumcentres of ∆AIA1 , ∆BIB1 and ∆CIC1 are collinear.
14
Singapore International Mathematical Olympiad Committee
1. Consider the set of integers A = {2a 3b 5c : 0 ≤ a, b, c ≤ 5}. Find the smallest number n
such that whenever S is a subset of A with n elements, you can find two numbers p, q in
A with p | q.
Soln. Identify a number 2a 3b 5c in A by its triple of indices abc. Thus 000 represents
20 30 50 while 213 represents 22 31 53 . Consider the following arrangement of numbers into
16 rows.
000
100
200 110
300 111 210 120
400 211 310 130 220
500 311 410 140 320 302 221
510 411 420 240 321 312 330 222
520 511 430 340 421 412 331 322
530 521 440 341 431 512 332 422
540 531 441 342 432 513 333 522
550 541 442 352 532 514 433
551 542 443 353 524
552 543 444 534
553 544
554
555
Turn the first column in 6 by considering the six permutations and turn each of the other
columns into 3 by considering the cyclic permutations. Note that (i) every number is one
of the rows, (ii) every two numbers in the same row do not divide each other (iii) for every
two numbers in the same column, one must divided the other. Since the row with the most
elements is 27, the answer is 28.
(Joel’s soln) The problem is equivalent to the following: Consider the ordered triples
of the form {(a, b, c) : 0 ≤ a, b, c ≤ 5}. Find the smallest number n such that if there are
n such triples, you can always find two (p, q, r) and (x, y, z) such that p ≥ x, q ≥ y, and
r ≥ z. If this is the case, we say that the two triples are comparable.
First we note that any two triples are comparable if their second and third elements
are the same. Let A be a set of pairwise noncomparable triples. Since there are 36 possible
combinations for (b, c), A has at most 36 triples. Examine the sequence of 11 triples in
order:
(a, 0, 0), (a, 0, 1), (a, 0, 2), (a, 0, 3), (a, 0, 4), (a, 0, 5), (a, 1, 5), (a, 2, 5), (a, 3, 5), (a, 4, 5), (a, 5, 5).
15
For these to be in A, the first elements have to be strictly decreasing. Thus at most 6
of these can be in A or at least 5 cannot be in A. Similarly, at least three of the following
triples
(a, 1, 0), (a, 2, 0), (a, 3, 0), (a, 4, 0), (a, 5, 0), (a, 5, 1), (a, 5, 2), (a, 5, 3), (a, 5, 4)
and at least 1 of the following triples:
(a, 1, 1), (a, 1, 2), (a, 1, 3), (a, 1, 4), (a, 2, 4), (a, 3, 4), (a, 4, 4)
are not in A. Thus A can have at most 27 triples and if 28 triples are chosen, then at least
two of them are comparable. Below is a collection of 27 pairwise noncomparable triples
(the sum of the 3 components is 7):
(0, 2, 5), (0, 3, 4), (0, 4, 3), (0, 5, 2), (1, 1, 5), (1, 2, 4), (1, 3, 3),
(1, 4, 2), (1, 5, 1), (2, 0, 5), (2, 1, 4), (2, 2, 3), (2, 3, 2), (2, 4, 1),
(2, 5, 0), (3, 0, 4), (3, 1, 3), (3, 2, 2), (3, 3, 1), (3, 4, 0), (4, 0, 3),
(4, 1, 2), (4, 2, 1), (4, 3, 0), (5, 0, 2), (5, 1, 1), (5, 2, 0)
So the answer is 28.
2. (Byelorrussian MO 95) “Words” are formed with the letters A and B. Using the words
x1 , x2 , . . ., xn we can form a new word if we write these words consecutively one next to
another: x1 x2 . . . xn . A word is called a palindrome, if it is not changed after rewriting its
letters in the reverse order. Prove that any word with 1995 letters A and B can be formed
with less than 800 palindromes.
Soln. Official solution: (The key idea is to find the longest word that can be formed using
at most 2 palindromes.) First of all, it is easy to check that any 5-letter word may be
formed with at most two palindromes. Indeed, (A and B are symmetric).
AAAAA = AAAAA, AAAAB = AAAA + B, AAABA = AA + ABA,
AAABB = AAA + BB, AABAA = AABAA, AABAB = AA + BAB,
AABBA = A + ABBA, AABBB = AA + BBB, ABAAA = ABA + AA,
ABAAB = A + BAAB, ABABA = ABABA, ABABB = ABA + BB,
ABBAA = ABBA + A, ABBAB = ABBA + B, ABBBA = ABBBA,
ABBBB = A + BBBB.
Let us consider an arbitrary word with 1995 letters and divide it into words with
5 letters each. Each of these 1995/5 = 399 words may be formed with at most two
palindromes. Thus any 1995-letter word may be formed with at most 399 × 2 = 798
palindromes.
16
Prove that x1 , x2 , . . . , xn−1 are integers, not all divisible by 4.
¡ n−i ¢
Soln. For each i ∈ {1, 2, . . . , n}, there are k−1 subsets, each with k elements and contains
¡n¢ ¡ i−1 ¢
i as the minimum element. (Note that k = 0 if k > n.) Also there are k−1 subsets,
each with k elements and contains i as the maximum element. Thus
1 X
xk = (min A + max A)
n + 1 A⊂M
|A|=k
· µ ¶ µ ¶ µ ¶
1 n−1 n−2 k−1
= 1 +2 + · · · + (n + 1 − k)
n+1 k−1 k−1 k−1
µ ¶ µ ¶ µ ¶¸
n−1 n−2 k−1
+n + (n − 1) + ··· + k
k−1 k−1 k−1
µ ¶
n
=
k
¡ ¢ ¡ n ¢
Thus x1 , . . . , xn−1 are all integers. Since x1 + · · · + xn−1 = n1 + · · · + n−1 = 2n − 2, not
all of x1 , . . . , xn−1 are divisible by 4.
(2nd Solution) For each k element subset A = {a1 , . . . , ak } with ai < aj if i < j, with
a1 = 1 + p and ak = n − q, let B = {b1 , . . . , bk }, where bi = ai + q − p. We have ¡ ¢
min A + min B + max A + max B = 2n + 2. Since the number of k element subsets is nk ,
we have µ ¶ µ ¶
1 X 1 n 2n + 2 n
xk = (min A + max A) = = .
n + 1 A⊂M n+1 k 2 k
|A|=k
A0 = {n + 1 − x|x ∈ A}
Then, min A = n + 1 − max A0 . Do the usual summation rewriting, and you can
compute xk explicitly to be some binomial function. The other bit follows readily too
using another standard trick.
Soln. Official solution: The maximum length of an ant’s path is equal to 24. First we
prove that the path along which the ant creeps, has at most 24 junctions of the shafts of
17
the cube frame. By the condition, any two consecutive shafts in the path (except possibly
for the first and the last shafts) are perpendicular. In particular, the ant’s path passes at
most one shaft on all but one edge of the cube. Thus there are at most 13 shafts along the
edges of the cube. However, each vertex in the path requires two shafts. Thus the path
misses at least two vertices of the cube. Hence the ant’s path passes through at most 25
junctions of the shafts. The ant’s path consists of an even number of junctions. This is
easily seen to be true by taking the starting point as the origin and the three mutually
perpendicular lines passing through it as the axes and assume that each shaft is of unit
length. Then each move by the ant causes exactly a change of one unit in one of the
coordinates. Thus the total number of moves is even. Thus the length of the path is at
most 24. A path of length 24 is shown in the figure.
•••••••••••••••••••••••••••••••••••••••••••••••• ••
••••• ••••••••
........................................................................................................................
...... . .
.... ... .... ... .... ...
•• • ••• ..... .
..... .... •
•• ..... .
..... ....
..... .
..... ....
•
••••••••••••••••••••••••••••••••••••••••••••••••• ••••• •
•••••• •••
..
.. ..
.. ..
..
......................................................................................................................................... ....
....... .... ....
.... ..
..... .... • ••• •
• .. ••• •
.
.... ..
..... .... ..
.
.... ..
..... ....
...
..
..
•• ••• ••• ••
.. ..
•••••••••••••••••••••••••••••••••••••••••••••••••
.
.... . . .
.... . . .
.... . ..
..
. . .
. ..
. . .
. ..
. . ...
................................................................................................................................. .
•
• .... .... •
••• ... ••• •• .... ... ... ....
.
.
...
..
..
• ••••••••••••••••••••••••••••••••••••••••••••••••••
•
.. . . . .
. . . .
. .
....
• .........................................................................................................................................
•
•
...
..
•••••• ..
... ............
.. . . .
.. .
.... . . .
....
• ...
•• ••• .... . .. .
. .
. .
. .
...
•••• •
. ... .
... ......... .... ... ......... ....
• .. .. ...... ... ..
••••••• •••
...
•
•
...
..
... ...... .
••
.. . ...
...............................................................................................................................................
•
. . ... ..
...
•
•
...
.. ......... ....
•• ••
..... . .
...
••
••• .
...
... ..
.. ..
. . .
.
.
.
... .
.. ..
.. ..
. . ..
..
• ... ..... .. .
••
••
....
. ...... ...
.
... ..... . .
.
.
. ...... ...
.
... ..... . ...
••••••••••••••••••••••••••••••••••••••••••••••••••• •••••
........ ... .
•...........................................................................................................................................
.. ... ... ..
. .
...
.
... ..
. .
..
.
...
..
..
...
••• •• ••
...
..
...
.. ••• ..
..
...
..
..
...
..
..
...
..
..
...
..
.
••• •••••••••••••••••••••••••••••••••••••••••••••••• •••
...
..
...
.. .. .. ................................................................................................................................
. . . . .
.
.
•••• ••
... ......... .. ..
•••• .... .. . . . .
...
•• .
... ........
..
.. .
... .
.... ........
..
..
•• •••••••••••••••••••••••••••••••••••••••••••••••••••••
.. .. ...... ..
...
• •
•
...
..
....... ..
.........................................................................................................................................
...... ......
5. Let n black and n white objects be placed on the circumference of a circle, and define
any set of m consecutive objects from this cyclic sequence to be an m-chain.
(a) Prove that for each natural number k ≤ n, there exists a chain of 2k consecutive
pieces on the circle of which exactly k are black.
(b) Prove that there are at least two such chains that are disjoint if
√
k ≤ 2n + 2 − 2
Soln. (a) Label the positions 1, 2, . . . , 2n in the clockwise direction. For each i, if the
object Pis i is black, let ai = 1. Otherwise, let ai = −1. Fix k ≤ n. Define the function
i+2k−1
g(i) = j=i aj . Then there is a chain of 2k consecutive pieces of which exactly k are
P2n
black iff g(i) = 0 for some i. Since i=1 g(i) = 0, and since g(i + 1) − g(i) = ±2, 0, such a
g(i) exists.
(b): Suppose on the contrary that there does not exist 2 disjoint chains. WLOG,
we may assume from (a) that g(2k) = 0 and g(2k + 1), . . . g(2n) are all nonzero, and
18
by the same continuity argument, they must all be of the same sign. WLOG, assume
g(2k + 1), . . . , g(2n) < 0 and thus g(2k), . . . , g(2n − 1) ≤ −2. Therefore, we have:
Thus, the sum on the left is maximum when g(1) = 0, g(1) = 2, . . . , g(k) = g(k + 1) =
2(k − 1), . . . , g(2k) = 0, that is, g(1), . . . , g(k) forms an increasing arithmetic progression
with a difference of 2, and g(k + 1), . . . , g(2k) forms a decreasing arithmetic progression
with a difference of −2. This yields:
6. We are given 1999 rectangles with sides of integer not exceeding 1998. Prove that
among these 1999 rectangles there are rectangles, say A, B and C such that A will fit
inside B and B will fit inside C.
Soln. (T& T Spring 1989 Q6, modified slightly). We partition the given set of rectangles
into 999 suitably chosen (pairwise disjoint) subsets S1 , . . ., S999 . These are defined as
follows:
For each i ∈ {1, . . . , 999}, let Si be the set of rectangles which have the following
properties:
(c) either the shorter side has length i or the longer side has length 1998 − i.
Any three rectangles in Si can always be arranged with the desired inclusion property.
Thus the result follows by the pigeon hole principle.
7. We are given N lines (N > 1) in a plane, no two of which are parallel and no three
of which have a point in common. Prove that it is possible to assign, to each region of
the plane determined by these lines, a non-zero integer of absolute value not exceeding N ,
such that the sum of the integers on either side of any of the given lines is equal to 0.
Soln. (Tournament of Towns Spring 1989 Q5). First note the regions can be painted
in two colours so that two regions sharing a common side have different colours. This is
19
trivial for N = 1. Assume that it is true for N = k. When the (k + 1)st line is drawn we
simply reverse the colours on exactly one side of this line.
Assign to each region an integer whose magnitude is equal to the number of vertices
of that region. The sign is + is the region is one of the two colours and − if it is the other
colour. Let L be one of the given lines. Consider an arbitrary vertex on one side of L. If
this vertex is on L, then it contributes = 1 to one region and −1 to a neighbouring region.
If it is not on L, it contributes = 1 to two regions and −1 to two regions. Thus the sum
of the numbers of one side of L is 0.
8. (APMO 99) Let S be a set of 2n + 1 points in the plane such that no three are collinear
and no four concyclic. A circle will be called good if it has 3 points of S on its circumference,
n − 1 points in its interior and n − 1 in its exterior. Prove that the number of good circles
has the same parity as n.
Soln. For any two points A and B, let P1 , P2 , . . ., Pk be points on one side of the line
AB and Pk+1 , . . ., P2n−1 be points on the other side. We shall prove that the number of
good circles passing through A and B is odd. Let
½
∠APi B if i = 1, . . . , k
θi =
180◦ − ∠APi B if i = k + 1, . . . , 2n − 1
It is easy to see that Pj is in the interior of the circle ABPi , if and only if
½
θj > θi for 1 ≤ j ≤ k
.
θj < θi for k + 1 ≤ j ≤ 2n − 1
Arrange the points Pi in increasing order of their corresponding angles θi . Colour the
points Pi , i = 1, . . . , k, black and the points Pi , i = k + 1, . . . , 2n − 1, white. For any point
X (different from A and B), let BX be the number of black points less than X minus the
number of black points greater than X and WX be the corresponding difference for white
points. (Note that black points which are greater than X are interior points of the circle
ABX while the white points greater than X are exterior points.) Define DX = BX − WX .
From the forgoing discussion we know that 4ABX is good if and only if DX = 0. We call
such a point good . If X < Y are consecutive points, then DX = DY if X and Y are of
different colours. (It is easy to show that DY − DX = −2 if X and Y are both white and
DY − DX = 2 if X and Y are both black. But we do not need these.)
If all the points are of the same colour, there is only one good point, namely the
middle point among the Pi ’s.
Now we suppose that there are points of either colour. Then there is a pair of adjacent
points, say X, Y , with different colour. Since DX = DY , either both are good or both are
not good. Their removal also does not change the value of DZ for any other point Z. Thus
the removal of a pair of adjacent points of different colour does not change the parity of
20
the number of good points. Continue to remove such pairs until only points of the same
colour are left. When this happens there is only one good point. Thus the number of good
circles through A and B is odd.
Now let gAB be the number of good circles through A and P B. Since each good circle
contains exactly three points, i.e., three pairs of points. Then gAB = 3g where g is total
number of good circles. Since there are a total of n(2n + 1) terms in the sum, and each
term is odd, we have g ≡ n (mod 2).
a b c 3
+ + ≥
b+c c+a a+b 2
Soln 2.
· ¸
a b c
+ + [a(b + c) + b(c + a) + c(a + b)] ≥ (a + b + c)2 .
b+c c+a a+b
Thus
a b c (a + b + c)2
+ + ≥ .
b+c c+a a+b 2(ab + ac + bc)
Also
(a + b + c)2 = (a2 + b2 + c2 ) + 2(ab + bc + ac) ≥ 3(ab + bc + ac).
Thus
(a + b + c)2 3
≥ .
a(b + c) + b(c + a) + c(a + b) 2
Soln 3. Since the inequality is symmetric about a, b, c, we may assume that a ≥ b ≥ c > 0.
Then,
1 1 1
≥ ≥ .
b+c a+c a+b
21
By rearrangement inequality, we have
a b c a b c
+ + ≥ + + ,
b+c c+a a+b c+a a+b b+c
a b c a b c
+ + ≥ + + .
b+c c+a a+b a+b b+c c+a
The inequality follows by adding these two inequalities.
1 1 1 3
+ + ≥ .
1+a 1+b 1+c 2
1 1 1 9 3
+ + ≥ ≥ .
1+a 1+b 1+c (1 + a) + (1 + b) + (1 + c) 2
Thus
1 1 1 9 3
+ + ≥ ≥ .
x y z 3+a+b+c 2
a b c 1
+ + ≥ .
10b + 11c 10c + 11a 10a + 11b 7
a b c 3
+ + ≥ ,
mb + nc mc + na ma + nb m+n
22
Since aA + bB + cC = (m + n)(ab + bc + ca) and 3(ab + bc + ca) ≤ (a + b + c)2 , the result
follows.
Soln 2. The inequality can also be proved by clearing the denominator and moving all
terms to the left hand side. After some simplification, the resulting inequality to be proved
is equivalent to
Soln. Use the same method as in the previous problem. The only extra thing that you
have to note is that X 1 X
a2i ≥ ai aj .
2
1≤i<j≤5
23. (SMO2001) Let n be a positive integer and let a1 , a2 , . . ., an be n positive real numbers
such that a1 + a2 + · · · + an = 1. Is it true that
Thus
a41 a42 a4n
+ + · · · +
a21 + a22 a22 + a23 a2n + a21
µ 4 ¶
1 a1 + a42 a42 + a43 a4n + a41
= + 2 + ··· + 2
2 a21 + a22 a2 + a23 an + a21
1
≥ [(a21 + a22 ) + · · · + (a2n + a21 )]
4
1 1
= (a21 + · · · + a2n ) ≥
2 n
23
which completes the proof. The last inequality follows as
r
a21 + · · · + a2n a1 + · · · + an 1
≥ = .
n n 2n
24
Soln 2. Let the denominators be A, B, C, D. Then
· 3 ¸
a d3
+ ··· + [aA + · · · + dD] ≥ (a2 + · · · + d2 )2 .
A D
x2 1
≥ , or 3x2 − x − 2 = (3x + 2)(x − 1) ≥ 0
2+x 3
25. (IMO 95) Let a, b and c be positive real numbers such that abc = 1. Prove that
1 1 1 3
+ 3 + 3 ≥ .
a3 (b+ c) b (a + b) c (a + b) 2
Soln. Let S be the right hand side and T = a(b + c) + b(a + c) + c(a + b) = 2(ab + ac + bc) ≥
6(abc)2/3 = 6. Then by Cauchy’s inequality, we have
µ ¶2
1 1 1 T2
ST ≥ + + = .
a b c 4
x2 y2 z2
S= + + .
y+z x+z x+y
By Cauchy’s inequality,
25
Find all values of n such that 11|an .
(a) What other condition does x1 need to satisfy so that all the numbers of the
sequence are positive?
Soln.
√
(a) For the sequence to be positive, we require 3 − 3x2n ≥ x2n , ie that |xn | ≤ 23 . One
√
can easily check by plugging into the recurrence relation that when |xn | ≤ 23 ,
√
then |xn+1 | ≤ 23 also, so the entire sequence will be positive. Hence, the required
√
condition is |x1 | ≤ 23 .
sequence is periodic.
28. Suppose that a function f defined on the positive integers satisfies f (1) = 1, f (2) = 2,
and
f (n + 2) = f (n + 2 − f (n + 1)) + f (n + 1 − f (n)), (n ≥ 1).
(c) Determine, with justification, all values of n for which f (n) = 210 + 1.
26
f (0) = 0 for convenience; this definition is consistent with the recurrence relation.) Once
we have established this claim, parts (a) and (b) follow essentially by induction. Also,
the answer to (c) is n = 211 . (Endnote: The key to solving this question would be to
list out the values and try to find a pattern, which can then be rigorously justified using
induction.)
29. Determine the number of all sequences {x1 , x2 , . . . , xn }, with xi ∈ {a, b, c} for i =
1, 2, . . . , n that satisfy x1 = xn = a and xi 6= xi+1 for i = 1, 2, . . . , n − 1.
30. Given is a prime p > 3. Set q = p3 . Define the sequence {an } by:
½
n for n = 0, 1, 2, . . . , p − 1,
an =
an−1 + an−p for n > p − 1.
Soln. (This remains an open question. I suspect the key is to prove that ap2 −1 ≡ a0 (mod
p), from which it follows that ap3 ≡ p − 1 (mod p).)
31. A and B are two candidates taking part in an election. Assume that A receives m
votes and B receives n votes, where m, n ∈ N and m > n. Find the number of ways in
which the ballots can be arranged in order that when they are counted, one at a time, the
number of votes for A will always be more than that for B at any time during the counting
process.
Soln. We model the vote counting process on the Cartesian plane. Starting at (0, 0), we
move to (1, 1) if the first vote goes to A, else we move to (1, −1) if the first vote goes to B
instead. Notice that this walk will end at (m + n, m − n). The number of ways of counting
in which the number of votes A gets is always more than that which B has is equal to the
number of paths on the plane from (0, 0) to (m+n, m−n) that do not cross the x-axis. We
count the complement of this set, ie the number of paths that do cross the x-axis. Notice
that the first vote must go to A. Starting from (1, 1), the number of paths that cross the
27
x-axis and end at (m+n, m−n) is equal to the number of paths (unrestricted) from (1, −1)
to (m + n, m − n). To see this, take any given path from (1, 1) to (m + n, m − n) and let
X be the first point of intersection with the x-axis. Reflect the portion of this path to the
left of X about the x-axis and we have a path from (1, −1) to (m + n, m − n). Vice versa,
we can take any such path from (1, −1) to (m + n, m¡− n) and ¢ reflect it back uniquely. The
number of paths from (1, −1) to (m + n, m − n) is m+n−1 m , since you need to choose m
steps upwards out¡ of a possible
¢ m + n − 1. Similarly, the number of paths from (1, 1) to
m+n−1
(m + n, m − n) is m−1 . Hence, the number of paths from (0, 0) to (m + n, m − n) that
¡ ¢ ¡m+n−1¢ m−n ¡m+n¢
do not cross the x-axis is m+n−1
m−1 − m = m+n m .
32. (Auckland MO 98) Find all prime numbers p for which the number p2 + 11 has exactly
6 different divisors (including 1 and the number itself.)
Soln. If p = 3, p2 + 11 has exactly 6 divisors. Now let p > 3. Then p is odd and p2 ≡ 1
(mod 4), thus p2 + 11 ≡ 0 (mod 4). Also p2 ≡ 1 (mod 3), thus p2 + 11 ≡ 0 (mod 3).
Thus p2 + 11 ≡ 0 (mod 12). Since every p2 + 11 > 12 and every divisor of 12 is also a
divisor of p2 + 11, it follows that p2 + 11 has more divisors than 12. Thus p2 + 11 has more
than 6 divisors. The only prime number with the desired property is therefore 3.
33. (IMO 98) Determine all pairs (a, b) of positive integers such that ab2 + b + 7 divides
a2 b + a + b.
For the second case, we note that b2 − 7a < ab2 + b + 7. Thus b2 − 7a < 0. If b ≥ 3,
then ab2 + b + 7 > 7a − b2 . Thus for ab2 + b + 7 to divide 7a − b2 , b = 1, 2. The case b = 1
requires that 7a − 1 be divisible by a + 8. The quotients are less than 7. Testing each of
the possibilities yields a = 49, 11. These are indeed solutions.
28
Soln. All the congruences are taken mod p. For each 0 < i < p, we have ip−1 ≡ 1. Thus
ip−2 ≡ 1i . So
µ ¶p−2
p−2 p−2 p−2 p−1 1 1 1
1 +2 +3 + ··· + ≡ + + ··· + .
2 1 2 (p − 1)/2
1 1
The last congruence follows because i + p−i ≡ 0.
Hence
(p−1)/2
X (p−1)/2
X µ ¶
p−2 p−2 2i p
i ≡− i
i=1 i=1
p 2i
(p−1)/2 µ ¶
2 X p−1 p
≡− i
p i=1 2i
(p−1)/2 µ ¶
2 X p
≡− (mod p) by Fermat’s Theorem.
p i=1 2i
The last summation counts the even-sized nonempty subsets of a p-element set, of which
there are 2p−1 − 1.
35. (Ukrainian MO 97) (10th grade) Let d(n) denote the greatest odd divisor of the natural
number n. Define the function f : N → N by f (2n − 1) = 2n , f (2n) = n + 2n/d(n) for all
n ∈ N. Find all k such that f (f (. . . (1) . . .)) = 1997 where f is iterated k times.
29
am+i+1 = (2i + 1)2j−i−1 + 2j−i = (2(i + 1) + 1)2j−i−1 .
Thus the result follows by induction. Since a1 = 20 , we can find write down a formula
for an as follows. If n = (0 + 1 + 2 + · · · + p) + 1 + q, where p ≥ 0, 0 ≤ q ≤ p, then
an = (2q + 1)2p−q . If (2q + 1)2p−q = 1997, then p = q = 998. Thus a499500 = 1997. Thus
k = 499499.
36. (Byelorrussian MO 95) Given three real numbers such that the sum of any two of
them is not equal to 1, prove that there are two numbers x and y such that xy/(x + y − 1)
does not belong to the interval (0, 1).
Soln. Official solution: Let a, b, c be the given numbers. Suppose that each of the numbers
ab ac bc
A= , B= , C= .
a+b−1 a+c−1 b+c−1
a2 b2 c2
ABC = > 0.
(a + b − 1)(a + c − 1)(b + c − 1)
Hence
D = (a + b − 1)(a + c − 1)(b + c − 1) > 0. (∗)
On the other hand, A − 1 < 0, B − 1 < 0, C − 1 < 0. Thus (A − 1)(B − 1)(C − 1) < 0. It
is easy to verify that
Consequently,
52. Determine all α ∈ R such that there exists a nonconstant function f : R → R such
that
f (α(x + y)) = f (x) + f (y).
αx
Soln. For α = 1, f (x) = x satisfies the functional equation. For α 6= 1, let y = 1−α , but
this implies f (y) = f (x) + f (y), hence f (x) = 0.
30
53. Let f : N → N be a function satisfying
Soln. Note that f (n0 + 1) = f (n0 + f (n0 )) = f (n0 ) = 1. Hence f (n) = 1 for all n ≥ n0 .
Let S = {n ∈ N|f (n) 6= 1}. If S = ∅, then we are done. So suppose S 6= ∅, let
M = supS = maxS, since S is a finite set. Then f (M + f (M )) = f (M ) 6= 1. Hence
M + f (M ) ∈ S, but M + f (M ) > M , contradicting the maximality of M . Hence S = ∅,
f (n) = 1 for all n ∈ N
Soln. Note that gcd(3f (n), 1 + 3f (n)) = 1, hence 3f (n)|f (2n). But f (2n) < 6f (n),
hence f (2n) = 3f (n), f (2n + 1) = 3f (n) + 1.Claim: If n = (b1 b2 · · · bn )2 , then f (n) =
(b1 b2 · · · bn )3 .This is a simple exercise in induction (induct on n, consider n odd and n
even separately) and is left to the reader. Then to find integers k, m such that f (k) +
f (m) = 2001, let us first write 2001 in its tenary representation. Now 2001 = 22020103 .
We note that the tenary representations of f (k), f (m) can only contain 0 and 1’s, hence
there are also no carry-overs from the addition. Hence the possible pairs of k, m are only
(11010002 , 11010102 ) and (11010102 , 11010002 ), i.e. (104, 106) and (106, 104).
Soln. Let x = y = 0 and we have f (0) = 0. Setting y = 0, we have f (x3 ) = x(f (x))2 ,
or equivalently, f (x) = x1/3 f (x1/3 )2 . In particular, x and f (x) have the same sign.Let S
be the set S = {a > 0 : f (ax) = af (x)∀x ∈ <}.Clearly 1 ∈ S, and we will show a1/3 ∈ S
whenever a ∈ S. In fact, axf (x)2 = af (x3 ) = f (ax3 ) = f ((a1/3 x)3 ) = a1/3 xf (a1/3 x)2 .
Hence (a1/3 f (x))2 = f (a1/3 x)2 . Since x and f (x) have the same sign, we conclude that
f (a1/3 x) = a1/3 f (x).Now we show that if a, b ∈ S, then a + b ∈ S. Indeed, we have
f ((a + b)x) = f ((a1/3 x1/3 )3 + (b1/3 x1/3 )3 )
= (a1/3 + b1/3 )x1/3 (f (a1/3 x1/3 )2 − f (a1/3 x1/3 )f (b1/3 x1/3 ) + f (b1/3 x1/3 )2 )
= (a1/3 + b1/3 )(a2/3 − a1/3 b1/3 + b2/3 )x1/3 f (x1/3 )2
= (a + b)f (x).
31
Hence in particular, we have f (2001x) = 2001f (x).
56. Find all functions f : N → N with the property that for all n ∈ N,
1 1 1 f (f (n))
+ + ··· + = .
f (1)f (2) f (2)f (3) f (n)f (n + 1) f (n + 1)
Soln. Let n = 1 and we have f (f (1))f (1) = 1; hence f (1) = 1. Replacing the equality for
n into one for n + 1 we obtain
f (f (n)) 1 f (f (n + 1))
+ = .
f (n + 1) f (n + 1)f (n + 2) f (n + 2)
This is equivalent to
Since n+1 > 1, f (n+1) > 1 and thus f (f (n+1)) > 1, which implies that f (n+2) > f (f (n+
1)).The problem is now reduced to finding all functions f such that f (f (n)) < f (n + 1).
f has a unique minimum at n = 1, for if n > 1, we have f (n) > f (f (n − 1)). By the same
reasoning, we see that the second smallest value is f (2), etc. Hence
Now since f (1) ≥ 1, we have f (n) ≥ n. Now suppose for some natural number n, we have
f (n) > n. Then f (n) ≥ n + 1. Since f is increasing, f (f (n)) ≥ f (n + 1), a contradiction.
Hence f (n) = n for all n ∈ N.
(a) f (x, x) = x,
32
(e) g(xy) = g(x) + g(y) + mg(f (x, y)).
Soln. We claim that f (x, y) = gcd(x, y). This is a simple exercise in induction. (induct
on the sum x + y).Now let a = b and we have g(a2 ) = (m + 2)g(a). Applying this again
we get g(a4 ) = (m + 2)g(a2 ) = (m + 2)2 g(a).On the other hand,
Now let a = 2001, and we have (m + 2)2 = 3m + 4 and hence m = 0, −1. For m = 0, an
example is given by
g(pα αn
1 · · · pn ) = α1 h(p1 ) + · · · + αn h(pn ),
1
where k is a prime factor of 2001, h(3) = 2002 and h(p) = 0 for all primes p 6= 3. For
m = −1, an example is given by
g(pα αn
1 · · · pn ) = h(p1 ) + · · · + h(pn ).
1
It is easy to check that these functions do indeed satisfy the given requirements.
Soln.
Pm 1Clearly,
Pm it 1is meaningful to consider the cases where m > n. If ¥mm ¦≤ 2n, then
i=n i < i=n n = 1. If m > 2n, then therePis a prime between m and 2 , which we
m
call p. From question 1, we can conclude that i=n 1i can never be an integer.
33
65. There is a tournament where 10 teams take part, and each pair of teams plays against
each other once and only once. Define a cycle {A, B, C} to be such that team A beats
team B, B beats C and C beats A. Two cycles {A, B, C} and {C, A, B} are considered the
same. Find the largest possible number of cycles after all the teams have played against
each other.
Soln. We label the teams T1 , T2 , ..., T10 . Let the number of times Ti wins be wi and the
number of losses be li . Instead of looking at the cycles as it is, we first look at the total
number of 3-combinations of teams and the number of 3-combinations which cannot be
rearranged to form a cycle. The total number of 3-combinations is 10 C3 = 120. Next,
we look at the 3-combinations that can be made to form cycles and those which cannot
(from here on, we call these non-cycles). The non-cycles consists of a single team beating
the other two teams and a single team losing to the other 2 teams, and if a 3-combination
consists of a single team beating the other two teams or a single team losing to the other
2 teams, it is a non-cycle. We can then count the non-cycles in a different manner. By
focusing
P10 ¡on ¢this¡lproperty
¢ of non-cycles, we can deduce that the number of non-cycles
wi
is + i
. Clearly, we have wi + li = 9, and rearranging the formula gives
P10 i=1
¡wi ¢ 2 ¡li ¢ 2 P10 ¡ 2 ¢
i=1 2 + 2 = i=1 wi − 9wi + 36 . And the number of non-cycles and the number
of cycles sum up to 120. We aim to minimise the number of non-cycles. This can be
achieved if wi = 4 or 5, and is certainly attainable. (The answer to this question is 40.)
The description of such a case is:
(i) If Ta and Tb are such a and b have the same parity, and a > b, then Ta beats Tb .
(ii) If Ta and Tb are such a and b have different parity, and a > b, then Ta loses to Tb .
66. Consider the recursions xn+1 = 2xn + 3yn , yn+1 = xn + 2yn with x1 = 2, y1 = 1.
Show that for each integer n ≥ 1, there is a positive integer Kn such that
³ ´
2
x2n+1 = 2 Kn2 + (Kn + 1) .
Therefore, it suffices to show that x2n+1 − 1 is an odd perfect square. Expressing xn+2
and yn+2 in terms of xn and yn , we have xn+2 = 7xn + 12yn and yn+2 = 4xn + 7xn .
As we try to express xn−2 in terms of xn and yn , we have xn−2 = 7xn − 12yn . Hence
xn+2 = 14xn − xn−2 .
34
To elaborate certain details. With xn+2 = 14xn − xn−2 , x1 = 2 and x3 = 26, we can
solve for xn to get:
1³ √ ´³ √ ´n 1 ³ √ ´³ √ ´n
x2n+1 = 2+ 3 7+4 3 + 2− 3 7−4 3
2 2
1³ √ ´³ √ ´2n 1 ³ √ ´³ √ ´2n
= 2+ 3 2+ 3 + 2− 3 2− 3
2 2
1 ³ √ ´ ³ √ ´ 2n 1³ √ ´³ √ ´2n
= 4+2 3 2+ 3 + 4−2 3 2− 3 −1+1
4 4
µ ³ ¶ µ ³ ¶
1 √ ´ 2³ √ ´2n 1 √ ´ 2³ √ ´2n
= 1+ 3 2+ 3 + 1− 3 2− 3 −1+1
2 2
µ ³ ¶ µ ³ ¶
1 √ ´³ √ ´n 2 1 √ ´³ √ ´n 2
= 1+ 3 2+ 3 + 1− 3 2− 3 −1+1
2 2
µ ³ ¶
1 √ ´³ √ ´n 1 ³ √ ´³ √ ´n 2
= 1+ 3 2+ 3 + 1− 3 2− 3 +1
2 2
67. Suppose that ABCD is a tetrahedron and its four altitudes AA1 , BB1 , CC1 , DD1
intersect at the point H. Let A2 , B2 , C2 be points on AA1 , BB1 , CC1 respectively such that
AA2 : A2 A1 = BB2 : B2 B1 = CC2 : C2 C1 = 2 : 1. Show that the points H, A2 , B2 , C2 , D1
are on a sphere.
Soln. Let G be the centroid of the triangle ABC. Prove that ∠HA2 G = ∠HB2 G =
∠HC2 G = ∠HD1 G = 90o . Thus, The centre of the sphere that we are looking for is at
the midpoint of H and G.
35
IMO Winter and Summer
Camp Trainings Test
Winter Camp 2008 Buffet Contest
Saturday, January 5, 2008
List of problems
A1. Find all functions f : R → R such that for all real numbers x and y,
f (xf (y) + x) = xy + f (x).
Solutions
A1. Find all functions f : R → R such that for all real numbers x and y,
a2 xy − abx − ax + b = xy − ax + b.
Equating coefficients, we get a = ±1 and b = 0, so f (x) = x or f (x) = −x. We can easily check both
are solutions.
A2. Let x, y, z be positive real numbers. Prove that
x y z
p + p + p ≤ 1.
x+ (x + y)(x + z) y+ (y + z)(y + x) z+ (z + x)(z + y)
√ √
Solution: By Cauchy, we have (x + y)(x + z) ≥ ( xy + xz)2 . Hence,
√
X x X x X x
p ≤ √ √ = √ √ √ = 1.
cyc x+ (x + y)(x + z) cyc
x + xy + xz cyc
x + y+ z
A3. Let p(x) be a polynomial with integer coefficients. Does there always exist a positive integer k such
that p(x) − k is irreducible?
(An integer polynomial is irreducible if it cannot be written as a product of two nonconstant integer
polynomials.)
Solution: Yes. Choose k such that the constant term of p(x)−k is −q, where q is some large prime (to
be specified later). Now, suppose p(x) − k = f (x)g(x) for some nonconstant integer polynomials f, g.
Since q is prime, looking at the constant term, we must have that either |f (0)| = 1 or |g(0)| = 1. Assume
without loss of generality |f (0)| = 1. Then if r1 , . . . , rk are the roots of f , we have |r1 · · · rk | = 1/|a| ≤ 1,
where a is the leading coefficient of f . Therefore, there is some ri such that |ri | ≤ 1.
Consequently, we must have that p(ri ) − k = 0. Let p(x) − k = an xn + an−1 xn−1 + · · · + a1 x − q, then
n
n n
X X X
k
q= ak ri ≤ |ak ||ri |k ≤ |ak |.
k=1 k=1 k=1
Pn
Therefore, if we pick k large enough so that q > k=1 |ak |, then we have a contradiction, which means
that p(x) − k is irreducible.
Source: MOP 2007
.....................................................................................................
1
C1. Let X be a finite set of positive integers and A a subset of X. Prove that there exists a subset B of X
such that A equals the set of elements of X which divide an odd number of elements of B.
First solution: We construct B in stages. Set B = ∅ and consider every number in X, starting
with the largest and going down. For each element x ∈ X, see whether it divides the correct parity of
elements in B. (That is, if x ∈ A, x divides an odd number of elements in B; if x ∈ X − A, x divides an
even number of elements in B.) If it does not, add it to B. Thus the first element added to B is the
largest element of A. Now, this procedure will not change the divisibility condition for any element
greater than x, and will fulfill the condition for x. Thus when all elements of X have been examined,
the divisibility conditions will be satisfied by all elements of X, and B will be as desired.
Second solution: Given B a subset of X, it is clear that there is a unique A such that A equals the
set of elements of X which divide an odd number of elements of B. Now, given two distinct subsets,
B1 , B2 ⊆ X, let us show that the corresponding subsets A1 , A2 ⊆ X are distinct too. If B1 and B2
are not disjoint, then we can simply replace B1 by B1 − B1 ∩ B2 and replace B2 by B2 − B1 ∩ B2 ,
as this would not change the distinctness of the two resulting A1 , A2 . So we assume that B1 and B2
are disjoint. Let n be the largest element in B1 ∪ B2 . Say that n ∈ B1 . Then n ∈ A1 but n ∈/ A2 . It
follows that B1 6= B2 implies A1 6= A2 .
It follows that the map sending B to A is a bijection from the subsets of X to itself. Thus for every
A, we can find a corresponding B.
Source: 102 Combinatorics Problems
C2. Let B be a set of more than 2n+1 /n distinct points with coordinates of the form (±1, ±1, . . . , ±1) in
n-dimensional space with n ≥ 3. Show that there are three distinct points in B which are the vertices
of an equilateral triangle.
Solution: Let S be the set of points with all coordinates equal to ±1. For each P ∈ B, let SP be the
set of points in S which differ from P in exactly one coordinate. Since there are more than 2n+1 /n
points in B, and each SP has n elements, the cardinalities of the sets SP sum to more than 2n+1 ,
which is to say, more than twice the number of points in S. By the Pigeonhole Principle, there must
be a point of S in at least three of the sets, say in SP , SQ , SR . But then any
√ two of P, Q, R differ
in exactly two coordinates, so P QR is an equilateral triangle of side length 2 2, by the Pythagorean
Theorem.
Source: Putnam 2000
C3. Let S be a set of n points on a plane, no three collinear. A subset of these points is called polite if they
are the vertices of a convex polygon with no points of S in the interior. Let ck denote the number of
polite sets with k points. Show that the sum
n
X
(−1)i ci
i=3
This clearly does not depend on the configuration of the points, and so we may call it f (n), where
n = |S|. (It is easy to see that f (n) = − 21 (n − 1)(n − 2), but we will not need this.)
Alternatively, let us calculate (†) by first grouping subsets of S into collection of subsets that share the
same convex hull. If the convex hull has t vertices on the boundary, and k points of S inside, then its
contribution to the sum in (†) will be
k
t
X k i
(−1) (−1) ,
i=0
i
2
which equals to zero if k ≥ 1 and (−1)t otherwise (consider the expansion of (1 − 1)k ). In other words,
if T is a subset of S whose vertices form a convex polygon, then the collection that T belongs to
contributes (−1)t to the sum if T is polite and 0 otherwise. Therefore,
n
X X
(−1)i ci = (−1)|T | = f (n).
i=3 T ⊂S,|T |≥3
G1. Let ABC be an acute triangle. The points M and N are taken on the sides AB and AC, respectively.
The circles with diameters BN and CM intersect at points P and Q respectively. Prove that P, Q and
the orthocenter H are collinear.
Solution: We need to show that H lies on the radical axis of the two circles, and thus it suffices to
show that it is equal powers with respect to the two circles. Let Y and Z be the feet of the altitudes
from B and C, respectively. Since ∠BY N = 90◦ , Y lies on the circle with diameter BN . Since BY
passes through H, it follows that the power of H with respect to this circle is HB · HY . Similarly, the
power of H with respect to the other circle is HC · HZ. On the other hand, HB · HY = HC · HZ
since B, C, Y, Z are concyclic. Thus, H has equal powers with respect to the two circles.
Source: Leningrad 1988
G2. Let ABC be a triangle with AC 6= AB, and select point B1 on ray AC such that AB = AB1 . Let ω be
the circle passing through C, B1 , and the foot of the internal bisector of angle CAB. Let ω intersect
the circumcircle of triangle ABC again at Q. Prove that AC is parallel to the tangent to ω at Q.
Solution: Let the angle bisector of ∠BAC meet BC at E and ω again at D. We have ∠ADB1 =
∠ADB = ∠ACB (this is true in both configurations) and it follows that C, B1 , E, D are concyclic.
Thus Q = D. Let ` be the line tangent to ω at Q. Then we have ∠(`, DA) = ∠ECD = ∠BCD =
∠BAD = ∠CAD. It follows that ` is parallel to AC.
Source: Russia 2001
G3. Let OAB and OCD be two directly similar triangles (i.e., CD can be obtained from AB by some
rotation and dilatation both centered at O). Suppose their incircles meet at E and F . Prove that
∠AOE = ∠DOF .
Solution: Let Ω1 be the incircle of OAB and Ω2 the incircle of OCD. Suppose p that Ω1 touches OA
at X, and Ω2 touches OC at Y . Consider an inversion about O with radius |OX| · |OY |. Suppose
that Ω1 gets sent to Ω01 and Ω2 gets sent to Ω02 . Note that the choice of the radius of inversion implies
that the radii of Ω1 and Ω02 are equal, and that the radii of Ω01 and Ω2 are equal. It follows that Ω01 is
the reflection of Ω2 about the angle bisector of ∠AOD, and likewise with Ω1 and Ω02 .
Let E 0 denote the image of E, so that E 0 is an intersection point of Ω01 and Ω02 . Then, E 0 is the
reflection of F across the angle bisector of ∠AOD. But O, E, E 0 are collinear. It follows that ∠AOE =
∠AOE 0 = ∠DOF .
Source: Tournament of Towns 2004 Fall
.....................................................................................................
3
N1. Let n > 1 be an odd integer. Prove that n does not divide 3n + 1.
Solution: Assume to the contrary that there is a positive odd integer n that divides 3n + 1. Let p be
the smallest prime divisor of n. Then p divides 3n + 1; that is, 3n ≡ −1 (mod p), so 32n ≡ 1 (mod p).
By Fermat’s little theorem, we also have 3p−1 ≡ 1 (mod p). It follows that
Since p is the smallest prime divisor of n, gcd(n, p − 1) = 1. Because n is odd, p − 1 is even. Hence
gcd(2n, p − 1) = 2. It follows that 32 ≡ 1 (mod p), or p divides 8, which is impossible as p is odd.
N2. Let S be a finite set of integers, each greater than 1. Suppose that for each integer n there is some
s ∈ S such that gcd(s, n) = 1 or gcd(s, n) = s. Show that there exist s, t ∈ S such that gcd(s, t) is
prime.
Solution: Let n be the smallest positive integer such that gcd(s, n) > 1 for all s in n; note that n has
no repeated prime factors. By the condition on S, there exists s ∈ S which divides n.
On the other hand, if p is a prime divisor of s, then by the minimality of n, n/p is relatively prime to
some element t of S. Since n cannot be relatively prime to t, t is divisible by p, but not by any other
prime divisor of s (any such prime divides n/p). Thus gcd(s, t) = p, as desired.
Source: Putnam 1999
N3. Let a positive integer k be given. Prove that there are infinitely many pairs of integers (a, b) with
|a| > 1 and |b| > 1 such that ab + a + b divides a2 + b2 + k.
a2 + b2 + k
= k + 1. (†)
ab + a + b
Let us rearrange this as a quadratic in a:
As a quadratic in a, the sum of the roots is (k + 1)(b + 1). Hence, if (a, b) is a solution to (†), then so
is ((k + 1)(b + 1) − a, b), and hence by symmetry of (†), so is (b, (k + 1)(b + 1) − a).
Define a sequence (an ) as follows: a1 = 0, a2 = 1, and
an = (k + 1)(an−1 + 1) − an−2
for all n ≥ 2. Then by the above reasoning, (a, b) = (an , an+1 ) is a solution of (†) for all n ≥ 0.
Furthermore, the sequence (an ) is increasing, giving an infinite number of positive integer solutions.
4
IMO Training Camp Buffet Contest
June 30, 2008
1. Let A be a subset of {1, 2, . . . , 2008}, such that for all x, y ∈ A with x 6= y, the sum x + y is not divisible
by 1004. Find, with proof, the maximum possible size of A.
Solution: We can group the 2008 numbers in 1003 pairs of the form {k, 2008−k}, with k ∈ {1, 2, . . . , 1003},
and the pair {1004, 2008}. Observe that A cannot contain two elements from the same pair, as they would
add up to a number divisible by 1004. Thus |A| ≤ 1004.
To see that |A| = 1004 is possible, construct A by taking all the elements of {1, 2, . . . , 2008} whose remainder
upon division by 1004 is less than or equal to 502, and removing the elements 1506 and 2008 from the
resulting set. This yields a 1004-element set A satisfying the desired properties (why?).
2. Find, with proof, all real number solutions to the following:
Solution: We have
which is positive unless a = 1 and b = 1, which forms a solution. Therefore, the only solution is a = b = 1.
and multiplying together yields [(a2 + 1)(b2 + 1)]2 ≥ [(ab + 1)(a + b)]2 , so we can find all the solutions to
the equation by consider the equality cases in the above inequalities.
3. Find all ordered pairs (x, y) of positive integers such that 2x = 3y + 7.
Solution: In mod 3, we have (−1)x ≡ 1 (mod 3), so that x is even. Let x = 2a. Now, in mod 4, we have
0 ≡ (−1)y − 1 (mod 4), so that y is even. Let y = 2b. Then 22a − 32b = 7, so that
(2a + 3b )(2a − 3b ) = 7.
Solution: The key observation is that for any n ≥ 6 and any side of P6 , some subsegment of this
side is a side of Pn (this can be easily proven using induction). So, for any Pn , we can select points
P1 , P2 , . . . , P6 on its perimeter so that Pi lies on the i-th side of P6 . Since Pn is convex, it contains the
1
1
hexagon P1 P2 P3 P4 P5 P6 . Therefore, it suffices to prove that the area of P1 P2 . . . P6 is at least 2 whenever
Pi lies on the i-th side of P6 for each i.
Consider this problem as a minimization problem, where we want to minimize the area of P1 P2 P3 P4 P5 P6
subject to the above condition. Observe that as Pi moves along the i-th side of P6 , the area of P1 P2 P3 P4 P5 P6
changes monotonically (in fact, linearly) as Pi moves form one end to the other. Therefore, the minimum
must occur when Pi coincides with a vertex of P6 . Therefore, we simply needs to search through the set
of (possibly degenerate) hexagons P1 P2 P3 P4 P5 P6 with the property that each Pi is one of the endpoints
of the i-th side of P6 . We wish to find the one with the minimum area. After some work, we see that the
minimum occurs when P1 P2 P3 P4 P5 P6 is an equilateral triangle, and its area is 21 .
Therefore, the area of Pn is at least 12 .
Remark: More elegantly, the bound can be proven using the inequality
x1 (1 − x2 ) + x2 (1 − x3 ) + x3 (1 − x4 ) + x4 (1 − x5 ) + x5 (1 − x6 ) + x6 (1 − x1 )
≤ (1 − x2 ) + x2 + (1 − x4 ) + x4 + (1 − x6 ) + x6
≤ 3.
I’ll leave the details to you.
Also, the bound 21 is not optimal. Can you find a better bound? What’s the best bound? (I don’t know
the answer to the last question.)
π
5. Let n be a positive integer. Suppose that θ1 , θ2 , . . . , θn are angles with 0 < θi < 2 for each i such that
2 2 2
cos θ1 + cos θ2 + · · · + cos θn = 1.
Prove that
tan θ1 + tan θ2 + · · · + tan θn ≥ (n − 1)(cot θ1 + cot θ2 + · · · + cot θn ).
1 1 1 1 1 1
≤ ai + + · · · + + · · ·
(n − 1)3/2 a1 a2 ai−1 ai+1 an
again by the Power-Mean inequality. Summing the above inequalities for i = 1, 2, . . . , n yields
n n X
X 1 X aj √ X aj
cot θi ≤ = n − 1 .
i=1
(n − 1)3/2 i=1 j6=i ai i6=j
ai
Therefore,
n n
X √ X aj X
tan θi ≥ n−1 ≥ (n − 1) cot θi .
i=1
ai i=1
i6=j
2
6. Let AA1 , BB1 , CC1 be the altitudes of an acute triangle ABC. Let O be an arbitrary point inside A1 B1 C1 .
Denote the feet of the perpendiculars from O to the lines AA1 and BC by M and N , respectively; the ones
from O to the lines BB1 and CA by P and Q, respectively; the ones from O to the lines CC1 and AB by
R and S, respectively. Prove that the lines M N , P Q, and RS are concurrent.
Solution: Consider a dilation about O with ratio 2. Let M 0 , P 0 , R0 be the image of M, P, R respectively.
The dilation sends line M N to line A1 M 0 , line P Q to line B1 P 0 , line RS to line C1 R0 . So it suffices to
prove that A1 M 0 , B1 P 0 , C1 R0 are concurrent.
Observe that in triangle A1 B1 C1 , A1 A is an angle bisector (why?), and ∠M 0 A1 A = ∠M N O = ∠OA1 A.
That is, line A1 M 0 is the reflection of A1 O across the angle bisector of ∠B1 A1 C1 . Therefore, A1 M 0 , B1 P 0 , C1 R0
concur at the isogonal conjugate of O with respect to triangle A1 B1 C1 .
7. Positive integers a and b are given such that 2a + 1 and 2b + 1 are relatively prime. Find all possible values
of the greatest common divisor of 22a+1 + 2a+1 + 1 and 22b+1 + 2b+1 + 1.
Proof. We use induction on the quantity k + n. If k + n = 1, the claim is obvious. Now, using the fact
that gcd(a, b) = gcd(a − b, b), we have (for k ≥ n)
Applying it to x = 2a gives us
It follows that the greatest common divisor d of 22a+1 + 2a+1 + 1 and 22b+1 + 2b+1 + 1 divides the greatest
common divisor of 24a+2 + 1 and 24b+2 + 1, and hence it divides the greatest divisor of 28a+4 − 1 and
28b+4 − 1, which, by the lemma, equals to 2(8a+4,8b+4) − 1 = 24 − 1 = 15, as we are given that 2a + 1 and
2b + 1 are relatively prime. Therefore, d divides 15.
Since 22a+1 + 2a+1 + 1 ≡ 2a+1 6≡ 0 (mod 3), 3 does not divide d. Hence d = 1 or d = 5. Both cases are
possible. For a = 4, b = 8 we obtain d = 5, and for a = 2, b = 3 we obtain d = 1.
Second solution: Suppose a prime p divides 22a+1 + 2a+1 + 1 and 22b+1 + 2b+1 + 1. Then 22a+1 ≡ 2a+1 + 1
(mod p). Squaring gives
Similarly, 24b+2 ≡ −1 (mod p). Since 2a + 1 and 2b + 1 are relatively prime, there are integers j, k such
that j(2a + 1) + k(2b + 1) = 1 (where j and k necessarily have opposite parity). Hence, we get
Hence, p = 5.
On the other hand, suppose 25 divides 2(22x+1 + 2x+1 + 1) = (2x+1 + 1)2 + 1. Since the only solutions
to y 2 + 1 ≡ 0 (mod 25) are y ≡ ±7 (mod 25), we must have 2x+1 + 1 ≡ ±7 (mod 25). But it is easily
checked that 2 is a primitive root mod 25, as its order is not φ(25)/2 = 10 or φ(25)/5 = 4. It follows that
there is there a unique solution in mod 20 for x for each of 2x+1 + 1 ≡ 7 (mod 25) and 2x+1 + 1 ≡ −7
3
(mod 25), namely x ≡ 7 (mod 20) for the former and x ≡ 12 (mod 20) for the latter. In either case,
we have 5 | 2x + 1. This means that if 25 divides the greatest common divisor of 22a+1 + 2a+1 + 1 and
22b+1 + 2b+1 + 1, then 5 must divide both 2a + 1 and 2b + 1, contradicting the hypothesis that they are
relatively prime.
It follows that the only possible values for the greatest common divisor are 1 and 5. They can be achieved
by (a, b) = (4, 8) and (2, 3) respectively.
Source: St. Petersburg 2002
8. Let X be a finite set, and suppose A1 , . . . , Am and B1 , . . . , Bm are subsets of X with |Ai | = r and |B
i| = s
for each i, such that Ai ∩ Bi = ∅ for every i and Ai ∩ Bj 6= ∅ whenever i 6= j. Prove that m ≤ r+s r .
Solution: (by Béla Bollobàs in 1965) Let |X| = n. There are n! ways to label the elements of X with
{1, 2, . . . , n}. Let us pick a random labeling (i.e. permutation), so that every labeling is equally likely to
be chosen. For 1 ≤ i ≤ m, let Ei denote the event that the highest label in Ai is less than the lowest label
in Bi (which we denote by Ai < Bi ).
−1
First, observe that Pr(Ei ) = r+s r (why?). Now, we claim that if i 6= j, then Ei ∩ Ej = ∅. That is, no
labeling of X will result in both Ai < Bi and Aj < Bj . Indeed, if this were the case, and, say, the highest
label of Ai is at least as great as the highest label of Aj (we may switch i and j needed), then we must
have Aj < Bi so that Ai ∩ Bj = ∅, contradiction.
Thus, E1 , E2 , . . . , Em are mutually disjoint events. It follows that
−1
r+s
1 ≥ Pr(E1 ∪ E2 ∪ · · · ∪ Em ) = Pr(E1 ) + Pr(E2 ) + · · · + Pr(Em ) = m .
r
4
Winter Camp 2008 Mock Olympiad
Monday, January 7, 2008
1 + 2 x 3y = z 2 .
3. In acute triangle ABC, ∠A < 45◦ . Point D lies in the interior of triangle ABC such that
BD = CD and ∠BDC = 4∠A. Point E is the reflection of C across line AB, and point F is
the reflection of B across line AC. Prove that AD ⊥ EF .
Solutions
1 + 2 x 3y = z 2 .
x3 + y 3 + z 3 ≥ x2 y + y 2 z + z 2 x.
1
Thus
a b c x3 + y 3 + z 3 x2 y + y 2 z + z 2 x
+ + = ≥ = a + b + c,
b c a xyz xyz
as desired.
3. In acute triangle ABC, ∠A < 45◦ . Point D lies in the interior of triangle ABC such that
BD = CD and ∠BDC = 4∠A. Point E is the reflection of C across line AB, and point F is
the reflection of B across line AC. Prove that AD ⊥ EF .
First solution: All angles are directed. Let RA be the rotation by angle 2∠CAB centered
at A and let RD be the rotation by angle ∠BDC centered at D. Then T = RA ◦ RD ◦ RA
is a translation because the sum of angles of RA , RD , RA is zero.
Second solution: We use complex numbers. Use lower case letters to denote the complex
number corresponding to a point. Place D at the center, and let B and C be on the unit circle
so that B and C are symmetric about the real axis. Let α denote the unit complex number
with argument ∠CAB. Then, b = α2 , c = α−2 . Since AE = AC and ∠CAE = 2∠CAB, we
see that e − a = α2 (c − a), which gives us
Similarly, f = a(1 − α−2 ) + 1. So f − e = a(α2 − α−2 ). Note that α2 − α−2 is purely imaginary
as α lies on the unit circle. It follows that EF is perpendicular to AD.
Source: MOP 2007
2
4. Let m, n be two positive integers with m ≥ n. Prove that
n
k m−k n
X
(−1) = 1.
n k
k=0
Second solution: (Combinatorial) For i = 1, 2, . . . , n, let Si denote the set of all n-element
subsets of {1, 2, . . . , m} that does not contain the element i. That is
Si = {A ⊂ {1, 2, . . . , m} : |A| = n, i ∈
/ A} .
Since S1 ∪ S2 ∪ · · · ∪ Sn contains all n-element subset of {1, 2, . . . , m} except for the subset
{1, 2, . . . , n}, we have
m
|S1 ∪ S2 ∪ · · · ∪ Sn | = − 1.
n
Also, the intersection Sa1 ∩ Sa2 ∩ · · · ∩ Sai , where 1 ≤ a1 < a2 < · · · < ai ≤ n, contains all
n-element subsets of {1, 2, . . . , m}\{a1 , a2 , . . . ai }. So
m−i
Sa1 ∩ Sa2 ∩ · · · ∩ Sai = .
n
By the Inclusion-Exclusion Principle,
X X X
|S1 ∪S2 ∪· · ·∪Sn | = |Si |− |Si ∩Sj |+ |Si ∩Sj ∩Sk |−· · ·−(−1)n |S1 ∩S2 ∩· · ·∩Sn |,
1≤i≤n 1≤i<j≤n 1≤i<j<k≤n
which is equivalent to
m m−1 n m−2 n m−3 n n m−n n
−1= − + − · · · − (−1) ,
n n 1 n 2 n 3 n n
and rearranging gives
n
X
k m−k n
(−1) = 1,
n k
k=0
3
as desired.
Third solution: (Generating function) We see that given sum equals to the coefficient of
xm in the product F (x)G(x), where
∞ ∞
k n k
X X
F (x) = (−1) xk , and G(x) = xk .
k n
k=0 k=0
and
∞ ∞ ∞
X k k X n+k k X −n − 1 k
G(x) = x = xn x = xn (−1)k x = xn (1 − x)−n−1 .
n n k
k=n k=0 k=0
And thus
F (x)G(x) = xn (1 − x)−1 = xn + xn+1 + xn+2 + · · · .
So the coefficient of xm is 1, and thus the sum equals to 1.
Source: MathLinks Contest
4
IMO Training Camp Mock Olympiad #2
July 3, 2008
1. Given an isosceles triangle ABC with AB = AC. The midpoint of side BC is denoted by M .
Let X be a variable point on the shorter arc M A of the circumcircle of triangle ABM . Let
T be the point in the angle domain BM A, for which ∠T M X = 90◦ and T X = BX. Prove
that ∠M T B − ∠CT M does not depend on X.
f (x + f (y)) = f (x + y) + f (y)
for all x, y ∈ R+ . (Symbol R+ denotes the set of all positive real numbers.)
3. For a prime p and a positive integer n, denote by νp (n) the exponent of p in the prime
factorization of n. Given a positive integer d and a finite set {p1 , . . . , pk } of primes. Show
that there are infinitely many positive integers n such that d | νpi (n!) for all 1 ≤ i ≤ k.
IMO Training Camp Mock Olympiad #3
July 5, 2008
1. A unit square is dissected into n > 1 rectangles such that their sides are parallel to the sides
of the square. Any line, parallel to a side of the square and intersecting its interior, also
intersects the interior of some rectangle. Prove that in this dissection, there exists a rectangle
having no point on the boundary of the square.
2. The diagonals of a trapezoid ABCD intersect at point P . Point Q lies between the parallel
lines BC and AD such that ∠AQD = ∠CQB, and line CD separates points P and Q. Prove
that ∠BQP = ∠DAQ.
3. Let n be a fixed positive integer. Find the maximum value of the expression
July 6, 2008
1. Let b, n > 1 be integers. Suppose that for each integer k > 1 there exists an integer ak such
that b − ank is divisible by k. Prove that b = An for some integer A.
f (m + n) ≥ f (m) + f (f (n)) − 1
3. Point P lies on side AB of a convex quadrilateral ABCD. Let ω be the incircle of triangle
CP D, and let I be its incenter. Suppose that ω is tangent to the incircles of triangles AP D
and BP C at points K and L, respectively. Let lines AC and BD meet at E, and let lines
AK and BL meet at F . Prove that points E, I and F are collinear.
IMO Training Camp Mock Olympiad #5
July 8, 2008
1. Given non-obtuse triangle ABC, let D be the foot of the altitude from A to BC, and let I1 , I2
be the incenters of triangles ABD and ACD, respectively. The line I1 I2 intersects AB and
AC at P and Q, respectively. Show that AP = AQ if and only if AB = AC or ∠A = 90◦ .
2. Let λ be the positive root of the equation t2 − 2008t − 1 = 0. Define the sequence x0 , x1 , . . .
by setting
x0 = 1 and xn+1 = bλxn c for n ≥ 0.
Find the remainder when x2008 is divided by 2008.
3. Let A0 = (a1 , . . . , an ) be a finite sequence of real numbers. For each k ≥ 0, from the sequence
Ak = (x1 , . . . , xn ) we construct a new sequence Ak+1 in the following way.
We choose a partition {1, . . . , n} = I ∪ J, where I and J are two disjoint sets, such that the
expression
X X
xi − xj
i∈I j∈J
attains the smallest possible value. (We allow the sets I or J to be empty; in this case the
corresponding sum is 0.) If there are several such partitions, one is chosen arbitrarily. Then
we set Ak+1 = (y1 , . . . , yn ), where yi = xi + 1 if i ∈ I, and yi = xi − 1 if i ∈ J.
Prove that for some k, the sequence Ak contains an element x such that |x| ≥ n2 .
IMO Training Camp Mock Olympiad #6
1. Let ABC be a triangle, and let M be the midpoint of side BC. Triangles ABM and ACM
are inscribed in circles ω1 and ω2 , respectively. Points P and Q are midpoints of arcs AB
and AC (not containing M , on ω1 and ω2 respectively). Prove that P Q ⊥ AM .
then
n X
X n
cjk = (a1 + · · · + an ) + (b1 + · · · + bn ).
j=1 k=1
X + a = {x + a|x ∈ X}.
1. Let T be a finite set of real numbers satisfying the property: For any two elements t1 and
t2 in T , there is a element t in T such that t1 , t2 , t (not necessarily in that order) are three
consecutive terms of an arithmetic sequence. Determine the maximum number of elements
T can have.
2. Let ABM be an isosceles triangle with AM = BM . Let O and ω denote the circumcenter
and circle of triangle ABM , respectively. Point S and T lie on ω, and tangent lines to ω at S
and T meet at C. Chord AB meet segments M S and M T at E and F , respectively. Point X
lies on segment OS such that EX ⊥ AB. Point Y lies on segment OT such that F Y ⊥ AB.
Line ` passes through C and intersects ω at P and Q. Chords M P and AB meet R. Let Z
denote the circumcenter of triangle P QR. Prove that X, Y, Z are collinear.
3. Let a1 , a2 , . . . be a sequence of positive integers satisfying the condition 0 < an+1 − an ≤ 2008
for all integers n ≥ 1 Prove that there exist an infinite number of ordered pairs (p, q) of
distinct positive integers such that ap is a divisor of aq .
IMO Training Camp Mock Olympiad #2 Solutions
July 3, 2008
1. Given an isosceles triangle ABC with AB = AC. The midpoint of side BC is denoted by M .
Let X be a variable point on the shorter arc M A of the circumcircle of triangle ABM . Let
T be the point in the angle domain BM A, for which ∠T M X = 90◦ and T X = BX. Prove
that ∠M T B − ∠CT M does not depend on X.
f (x + f (y)) = f (x + y) + f (y)
for all x, y ∈ R+ . (Symbol R+ denotes the set of all positive real numbers.)
3. For a prime p and a positive integer n, denote by νp (n) the exponent of p in the prime
factorization of n. Given a positive integer d and a finite set {p1 , . . . , pk } of primes. Show
that there are infinitely many positive integers n such that d | νpi (n!) for all 1 ≤ i ≤ k.
1
IMO Training Camp Mock Olympiad #3
July 5, 2008
1. A unit square is dissected into n > 1 rectangles such that their sides are parallel to the sides
of the square. Any line, parallel to a side of the square and intersecting its interior, also
intersects the interior of some rectangle. Prove that in this dissection, there exists a rectangle
having no point on the boundary of the square.
2. The diagonals of a trapezoid ABCD intersect at point P . Point Q lies between the parallel
lines BC and AD such that ∠AQD = ∠CQB, and line CD separates points P and Q. Prove
that ∠BQP = ∠DAQ.
3. Let n be a fixed positive integer. Find the maximum value of the expression
a+b 2
≤ 14 , and
Solution: First consider the case when n ≥ 2. By AM-GM, we have ab ≤ 2
similarly ab, bc, ca ≤ 14 . So
Thus, we have to prove that (ab)n + (bc)n + (ca)n ≤ 41n . Without loss of generality, suppose
that a is the maximum among a, b, c. We have a(1 − a) ≤ 14 , so
1
≥ an (1 − a)n = an (b + c)n ≥ an bn + an cn + nan bn−1 c ≥ an bn + bn cn + cn an .
4n
So we have proved that in this case the maximum is at most 3·41n−1 . Furthermore, this bound
can be attained when a = b = 12 , c = 0. Therefore, the maximum value of the expression is
1
3·4n−1
when n ≥ 2.
Now, suppose that n = 1. The value of 83 can be attained by a = b = c = 13 . So it remains to
prove that
ab bc ca 3
+ + ≥ .
1 − ab 1 − bc 1 − ca 8
Since a + b + c = 1, it is equivalent to prove that
ab bc ca 3
2
+ 2
+ 2
≥ .
(a + b + c) − ab (a + b + c) − bc (a + b + c) − ca 8
2
Now, clearing the denominators and simplifying (details omitted here . . . ), we see that the
above inequality is equivalent to
Now, (†) is true as it is the sum of the following inequalities, each of which is true due to
Muirhead’s Inequality:
3
IMO Training Camp Mock Olympiad #4
July 6, 2008
1. Let b, n > 1 be integers. Suppose that for each integer k > 1 there exists an integer ak such
that b − ank is divisible by k. Prove that b = An for some integer A.
f (m + n) ≥ f (m) + f (f (n)) − 1
3. Point P lies on side AB of a convex quadrilateral ABCD. Let ω be the incircle of triangle
CP D, and let I be its incenter. Suppose that ω is tangent to the incircles of triangles AP D
and BP C at points K and L, respectively. Let lines AC and BD meet at E, and let lines
AK and BL meet at F . Prove that points E, I and F are collinear.
4
IMO Training Camp Mock Olympiad #5
July 8, 2008
1. Given non-obtuse triangle ABC, let D be the foot of the altitude from A to BC, and let I1 , I2
be the incenters of triangles ABD and ACD, respectively. The line I1 I2 intersects AB and
AC at P and Q, respectively. Show that AP = AQ if and only if AB = AC or ∠A = 90◦ .
P I2
I1
B C
D
Conversely, suppose that AP = AQ. Let D0 be the point on AD such that AD0 = AP = AQ.
If D 6= D0 , then triangles AP I1 and AD0 I1 are congruent, and triangles AQI2 and AD0 I2
are congruent, so ∠I1 D0 A = ∠I2 D0 A. Then ∠I1 D0 D = ∠I2 D0 D, and since we also know
that ∠I1 DD0 = ∠I2 DD0 (they are either both 45◦ or both 135◦ , depending on whether D0
is on the segment AD or the ray AD past D), we have that triangles I1 D0 D and I2 D0 D
are congruent. Thus triangles AD0 I1 and AD0 I2 are congruent, so in particular ∠BAD =
2∠I1 AD0 = 2∠I2 AD0 = ∠CAD, hence triangle ABC is isosceles with AB = AC.
If D = D0 , then ∠AP I1 = ∠ADI1 = ∠ADI2 = ∠AQI2 = 45◦ , so ∠A = 90◦ .
Comment: Various trigonometric solutions are available, and they are generally pretty easy.
6
2. Let λ be the positive root of the equation t2 − 2008t − 1 = 0. Define the sequence x0 , x1 , . . .
by setting
x0 = 1 and xn+1 = bλxn c for n ≥ 0.
Find the remainder when x2008 is divided by 2008.
5
Because xn = bxn−1 λc and xn−1 is an integer and λ is irrational, we have xn = xn−1 λ − ,
where 0 < < 1 is the fractional part of xn−1 λ. Since λ > 1, we have 0 ≤ λ < 1, and so
jx k j k
n
= xn−1 − = xn−1 − 1.
λ λ
It follows that xn+1 ≡ xn−1 −1 (mod 2008). Therefore, by induction x2008 ≡ x0 −1004 ≡ 1005
(mod 2008).
3. Let A0 = (a1 , . . . , an ) be a finite sequence of real numbers. For each k ≥ 0, from the sequence
Ak = (x1 , . . . , xn ) we construct a new sequence Ak+1 in the following way.
We choose a partition {1, . . . , n} = I ∪ J, where I and J are two disjoint sets, such that the
expression
X X
xi − xj
i∈I j∈J
attains the smallest possible value. (We allow the sets I or J to be empty; in this case the
corresponding sum is 0.) If there are several such partitions, one is chosen arbitrarily. Then
we set Ak+1 = (y1 , . . . , yn ), where yi = xi + 1 if i ∈ I, and yi = xi − 1 if i ∈ J.
Prove that for some k, the sequence Ak contains an element x such that |x| ≥ n2 .
6
IMO Training Camp Mock Olympiad #6
July 11, 2008
1. Let ABC be a triangle, and let M be the midpoint of side BC. Triangles ABM and ACM
are inscribed in circles ω1 and ω2 , respectively. Points P and Q are midpoints of arcs AB
and AC (not containing M , on ω1 and ω2 respectively). Prove that P Q ⊥ AM .
Second solution: Consider a rotation about P that brings B to A, and let the image of M be
T . Since ∠P BM + ∠P AM = 180◦ , we see that M, A, T are collinear and M B = AT = M C.
Similarly, consider a rotation about Q that brings C to A, this must also bring M to T as
the triangles QCM and QAT have equal side lengths.
Now, notice that in quadrilateral M P T Q we have P M = P T and QM = QT , so it is a kite,
and thus P Q ⊥ M T . The result follows from the fact that A lies on line M T .
then
n X
X n
cjk = (a1 + · · · + an ) + (b1 + · · · + bn ).
j=1 k=1
Pn
Solution: Let ri = k=1 cjk . Then
n n
X X cjk n X X cjk n n
X rj
= = = 1, (1)
ai + bj ai + bj ai + bj
j=1 j=1 k=1 k=1 j=1
Then R(x) = P (x)/Q(x) where Q(x) = (x + b1 )(x + b2 ) · · · (x + bn ) and P (x) has degree at
most n − 1. By (1), R(a1 ) = R(a2 ) = · · · = R(an ) = 1, so if we write
S(x)
R(x) = 1 − ,
Q(x)
7
then S(x) is a monic polynomials of degree n and S(a1 ) = S(a2 ) = · · · = S(an ) = 0. Hence
S(x) = (x − a1 )(x − a2 ) · · · (x − an ).
Consider the coefficient of xn−1 in P (x) = Q(x)−S(x). Form (2), this coefficient is r1 +· · ·+rn .
On the other hand,
From this we see that coefficient xn−1 in Q(x) − S(x) equals to (a1 + a2 + · · · + an ) + (b1 +
b2 + · · · + bn ). Hence we have our desired result.
X + a = {x + a|x ∈ X}.
Solution:
Assume wlog that 0 = a1 < a2 < · · · < an . Define a map f : Z → {a1 , a2 , . . . , an } thus: for
r ∈ Z, f (r) is the unique ai such that r = X + ai .
Suppose we are given f (r), f (r + 1), f (r + 2), . . . , f (r + an ) for some r ∈ Z. Then the value
of f (r + an + 1) is determined: if there exists ai > 0 such that f (r + an + 1 − ai ) = 0, then
f (r + an + 1) = ai ; otherwise, f (r + an + 1) = 0 = a1 . The value of f (r − 1) is also determined:
if there exists ai < an such that f (r − 1 + an − ai ) = an , then f (r − 1) = ai ; otherwise,
f (r − 1) = an .
By the Pigeonhole principle, there must exist distinct integers r and s such that
The preceding paragraph, along with a straightforward induction argument shows that that
f is periodic with period |r − s|, and therefore X = X + |r − s|.
8
IMO Training Camp Mock Olympiad #7
July 13, 2008
1. Let T be a finite set of real numbers satisfying the property: For any two elements t1 and
t2 in T , there is a element t in T such that t1 , t2 , t (not necessarily in that order) are three
consecutive terms of an arithmetic sequence. Determine the maximum number of elements
T can have.
2. Let ABM be an isosceles triangle with AM = BM . Let O and ω denote the circumcenter
and circle of triangle ABM , respectively. Point S and T lie on ω, and tangent lines to ω at S
and T meet at C. Chord AB meet segments M S and M T at E and F , respectively. Point X
lies on segment OS such that EX ⊥ AB. Point Y lies on segment OT such that F Y ⊥ AB.
Line ` passes through C and intersects ω at P and Q. Chords M P and AB meet R. Let Z
denote the circumcenter of triangle P QR. Prove that X, Y, Z are collinear.
Solution: Consider a homothety centred at S that sends segment OM to segment XE. Let
ω0 be the image of ω under the homothety. We see that ω0 is centred at X and passes through
E and S. Furthermore, because ω is tangent to CS, ω0 is tangent to CS as well.
9
M
A R E B
X
Z S
Q
P
CP · CQ = CS 2 .
It follows that both M and C have equal powers with respect to ω0 and the circumcircle
of P QR. That is, M C is the radical axis of these two circles. Since the radical axis is
perpendicular to the line joining the centres of the two circles, we have that ZX ⊥ M C.
Similarly, ZY ⊥ M C. Therefore, X, Y, Z are collinear.
Source: China TST 2007
3. Let a1 , a2 , . . . be a sequence of positive integers satisfying the condition 0 < an+1 − an ≤ 2008
for all integers n ≥ 1 Prove that there exist an infinite number of ordered pairs (p, q) of
distinct positive integers such that ap is a divisor of aq .
Solution: Consider all pairs (p, q) of distinct positive integers such that ap is a divisor of aq .
Assume, by way of contradiction, that there exists a positive N such that q < N for all such
pairs.
We prove by induction on k that for each k ≥ 1, there exist
10
Tk+1 , like Tk , consists of 2008 consecutive positive integers greater than or equal to aN —
in fact, greater than or equal to Q max Sk . Also, at least k elements of Tk+1 are divisible by
some element of Sk : namely, t + s∈Sk s for each of the elements t ∈ Tk which are divisible
by some element of Sk .
By the given condition 0 < an+1 − an ≤ 2008, and because the elements of Tk+1 are greater
than or equal to aN , we have that aq ∈ Tk+1 for some q ≥ N . Because the elements of Tk+1
are greater than max Sk , we have aq 6∈ Sk . Thus, by the definition of N , no element of Sk
divides aq .
Hence, at least k + 1 elements of Tk+1 are divisible by some element of Sk ∪ {aq }: at least k
elements of Tk+1 are divisible by some element of Sk , and in addition aq is divisible by itself.
Therefore, setting Sk+1 = Sk ∪ {aq } completes the inductive step.
Setting k = 2009, we have the absurd result that T2009 is a set of 2008 elements, at least 2009
of which are divisible by some element of S2009 . Therefore, our original assumption was false,
and for each N there exists q > N and p 6= q such that ap | aq . It follows that there are
infinitely many ordered pairs (p, q) with p 6= q and ap | aq .
Source: Vietnam TST 2001
11
Winter Camp 2009
Pre-camp problem set
3. The first five terms of a sequence are 1, 2, 3, 4, 5. From the sixth term on, each term is 1 less
than the product of all the preceding ones. Prove that the product of the first 70 terms is
equal to the sum of their squares.
5. Let ABC be an equilateral triangle of altitude 1. A circle, with radius 1, and center on the
same side of AB as C, rolls along the segment AB. Prove that the length of the arc of the
circle that is inside the triangle remains constant.
7. Let S be a finite set of points in the plane, and let r be a positive real number. Suppose it is
possible to place several circular discs of radius r on the plane such that: (a) every point in
S is covered by exactly one disc, and (b) the center of every disc is a point in S. Prove that,
no matter how this is done, the number of discs used will be constant.
a3 b3 c3 d3 1
+ + + ≥ .
b+c c+d d+a a+b 8
9. Fix a prime p > 2. An integer x is called a quadratic residue mod p if there exists an integer
y so that y 2 − x is a multiple of p.
(a) Determine the number of integers x in {0, 1, 2, . . . , p − 1}, for which both x and x − 1
are quadratic residues mod p.
p−1
(b) Prove that −1 is a quadratic residue mod p if and only if 4 is an integer.
10. Prove that there exists a unique function f defined on the positive reals such that f (f (x)) =
6x − f (x) and f (x) > 0 for all positive x.
1
11. Let O be the circumcenter of an acute-angled triangle ABC and let A1 be a point on the
smaller arc BC of the circumcircle of 4ABC. Let A2 and A3 be points on the sides AB and
AC respectively, such that ∠BA1 A2 = ∠OAC and ∠CA1 A3 = ∠OAB. Show that the line
A2 A3 passes through the orthocenter of 4ABC.
12. The numbers 1, 2, . . . , 22008 are stored in 2009 memory locations of a computer. Two pro-
grammers take turns choosing five memory locations, and then subtracting 1 from each of
these locations. If any location ever acquires a negative number, the computer breaks and
the guilty programmer pays for the repairs. Which programmer can ensure himself financial
security, and how?
(a) Let ω be the circumcircle of 4CF R, and let G and H be the second intersections of
lines BC and EF with ω. Prove that triangles RGH and P BE have parallel sides.
(b) Prove that P , Q, and R lie on a line.
14. A positive integer is written in each square of an 8 × 8 chessboard. One is allowed to choose
a 3 × 3 or a 4 × 4 square on the chessboard and increase all numbers in it by 1. Is it always
possible, applying such operations several times, to arrive at a situation where all squares
contain multiples of 10?
15. Let P and Q be points in the plane and let ω1 , ω2 , and ω3 be circles passing through both.
If A, B, C, D, E, and F are points on a line in that order so that A and D lie on ω1 , B and
E lie on ω2 , and C and F lie on ω3 , prove that AB · CD · EF = BC · DE · F A.
17. Find all positive integers m, n such that m|n2 + 1 and n|m2 + 1.
18. For every finite set of positive integers A, define the collection A2 to be
A2 = {a + b|a, b ∈ A, a 6= b}.
For example, if A = {1, 2, 3, 4}, then A2 = {3, 4, 5, 5, 6, 7}. (Note that we are allowed to
repeat elements in a collection.) Suppose there exist two finite sets A, B such that A2 = B2
but A 6= B. Prove that the number of elements in A and in B are the same and this number
is a power of 2.
2
Solutions
2k−1 √
√ p
1. Note that √ = k 2 + 1 − (k − 1)2 + 1. Therefore,
(k−1)2 +1+ k2 +1
n
X n p
X p p
2k − 1
p √ = k 2 + 1 − (k − 1)2 + 1 = n2 + 1 − 1,
k=1
(k − 1)2 + 1 + k 2 + 1 k=1
√
and the result follows from the fact that n < n2 + 1 < n + 1.
4. For any integers x, y, one can check y 2 ∈ {0, 1, 3, 4, 5, 9} mod 11 and x5 ∈ {−1, 0, 1} mod 11.
Therefore, the equation has no solutions mod 11, and hence no integer solutions.
5. Suppose the circle is centered at O. We assume without loss of generality that O is closer to
B than A. Since O and C are both distance 1 from AB, we know CO is parallel to AB, and
hence ∠OCB = 60◦ .
Now let P and Q be where the circle hits CA and CB respectively. Also let Q0 be the point
−−→
on CB such that ∠P OQ0 = 60◦ . Note that ∠P OQ0 = 60◦ = ∠P CQ0 , so P COQ0 is cyclic.
Therefore, ∠OP Q0 = ∠OCQ0 = ∠OCB = 60◦ , which implies that 4OP Q0 is equilateral and
−−→
OQ0 = OP = 1. However, we know Q is the unique point on CB such that OQ = 1, which
means Q0 = Q, and hence ∠P OQ = ∠P OQ0 = 60◦ .
Therefore, the arc contained within 4ABC is always one-sixth of the circumference, regardless
of where O is.
7. Let X and Y be two sets of discs, each fulfilling the given requirements. We define a function
f : X → Y as follows. Fix a disc C ∈ X. Then, its center must be covered by some unique
disc D in Y . Define f (C) = D. Suppose f (C) = f (C 0 ) = D for some discs C, C 0 . Then, the
center of D is within distance r of the centers of C and C 0 , meaning C and C 0 both cover
the center of D, which is impossible. Therefore, f is one-to-one and |Y | ≥ |X|. Similarly,
|X| ≥ |Y |, and the result follows.
µq q q q ¶
a3 b3 c3 d3
8. By the Cauchy-Schwarz inequality applied to the vectors b+c , c+d , d+a , a+b and
3
√ √ √ √
( b + c, c + d, d + a, a + b), we have
¡ 1.5 ¢2
a3 b3 c3 d3 a + b1.5 + c1.5 + d1.5
+ + + ≥
b+c c+d d+a a+b (b + c) + (c + d) + (d + a) + (a + b)
µ 1.5 ¶2
a + b1.5 + c1.5 + d1.5
= 8· ,
4
¡ a+b+c+d ¢3
which, by the power-mean inequality is at least 8 · 4 = 18 .
9. Let n denote the number of consecutive quadratic residues mod p, and let m denote the
number of solutions (x, y) to x2 − y 2 ≡ 1(mod p). Since every quadratic residue except for 0
has exactly two square roots mod p, every pair of consecutive quadratic residues corresponds
to 4 solutions to x2 −y 2 ≡ 1(mod p), unless one of the residues is 0, in which case it corresponds
to 2 solutions. Therefore, m = 4n − 4 if −1 is a quadratic residue and m = 4n − 2 otherwise.
We now calculate m. Note that x2 −y 2 ≡ 1(mod p) is equivalent to there existing z 6= 0 so that
−1
x + y ≡ z(mod p) and x − y ≡ z −1 (mod p), which in turn is equivalent to x ≡ z+z2 (mod p)
−1
and y ≡ z−z2 (mod p). Thus, there is one unique solution (x, y) for each z 6= 0, and m = p−1.
Therefore, n = p+3 p+1
4 or 4 depending on whether −1 is a quadratic residue. Since § n¨ is an
integer, −1 is a quadratic residue if and only if p ≡ 1(mod 4), and either way, n = p4 .
10. For any x > 0, define the sequence {xn } by x0 = x and xn = f (xn−1 ) for n > 0. Then,
f (f (xn−2 )) = 6xn−2 − f (xn−2 ) =⇒ xn = 6xn−2 − xn−1 for all n. It follows1 that there exist
constants A, B so that xn = A · 2n + B · (−3)n . Since xn > 0 for all n, we must have B = 0,
and since x0 = x, we then have A = x. Therefore, f (x) = x1 = 2x. Conversely, it is easy to
check f (x) = 2x does satisfy the given condition.
11. Let B 0 and C 0 be where the altitudes from B and C hit the circumcircle of 4ABC. Then,
∠BA1 A2 = ∠OAC = 90◦ − ∠B = ∠BCC 0 = ∠BA1 C 0 , so A2 is the intersection of AB and
A1 C 0 . Similarly, A3 is the intersection of AC and A1 B 0 . Now, Pascal’s theorem (see #13),
applied to the “hexagon” ABB 0 A1 C 0 C, states that A2 , H, and A3 are collinear.
Remark: Instead of applying Pascal’s theorem, one can show HA2 = C 0 A2 and HA3 = B 0 A3 ,
and then angle-chase.
12. Let us call the programmers Alice and Biff. Also label the first 2005 memory locations as
good, and the remaining ones as bad. Alice can guarantee a win as follows. On her first turn,
she chooses the first good memory location and the four bad ones. From then on, she always
copies Biff’s last move.
This ensures that at the end of every one of Alice’s turns, all good memory locations have
even parity. On the other hand, there are only 4 bad memory locations, so each turn must
decrease a good memory location. Therefore, a good location will become negative in at most
1 + 1 + 2 + 4 + . . . + 22004 = 22005 turns. Conversely, no bad location can become negative
that quickly.
Since all good memory locations are even at the end of every one of Alice’s turns, it follows
that Biff must be the first one to turn one of these location negative.
1
We
Pare using a general theorem here: let c1 , c2 , . . . , cnn be P
constants, and let {x1 , x2 , . . .} be a sequence satisfying
xm = n i=1 ci xm−i for m ≥ n. Suppose the polynomial Pnx = i=1
n
ci xn−i has n distinct roots α1 , α2 , . . . , αn . Then,
m
there exist constants C1 , C2 , . . . , Cn such that xm = i=1 Ci · αi for all m.
4
13. It is important to correctly handle all possible configurations while doing part (a).
(a) Claim: Let circles ω1 and ω2 intersect at X and Y . If AB is a chord on ω1 , and C and
D are the second intersections of AX and BY with ω2 , then AB and CD are parallel.
Proof: Using directed angles2 , we have ∠XAB = ∠XY B = ∠XY D = ∠XCD. There-
fore, lines AB and CD make the same angle with line AXC, and thus are parallel.
The result follows from applying this claim to chords BA, BE, and DE.
(b) Since triangles RGH and P BE have parallel sides, P R, BG, and EH all meet at a
common point, namely Q, so Q lies on P R.
14. It is not always possible. We consider all positions modulo 10. In this setting, adding 1 to a
square 10 times has no effect.
If a position A can reach the zero position by adding 1 to various squares, the zero position
can reach A by subtracting 1 from the same squares. Since there are 52 + 62 = 61 squares
to choose from, and each square can be subtracted from between 0 and 10 times, a total of
1061 < 1064 positions can be reached from the zero position in this way.
However, there are 1064 positions altogether modulo 10, so not all of them can be reached.
15. Let R = P Q ∩ AF , and let a = AR, b = BR, c = CR, d = DR, e = ER, f = F R denote
−−→
signed distances along the line AB. Also let x denote the signed product P R · QR. By power
of a point on each circle, we have ad = be = cf = x. Now:
AB · CD · EF − BC · DE · F A = (b − a) · (d − c) · (f − e) − (c − b) · (e − d) · (f − a)
= bdf − bde − bcf + bce − adf + ade + acf − ace −
cef + ace + cdf − acd + bef − abe − bdf + abd
= bdf − dx − bx + cx − f x + ex + ax − ace −
ex + ace + dx − cx + f x − ax − bdf + bx = 0.
Remark: A slicker solution is to perform an inversion around P , and then use angle-Ceva’s
theorem to exploit the fact that lines A0 D0 , B 0 E 0 , and C 0 F 0 concur at Q0 .
which implies:
5
17. Let Fk denote the Fibonacci sequence. Then, the full set of solutions for (m, n) is:
Dividing through by (x − 1)t , we have h(x) · (f (x) + g(x)) = h(x2 ) · (x + 1)t . Now, letting
x = 1 and dividing through by h(1) (which we know is not 0), we have 2n = f (1) + g(1) = 2t .
It follows that n is a power of 2.
6
Winter Camp 2009
Buffet contest
A1. Show that for any positive integer n, there exists a positive integer m such that
√ √ √
(1 + 2)n = m + m + 1.
A2. For every ordered pair of positive integers (x, y), define f (x, y) recursively as follows:
f (x − y, y) + 1 for x > y,
f (x, y) = f (x, y − x) + 1 for y > x,
x for x = y.
For example, f (5, 3) = f (2, 3) + 1 = f (2, 1) + 2 = f (1, 1) + 3 = 4. If f (x, y) ≤ 15, show that
x + y < 2009.
A3. Let x, y, z ≥ 0 be such that x + y + z = 3. Prove that
x3 y3 z3 1 2
3
+ 3
+ 3
≥ + · (xy + yz + zx).
y +8 z +8 x +8 9 27
When does equality hold?
A4. We assign a real number tx,y between 0 and 1 to every point on the plane (x, y) with integer
t +t +t +t
coordinates. This is done in such a way that tx,y = x−1,y x,y−1 4 x+1,y x,y+1 for all x, y. Show
that all the numbers are equal.
C1. A deck contains 52 cards of 4 different suits. Vanya is told the number of cards in each suit.
He picks a card from the deck, guesses its suit, and sets it aside; he repeats until the deck is
exhausted. Show that if Vanya always guesses a suit having no fewer remaining cards than
any other suit, he will guess correctly at least 13 times.
C2. A mathematics competition has n contestants and 5 problems. For each problem, each
contestant is assigned a positive integer score which is at most seven. It turns out every
pair of contestants have at most one problem whose scores are common. Find the maximum
possible value of n.
C3. Let n ≥ 2 be an integer and Tn be the number of non-empty subsets S of {1, 2, . . . , n} with
the property that the average of the elements in S is an integer. Prove that Tn − n is always
even.
C4. For n an odd positive integer, the unit squares of an n × n chessboard are coloured alternately
black and white, with the four corners coloured black. A tromino is an L-shape formed by
three connected unit squares. For which values of n is it possible to cover all the black
squares with non-overlapping trominos? When it is possible, what is the minimum number
of trominos needed?
1
G1. In 4ABC, points D and F are selected on sides BC and AB respectively so that AD · BC =
AB · CF . Let AD and CF intersect at P . Prove that either quadrilateral BF P D is cyclic or
quadrilateral F ACD is cyclic.
G2. Let ABC be a scalene triangle and let A0 , B 0 , and C 0 (respectively) be the points of intersection
of the interior angle bisectors A, B, and C (respectively) with the opposite sides of the triangle.
Now let:
G3. Let ω1 and ω2 be concentric circles with ω2 inside ω1 . Let ABCD be a parallelogram with
B, C, D on ω1 and A on ω2 . If BA intersects ω2 again at E and CE intersects ω2 again at
P , prove that CD = P D.
G4. Convex hexagon ABCDEF has area 1. Prove that at least one triangle out of ABC, BCD,
CDE, DEF , EF A, and F AB has area at most 16 .
N1. Find all positive integers n less than 1000 such that n2 is equal to the cube of the sum of its
digits.
N2. Find all integers a, b, c greater than 1 for which ab − 1 is divisible by c, bc − 1 is divisible by
a, and ca − 1 is divisible by b.
N3. The sequence of natural numbers a1 , a2 , a3 , . . . , satisfies the condition an+2 = an+1 an + 1 for
all n. Prove that an − 22 is composite for all n > 10, no matter what a1 and a2 are.
N4. Find all positive integers that can be written in the form
a2 + b2 + 1
ab
where a, b are positive integers.
2
Solutions
√ n
A1. Applying
√ nthe binomial
√ theorem to√(1 n+ 2) , we√ see there exist integers a and b 2such that
(1 2
¡ + √2) = a√+ ¢bn 2 and (1 − 2) = a − b 2. Multiplying these, we get a − 2b =
(1 + 2)(1 − 2) = ±1.
√ √ √ √ √ √
Setting m = min(a2 , 2b2 ), we have m + m + 1 = a2 + 2b2 = a + b 2 = (1 + 2)n .
Source: Romanian Math Stars Competition 2007, #1. Also see CMO 1994, #2.
A4. Define dx,y,n = tx+n,y+n − tx,y . Let C, ² be constants so that dx,y,1 ≥ C for some x, y
but dx,y,1 ≤ C + ² for all x, y. For any x, y, note that tx,y,n = tx,y,1 + tx+1,y+1,1 + . . . +
tx+n−1,y+n−1,1 ≤ n(C + ²).
We prove by induction that for all n, there exist x, y so that dx,y,n ≥ nC − 3n ². For n = 1, the
claim is trivial. Now suppose the result holds for n, and choose x, y so that dx,y,n ≥ nC − 3n ².
Using the given relation on t, we have:
dx−1,y,n+1 + dx,y−1,n+1 + dx+1,y,n−1 + dx,y+1,n−1
= dx,y,n ≥ nC − 3n ²
4
dx−1,y,n+1 + dx,y−1,n+1
=⇒ ≥ 2nC − 2 · 3n ² − (n − 1)(C + ²) ≥ (n + 1)C − 3n+1 ².
2
Therefore, one of dx−1,y,n+1 or dx,y−1,n+1 is at least (n + 1)C + 3n+1 ², and the claim is proven.
Now suppose dx,y,1 = C > 0 for some x, y. Fix ² > 0 and let m be the largest integer so that
there exists x, y for which dx,y,1 ≥ C + m². Then, as shown above, for each n, there exist
3
x, y so that dx,y,n ≥ nC + m² − 3n ² ≥ nC − 3n ². Choosing n large and then ² small, we have
dx,y,n > 1, which is impossible.
Therefore, dx,y,1 ≤ 0 for all x, y. Similarly, dx,y,1 ≥ 0 for all x, y, and hence tx,y = tx+1,y+1
for all x, y. Similarly, tx,y = tx+1,y−1 for all x, y. The original relation now implies that
tx,y = tx−1,y = tx,y−1 = tx+1,y = tx,y+1 , and the result follows.
Source: Iberoamerican Olympiad, miscellaneous problem
Remark: You cannot assume that there exist x, y for which dx,y,n is maximal. However, there
is a theorem in analysis saying there exist real numbers Mn such that dx,y,n gets arbitrarily
close to Mn without exceeding Mn . If you know this theorem, the proof becomes a lot cleaner.
C1. Let M denote the maximum number of cards remaining in any single suit. As Vanya proceeds,
M will only decrease if the current card is in a suit with M cards remaining, and no other suit
has M cards remaining. In this case, however, Vanya will correctly guess that suit. Therefore,
Vanya will guess correctly every time M decreases.
52
Since M ≥ 4 = 13 initially and it is 0 by the end, Vanya will be correct at least 13 times.
Source: Russia, 1998
§ ¨
C2. There are 7 possible scores on each question. If n ≥ 50, then at least 50
7 = 8 contestants
got the same score on problem 1. But then two of those contestants must have gotten the
same score on problem 2, which is impossible.
Now, for 1 ≤ i, j ≤ 7, let xi,j,k denote the value in {1, 2, . . . , 7} that is congruent to i +
jk (mod 7). Consider 49 contestants Ci,j where contestant Ci,j receives score xi,j,k on problem
k. Suppose that two contestants Ci1 ,j1 and Ci2 ,j2 got the same scores on questions k1 and
k2 . Then i1 − i2 + (j1 − j2 )k1 ≡ i1 − i2 + (j1 − j2 )k2 ≡ 0 (mod 7). Subtracting, we have
(j1 − j2 )(k1 − k2 ) ≡ 0 (mod 7) =⇒ j1 ≡ j2 (mod 7) =⇒ j1 = j2 . But then we must also
have i1 = i2 , which is a contradiction.
Therefore, it is possible to satisfy the required condition with 49 contestants, and hence 49
is the maximum possible value for n.
C3. Let S 0 denote the subsets of {1, 2, . . . , n} with at least two elements and with integer average.
For each set X ∈ S 0 that contains its average x, we pair it with the set X\{x}, and conversely
for each set Y ∈ S 0 that does not contain its average y, we pair it with the set Y ∪ {y}. This
is a proper pairing, so S 0 must contain an even number of sets.
Therefore, Tn has the same parity as the number of singleton sets with integer average, of
which there are exactly n.
Source: Putnam 2002, A3
C4. Let Xn denote the minimum number of trominos required to cover an n × n board in this
2
way. We claim any n ≥ 7 is possible, and Xn = (n+1)
4 .
Indeed, let B denote the set of all squares that are an even number of rows and even number
2
of columns away from the bottom-left square. There are exactly (n+1)
4 such squares, and they
(n+1)2
are all black. Furthermore, each tromino can cover at most one square in B, so Xn ≥ 4 .
(n+1)2
However, if n ∈ {1, 3, 5}, then 3 · 4 > n2 , so it is impossible to place this many trominos
on the board.
2
It remains to show that if n ≥ 7, there exists a valid tiling with (n+1)
4 trominos. For n = 7,
a valid tiling is shown below. Now, suppose there is a valid tiling of an n × n square using
4
2
exactly (n+1)
4 trominos. For k > 1 odd, we can also tile a 2 × k rectangle with k+1
2 trominos
by using two tronominos for the first 2 × 3 rectangle, and 1 tromino for each following 2 × 2
rectangle. Since an (n+2)×(n+2) rectangle can be partitioned into an n×n rectangle, a 2×n
2 (n+3)2
rectangle, and a 2×(n+2) rectangle, it can therefore by tiled with (n+1)
4 + n+1 n+3
2 + 2 = 4
trominos. The result now follows by induction.
G1. Let R denote the circumradius of 4ABC. By the extended sine law, we have AD ·BC = AB ·
4R2 ·(sin A)·(sin B)·(sin C) 2
sin B
sin ADB · BC = sin ∠ADB . Similarly, AB · CF = 4R ·(sinsin
A)·(sin B)·(sin C)
∠CF B . Equating
these, we get sin ∠ADB = sin ∠CF B, which implies ∠ADB = ∠CF B or ∠ADB = 180◦ −
∠CF B. In the former case, F ACD is cyclic; in the latter case, BF P D is cyclic.
G2. Assume without loss of generality that A00 lies on the same side of A0 as C does. Then,
∠A00 AA0 = ∠A00 A0 A = ∠CA0 A = 180◦ − ∠A0 AC − ∠A0 CA = ∠A 2 + ∠B. It follows that
00 00
∠CAA = ∠B, and hence AA is tangent to the circumcircle ω of 4ABC at A. Therefore,
A00 is the intersection of BC and the tangent to ω at A. Similar statements hold for B 00 and
C 00 .
The problem is now equivalent to Pascal’s theorem on the degenerate hexagon AABBCC.
Source: Iberoamerican Olympiad 2004
G3. Let O denote the center of ω1 and ω2 . The perpendicular bisectors of AE and CD are parallel
and both pass through O, so they are in fact identical. Therefore, the quadrilateral AEDC
is symmetric about this line, and BC = AD = EC.
Now, let B 0 and C 0 denote the second intersections of BE and CE with ω1 . ∠EC 0 B 0 = ∠EBC
and ∠B 0 EC 0 = ∠CEB so 4C 0 B 0 E ∼ 4BCE, and hence, C 0 B 0 = EB 0 .
Applying the symmetry argument to B 0 EAB and C 0 EP C, we also have B 0 E = AB = DC
and C 0 E = P C. Also, ∠C 0 EB 0 = ∠P CD since B 0 E and DC are parallel. Therefore,
4C 0 B 0 E ∼
= 4P DC, and the result follows.
Remark: there are two configurations, depending on which of A or E is closer to B, but this
argument works without change in either case.
G4. Let P be the intersection of AD and BE, Q be the intersection of BE and CF , and R be the
intersection of CF and AD. Also assume without loss of generality that P is on the same
side of CF as A and B; i.e., P is between A and R. Then, it is easy to check that R must be
between C and Q, and Q must be between E and P .
In this case, triangles ABR, BCR, CDQ, DEQ, EF P , and F AP are all disjoint, so their
total area is at most 1. It follows that one of them has area at most 16 . Regardless of which
triangle it is, we have found four adjacent vertices P, Q, R, S on the hexagon and a point X on
5
segment P S for which 4QRX has area at most 16 . Note that the area of 4QRX is bounded
between the area of 4QRP and the area of 4QRS. Therefore, one of these triangles also
has area at most 16 , and the result follows.
N2. Note that a, b and c are all relatively prime, since if p|a, b, then a cannot divide bc − 1. Now
the given condition implies:
N3. For any n ≥ 3, we have an ≡ 0 (mod an ) and an+1 = an an−1 +1 ≡ 1 (mod an ). We now apply
the recurrence to calculate the sequence {an , an+1 , . . . an+6 } ≡ {0, 1, 1, 2, 3, 7, 22} (mod an ).
Therefore, an+6 − 22 must be a multiple of an .
For n ≥ 3, we have an = an−1 an−2 + 1 > 1. It is also easy to check that an+6 − an > 22.
Therefore, an and an+6
an
−22
are both integers greater than 1, and hence an+6 − 22 is not prime.
Source: American Math Olympiad Program 1998
a2 +b2 +1
N4. Suppose k can be expressed in this form, and let (a, b) be such that ab = k and a + b is
as small as possible.
a2 +1
Suppose a < b. Then b = ka − b is an integer. Denoting this quantity by b0 , we have
(b−1)2 +1
b0 ≤ b < b, and
³ ´2
a2 +1
a2 + (b0 )2 +1 a2 + b +1 a2 + b2 + 1
= = = k,
ab0 a· a2 +1 ab
b
which contradicts the minimality of (a, b). Similarly, b < a is impossible so we must have
2 2 +1
a = b. In this case, a +b
ab is only an integer if a = b = 1 and k = 3.
Therefore, 3 is the only integer that can be expressed in this form.
2 2
Remark: The equation for b0 is found by root-flipping. We interpret a +bab
+1
= k as a quadratic
0
equation in b, and note that if b is one root, then b is the other one.
6
Winter Camp 2009
Mock Olympiad
1. At a party, one or more pairs of people shook hands with each other. We say two people were
“close” if either they shook hands with each other or if there was a third person they both
shook hands with. Show there were two people who (a) were close, and (b) shook hands with
the exact same number of people.
2. Let ABCD be a cyclic quadrilateral such that AD + BC = AB. Prove that the bisectors of
the angles ADC and BCD meet on the line AB.
3. Find all integer-valued functions, f and g, defined on the integers, for which g is one-to-one
and
4. Prove that any set of 10 positive integers, ranging between 1 and 2009, contains three distinct
elements a, b, c such that gcd(a, b) divides c.
1
Solutions
1. For each person p, let np denote the number of people that p shook hands with. Let M =
max np , let x0 be a person for which np = M , and let {x1 , x2 , . . . , xM } be the people that x0
shook hands with.
For each i, we have nxi ≥ 1 since each person xi shook hands with x0 , and nxi ≤ M by defi-
nition. Therefore, the Pigeonhole Principle implies that two of the values {nx0 , nx1 , . . . , nxM }
are equal. But if nxp = nxq , then xp and xq have the property we are looking for.
2. Choose E on AB so that AE = AD and hence BE = BC. Also let F denote the second
intersection of AB and the circumcircle of 4CDE. Note that F lies between A and B.1 Also
note that regardless of configuration, ∠AF D = ∠ECD.
Therefore, ∠ADC = 180◦ − ∠EBC = 2∠BCE = 2∠BCD − 2∠ECD = 360◦ − 2∠F AD −
2∠AF D = 2∠ADF . Similarly, ∠BCD = 2∠BCF , so F is the intersection of the bisectors of
∠BCD and ∠ADC.
We know that F lies on AB, however, so the result follows.
Source: American Math Olympiad Program 1999
3. Let m = f (0) and n = g(0). Setting x = 0 in the given equation yields f (n + y) = g(f (y))
(1). Setting y = 0 in the given equation yields g(m + x) = f (g(x)) (2).
Setting y = g(x) in equation (1) gives us g(f (g(x))) = f (n + g(x)), which we know from
the original equation is equal to g(f (n) + x). Since g is one-to-one, it follows that f (g(x)) =
f (n) + x. Combining this with (2), we have g(m + x) = f (n) + x. In particular, there exists
a constant C so that g(x) = x + C.
Substituting this into (2) yields m + x + C = f (x + C), which implies there exists a constant
B so that f (x) = x + B.
Conversely, it is easy to check that f (x) = x + B and g(x) = x + C satisfies the original
functional equation.
Source: Latvia
4. Consider a set {x1 , x2 , . . . , x10 } of positive integers for which gcd(xi , xj ) does not divide xk
for all distinct i, j, k. We claim max(xi ) > 2009. If each xi is even, we can divide through by
2 to achieve a set with smaller elements. Therefore, we may assume without loss of generality
that the set contains an odd element, say x1 .
For i > 1, let gi = gcd(x1 , xi ). If gi |gj for any i 6= j, then gcd(x1 , xi )|xj , which is impossible.
Therefore, x1 has 9 factors (g2 , g3 , . . . , g9 ), none of which divide each other.
Now, let p1e1 · pe22 · . . . · pekk be the prime factorization of x1 , ordered so that e1 ≥ e2 ≥ . . . ≥ ek .
e e e
Each gi can be written in the form p1i,1 · p2i,2 · . . . · pki,k . If ei,t = ej,t for allQ
t > 1, then one of gi
or gj must divide the other, which is impossible. However, there are onlyQ j>1 (ej +1) possible
values for {ei,2 , ei,3 , . . . , ei,t }, so if these are all distinct, we must have j>1 (ej + 1) ≥ 9.
1
For example, if B, E, A, F occur on line BA in that order, then by continuity, there exists X between E and A
for which BX · XA = EX · XF . This point X must lie on the radical axis of circle ABCD and circle CDE, which
is CD. Therefore, CD intersects AB between A and B, which is impossible.
2
Now, if k = 2, then e2 ≥ 8 and x1 ≥ 38 · 58 . If k = 3, then e2 , e3 ≥ 2 and x1 ≥ 32 · 52 · 72 ,
or e2 ≥ 4 and x1 ≥ 34 · 54 · 7. If k = 4, then a2 ≥ 2 and x1 ≥ 32 · 52 · 7 · 11. If k = 5, then
x1 ≥ 3 · 5 · 7 · 11 · 13. Regardless, x1 > 2009, and the result is proven.
Source: Romanian Math Stars Competition 2007, #8, except they consider 27 integers instead
of 10. They also conjecture the result holds even for 6 integers.
3
1 Mock CMO
1. (Lithuania team contest 2000, #3)
Solve the equation
p p p
−3x2 + 18x + 37 + −5x2 + 30x − 41 = x2 − 6x + 109.
Solution: √ √ √
After setting t = (x − 3)2 , the equation reduces to 64 − 3t + 4 − 5t = t + 100. t = 0 is
one solution, and the left-hand side is decreasing in t while the right-hand side is increasing in t.
Therefore, t = 0 (and hence x = 3) is the unique solution.
2. (Ukraine 10th grade 2008, #2)
Let ABCD be a parallelogram with 6 BCD > 90◦ . Extend line BC to K so that DK = DC, and
extend line DC to L so that BL = BC. Let the bisectors of 6 CDK and 6 CBL meet at point Q. If
6 BQD = α and KL = a, express the circumradius of triangle AKL in terms of a and α.
Solution:
ADLB and ADKB are isosceles trapezoids, so K, L lie on the circumcircle of 4ABD. 6 BAD =
6 LCK = 180◦ − 6 BQD since BQ and QD are perpendicular to LC and CK. Therefore, ABLQKD
is a cyclic hexagon. Now, 6 LAK = 6 LAQ + 6 KAQ = 6 LDQ + 6 KBQ = 180◦ − 2α. Applying
the extended sine law, we have the radius is 2 sina 2α .
3. (Japan junior math olympiad 2008, #10)
Consider a board consisting of 9 squares of length 1, arranged in 3 rows and 3 columns as in the figure
below. Let us number squares as indicated in the figure.
1 2 3
8 4
7 6 5
Let A, B, C, D be points chosen from the interior (not on the boundary) of the square 1, 3, 5, 7,
respectively. Denote by X the sum of the areas of the intersection of the quadrilateral ABCD with the
squares 1, 3, 5, and 7. Denote also by Y the sum of the areas of the intersections of the quadrilateral
ABCD with the squares 2, 4, 6, and 8. Prove that X < Y .
Solution:
Suppose that ABCD intersects the boundary of square i and square i + 1 at point Xi , and that
Xi is distance xi from P8 the central square. Then, the area of ABCD within square 2i is exactly
x2i−1 +x2i 1
2 , so Y = 2 i=1 xi . Now let Yi and Zi denote the outer and inner corners of square i for
i = 1, 3, 5, 7. Then, the area of ABCD within square 2i − 1 is less than the area of quadrilateral
X2i−2 Y2i−1 X2i−1 Z2i−1 , which is exactly 1 − 1−x22i−2 − 1−x22i−1 = x2i−2 +x
2
2i−1
. Therefore, X <
1
P8
2 i=1 ix = Y .
1
0 (mod d + 1). Equivalently m ≡ d (mod lcm(d + 1, d + 2)). Since d + 1 and d + 2 are relatively
prime, it follows that m = C(d + 1)(d + 2) + d, which gives n = C(2d + 3) + 2.
Clearly, n = 2 can be expressed in this form but n = 1 cannot. For n > 2, the condition is
equivalent to requiring that n − 2 have an odd divisor greater than 3. This fails only for n = 2 + 2k
and n = 2 + 3 · 2k . Altogether, that gives only 1 + 11 + 10 numbers that are not reducible, so the
answer is 1978.
5. (Japan math olympiad final 2008, #5)
Does there exist a positive integer n satisfying the following condition?
Condition: For an arbitrary rational number r, there exists an integer b and nonzero integers a1 , a2 , . . . , an
such that r = b + a11 + . . . + a1n .
Solution:
No. We prove by induction on n that for each integer n and rational number q, there exists a
rational number q 0 < q such that no number in the interval (q 0 , q) can be expressed in this form.
For n = 1, the claim is trivial.
Now suppose the claim has been proven for n − 1. Fix an arbitrary q, and let f denote the
fractional part of q (except if q is an integer, we take f = 1). Then, if we are to express some
number y ∈ (q − f2 , q) in the required form, we must have some a1i ≥ f2 =⇒ ai ≤ f2 . This leaves
only a finite number of options for ai . Once ai is fixed to some value u, we have reduced the problem
to the n − 1 case, so by the inductive hypothesis, there exists qu0 such that no number in (qu0 , q) can
be achieved, given that ai = u. The inductive step follows from taking q 0 = max(q − f2 , maxu qu0 ).
2
2 Mock IMO Day 1
1. (Ukraine 11th grade 2008, #3)
A point O is placed inside triangle ABC so that 6 BOC = 90◦ and 6 BAO = 6 BCO. If M and N are
the midpoints of the segments AC and BC respectively, prove that 6 OM N = 90◦ .
Solution:
Let P be the midpoint of OC. N is the circumcenter of right triangle BOC since it is the midpoint
of the hypoteneuse, so 6 BCO = 6 N CP = 6 N OP . Since M P is parallel to AO and M N is parallel
to AB, we have 6 N M P = 6 BAO = 6 BCO = 6 N OP . Therefore, OM P N is a cyclic quadrilateral
and 6 OM N = 6 OP N = 6 BOC = 90◦ .
2. (Japan math olympiad final 2008, #4)
Determine all real-valued functions f defined on the real line, which satisfy
3
3 Mock IMO Day 2
4. (IMO short list 2008, N1) Let n be a positive integer and let p be a prime number. Prove that if a, b, c
are integers (not necessarily positive) satisfying the following equations
an + pb = bn + pc = cn + pa,
then a = b = c.
Solution:
If two values are equal, the claim is trivial. Otherwise, multiply the equations an − bn = p(c − b) to
n
−bn bn −cn cn −an
get a a−b · b−c · c−a = −p3 . If n is odd, an − bn has the same sign as a − b, and the left-hand
side is positive, which is impossible. Otherwise, two of a, b, c are the same parity, and in this case,
n
−bn
the corresponding a a−b term is even, which implies p = 2.
n n
−b
The original equation now implies a, b, c must all be the same parity, and hence a a−b is even (as
3
above). Since the product of these terms is −2 , each such term is ±2. Comparing with the original
equation, we have a − b = ±(b − c). If a − b, b − c, c − a are all the same sign, they are all equal,
but their sum is 0 so then a = b = c. Otherwise, a − b = c − b or some shift thereof, and two values
are equal. Either way, we have a contradiction.
5. (Ukraine 11th grade 2008, #7)
Prove that the inequality
x y z √
p +p +p ≤ 3
x2 + y + z x + y2 + z x + y + z2
4
Mock Olympiad #1
1. Given trapezoid AY CD with parallel sides AY and CD, assume that there exist points E on
line AD outside segment AD, and F inside segment Y C such that ∠Y CE = ∠ADF . Denote
by I the point of intersection of AY and EF , and by J the point of intersection of CD and
EF . Let K be the midpoint of segment EF , assume it does not lie on line CD. Prove that
I belongs to the circumcircle of CDK if and only if K belongs to the circumcircle of AY J.
2. Let a0 , a1 , a2 , . . . be a sequence of positive integers such that the greatest common divisor of
any two consecutive terms is greater than the preceding term; in symbols, gcd(ai , ai+1 ) > ai−1 .
Prove that an ≥ 2n for all n ≥ 0.
3. Prove that for any four positive real numbers a, b, c, d the inequality
1
Canada Day Mock Olympiad 2009
Solutions
C J D
A
I Y
1
which is equivalent to JI · KI = F I · EI. Writing JI = F I + F J, KI = F I + 12 F E, and
EI = F I + F E, this occurs if and only if F J = 2FF I·F E
I+F E (4).
The result follows from the fact that conditions (3) and (4) are identical.
2. (IMO Short list 2008, N3)
We first prove the result for small values of n = 0, 1, 2, 3. Since ai ≥ gcd(ai , ai+1 ) > ai−1 , the
sequence is increasing, so a1 ≥ 2. For each i ≥ 1, we also have ai+1 ≥ gcd(ai , ai+1 ) + ai ≥
ai + ai−1 + 1, hence a2 ≥ 4, a3 ≥ 7. a3 = 7 is impossible, because then 2 ≤ a1 < gcd(a2 , 7),
which contradicts the fact that a2 < 7. So a3 ≥ 8 and the claim is proven for n = 0, 1, 2, 3.
We now proceed be induction.
Let n ≥ 3 and assume that ai ≥ 2i for i = 0, 1, . . . , n. We must show that an+1 ≥ 2n+1 .
We will in fact show that an+1 ≥ min(2an , 4an−1 , 8an−2 , 16an−3 ), which is clearly sufficient.
Suppose this is not the case. Then we must have an+1 < 2an , so gcd(an+1 , an ) < an+12 , and
an+1 an+1
< an−1 < gcd(an , an+1 ) < .
4 2
an+1 2an+1
So we must have that gcd(an+1 , an ) = 3 and an = 3 .
Now look at an−2 . By assumption,
an+1 3an−1 an−1
an−2 >
> > .
8 8 3
But we also know that an−2 < gcd(an−1 , an ), and so gcd(an−1 , an ) = an−12 . Hence
an an+1 an−1 an
< < an−2 < gcd(an−1 , an ) = < ,
6 8 2 4
an 2an 4an+1
so we conclude that gcd(an−1 , an ) = 5 , and an−1 = 5 = 15 .
Finally, we see that
an+1 4an+1
< an−3 < gcd(an−2 , an−1 ) = gcd an−2 , .
16 15
4an+1 an+1 2an+1
From before, we know that 45 < 8 < an−2 < gcd(an−1 , an ) = 15 , hence an−2 doesn’t
divide 4a15
n+1
, and so
4an+1 an−2 an+1
gcd an−2 , ≤ < ,
15 2 15
but then
4an+1 4an+1 an+1
an−3 < gcd an−2 , ≤ <
15 75 16
which is a contradiction. This completes the induction, so the claim is proven.
3. (IMO Short list 2008, A7)
2 (c−a)2
Note that 2(a−b)(a−c)
a+b+c = (a−c)
a+b+c +
(a−c)(a−2b+c)
a+b+c and 2(c−d)(c−a)
c+d+a = c+d+a + (c−a)(c−2d+a)
c+d+a . There-
2(a−b)(a−c) 2(c−d)(c−a)
fore, a+b+c + can be written as
c+d+a
2 1 1 a − 2b + c a − 2d + c
(a − c) · + + (a − c) · −
a+b+c c+d+a a+b+c a+d+c
2 2a + b + 2c + d 3a + 3c
= (a − c) · + (a − c) · (d − b) · .
(a + b + c)(c + d + a) (a + b + c)(c + d + a)
2
2(b−c)(b−d) 2(d−a)(d−b)
Similarly, b+c+d + d+a+b can be written as
a + 2b + c + 2d 3b + 3d
(b − d)2 · − (a − c) · (d − b) · .
(b + c + d)(d + a + b) (b + c + d)(d + a + b)
where we used the fact here that b, d ≥ 0 in each factor. Furthermore (a + c)2 ≥ 4ac, so this
is at least
p √
8(a + c)2 · a2 c2 4 2 · |(a − c)(b − d) · ac(a + c)|
2 · |(a − c) · (b − d)| · = .
(S − a)(S − b)(S − c)(S − d) (S − a)(S − b)(S − c)(S − d)
3·|(a−c)(b−d)·ac(a+c)|
This is clearly at least (S−a)(S−b)(S−c)(S−d) , which completes the proof. For equality to hold
at the end, we must have a = c or b = d. For equality to hold during our initial application
of AM-GM, we must then have both a = c and b = d. Conversely, it is clear that if a = c and
b = d, then equality does indeed hold.
3
Mock Olympiad #2
1. Let p be the product of two consecutive integers greater than 2. Show that there are no
integers x1 , x2 , . . . , xp satisfying the equation
p
" p
#2
! 4 !
x2i − xi = 1.
4·p+1
i=1 i=1
2. Let S = {x1 , x2 , . . . , xk+l } be a (k + l)-element set of real numbers contained in the interval
[0, 1]; k and l are positive integers. A k-element subset A ⊂ S is called nice if
$ $
$ $
$1 ! 1 ! $ k+l
$ xi − xj $$ ≤
$k l 2kl
$
xi ∈A xj ∈S\A $
% &
2 k+l
Prove that the number of nice subsets is at least .
k+l k
3. In $ABC, let AA0 , BB0 , CC0 be altitudes. Let A1 be a point inside $ABC such that
1
Mock Olympiad #2 Solutions
July 4, 2009
p p
!2
X 4 X
LHS = x2i − · xi
4p + 1
i=1 i=1
!2
p p p
1 X 4 X X
= · x2i + · p · x2i − xi
4p + 1 4p + 1
i=1 i=1 i=1
p
1 X 4 X
= · x2i + · (xi − xj )2 .
4p + 1 4p + 1
i=1 1≤i<j≤p
Suppose xi takes on at least 3 values, with a numbers taking on the minimum value, c
numbers taking on the maximum value, and b numbers taking on intermediate values. Then,
2 2 2 2
P
1≤i<j≤p (xi − xj ) ≥ ac · 2 + ab · 1 + bc · 1 ≥ 4a + b + c ≥ p + 3, and LHS > 1. Therefore,
xi can take on at most 2 different values.
pn 2
If all the xi are equal to some value n, then LHS = 4p+1 . If p = 1, then this has no solutions
because 5 is not a square. If p > 1, then the factor of p in the numerator can never be
canceled out, so there are no solutions in this case either. Assume now that p > 2.
Let a numbers take the value A, and b numbers take the value B. Suppose that |A − B| > 1,
then
4 16(p − 1)
LHS ≥ · (4ab) ≥ >1
4p + 1 4p + 1
since p > 1. So we can assume that |A − B| = 1.
Now assume that neither A nor B are equal to 0. Then
(p − 1) + 4 4(p − 1) 5p − 1
LHS ≥ + =
4p + 1 4p + 1 4p + 1
which is bigger than 1 since p > 2.
So we can further assume A = 0. Then
b + 4ab
LHS = .
4p + 1
1
4p−3
If b = 1, then a = p − 1 and LHS = 4p+1 6= 1. If b ∈ [2, p − 2], then p ≥ 4, ab ≥ 2p − 4, and
8p−14 5p−5
LHS ≥ 4p+1 > 1. If b = p − 1, then a = 1 and LHS = 4p+1 , which is 1 only if p = 6.
This proves that p must equal 2 or 6. Conversely, if p = 2, we can take {x1 , x2 } = {1, 2}, and
if p = 6, we can take {x1 , x2 , . . . , x6 } = {0, 1, 1, 1, 1, 1}.
Notice that f (A1 ) + f (A2 ) + · · · + f (Ak+l ) = 0. (This is because each element xj appears in
Ai for k different values of i, and it appears in S\Ai for l different values of i.) Also,
xi+k − xi xi+k − xi 1 1
|f (Ai+1 ) − f (Ai )| = + ≤ + .
k l k l
Therefore if Ai and Ai+1 are of different signs1 , then either |f (Ai )| or |f (Ai+1 | is at most
1 1 1 k+l
2 · k + l = 2kl , and therefore one of Ai , Ai+1 is nice.
Since the sum of the f (Ai ) is 0, we must have at least 1 negative and 1 positive sign (unless
they’re all 0 which is silly). If there exist 2 disjoint sets {i, i + 1}, {j, j + 1} such that
f (Ai ), f (Ai+1 ) and f (Aj ), f (Aj+1 ) are of opposite signs, then by above we have at least two
nice sets. Otherwise, exactly one f (Ai ) is of a different sign from the rest. Assume wlog that
f (A1 ) ≥ 0 and for i 6= 1, f (Ai ) < 0. If A1 is not nice, than bothP Ak+l and A2 are nice,k+l so we
have found our two nice sets. Otherwise, f (A1 ) ≤ k+l 2kl , and i6=1 |f (A i )| = f (A 1 ) ≤ 2kl , so
every set must be nice. This finishes the proof of Claim 1.
Now, consider choosing a random permutation {y1 , y2 , . . . , yk+l }, and then choosing a random
Ai corresponding to this permutation. By Claim 1, this chooses a nice set with probability
2
at least k+l . On the other hand, this is equivalent to first choosing the shift i and then the
permutation {y1 , y2 , . . . , yk+l }, but once i is fixed, we will be equally likely to choose any
possible set. Therefore, this entire process chooses a set uniformly at random, so it follows
2
that at least k+l · k+l
k sets are nice.
3. Ukraine 2008,11.8
Solution 1:
Denote the angles of triangle ABC by a, b, c, and let ∠A1 BC = ∠A1 AB = x,∠A1 CB =
∠A1 AC = y. Then
∠ABA1 = b − x, and so ∠BA1 A = 180 − b. Similarly, ∠CA1 A = 180 − c, and so ∠BA1 C =
180 − a. Therefore, if we let H be the orthocenter of triangle ABC, CBA1 H are concyclic.
1
We consider 0 to be of positive sign.
2
Denote the circle they lie on by S1 . Do a dilation centered at A with factor 21 , and let
S1 transform to S2 . Then denoting the midpoints of ABC by A0 , B 0 , C 0 , we know that S2
contains B 0 , C 0 , and the midpoint of AH, so it must be the nine-point circle. So S2 passes
through B0 , B 0 , C0 and C 0 as well. Notice that since S1 passed through A1 , A2 lies on the
nine-point circle.
We will prove A2 A0 , B2 B0 , C2 C0 are concurrent using Sine-Ceva’s theorem on triangle A0 B0 C0 .
Let B 00 denote the image of B0 under the dilation centered at A with factor 2. We have
AA1
Similarly, sin ∠A2 A0 C0 = AB sin b.
Applying Sine-Ceva to A0 B0 C0 ,
A2
B0
C’ B’
C0 A1
H B’’
B C
A0
C’’
Solution 2:
Let A0 denote the intersection of AA2 and BC. The given condition implies that the circum-
circle of 4AA1 B is tangent to BC at B, and the circumcircle of 4AA1 C is tangent to BC at
3
C. Since A0 is on the radical axis of these two circles, it follows that BA0 = CA0 , and hence
A0 is the midpoint of BC. Also let B 0 and C 0 denote the midpoints of AC and AB.
Now, as in the other solution, note that A2 lies on the nine-point circle, and hence C0 , A2 , B0 , A0 ,
and A0 are concyclic. Therefore, ∠C0 A0 A2 = ∠C0 A0 A2 = ∠C0 A0 A. By the sine law,
0
sin ∠C0 A0 A = AC0 · sinA∠BAA
0C
0
. Now, BC0 C is a right triangle with circumcenter A0 so
0 0B
A0 C0 = A0 B, and sin ∠C0 A0 A = (AC cos A) · sin ∠BAA
A0 B = (AC cos A) · sin ∠AA
AB .
sin ∠AA0 C sin ∠C0 A0 A AC 2
Similarly, sin ∠B0 AA0 = (AB cos A) · AC , so sin ∠B0 A0 A = AB 2
. Therefore,
A2
B0
C0 A1
B C
A0 A’
4
Mock Olympiad #3
3. (IMO Short list 2001, N4) Let p ≥ 5 be a prime number. Prove that there exists an integer
a with 1 ≤ a ≤ p − 2 such that neither ap−1 − 1 nor (a + 1)p−1 − 1 is divisible by p2 .
1
Mock Olympiad #3 Solutions
July 6, 2009
(n − 2)|2(a1 + a2 + · · · + an − an − an−1 ),
so
(n − 2)|n2 − 1 − 2an−1 .
But this implies 2an−1 = 3(mod n − 2), and since n − 2 is odd, this has the unique solution
an−1 = n+12 , which contradicts an−1 6= an . So we must have an = 1 or an = n. This means
that Xn = Xn−1 + Yn−1 = 2Xn−1 . For n ≥ 3, we therefore have Xn = 3 · 2n−2 .
d−a b2 c2
Proof: Set z = 2 ,x = 4z , y = 4z . These values are all positive since d > a. Further-
1
more,
b2 + c2 (d2 − a2 ) − (d2 + a2 − 2ad)
x+y−z = −z = = a,
4z 2(d − a)
r r
√ b2 √ c2
2 xz = 2 z · = b, 2 yz = 2 z · = c, and
4z 4z
b2 + c2 (d2 − a2 ) + (d2 + a2 − 2ad)
x+y+z = +z = = d.
4z 2(d − a)
Since x + y − z = a > 0, we can substitute x, y, z into the given equation to get f (a) + f (b) +
f (c) = f (d), which completes the proof of the claim.
√
Now let g(x) = f ( x). We have proven that g(a) + g(b) + g(c) = g(a + b + c) for all a, b, c > 0.
Taking a = b = c = x, we have g(3x) = 3g(x). Taking a = b = 3x and c = x, we then
have g(7x) = 7g(x). Taking a = b = 2x and c = 3x, we then have g(2x) = 2g(x). Now,
taking a = b = x and c = (n − 2)x, we have g(nx) = 2g(x) + g((n − 2)x), so it follows that
g(nx) = ng(x) for all positive integers n. For any positive integers p, q, we therefore have
g( pq ) = 1q · g(p) = pq · g(1) (*).
Also, if a < d, then g(d) − g(a) = g(b) + g(c) > 0, so g is increasing. Since the rationals
are dense in the reals, it now follows from (∗) that g(x) = Cx for some constant C > 0, and
√ √
hence f (x) = Cx2 . Conversely, if f (x) = Cx2 , then f (x + y − z) + f (2 xz) + f (2 yz) =
√ √
(x + y − z)2 + (2 xz)2 + (2 yz)2 = (x + y + z)2 = f (x + y + z).
Remark: It isn’t necessary to calculate x, y, z explicitly to prove the claim. One may also
proceed as follows: First fix the products xz and yz, and let z vary. Then if z is very small,
x + y − z will approach infinity, and if z is very big, x + y − z will approach minus infinity,
so by continuity of x + y − z as a function of z, x + y − z will take every possible value. If
you find this sort of argument confusing and want to understand it better, feel free to speak
to one of the trainers!
since p 6 |x, p − 1.
Claim 2: If x ∈ [1, p − 1], then xp−1 6≡ (x + 2p)p−1 (mod p2 ).
Proof: Again, the binomial theorem gives
since p 6 |x, p − 1.
Let hus callia number x “good” if xp−1 ≡ 1(mod p2 ). By pairing up x and p − x for all
x ∈ 1, p−1 p−1
2 , Condition 1 implies that at most 2 numbers in [1, p−1] are good. Furthermore,
2
we already know that 1 is good. Now, if two consecutive numbers in [1, p] are not good, then
the problem is done. Otherwise, the good numbers in [1, p] must be exactly 1, 3, 5, . . . , p − 2.
Now, (3p − 6)p−1 ≡ 3p−1 · (p − 2)p−1 ≡ 1(mod p2 ), so 3p − 6 is good. However, we already
know p − 6 is good, so this contradicts Claim 2. The proof is now complete.
3
Mock Olympiad #4
July 9, 2009
a b c d
abcd = 1 and a + b + c + d > + + + .
b c d a
Prove that
b c d a
a+b+c+d< + + + .
a b c d
2. In an acute triangle ABC, segments BE and CF are altitudes. Two circles passing through
the points A and F are tangent to the line BC at the points P and Q so that B lies between
C and Q. Prove that the lines P E and QF intersect on the circumcircle of triangle AEF .
3. For every n ∈ N, let d(n) denote the number of (positive) divisors of n. Find all functions
f : N → N with the following properties:
1
1 Solutions
1. (IMO 2008 Short list, A5)
We will prove that
X a b
2a + 2b + 2c + 2d ≤ + . (1)
cyc
b a
Q B P C
We claim that AEP Q is a cyclic quadrilateral. Let a, b, c denote the angles of 4ABC. By
power of a point, BP 2 = BF · BA = BQ2 . Therefore, CP · CQ = (CB + BQ) · (CB −
2
BP ) = CB 2 − BF · BA = CB 2 − (CB · cos b) · CB sin c 2 · sin(b+c)−(cos b)(sin c)
sin a = CB sin a =
(cos c)(sin b) CB sin b
CB 2 ·
sin a = (CB cos c) · sin a = CE · CA. This proves AEP Q is cyclic by power
of a point.
Now, ∠F RE = ∠QRP = 180◦ − ∠RP Q − ∠RQP = ∠QAC − ∠QAB. Here, we used the
fact that AEP Q is cyclic and the circumcircle of AF Q is tangent to BC at Q. Therefore,
∠F RE = ∠BAC = ∠F AE, which completes the proof.
Remark: Here is another way to prove AEP Q is cyclic. Let H be the orthocentre of ABC.
Note that AF HE is cyclic. By power of a point, BP 2 = BQ2 = BF · BA = BH · BE, so the
circumcircle of QEH is tangent to line BC. Likewise, the circumcircle of P EH is tangent to
line BC. Now invert around C, preserving circle AF HE. The circles passing through AF
tangent to line BC goes to the circle passing through EH tangent to line BC, so P and Q
go to each other.
3
prime factors. Let r be a prime dividing
and let p 6= r be a prime dividing n. Taking
f (n),
n n n−1 n
x = p , y = p, we get f (n)| p − 1 p f p . We know r must divide n, so it doesn’t divide
n n−1 , and hence it divides f ( n ). By our induction claim, the exponent of r dividing
p −1 p p
e1 e2 em −1
p −1 p −1
f (n) is bounded by rk − 1, where rk ||n. Hence, f (n) divides p11 · p22 · · · ppmm . The
claim now follows from the fact that d(f (n)) = n.
It’s straightforward to check this f actually satisfies the two conditions.
4
Mock Olympiad #5
1. An eccentric mathematician has a ladder with n rungs that he always ascends and descends
in the following way: When he ascends, each step he takes covers a rungs of the ladder, and
when he descends, each step he takes covers b rungs of the ladder, where a and b are fixed
positive integers. By a sequence of ascending and descending steps he can climb from ground
level to the top rung of the ladder and come back to ground level again. Find, with proof,
the minimum value of n, expressed in terms of a and b.
2. Let A0 , B 0 , C 0 , D0 , E 0 , F 0 be midpoints of the sides AB, BC, CD, DE, EF, F A of convex hexagon
ABCDEF . Also let p denote the perimeter of ABCDEF and p0 denote the perimeter of
A0 B 0 C 0 D0 E 0 F 0 . If all inner angles of hexagon A0 B 0 C 0 D0 E 0 F 0 are equal, prove that
√
2· 3
p≥ · p1 .
3
When does equality hold?
3. Let X be a set of 2k elements and F a family of subsets of X each of cardinality k such that
each subset of X of cardinality k − 1 is contained in precisely one member of F. Show that
k + 1 is prime.
1
1 Solutions
1. (IMO 1990 Short list, #13)
We claim the minimum value of n is a + b − gcd(a, b). We will prove this first in the case
where a and b are relatively prime.
Claim: If the mathematician makes m moves beginning at some arbitrary rung and ends up
where he started, then m ≥ a + b (assuming a, b are relatively prime).
Proof of claim: If he ascends A times and descends B times, we must have Aa = Bb.
Since a and b are relatively prime, it follows that a|B and b|A, so B ≥ a, A ≥ b, and
m = A + B ≥ a + b.
Consider a ladder with n < a + b − 1 rungs, and suppose the mathematician can make a
sequence of moves and end up where he started. By the above claim, this number of moves
must be at least a + b > n + 1. However, there are only n + 1 different positions for the
mathematician, so this is impossible.
Next we show a ladder with n = a + b − 1 rungs is always possible. From any position, the
mathematician always has a legal move: if he is on rung 0, 1, . . . , b − 1, he can ascend, and if
he is on rung b, b + 1, . . . , a + b − 1, he can descend. Therefore, he can start on the ground
floor and continue to make moves until he returns to a position he has already been to. By
the original claim, he can only repeat a position after visiting all other a + b − 1 positions
first. Therefore, he must return to the ground floor before repeating another position, and he
must have visited the top rung during that time. Thus, n = a + b − 1 rungs is indeed always
possible, which completes our solution in the case where a, b are relatively prime.
Finally, suppose a and b have greatest common divisor g > 1. Then the mathematician can
only visit rungs with position a multiple of g. Ascending is equivalent to climbing up ag such
rungs, and descending is equivalent to going down gb such rungs. Since ag and gb are relatively
prime, we know the minimum possible n for which is possible to visit the top rung and return
to the start in this scenario satisfies ng = ag + gb − 1. Equivalently, n = a + b − gcd(a, b).
2
For equality to hold, ∠AA0 A0 = ∠BA0 B0 = 30◦ , and so on. Therefore, ABCDEF is equian-
gular and triangles A0 BB 0 , B 0 CC 0 , . . . , F AA0 are all isosceles, so AA0 = A0 B = BB 0 = . . . =
F F 0 = F 0 A, and ABCDEF is equilateral. Thus, equality holds if and only if ABCDEF is
regular.
Proof of claim: There are k + r elements not in S. Therefore, there are (k + r)(k + r −
1) · · · (k + 2) ways to add elements to S (in order) to make S size k − 1. Let the resulting set
be S 0 . By the property of F, there exists a unique element not in S 0 that we can add to S 0
so that the resulting set is in F. For each set F in F that contains S, there are r! ways to
add r elements to S to make the resulting set in F . Therefore, there are
(k + r)(k + r − 1) · · · (k + 2)
r!
sets from F that contains S. This proves the claim.
We now have that
(k + r)(k + r − 1) · · · (k + 2)
(1)
r!
is an integer for all 2 ≤ r ≤ k − 1. Let p be a prime factor of k + 1. Then p | k + 1 and p does
not divide any of the numbers k + 2, k + 3, · · · , k + p, which means
(k + p)(k + p − 1) · · · (k + 2)
(2)
p!
is not an integer. If k + 1 is not prime however, then 2 ≤ p ≤ k − 1, and (2) contradicts (1)
when r = p. We conclude that k + 1 must be prime.
3
IMO Winter Camp 2010 Warm-Up Problems 1
A1 Let x, y, z ∈ R be non-negative real numbers such that 0 ≤ x, y, z ≤ 1. Find the maximum possible value
of
x + y + z − xy − yz − zx.
Determine all triples (x, y, z) for which this maximum is attained.
A2 For any positive integer k, prove that
2k p
X 1
2k 2 + k < k 2 + j < 2k 2 + k + .
j=1
2
A4 P (x) is a polynomial of odd degree with real coefficients. Show that the equation P (P (x)) = 0 has at
least as many distinct real roots, as the equation P (x) = 0.
A5 Find all functions f : R 7→ R such that ∀x, y, z ∈ R, we have that if x3 + f (y) · x + f (z) = 0, then
f (x)3 + y · f (x) + z = 0.
Number Theory
N1 Let n be a positive integer. The sum of the positive divisors of n is s. Prove that the sum of the reciprocal
of these divisors is ns .
N2 A rational number x is written on a blackboard. In each step, you erase x and replace it with either x + 1
or − x1 . (If x = 0, you must choose x + 1). Prove that for any rational number p, if p currently appears on
the blackboard, then you can make 0 appear after a finite number of steps.
N3 Find all triples of positive integers (a, b, c) such that a2 + 2b+1 = 3c .
m 2 + n2 + 1
,
mn
for all positive integers m, n.
N5 Find all prime numbers p such that the following statement is true: there are exactly p ordered pairs of
integers (x, y) such that 0 ≤ x, y < p and y 2 ≡ x3 − x mod p.
1
IMO Winter Camp 2010 Warm-Up Problems 2
Geometry
G1 A circle C and a point P are given on the same plane. Given any point Q on the circumference of C, let
M be the midpoint of P Q. Find the locus of point M , i.e. find all possible locations for point M .
G2 Let A be a point outside of a circle C. Two lines pass through A, one intersecting C at B, C, with B closer
to A than C, and the other intersecting C at D, E, with D closer to A than E. The line passing through
D parallel to AC intersects C a second time at F and the line AF intersects C a second time at G. Let
M = EG ∩ AC. Prove that
1 1 1
= + .
|AM | |AB| |AC|
G3 Let ABC be a triangle and D be a point on side BC. The internal angle bisector of ∠ADB and that of
∠ACB intersect at P . The internal angle bisector of ∠ADC and that of ∠ABC intersect at Q. Let M
be the midpoint of P Q. Prove that |M A| < |M D|.
G4 A convex quadrilateral ABCD has |AD| = |CD| and ∠DAB = ∠ABC < 90◦ . The line through D and
the midpoint of BC intersects line AB at point E. Prove that ∠BEC = ∠DAC.
G5 Let ABC be a triangle with |AB| > |AC|. Let its incircle touch side BC at E. Let AE intersect this
incircle again at D. Let F be the second point on AE such that |CE| = |CF |. Let CF intersect BD at
G. Prove that |CF | = |F G|.
Combinatorics
C1 Find the number of subsets of {1, 2, · · · , 10} that contain its own size. For example, the set (1, 3, 6) has 3
elements and contains 3.
C2 A sequence of non-negative integers is defined by G0 = 0, G1 = 0 and Gn = Gn−1 + Gn−2 + 1 for every
n ≥ 2. Prove that for every positive integer m, there exists a positive integer a such that Ga , Ga+1 are
both divisible by m.
C3 A set S of ≥ 3 points in a plane has the property that no three points are collinear, and if A, B, C are
three distinct points in S, then the circumcentre of ∆ABC is also in S. Prove that S is infinite.
C4 Let n, k be positive even integers. A survey was done on n people where on each of k days, each person
was asked whether he/she was happy on that day and answered either ”yes” or ”no”. It turned out that
on any two distinct days, exactly half of the people gave different answers on the two days. Prove that
there were at most n − nk people who answered ”yes” the same number of times he/she answered ”no”
over the k days.
C5 There are n ≥ 5 people in a room, where each pair is classified as friends or strangers. No three people are
mutually friends. There also exist an odd number of people P1 , · · · , Pm such that Pi is friends with Pi+1
for all i ∈ {1, · · · , m}, where the indices are taken modulo m. Prove that there exists one person who is
friends with at most 2n/5 people.
2
IMO Winter Camp 2010 Warm-Up Problems 3
A1 Let x, y, z ∈ R be non-negative real numbers such that 0 ≤ x, y, z ≤ 1. Find the maximum possible value
of
x + y + z − xy − yz − zx.
Determine all triples (x, y, z) for which this maximum is attained.
Solution: The maximum possible value is 1 and is attained when (x, y, z) = (1, t, 0) for any 0 ≤ t ≤ 1, or
any of its permutation solutions.
Note that
x + y + z − xy − yz − zx = −(1 − x)(1 − y)(1 − z) + 1 − xyz ≤ 1.
Equality holds when at least one of x, y, z is equal to 1 and at least one of x, y, z is equal to 0. Clearly,
this is possible. Therefore, the maximum possible value of x + y + z − xy − yz − zx is 1 and is attained
when (x, y, z) = (1, t, 0) for any 0 ≤ t ≤ 1, and any of its permutation solutions.
Comments: When the expressions x + y + z, xy + yz + zx, xyz appear, you should always consider poly-
nomials of degree three and/or terms of the form (c − x)(c − y)(c − z) for some constant c. This may even
help you factor terms in other similar problems.
Note that
p j
k2 + j − k = p ,
k2 + j + k
and
j j j
<p < .
2k + 1 k2 + j + k 2k
Therefore,
2k 2k 2k
X j X j X j
< p < .
j=1
2k + 1 j=1 k 2 + j + k j=1
2k
3
IMO Winter Camp 2010 Warm-Up Problems 4
Hence,
2k
2k(2k + 1) X j 2k(2k + 1)
< p < .
2(2k + 1) j=1
2
k +j+k 2(2k)
j
p
Since √ = k 2 + j − k, we conclude that
k2 +j+k
2k p
X 1
k< k2 + j − k < k + .
j=1
2
Comments: The appearance of the square root suggests that p you rationalize the pnumerator of some
you to consider the term k 2 + j − k, since k < k 2 + j < k + 1 for
expression. It should be natural for p
j j
1 ≤ j ≤ 2k. Bounding the term j/( k 2 + j + k) between 2k+1 and 2k should also be natural since you
j
are looking for rational lower and upper bounds of √ 2 .
k +j+k
as desired.
Solution 2: By adding the terms on the left-hand side and cross multiplying, the inequality becomes
equivalent to
3 + (ab + bc + ca) ≥ abc(a + b + c) + 3a2 b2 c2 .
By homogenizing (i.e. using the condition a2 + b2 + c2 = 3 to make every term have the same degree), we
have
(a2 + b2 + c2 )3 (a2 + b2 + c2 )2 (ab + bc + ca) (a2 + b2 + c2 )abc(a + b + c)
3· + ≥ + 3a2 b2 c2 .
27 9 3
By clearing denominators, this inequality becomes equivalent to
(a2 + b2 + c2 )3 + (a2 + b2 + c2 )2 (ab + bc + ca) ≥ 3(a2 + b2 + c2 )abc(a + b + c) + 27a2 b2 c2 .
For any non-negative real numbers x, y, z, let
X
[x, y, z] = ax by cz .
sym
4
IMO Winter Camp 2010 Warm-Up Problems 5
We recall Muirhead’s Majorization inequality, which states that if x, y, z, u, v, w are non-negative real
numbers such that x ≥ y ≥ z, u ≥ v ≥ w, x + y + z = u + v + w, x ≥ u, x + y ≥ u + v, then
[x, y, z] ≥ [u, v, w]. [x, y, z] is said to majorize [u, v, w]. Note that
1
(a2 + b2 + c2 )3 = [6, 0, 0] + 3[4, 2, 0] + [2, 2, 2],
2
1
(a2 + b2 + c2 )2 (ab + bc + ca) = [5, 1, 0] + [4, 1, 1] + [3, 3, 0] + 2[3, 2, 1],
2
3
3(a2 + b2 + c2 )abc(a + b + c) = [4, 1, 1] + 3[3, 2, 1],
2
9
27a2 b2 c2 = [2, 2, 2].
2
Hence, after cancellation of terms, the previous inequality becomes equivalent to
1 7
[6, 0, 0] + 3[4, 2, 0] + [5, 1, 0] + [3, 3, 0] ≥ [4, 1, 1] + [3, 2, 1] + [2, 2, 2].
2 2
By Muirhead’s inequality, [5, 1, 0] ≥ [4, 1, 1], [3, 3, 0] ≥ [3, 2, 1], 21 [6, 0, 0] + 3[4, 2, 0] ≥ 21 [2, 2, 2] + 3[2, 2, 2] =
7
2 [2, 2, 2]. Hence, the inequality holds.
Comments: You should always be able to solve any three-variable symmetric inequality, that can be
homogenized, has equality case a = b = c and/or a = b, c = 0 and its permutation solutions, and whose
variables are non-negative real numbers, using Muirhead’s and Schur’s inequality. Muirhead’s inequality
states that [x, y, z] ≥ [u, v, w] whenever [x, y, z] majorizes [u, v, w]. An equivalent version of Schur’s in-
equality is [x + 2, 0, 0] + [x, 2, 0] ≥ 2[x + 1, 1, 1]. For proofs of these inequalities, see Chapter 3.2 of [2].
You may be thinking that solving problems in this manner is not elegant and can be considered ugly.
But the truth of the matter is that a lot of mathematics is initially done by getting your hands dirty and
working with cumbersome expressions. Muirhead’s and Schur’s inequality is in the arsenal of every top
math olympian in the world and should always be used as a last resort to solve inequalities of this type.
To test your ability to use these inequalities, do try the following problem.
A4 P (x) is a polynomial of odd degree with real coefficients. Show that the equation P (P (x)) = 0 has at
least as many distinct real roots, as the equation P (x) = 0.
Solution: Let x1 , · · · , xn be all of the distinct roots of the equation P (x) = 0. We want to show that
P (P (x)) = 0 has at least n distinct real roots.
For each i = 1, · · · , n, consider the equation P (x) = xi . It has at least one real root ai since P is a
polynomial of odd degree. Now, for i, j ∈ {1, 2, · · · , n}, i 6= j, if ai = aj , then P (ai ) = P (aj ). Therefore,
5
IMO Winter Camp 2010 Warm-Up Problems 6
xi = xj , which is impossible since x1 , · · · , xn are pairwise distinct. Therefore, the ai ’s are also pairwise
distinct. But each ai is a solution to P (P (x)) = 0, since P (P (ai )) = P (xi ) = 0. Hence the equation
P (P (x)) = 0 has at least n distinct real roots a1 , a2 , ..., an . The result follows.
Comments: When polynomials in question have certain properties such as having odd degree, a question
you should ask yourself is what are the differences between polynomials of odd degree and those of even
degree? The answer will often give you clues as to how to approach a problem. If you have not used the
fact that the polynomial has odd degree, you may not be approaching the problem in a correct way.
A5 Find all functions f : R 7→ R such that ∀x, y, z ∈ R, we have if x3 + f (y) · x + f (z) = 0, then
f (x)3 + y · f (x) + z = 0.
Solution: We will call x3 + f (y) · x + f (z) = 0 relation (A) and f (x)3 + y · f (x) + z = 0 relation (B).
We first prove that f is surjective. Let m be an arbitrary real number. We will show that f (x) = m
for some x ∈ R. Since (A) is a cubic polynomial in x, given any fixed y, z ∈ R, there exists x ∈ R such
that (x, y, z) satisfies (A). We choose y = 1, z = −m3 − m. We can choose x ∈ R such that (x, y, z)
satisfies (A). Therefore, (x, y, z) satisfies (B), i.e. f (x)3 + f (x) − (m3 + m) = 0. This implies that
(f (x) − m)(f (x)2 + m · f (x) + m2 + 1) = 0. The latter factor, as a quadratic equation in f (x), has negative
discriminant, hence cannot be zero. Therefore, f (x) = m. This proves that f is surjective.
We now show that f (x) = 0 if and only if x = 0. Since f is surjective, f (r) = 0 for some r ∈ R.
Note that (x, y, z) = (0, a, r) satisfies (A) for any choice of a ∈ R. Hence, (0, a, r) satisfies (B), i.e.
f (0)3 + af (0) + r = 0 holds for all a ∈ R. Therefore, af (0) = r − f (0)3 for all a ∈ R. The right hand side
is a constant. Therefore, f (0) = 0. This implies r = 0. Hence, f (x) = 0 if and only if x = 0.
We now show that f is injective. Assume f (a) = f (b) for some a, b ∈ R. Take any z 6= 0. Then f (z) 6= 0.
There exists x for which x3 + f (a)x + f (z) = 0. Note that x 6= 0, since f (z) 6= 0. Then (x, a, z) and
(x, b, z) satisfy (A), since f (a) = f (b) and x3 + f (a)x + f (z) = 0. Therefore, (x, a, z), (x, b, z) satisfy (B),
i.e. x3 + af (x) + z = x3 + bf (x) + z = 0, hence (a − b)f (x) = 0. But x 6= 0, implying f (x) 6= 0. Hence
a = b. Therefore, f is injective.
We now show that (A) and (B) are equivalent, i.e. (x, y, z) satisfies (A) if and only if (x, y, z) satisfies (B).
From the condition given in the problem, (x, y, z) satisfies (A) implies (x, y, z) satisfies (B). To prove the
converse, let x, y, z ∈ R be a triple such that (x, y, z) satisfies (B), i.e. we have have f (x)3 +y ·f (x)+z = 0.
Let z 0 ∈ R such that (x, y, z 0 ) satisfies (A). This is possible since f is surjective. Therefore, (x, y, z 0 ) sat-
isfies (B), i.e. z 0 = −f (x)3 − yf (x). But this term is also equal to z. Therefore, z 0 = z, implying (x, y, z)
satisfies (A). Therefore, (A) and (B) are equivalent.
Take (x, y, z) such that x = 1, f (y) = −1, z = 0. Choosing such a y is possible since f is surjective. Then
(x, y, z) satisfies (A). Hence, f (1)3 + yf (1) = 0. Therefore, y = −f (1)2 , which implies f (−f (1)2 ) = −1,
by the definition of y. Take (x, y, z) such that x = 1, y = 0, f (z) = −1. Then (x, y, z) satisfies (A). Hence,
f (1)3 + z = 0, implying z = −(f (1))3 . Therefore, f (−f (1)3 ) = −1. But f is injective and f (z) = 1.
6
IMO Winter Camp 2010 Warm-Up Problems 7
Therefore, −f (1)2 = −f (1)3 , or equivalently, f (1)2 (f (1) − 1) = 0. Since f is injective, f (1) 6= f (0) = 0.
Therefore, f (1) = 1.
Take x = 1, z = −y − 1. Since f (1) = 1, (x, y, z) satisfies (B). Since (A) and (B) are equivalent, (x, y, z)
satisfies (A), i.e.
f (−y − 1) = −f (y) − 1, ∀y ∈ R. (1)
Take x = −1, z = y + 1. Since f (−1) = −1, (x, y, z) satisfies (B), which implies (x, y, z) satisfies (A), i.e.
f (y + 1) = f (y) + 1, ∀y ∈ R. Inductively, we can show that
f (y + k) = f (y) + k, ∀y ∈ R, k ∈ Z. (3)
f (x) = x, ∀x ∈ Z. (4)
From (1) and (3), we get that f (−y − 1) = −f (y) − 1 = −(f (y + 1) − 1) − 1 = −f (y + 1). Therefore,
Let x, y be any fixed real numbers and let z = −f (x)3 − yf (x). Then (x, y, z) satisfies (A). Hence, (x, y, z)
satisfies (B), i.e. x3 + f (y)x + f (z) = 0. Therefore, f (z) = −x3 − f (y)x. By the definition of z, we now
have
f (−f (x)3 − yf (x)) = −x3 − f (y)x, ∀x, y ∈ R. (6)
If x is an integer, then by (4), f (x) and x3 are also integers. Therefore, by (3) and (7), we have x3 +xf (y) =
f (f (x)3 + yf (x)) = f (x)3 + f (yf (x)) = x3 + f (yx). (We substitute y ← yf (x) and k ← f (x)3 into (3) for
the second last assertion and the fact that f (x) = x for all x ∈ Z, for the last assertion.) Therefore,
Substituting y = −f (x)2 into (7) gives us 0 = f (0) = −x3 − f (−f (x)2 )x. Therefore, f (−f (x)2 ) = −x2 .
By (5), we have
f (f (x)2 ) = x2 , ∀x ∈ R. (11)
7
IMO Winter Camp 2010 Warm-Up Problems 8
Let c > 0 be an arbitrary real number. Then since f is surjective, there exists d ∈ R such that c = f (d)2 .
Since c 6= 0, f (0) = 0 and f is injective, d 6= 0. Therefore, by (11), we have f (c) = f (f (d)2 ) = d2 > 0.
Hence,
c > 0 ⇒ f (c) > 0, ∀c ∈ R (12)
Assume for some x, we have f (x) < x. Then there exists a rational number r for which f (x) < r < x.
Then by (12), we have f (x − r) > 0 and by (9), we have r > f (x) = f (r + (x − r)) = r + f (x − r) > r.
This is a contradiction. So we cannot have f (x) < x. Similarly, we cannot have f (x) > x. So the only
possibility for f is f (x) = x for all x ∈ R. It is easy to check f (x) = x satisfies the conditions of the
problem and so is the only solution.
Comments: This problem is difficult in the sense that the solution requires many steps. However, every
step demonstrates an important technique that you should know to solve functional equations.
The first thing you should almost always do is to find solutions to the functional equation by inspection.
The most common ones are f (x) = c for some constant c, f (x) = x + c for some constant c, quadratic
equations and f (x) = x1d for some integer d.
You next want to establish properties of the functional equation, i.e. what is f (0)? If f (x) = 0, does it
mean x = 0? Is f one-to-one? Is f surjective? (Don’t do the latter two questions if one of the solutions
to the functional equation is, say, f (x) = x2 . You already know that this function is neither one-to-one
or surjective over the reals.)
Another technique is solving the problem over the rationals first, and then use detailed (and careful)
continuity arguments to solve the problem over all of the reals.
8
IMO Winter Camp 2010 Warm-Up Problems 9
Number Theory
N1 Let n be a positive integer. The sum of the positive divisors of n is s. Prove that the sum of the reciprocal
of these divisors is ns .
Solution: Let d1 , · · · , dt be the positive divisors of n with 1 = d1 < d2 < · · · < dt = n. Note that
n = di dt+1−i for each i. Therefore, d1i = dt+1−i
n . Hence,
t t t t
X 1 X dt+1−i 1X 1X s
= = dt+1−i = di = .
d
i=1 i i=1
n n i=1 n i=1 n
Source: Original
Comments: Always remember that the positive divisors√of n come in pairs as shown in this solution.
The exception is when n is a perfect square. In this case, n is paired with itself.
N2 A rational number x is written on a blackboard. In each step, you erase x and replace it with either x + 1
or − x1 . (If x = 0, you must replace x with x + 1). Prove that for any rational number p, if p currently
appears on the blackboard, then you can make 0 appear in a finite number of steps.
Comments: This is motivated by the Euclidean algorithm and using a general technique in number
theory called descent. A strategy for this type of problem generally involves solving this problem for
certain values of p with small numerator and denominator. It should be pretty natural to try to lower the
numerator and/or denominator of the number on the board. I will leave as an exercise that starting at 0,
you can make p appear after a finite number of steps.
Another note is that we used a variant of the division algorithm. Instead of writing n = qm + r with
0 ≤ r < m, which is the more traditional method, we wrote it as n = qm − r with 0 ≤ r < m to make the
problem easier. The other useful variant is writing n as n = qm + r where −m/2 < r ≤ m/2.
9
IMO Winter Camp 2010 Warm-Up Problems 10
Since 2b+1 is even and 3c is odd, a2 is odd. Therefore, a2 ≡ 1 mod 4. Since 2b+1 ≡ 0 mod 4, 3c ≡ 1 mod
4. This implies 3c ≡ 1 mod 4. Hence, (−1)c ≡ 1 mod 4. We conclude that c is even. Therefore,
c c
2b+1 = (3 2 − a)(3 2 + a).
c c
Hence, 3 2 − a and 3 2 + a are both powers of 2. Since a is odd, both of these terms are even, so none
c c
of these terms are 1. Hence, we can let s, t be positive integers such that 3 2 − a = 2s and 3 2 + a = 2t .
t s t−1 s−1
Subtracting the former from the latter yields 2a = 2 − 2 . Hence, a = 2 − 2 . Since a is odd, s = 1.
c c
Therefore, a = 2t−1 − 1. Substituting this into 3 2 + a = 2t yields 3 2 + 2t−1 − 1 = 2t , or equivalently,
c
3 2 − 1 = 2t−1 .
c c
If t = 1, then 3 2 = 2, which has no integer solutions. If t = 2, then 3 2 − 1 = 2. Hence, c = 2,
a = 2t−1 − 1 = 22−1 − 1 = 1. Therefore, 1 + 2b+1 = 32 , implying b = 2. Hence, (1, 2, 2) is a solution to the
c
equation and is easily verified to be a solution. Otherwise, t > 2, implying 4 | 2t−1 . Hence, 3 2 ≡ 1 mod
c
4, which implies (−1) 2 ≡ 1 mod 4. Hence, 2c is even. Therefore,
c c
(3 4 − 1)(3 4 + 1) = 2t−1 .
c c
From this, we have that both 3 4 − 1, 3 4 + 1 are powers of 2 and are consecutive even integers. This is
c
only possible when the smaller integer is 2 and the larger integer is 4. Therefore, 3 4 − 1 = 2, implying
c = 4. Therefore, 32 − 1 = 2t−1 = a + 1, or equivalently, a = 7. Finally, 72 + 2b+1 = 34 , implying b = 4.
Therefore, (7, 4, 4) is a solution to the equation and is easily verified to be a solution.
Comments: One technique to solve Diophantine equations is consider reduce everything modulo a cer-
tain positive integer. The existence of a square and a power of 2 suggests that mod 4 and mod 8 are
possibilities. Problems like this test your comfort level with divisibility problems. You should always try
to find as many solutions by inspection as you can as a first step. (However, I do not expect you to initially
find the solution (7, 4, 4).)
Exercise: Let m, n be positive integers. Find the minimum possible value of |12m − 5n |.
m2 + n2 + 1
f (m, n) = .
mn
10
IMO Winter Camp 2010 Warm-Up Problems 11
We next consider the case m = n. If (m, n) ∈ S and m = n, then m2 divides 2m2 + 1. Therefore, m2 | 1.
Therefore, m = 1. Hence, if (m, m) ∈ S, then m = 1. This also yields that f (m, n) = 3.
m2 +n2 +1
Now, suppose (m, n) ∈ S with 1 < m < n. Suppose k = mn . I claim there exists 0 < n0 < m such
that (m, n0 ) ∈ S. Rewriting the equation gives us
n2 − kmn + (m2 + 1) = 0.
Hence, n is an integer root of the equation x2 − kmx + (m2 + 1) = 0. Let n0 be the other root. Since
n + n0 = km, which is an integer, n0 is an integer. Since nn0 = m2 + 1 > 0 and n > 0, n0 > 0. Note that
f (m, n) = f (m, n0 ), by rewriting the quadratic equation back to the form f (m, n0 ). Therefore, (m, n0 ) ∈ S.
I claim that n0 < m. Suppose on the contrary that n0 ≥ m. Since (m, n0 ) ∈ S and n, m > 1, n0 6= m by
the argument in the case m = n. Suppose n0 > m. Let n = m + a, n0 = m + b for some positive integers
a, b. Then nn0 = (m + a)(m + b) = m2 + 1, since the product of the roots of x2 − kmx + (m2 + 1) is m2 + 1.
Hence, m(a + b) + ab = 1, which is impossible since a, b ≥ 1. Therefore, n0 < m. This proves the claim.
Hence, (m, n0 ) ∈ S, which implies (n0 , m) ∈ S. Note that k = f (m, n) = f (m, n0 ) = f (n0 , m). If n0 = 1,
then from an earlier argument, k = 3. Otherwise, we repeat this process for 1 < n0 < m. Every time we
perform the previous paragraph, min{m, n} strictly decreases and remains positive and f (m, n) remains
constant. Hence, eventually, at least one of m, n = 1. By an earlier argument, this results in k = 3. Since
f (m, n) remains constant at every step, we conclude that f (m, n) = 3. This completes the problem.
Comments: The technique used in this problem is called Vieta-jumping or descent. It is extremely
helpful in solving Diophantine equations with quadratic terms. By letting S be a solution set, you can
claim statements of the form if (a, b) ∈ S, then (c, d) ∈ S for certain choices of a, b, c, d. By doing so, you
can create a series of solutions for which min{m, n} is decreasing. Try to solve the following problem from
the 1988 International Mathematical Olympiad using Vieta-jumping.
Exercise: Suppose m, n are positive integers such that mn + 1 divides m2 + n2 . Prove that
m 2 + n2
mn + 1
is a perfect square.
N5 Find all prime numbers p such that the following statement is true: there are exactly p ordered pairs of
integers (x, y) such that 0 ≤ x, y < p and y 2 ≡ x3 − x mod p.
Solution: The answers are p = 2 and all primes p such that p ≡ 3 mod 4.
11
IMO Winter Camp 2010 Warm-Up Problems 12
If p = 2, then we can check that the only pairs that work are (0, 0) and (1, 0). There are exactly two
solutions. Hence p = 2 is an answer.
We may assume now that p is odd. For a ∈ {0, 1, · · · , p − 1}, we call a a quadratic residue modulo p if
and only if n2 ≡ a mod p for some integer n. Otherwise, we call a a quadratic non-residue. We recall
that the product of two quadratic residues is a quadratic residue, and the product of two quadratic non-
residues is a quadratic residue. Also, the product of a quadratic residue and a quadratic non-residue, is a
quadratic non-residue. Also recall that −1 is a quadratic residue of p if and only if p ≡ 1 mod 4 or p = 2. (*)
Note that (−1, 0), (0, 0), (1, 0) are the only solutions involving y = 0 and x ∈ {−1, 0, 1}. There are three
such solutions. We will henceforth only consider the cases when x ∈ {2, · · · , p − 2} and y 6= 0.
If p ≡ 1 mod 4, then note that x3 − x = −1 · ((−x)3 − (−x)). By the arguments made in (*), since −1 is
a quadratic residue mod 4, x is solvable if and only if −x is solvable. For each x ∈ {2, · · · , p − 2} which is
solvable, there are exactly two values of y such that (x, y) is a solution to the original equation. This is
because (x, y) is a solution if and only if (x, −y) is a solution. Note that x 6≡ −x and y 6≡ −y mod p since
p is odd and y 6= 0. Hence the number of values of x ∈ {2, · · · , p − 2} which is solvable is even and each
solvable x yields two values of y. Hence, the number of ordered pair solutions with x ∈ {2, · · · , p − 2} is
divisible by 4. Including the three aforementioned solutions, the number of solutions (x, y) is congruent
to 3 mod 4. But p ≡ 1 mod 4. Therefore, the number of solutions (x, y) cannot equal to p.
Comments: The facts in the paragraph marked (*) are important topics to learn for number theory,
especially the fact that n2 ≡ −1 mod p has a solution if and only if p = 2 or p ≡ 1 mod 4. Please read
Theorem 4.9 and Chapter 7 of [3] for more details.
Another important point is that the very least you should be able to do for this problem, even without
knowledge of quadratic residues and of fact (*), is to conjecture your answer for p by solving this problem
for small primes p, say up to p = 17. The motivation behind this given solution arises from the work you do
for small values of p. Only then would you be able to consider the differences between primes 1 and 3 mod
4. The least that you should be able to write, is something along the lines of ”I conjecture that the solution
to this problem are p = 2 and all primes p ≡ 3 mod 4”. Of course, now that you have read and under-
stood this solution, I will expect much more than this. :) Using fact (*), you can try the following exercise.
Exercise: Prove that there are no pairs of positive integers a, b such that 4ab − a − b is a perfect square.
12
IMO Winter Camp 2010 Warm-Up Problems 13
Geometry
G1 A circle C and a point P are given on the same plane. Given any point Q on the circumference of C, let
M be the midpoint of P Q. Find the locus of point M , i.e. find all possible locations for point M .
Solution 1: The locus of M is exactly the dilation of C about point P with ratio 12 . The image is therefore
the circle with radius half of that of C whose centre is the midpoint of P and O, where O is the centre of C.
|P M | |P N |
Solution 2: Let O be the centre of C, r the radius of C and N the midpoint of P O. Since |M Q| = |N O| = 1,
|M N | 1 r r
we have |QO| = 2. Hence, |M N | = 2. Therefore, M lies on the circle with radius 2 centered at N .
Conversely, let M be any point on the circle centered at N with radius 2r . Let Q be on the circumference
of C such that QO is parallel to M N . Then |M N| |P N | 1
|QO| = |P O| = 2 , which implies P, M, Q are collinear and
|P M |
|P Q| = 12 . Therefore, M is the midpoint of P Q.
Source: Original
G2 Let A be a point outside of a circle C. Two lines pass through A, one intersecting C at B, C, with B closer
to A than C, and the other intersecting C at D, E, with D closer to A than E. The line passing through
D parallel to AC intersects C a second time at F and the line AF intersects C a second time at G. Let
M = EG ∩ AC. Prove that
1 1 1
= + .
|AM | |AB| |AC|
Solution: Note that ∠GAM = ∠F AM = ∠AF D = ∠GF D = ∠GED, the latter assertion following
|M A| |M E|
from that DEF G is cyclic. Therefore, ∆GAM ∼ ∆AEM . Hence, we have |M G| = |M A| , which implies
2
|M A| = |M G| · |M E| = |M B| · |M C|. The assertion follows from that DEF G is cyclic. We shall call
this property (*).
Note that M lies on the segment AB. Therefore, |AB| = |M A| + |M B| and |AC| = |M A| + |M C|. Hence,
This equality holds if and only if (|M A| + |M B|)(|M A| + |M C|) = 2|M A|2 + |M A||M B| + |M A||M C| ⇔
|M B| · |M C| = |M A|2 , which is true by property (*).
Comments: If you are not familiar with Power of a Point, please read Chapter 6 of [1]. It is an important
and powerful technique to solve certain geometry problems, especially those involving circles and tangent
lines. Power of a Point is the motivation for this solution.
13
IMO Winter Camp 2010 Warm-Up Problems 14
G3 Let ABC be a triangle and D be a point on side BC. The internal angle bisector of ∠ADB and that of
∠ACB intersect at P . The internal angle bisector of ∠ADC and that of ∠ABC intersect at Q. Let M
be the midpoint of P Q. Prove that |M A| < |M D|.
Solution: Note that P is the excentre of 4ACD opposite C, and Q is the excentre of 4ABD opposite
B. Therefore, ∠P AD = 180−∠CAD
2 and ∠QAD = 180−∠BAD
2 .
Note also that ∠QDP = ∠QDA + ∠ADP = 12 (∠BDA + ∠CDA) = 90o . Therefore, |M P | = |M Q| =
|M D|. Consider the circle centred at M passing through P, Q, D. To prove that |M A| < |M D|, it suffices
to show that A is in the interior of the circle, i.e. ∠QAP is obtuse. But
180 − ∠BAD 180 − ∠CAD ∠BAD + ∠CAD
∠QAP = ∠QAD + ∠P AD = + = 180 − = 180 − ∠BAC
2 2 2
and this angle is greater than 90o (and less than 180o ). Therefore, |M A| < |M D|.
Comments: It is important to observe excentres in a diagram. Their existence, along with incentres,
often simplifies problems involving angle bisectors.
G4 A convex quadrilateral ABCD has |AD| = |CD| and ∠DAB = ∠ABC < 90◦ . The line through D and
the midpoint of BC intersects line AB in point E. Prove that ∠BEC = ∠DAC.
Solution: Let F be on side AB such that |DA| = |DF |, which is also equal to |DC|. Since ∠AF D =
∠DAF = ∠DAB = ∠ABC, DF ||BC. Extend DF to meet EC at G. Since DF ||BC and ED passes
through the midpoint of BC, D is the midpoint of F G. Hence, |DA| = |DC| = |DF | = |DG|.
Hence, ACGF is cyclic. Then ∠F AC = ∠F GC = ∠DGC = ∠BCE. Since ∠DAB = ∠ABC,
∠DAC = ∠DAB − ∠F AC = ∠ABC − ∠BCE = ∠BEC, as desired.
Comments: Construction problems in geometry are fun. Of course, there are solutions that does not
involve extra constructions given in this solution. However, the construction of point F on AB is quite nat-
ural given that |AD| = |CD| and ∠DAB = ∠ABC. You will be given three equal lengths in AD, CD, F D
and that F D is parallel to AB. Many new properties of the diagram arise from this one point. Wow! :)
G5 Let ABC be a triangle with |AB| > |AC|. Let its incircle touch side BC at E. Let AE intersect this
incircle again at D. Let F be the second point on AE such that |CE| = |CF |. Let CF intersect BD at
G. Prove that |CF | = |F G|.
Solution: Let the line tangent to the incircle at D intersect BC (extended) at P . Since |P E| = |P D|
and |CE| = |CF |, CF ||P D.
14
IMO Winter Camp 2010 Warm-Up Problems 15
|CP | |CE|
= .
|P B| |EB|
Note that the polar of P with respect to the incircle is line DE, which also contains A. Therefore, the
polar of A contains P . Hence, the polar of A is XY , where X, Y are points where the incircle touches
AC, AB, respectively. Hence, P lies on XY . Since |AX| = |AY |, |BY | = |BE|, |CE| = |CX|, we have
that
AY BE CX
· · = 1. (13)
Y B EC XA
Hence, AE, BX, CY are concurrent by Ceva’s Theorem. Since X, Y, P are collinear, by Menelaus’ Theorem
on ∆ABC, we have
AY BP CX
· · = −1. (14)
Y B P C XA
Comparing equations (13) and (14), yields
|CP | |CE|
= ,
|P B| |EB|
as desired.
Comments: Ceva’s and Menealus’ Theorem are essential pieces of knowledge for proving concurrency
and collinearity involving triangles. Please learn these theorems if you haven’t done so already.
Poles and polars have come to the rescue. Those familiar with poles and polars and harmonic conjugates
will realize that (E, P ) divides (B, C) harmonically. To learn more about these powerful techniques, please
consult Chapter 10 and 11 of [1].
15
IMO Winter Camp 2010 Warm-Up Problems 16
Combinatorics
C1 Find the number of subsets of {1, 2, · · · , 10} that contain its own size. For example, the set (1, 3, 6) has 3
elements and contains 3.
For each k ∈ {0, 1, · · · , 10}, we want to find how many elements there are of size k. Note that k 6= 0
since 0 6∈ {1, · · · , 10}. Every
subset of size k must contain k. The other k − 1 elements can be anything.
9
Therefore, there are k−1 subsets of size k that contains k. Therefore, the number of subsets that contain
its own size is
10 X 9
X 9 9
= = 29 = 512.
k−1 k
k=1 k=0
Source: Original
Solution: Consider each pair (Gt , Gt+1 ) modulo m. Since there are at m2 pairs of integers (x, y) mod-
ulo m, the sequence (Gt , Gt+1 ) modulo m eventually repeats, by Pigeonhole Principle. It suffices to
prove that (Ga , Ga+1 ) = (0, 0) = (G0 , G1 ) modulo m, for some positive integer a. Suppose the sequence
{(Gt , Gt+1 } repeats the first time at index t = a. Then (Ga , Ga+1 ) = (Gj , Gj+1 ) modulo m for some
0 ≤ j < t. If j = 0, then (Ga , Ga+1 ) = (0, 0), implying Ga , Ga+1 are both divisible by m and we are
done. Otherwise, note that the pair that appears before (x, y) is (y, x + y − 1). In other words, given a
pair of consecutive terms in the sequence, the previous pair of consecutive terms is determined. Hence,
(Ga , Ga+1 ) = (Gj , Gj+1 ) implies (Gj−1 , Gj ) = (Ga−1 , Ga ), contradicting that the sequence repeats the
first time at index a. Therefore, Ga , Ga+1 are both divisible by m.
Comments: Pigeonhole Principle is a powerful tool to solve certain existence type of combinatorics prob-
lems. It is an art to determine what are the pigeons and what are the holes.
C3 A set S of ≥ 3 points in a plane has the property that no three points are collinear, and if A, B, C are
three distinct points in S, then the circumcentre of ∆ABC is also in S. Prove that S is infinite.
Solution: Suppose on the contrary that S is finite. Consider the convex hull P of S, i.e. the smallest poly-
gon containing all of S (on the polygon’s boundary and interior). Since |S| ≥ 3, P contains at least three
vertices. Let AB be a side of P and C be any other vertex of P. Note that P is contained in the halfplane
of AB containing C and there are no points in S in the halfplane not containing C. If ∠ACB > 90◦ , then
the circumcircle of ∆ACB lies in the halfplane of AB not containing C, which does not contain any point
16
IMO Winter Camp 2010 Warm-Up Problems 17
in S. Hence, the circumcircle of ∆ACB is not in S. This contradicts the assumption given in the problem.
Otherwise, ∠ACB is acute. We let O1 be the circumcentre of ∆ACB and recursively, define Oi+1 to be
the circumcentre of ∆AOi B. Hence, Oi ∈ S for all i ≥ 1. Hence, ∆AO1 B = 2∠ACB. We choose the
smallest index k such that ∠AOk B ≥ 90◦ . Therefore, ∠AOk B = 2∠AOk−1 B. Since ∠AOk−1 B ≤ 90◦ by
our choice of k, ∠AOk−1 B < 180◦ . If ∠AOk B = 90◦ , then Ok+1 lies on AB, contradicting the fact that
no three points in S are collinear. (Note that Oi 6= A, B for any i, since the circumcentre of a triangle
cannot be any of the vertices of the triangle.) Otherwise, 90 < ∠AOk B < 180◦ , which implies Ok+1 lies
on the halfplane of AB not containing Ok . Hence, Ok+1 cannot lie in S, but this contradicts the fact that
Oi ∈ S for all i ≥ 1. Hence, S cannot be finite, implying S is infinite.
Source: Unknown
Comments: When you are given a finite set of points in the plane in a problem, you should always
consider the convex hull of the set. It is likely to simplify the problem. Try the following problem from
the 1999 International Mathematical Olympiad by considering the convex hull.
Exercise: Find all finite set of points S such that for every pair of points A, B ∈ S, the perpendicular
bisector of A and B is an axis of symmetry for S.
C4 Let n, k be positive even integers. A survey was done on n people where on each of k days, each person was
asked whether he/she was happy on that day and answered either ”yes” or ”no”. It turned out that on any
two distinct days, exactly half of the people gave different answers on the two days. Prove that there were
at most n− nk people who answered ”yes” the same number of times he/she answered ”no” over the k days.
Solution: Consider a grid with k rows and n columns. Label the days from 1 to k and the people from
1 to n. We place a 1 in the ith row and j th column if person j is happy on day i and a 0 otherwise. Since
on any two days, exactly half of the people gave different answers on the two days, we have that for any
two rows, exactly half of the columns differ. Hence, the number of pairs of squares in the same column
that differ, is k2 · n2 . For each person that answered yes the same number of times as he/she answered no,
the number of pairs of squares in the person’s column that differ is ( k2 )2 . Hence, the maximum number of
people that could have answered yes the same number of times as he/she answered no, is
k n
nk(k−1)
2 2 4 n(k − 1) n
= k2
= =n− ,
( k2 )2 4
k k
as desired.
Comments: An important key to solving combinatorics problem is find out what you are suppose to be
counting. If you can count a certain object in two different ways, you can yield some extremely neat and
powerful identities and relationships. If you take the time to solve enough combinatorics problems, it will
become more natural as to what objects you should be counting. Use this technique to solve the following
problem from the 1998 International Mathematical Olympiad
17
IMO Winter Camp 2010 Warm-Up Problems 18
Exercise: In a contest, there are m candidates and n judges, where n ≥ 3 is an odd integer. Each
candidate is evaluated by each judge as either pass or fail. Suppose that each pair of judges agrees on at
most k candidates. Prove that
k n−1
≥ .
m 2n
C5 There are n ≥ 5 people in a room, where each pair is classified as friends or strangers. No three people are
mutually friends. There also exist an odd number of people P1 , · · · , Pm such that Pi is friends with Pi+1
for all i ∈ {1, · · · , m}, where the indices are taken modulo m. Prove that there exists one person who is
friends with at most 2n/5 people.
Solution: Given a positive integer m ≥ 3, we will call an ordered set of m people P1 , · · · , Pm a cycle if
Pi is friends with Pi+1 for all i ∈ {1, · · · , m}, where the indices are taken modulo m. If m is odd, we call
a cycle an odd cycle. Given the condition in the problem, an odd cycle exists. Consider the odd cycle in
the room P1 , · · · , Pg with the smallest number of people. Since no three people who are mutually friends,
the odd cycle must contain at least five people, i.e. g ≥ 5
I claim that every person outside of the cycle is friends with at most two people inside the cycle. Suppose
there exists a person A outside of the cycle who is friends with three people in the cycle, say Pi , Pj , Pk
with i < j < k. Note that no two of Pi , Pj , Pk are friends, since otherwise, A and these two people are
three mutual friends, which is not allowed. Consider the cycle drawn as a graph with Pi , Pj , Pk appearing
in clockwise order. Let x be the distance of the path between Pi and Pj on the cycle not containing
Pk . Define y, z analogously. Therefore, x + y + z = g. Since no two of Pi , Pj , Pk are friends, x, y, z > 1.
Therefore, max{x, y, z} < g−2. Since g is odd, at least one of x, y, z is odd. Without loss of generality, sup-
pose x is odd. Then APi Pi+1 · · · Pj A is an odd cycle of length x + 2 < g, contradicting the minimality of g.
Therefore, every person outside of the cycle is friends with at most two people inside the cycle. Hence,
the number of pairs of friends with one person in the pair in the cycle and the other person in the
pair outside of the cycle, is at most 2(n − g). Therefore, there exists a person inside the cycle who
is friends with at most 2(n−g)
g friends outside of the cycle. Since this person is friends with exactly two
2(n−g) 2n 2n
people inside the cycle, this person is friends with at most g +2 = g ≤ 5 , since g ≥ 5, as desired.
Source: Unknown
Comments: Knowledge in graph theory is of utmost importance to have to solve this problem. Please
read [4] to learn what graphs are and their important properties.
18
Winter Camp 2010 Buffet Contest 1
A1 From an infinite arithmetic sequence a1 , a2 , a3 · · · of positive real numbers, some (possibly infinitely many)
terms are deleted, obtaining an infinite geometric sequence 1, r, r2 , r3 , · · · for some real number r > 0. Prove
that r is an integer.
for all x, y ∈ R.
A3 Find all finite sets A of distinct non-negative real numbers for which:
Number Theory
N1 Let a, b be rational numbers such that a + b and a2 + b2 are integers. Prove that a, b are both integers.
N2 Find all pairs of positive integers (a, b) such that the sequence of positive integers a1 , a2 , a3 , · · ·, formed
by a1 = a, a2 = b and
an−1 + an−2
an = ,
gcd(an−1 , an−2 )
for n ≥ 3, is bounded. (A bounded sequence is a sequence for which there exists a positive real number
M such that an ≤ M for all n ∈ N.)
N3 For a positive integer n, let f (n) be the largest prime divisor of n. Prove that there are infinitely many
positive integers n such that
f (n) < f (n + 1) < f (n + 2).
1
Winter Camp 2010 Buffet Contest 2
Combinatorics
C1 Let n ≥ 2 be a positive integer. An n × n grid is filled with the integers 1, 2, · · · , n2 . Let t be the maximum
of the (positive) difference of the entries of two neighbouring squares, where two squares are said to be
neighbours if they share at least one vertex. Determine the minimum possible value of t in terms of n.
C2 A chessboard is tiled with 32 dominoes. Each domino covers two adjacent squares, a white and a black
square. Show that the number of horizontal dominoes with the white square on the left of the black square
equals the number of horizontal dominoes with the white square on the right of the black square.
C3 Let n > 1 be a positive integer. On each of 2n points around a circle we place a disk with one white side
and one black side. We may perform the following move: select a black disk, and reverse its two neighbors.
Find all initial configurations from which some sequence of such moves leads to a position where all disks
but one are white.
Geometry
G1 The altitude from A of triangle ABC intersects the side BC at D. A circle is tangent to BC at D,
intersects AB at M and N , and intersects AC at P and Q. Prove that
AM + AN AP + AQ
= .
AC AB
G2 Let P be a convex 2010-gon. The 1005 diagonals connecting opposite vertices and the 1005 lines connect-
ing the midpoints of opposite sides are concurrent. (i.e. all 2010 lines are concurrent.) Prove that the
opposite sides of P are parallel and have the same length.
G3 Two circles meet at A and B. Line ` passes through A and meets the circles again at C and D respectively.
Let M and N be the midpoints of arcs BC and BD which do not contain A, and let K be the midpoint
of CD. Prove that ∠M KN = 90◦ .
2
Winter Camp 2010 Buffet Contest 3
A1 From an infinite arithmetic sequence a1 , a2 , a3 · · · of positive real numbers, some (possibly infinitely many)
terms are deleted, obtaining an infinite geometric sequence 1, r, r2 , r3 , · · · for some real number r > 0. Prove
that r is an integer.
Solution: The arithmetic sequence contains 1. Let 1, d + 1, 2d + 1, · · · be the given arithmetic sequence
starting at 1. This sequence contains 1, r, r2 . Then r = 1 + md and r2 = 1 + nd for some integers
2
m, n. If d = 0, then r = 1, which is clearly an integer. Otherwise, m = r−1 d and n = r d−1 are in-
2
tegers. But r d−1 = r−1d · (r + 1). This implies r + 1 is a rational number. Hence, r is rational. But
r is a term in the arithmetic sequence 1, d + 1, 2d + 1, · · ·. This implies this arithmetic sequence has a
rational common difference and consequently, contains only rational numbers. Let f be the denominator
of the rational common difference. Then all terms in the arithmetic sequence have denominator at most f .
If r is not an integer, then if we write r = ab with gcd(a, b) = 1 and b > 1, then the denominators of
1, r, r2 , r3 , · · · are strictly increasing, and will eventually exceed f . But then these terms whose denomi-
nator is larger than f cannot be in the given arithmetic sequence. This is a contradiction. Therefore, r
must be an integer.
for all x, y ∈ R.
Substituting y = x2 yields
4f (x)2 = 4f (x)x2
for all x ∈ R. Equivalently, f (x)(f (x) − x2 ) = 0. Therefore, for each x ∈ R, f (x) = 0 or f (x) = x2 . I
claim that exactly one of these conditions hold for all x ∈ R. Suppose f (a) = 0 for some a 6= 0. I claim
that f ≡ 0. Then substituting x = a yields
3
Winter Camp 2010 Buffet Contest 4
a2 z2 2
Since 2 6= 2 , f ( a2 ) = 0. Hence, f ≡ 0.
Therefore, f (x) ≡ 0, x2 are the only candidate solutions. It remains to verify that both solutions work. If
f (x) ≡ 0, then f (f (x) + y) = 0 and f (x2 − y) + 4f (x)y = 0. If f (x) ≡ x2 , then f (f (x) + y) = (x2 + y)2
and f (x2 − y) + 4f (x)y = (x2 − y)2 + 4x2 y = (x2 + y)2 , as desired.
Comments: In the step f (x)(f (x) − x2 ) = 0, a costly mistake is to conclude here that f (x) ≡ 0 and
f (x) ≡ x2 are the only solutions. It is still possible that f is a function that takes on 0 for some non-trivial
values of x and x2 for the other values of x. You must handle this with care in similar problems. Try
the following problem from the 2008 International Mathematical Olympiad where you have to handle a
similar situation.
Exercise: Find all functions f : (0, ∞) → (0, ∞) ( so f is a function on the positive real numbers) such
that
(f (w))2 + (f (x))2 w2 + x2
= ,
f (y 2 ) + f (z 2 ) y2 + z2
for all positive real numbers w, x, y, z such that wx = yz.
A3 Find all finite sets A of distinct non-negative real numbers for which:
Lemma: Let a, b, c, d be non-negative real numbers such that a < b < c < d. Then ad + bc < ac + bd <
ab + cd.
Proof of Lemma: This follows from the fact that (ab + cd) − (ac + bd) = (d − a)(c − b) > 0 and
(ac + bd) − (ad + bc) = (b − a)(d − c) > 0. End Proof of Lemma 1
Suppose n = |A| ≥ 5. Let a1 , · · · , an ∈ A with a1 < a2 < · · · < an . Then by the Lemma,
and
a1 a2 + a3 an < a1 a2 + a4 an < · · · < a1 a2 + an−1 an ,
and
a1 a2 + an−1 an < a1 a3 + an−1 an < · · · < a1 an−2 + an−1 an
and
a1 an−2 + an−1 an < a2 an−2 + an−1 an < · · · < an−3 an−2 + an−1 an .
There are (n − 2) + (n − 4) + (n − 4) + (n − 4) = 4n − 14 elements in these inequalities. Since n ≥ 5,
4n − 14 > n. Hence, the 4n − 14 terms are pairwise distinct and all in A. This is impossible since |A| = n
1 This result is simply the rearrangement inequality on four variables.
4
Winter Camp 2010 Buffet Contest 5
and 4n − 14 > n.
Therefore, |A| = 4. Let A = {a, b, c, d} with a < b < c < d. Let x = ad + bc, y = ac + bd, z = ab + cd.
Then by the Lemma, x < y < z. By property (b), x, y, z ∈ A. Since A = {a, b, c, d}, {x, y, z} ⊆ {a, b, c, d}.
Therefore, x = a or x = b.
If x = a, then ad + bc = a. Hence, bc = a(1 − d). Since bc > 0, d < 1. Note that ac + bd = b or c, implying
ac+bd ≥ b. Then b−a ≤ (ac+bd)−(ad+bc) = (b−a)(d−c). Hence, d−c ≥ 1, contradicting d < 1. Hence,
x 6= a. Since x < y < z and x, y, z ∈ A and A = {a, b, c, d} and x 6= a, we conclude that x = b, y = c, z = d.
Number Theory
N1 Let a, b be rational numbers such that a + b and a2 + b2 are integers. Prove that a, b are both integers.
Solution 1: Let a = xz for some integers x, z with z, > 0 and gcd(x, z) = 1. Then b = k − xz = kz−x z
for some integer k. Since gcd(kz − x, z) = gcd(x, z) = 1, the denominator of b in lowest terms is also z.
Hence, we can let b = yz with gcd(y, z) = 1.
Solution 2: Using the same notation as Solution 1, we note that 2(a2 + b2 ) − (a + b)2 = (a − b)2 is an
integer. Since a, b is rational, a − b is rational. Hence, a − b is an integer. Since a + b is an integer, 2a, 2b
are integers. If at least one of a, b is an integer, then since a + b is an integer, the other of a, b is also an
integer and we are done. Otherwise, a, b are both not integers, implying a, b have denominator 2 when
expressed in lowest terms. Let a = x2 , b = y2 , where x, y are odd integers. Since a2 + b2 is an integer, 4
divides x2 + y 2 . But x2 , y 2 ≡ 1 mod 4 since x, y are odd. Hence, x2 + y 2 ≡ 2 mod 4, implying 4 - x2 + y 2 .
This is a contradiction. Therefore, a, b are integers.
N2 Find all pairs of positive integers (a, b) such that the sequence of positive integers a1 , a2 , a3 , · · ·, formed
by a1 = a, a2 = b and
an−1 + an−2
an = ,
gcd(an−1 , an−2 )
for n ≥ 3, is bounded. (A bounded sequence is a sequence for which there exists a positive real number
M such that an ≤ M for all n ∈ N.)
5
Winter Camp 2010 Buffet Contest 6
First note if for two consecutive terms ak , ak+1 satisfy gcd(ak , ak+1 ) = 1, then gcd(ak+2 , ak+1 ) = gcd(ak +
ak+1 , ak+1 ) = gcd(ak , ak+1 ) = 1. Iteratively, we have that gcd(an , an+1 ) = 1 for all n ≥ k. Hence, for
all n ≥ k + 2, an = an−1 + an−2 and so the sequence is strictly increasing since each term is a positive
integer. Hence, the sequence is not bounded. Henceforth, we will assume that no two consecutive terms
in the sequence are coprime.
Hence am = am+1 = am+2 = · · · = t and gcd(am , am+1 ) = 2. Therefore, t = 2. Finally, we now find the
am−1 +am am−1 +2
values of am−1 , am−2 , · · · , a1 . Since 2 = am+1 = gcd(am−1 ,am )
= gcd(am−1 ,2)
≥ am−1
2
+2
. Hence, am−1 ≤ 2.
If am−1 = 1, then gcd(am−1 , am ) = 1, which is not allowed. Therefore, am−1 = 2. Hence, iteratively, we
have that am−1 = am−2 = · · · = a1 = 2.
So the only such sequence is the sequence where all terms are equal to 2. Hence, (a, b) = (2, 2) is the only
solution.
N3 For a positive integer n, let f (n) be the largest prime divisor of n. Prove that there are infinitely many
positive integers n such that
f (n) < f (n + 1) < f (n + 2).
k k
Solution 1: Let p ≥ 5 be an odd prime number and let mk = p2 − 1 and nk = p2 + 1 (= mk + 2).
Clearly, f (mk + 1) = p. Note also that
k−1
nk − 2 = mk = (p − 1)(p + 1)(p2 + 1) · · · (p2 − 1) = (p − 1)n0 n1 · · · nk−1 .
k
Note that nk = p2 + 1 ≡ 2 mod 4. Hence, 4 - nk and nk > 4. Therefore, nk contains an odd prime factor.
Suppose q is an odd prime such that q | nk , ni for some 0 ≤ i < k − 1. Then q|2, which is impossible.
Therefore, nk contains an odd prime factor which is not a prime factor of n0 , n1 , · · · , nk−1 . Hence, let k be
the smallest non-negative integer such that f (nk ) > p. Since n0 = p + 1 and p + 1 is composite, implying
f (n0 ) < p. Therefore, k ≥ 1. Hence, f (mk + 2) > p = f (mk + 1).
Finally, note that mk = (p − 1)n0 n1 n2 · · · nk−1 . By the choice of k and the fact that p - ni , we have
f (ni ) < p for each i ∈ {0, 1, · · · , k − 1}. p + 1 is composite, which implies f (p + 1) < p. Hence, f (mk ) < p.
Therefore, f (mk ) < f (mk + 1) < f (mk + 2). We now choose another prime p such that p > mk to generate
another integer n larger than mk such that f (n) < f (n + 1) < f (n + 2). We repeat this process similarly
to generate infinitely many such positive integers. This completes the problem.
Solution 2: Suppose the statement is false, i.e. there are only finitely many integers n such that
f (n) < f (n + 1) < f (n + 2). Consider the number n = 2k for some positive integer k. Note that f (n) = 2
and f (n + 1) > 2 = f (n). Since there are only finitely many k such that f (2k ) < f (2k + 1) < f (2k + 2),
there exists a positive integer m such that k ≥ m implies f (2k ) < f (2k + 1) and f (2k + 2) ≤ f (2k + 1).
Since gcd(2k + 1, 2k + 2) = 1, f (2k + 2) < f (2k + 1).
For k ≥ m, note that f (2k + 2) = f (2k−1 + 1), since every odd factor of 2k + 2 (which is ≥ 2) is also
an odd factor of 2k−1 + 1. Therefore, for k ≥ m, f (2k−1 + 1) < f (2k + 1). Hence, f (2m + 1), f (2m+1 +
1), f (2m+2 + 1), · · · is a strictly increasing sequence of integers. (*)
6
Winter Camp 2010 Buffet Contest 7
Since 2(2k − 1) > k ≥ m, this contradicts (*). Therefore, f (22(2k−1) + 1) = f (22k−1 − 2k + 1) for all
k ≥ m. Since f (22(2k−1) + 1) > f (2k + 1) (since k ≥ m), we have that f (22k−1 − 2k + 1) > f (2k + 1).
Let n = 22k − 2k+1 = 2k+1 (2k−1 − 1). Then f (n) = f (2k−1 − 1), f (n + 1) = f (22k − 2k+1 + 1) =
f ((2k − 1)2 ) = f (2k − 1) and f (n + 2) = f (22k − 2k+1 + 2) = f (22k−1 − 2k + 1) > f (2k − 1) = f (n + 1).
If f (n) < f (n + 1) for infinitely many choices of k, then we are done. Otherwise, there exists a positive
integer M such that M > m and k ≥ M implies f (n) ≥ f (n + 1), i.e. f (2k−1 − 1) ≥ f (2k − 1). Since
(2k − 1) − 2(2k−1 − 1) = 1, gcd(2k−1 − 1, 2k − 1) = 1. Hence, f (2k−1 − 1) > f (2k − 1) for all k ≥ M .
Hence, for f (2M − 1), f (2M +1 − 1), f (2M +2 − 1), · · · is an infinite strictly decreasing sequence of positive
integers, which is absurd. Hence, the statement of the problem is indeed true.
Combinatorics
C1 Let n ≥ 2 be a positive integer. An n × n grid is filled with the integers 1, 2, · · · , n2 (with each number
used exactly once). Let t be the maximum of the (positive) difference of the entries of two neighbouring
squares, where two squares are said to be neighbours if they share at least one vertex. Determine the
minimum possible value of t in terms of n.
The number of squares in the shortest sequence of successive neighbouring squares between any two squares
is at most n − 1. Since both 1, n2 are in the grid and differ by n2 − 1, there are two neighbouring squares
2
−1
that differ by at least nn−1 = n + 1. Hence, t ≥ n + 1. Consider the grid filled in order row by row
1, 2, · · · , n , i.e. place (i − 1)n + j in the ith row and the j th column. Two entries in such a configuration
2
Source: Unknown
C2 A chessboard is tiled with 32 dominoes. Each domino covers two adjacent squares, a white and a black
square. Show that the number of horizontal dominoes with the white square on the left of the black square
equals the number of horizontal dominoes with the white square on the right of the black square.
Solution: Let Si be the set of horizontal dominoes contained in columns i and i + 1 (obviously, all Si are
disjoint). Then Si = Li ∪ Ri , where Li is the set of such dominoes with a white square on the left, and
Ri is the set of such dominoes with a white square on the right. Since each column has an even number
of rows, and an even number of those rows is taken up by vertical dominoes (each of them takes 2 rows),
the horizontal dominoes must also occupy an even number of rows, so |Si−1 ∪ Si | is even for each i. Since
|S1 | is even, we must have that |Si | is even for each i.
Consider the horizontal dominoes that intersect the first column of the board. The space between any
consecutive pair of horizontal dominoes is taken up by vertical dominoes. Each of those occupies 2 rows,
so there is an even number of rows between the horizontal dominoes. Thus, if one of those dominoes is in
L1 , the other is in R1 , and vice-versa. Since |S1 | is even, we must have |L1 | = |R1 |.
7
Winter Camp 2010 Buffet Contest 8
We prove by induction that |Li | = |Ri | for each i. The base case is i = 1, shown above. Suppose
|Li−1 | = |Ri−1 |. The set of horizontal dominoes that intersect column i is Si−1 ∪ Si . As in the i = 1 case,
any consecutive pair of these dominoes has an even number of rows between them. Thus, the number of
dominoes that use a black square in column i (|Li−1 ∪ Ri |) is the same as the number of dominoes that
use a white square in column i (|Ri−1 ∪Li |). However, since |Li−1 | = |Ri−1 |, we must have that |Li | = |Ri |.
Therefore, the total number of horizontal dominoes with a white square on the left (| ∪i Li |) equals the
total number of horizontal dominoes with a white square on the right (| ∪i Ri |).
C3 Let n > 1 be a positive integer. On each of 2n points around a circle we place a disk with one white side
and one black side. We may perform the following move: select a black disk, and reverse its two neighbors.
Find all initial configurations from which some sequence of such moves leads to a position where all disks
but one are white.
Solution: The answer is when the number of black disks initially is odd.
Note that each move preserves the parity of the number of black disks on the circle. Hence, if the initial
number of black disks is even, then we can never have one black disk remaining.
Suppose there is an odd number of black disk. We define a group to be a maximal set of black disks that
appear on consecutive points on the circle. Let k be the number of groups initially. I claim that if k > 1,
after a finite number of moves, we can decrease the number of groups. Since there are an odd number of
black disks, one group contains an odd number of disks. Suppose there are 2m − 1 disks in this group for
some positive integer m. We label the disks 1, 2, · · · , 2m − 1 in clockwise order. By assumption, the disk
next to 1 (which is not 2), which we now call 0, is white and the disk next to 2m − 1 (which is not 2m − 2),
which we will call 2m, is also white. Note that since k > 1, disks 0 and 2m are distinct. We perform a
move on disks 1, 3, 5, · · · , 2m − 1, in this order. In doing so, the disks 2, 4, · · · , 2m − 2 are each flipped
twice, and therefore, remain black. The only disks flipped are 0 and 2m, which are now flipped to black.
Hence, this group increased by size at least 2. If it increased by more than 2, then this group merged with
another group and the number of groups decreased. Otherwise, this group still has odd cardinality and
we can repeat this process. Since there is another group, eventually, this group will merge with another
group. Hence, we have decreased the number of groups. We repeat this process (by choosing another odd
group). Eventually, the number of groups becomes one.
By performing the steps in the previous paragraph, we have one group of black disks. There are an
odd number of disks in this group, say 2m − 1 for some positive integer m. We again label the disks
1, 2, · · · , 2m − 1. If m = 1, then only one black disk remain and we are done. Otherwise, we choose the
disks 2, 4, · · · , 2m − 2. Hence, disks 3, 5, · · · , 2m − 3 are flipped twice, and thus remain the same colour.
But disks 1 and 2m − 1 are changed to white. Hence, the group size decreased by 2. We can repeat this
procedure until one black disk remains. We are done.
Geometry
G1 The altitude from A of triangle ABC intersects the side BC at D. A circle is tangent to BC at D,
intersects AB at M and N , and intersects AC at P and Q. Prove that
AM + AN AP + AQ
= .
AC AB
8
Winter Camp 2010 Buffet Contest 9
Solution 2: Let O be the centre of the circle. Since the circle is tangent to BC at D and AD ⊥ BC, O
lies on AD. Let U , V be the feet of the perpendicular on AB, AC from O, respectively. Therefore, U, V
are midpoints of M N, P Q, respectively. This implies AM + AN = 2 · AU and AP + AQ = 2 · AV . Hence,
it suffices to show that
AU AV
= .
AC AB
AD AD
Since AU = AO cos ∠BAO, AC = cos ∠CAO , AV = AO cos ∠CAO, AB = cos ∠BAO , this equation is true,
as desired . 2
G2 Let P be a convex 2010-gon. The 1005 diagonals connecting opposite vertices and the 1005 lines connect-
ing the midpoints of opposite sides are concurrent. (i.e. all 2010 lines are concurrent.) Prove that the
opposite sides of P are parallel and have the same length.
Lemma: Let ABCD be a convex quadrilateral, M the midpoint of AB, N the midpoint of CD. Suppose
AC, BD, M N are concurrent at a point P . Then AB||CD and ∆P AB ∼ ∆P CD.
Proof of Lemma: Let l be a line passing through B parallel to CD and intersecting P A at a point A0 . Let
M 0 be the midpoint of A0 B. Since ∆P A0 B ∼ ∆P CD, ∆P AM 0 ∼ ∆P CN . Hence, ∠A0 P M 0 = ∠CP N ,
i.e. M 0 lies on P N . Suppose A 6= A0 . Then M 6= M 0 . Since M also lies on P N , M M 0 is parallel to
AP . But they intersect at P , contradicting the fact that they are parallel. Therefore, A = A0 . Therefore,
AB||CD. Subsequently, ∠P BA = ∠P DC and ∠P AB = ∠P CD, we have that ∆P AB ∼ ∆P CD. End
Proof of Lemma
Let A1 , A2 , · · · , A2010 be the vertices appearing clockwise of the 2010-gon and Mi the midpoint of Ai and
Ai+1 . Then Ai and Ai+1005 are opposite vertices. Let P be the concurrent point of the 2010 lines. Since
Ai Ai+1005 , Ai+1 Ai+1006 , Mi M1005 are concurrent at P , consider the quadrilateral Ai Ai+1 Ai+1005 Ai+1006 .
Since the polygon is convex, this quadrilateral is convex. By the Lemma, we have that Ai Ai+1 is parallel
to Ai+1005 Ai+1006 . Hence, opposite sides of the polygon are parallel.
9
Winter Camp 2010 Buffet Contest 10
AP CP
Alternate Proof of Lemma: It suffices to prove BP = DP . By Sine Law, we have that
G3 Two circles meet at A and B. Line ` passes through A and meets the circles again at C and D respectively.
Let M and N be the midpoints of arcs BC and BD which do not contain A, and let K be the midpoint
of CD. Prove that ∠M KN = 90◦ .
Solution 1: Let X, Y be the midpoints of BC, BD, respectively. Note that M X ⊥ BC and N Y ⊥ BD.
Also note that KY ||BC and KX||BD. Hence, KXBY is a parallelogram.
|M X| |M X| |BY | |XK|
= = = .
|KY | |XB| |Y N | |Y N |
Solution 2: (Inversion solution:) Since M, N are midpoints of the arcs BC and BD, respectively, AM
bisects ∠BAC and AN bisects ∠BAD. Therefore, ∠M AN = 90◦ . To prove ∠M KN = 90◦ , it suffices to
prove that AM N K is cyclic.
We will invert the diagram about the point A with radius 1. For each object X, let X 0 be its image under
the inversion. Since ABM C is cyclic, B 0 , M 0 , C 0 are collinear. Since AM bisects ∠BAC, AM 0 bisects
∠B 0 AC 0 . Similarly, B 0 , N 0 , D0 are collinear and AN 0 bisects ∠B 0 AD0 . The point K 0 lies on C 0 D0 external
to segment C 0 D0 . To prove that M 0 , N 0 , K 0 are collinear, by Menelaus’ Theorem, it suffices to prove that
|C 0 M 0 | |B 0 N 0 | |D0 K 0 |
· · = 1.
|M 0 B 0 | |N 0 D0 | |K 0 C 0 |
|C 0 M 0 | |AC 0 | |B 0 N 0 | |AB 0 |
By angle bisector theorem, we have that |M 0 B 0 | = |AB 0 | and |N 0 D 0 | = |AD 0 | . Hence, it suffices to show
that
|C 0 A| |C 0 K 0 |
= .
|AD0 | |K 0 D0 |
10
Winter Camp 2010 Buffet Contest 11
as desired.
Solution 3: (Harmonic Division solution:) Using the notation of Solution 2, it suffices to prove that
|C 0 A| |C 0 K 0 |
= ,
|AD0 | |K 0 D0 |
i.e. (K 0 , A) divides (C 0 , D0 ) harmonically. But since K is the midpoint of CD, (K, ∞) divides (C, D)
harmonically. Since the property that a harmonic quadruple is preserved under inversion of a point on
the same line as the quadruple3 , this implies (K 0 , A) divides (C 0 , D0 ) harmonically, as desired.
11
IMO Winter Camp Mock Olympiad 2010
Time: 4 Hours
1. Let ABCD be a parallelogram with AC/BD = k. The bisectors of the angles formed by AC and
BD intersect the sides of ABCD at K, L, M, N . Prove that the ratio of the areas of KLM N
and ABCD is 2k/(k + 1)2 .
Prove that there is at least one square whose edges are oriented clockwise.
3. Let Z∗ denote the set of non-zero integers. A function f : Z∗ → Z≥0 satisfies the following
properties:
1. Let ABCD be a parallelogram with AC/BD = k. The bisectors of the angles formed by AC and
BD intersect the sides of ABCD at K, L, M, N . Prove that the ratio of the areas of KLM N
and ABCD is 2k/(k + 1)2 .
Solution: Suppose K, L, M, N are on AB, BC, CD, DA, respectively. Let P = AC ∩ BD. Since
ABCD is a parallelogram, AP = P C and BP = P D. For any polygon X1 X2 · · · Xn , we denote
its area by [X1 X2 · · · Xn ].
k2 k2
[AKN ] = [ABD] = [ABCD].
(k + 1)2 2(k + 1)2
Similarly,
1 k2 1
[BKL] = [ABCD], [CLM ] = [ABCD], [DM N ] = [ABCD].
2(k + 1)2 2(k + 1)2 2(k + 1)2
Hence,
[KLM N ] = [ABCD] − [AKN ] − [BKL] − [CLM ] − [DM N ]
k2
1 2k
= 1− − [ABCD] = [ABCD],
(k + 1)2 (k + 1)2 (k + 1)2
as desired.
Prove that there is at least one square whose edges are oriented clockwise.
Solution: Let a square be called half-clockwise if its top and left sides are oriented clockwise.
Suppose that there is no clockwise square in the grid.
Consider a half-clockwise square S. Since it is not a clockwise square, either the right or bottom
edge must be oriented counterclockwise. If it is the right edge, let the square to the right of S
be T (if there are no squares to the right of S, then the right edge of S is on the border and
thus must be oriented clockwise).
Consider the top right vertex of S - it is either in the interior or on the top border of the grid.
If it is an interior vertex, it has the top and right edges of S going into it, so its other edges are
going out, in particular the top edge of T . If it is on the top border, then the top edge of T is
clockwise. In both cases, T is a half-clockwise square.
If the bottom edge of S is oriented counterclockwise, we can similarly show that the square T
below S is a half-clockwise square.
Since the top left square in the grid is half-clockwise, we can make a path of half-clockwise
squares by stepping either to the right or down. This implies that the bottom right square in the
grid is half-clockwise. However, the bottom and right edges of this square are clockwise-oriented,
so the square is clockwise. Contradiction.
3. Let Z∗ denote the set of non-zero integers. A function f : Z∗ → Z≥0 satisfies the following
properties:
Solution: The minimum possible value is 30 and the maximum possible value is 2002.
Let p be the smallest integer such that f (p) = 1. This is well defined since f (2010) = 1 > 0. I
claim that p is a prime. Suppose p = ab for some 1 < a, b < p. Then f (p) = f (a) + f (b) by (b).
Hence, at least one of f (a), f (b) = 1. This contradicts p being the smallest integer such that
f (p) > 0. Therefore, p is prime and f (1) = f (2) = · · · = f (p − 1) = 0.
For any positive integer m, write m = pr ·s where r ∈ Z≥0 and s - p. Then f (m) = f (pr )+f (s) =
r · f (p) = r. Hence, f (m) is the number of times p divides into m for a fixed prime p. We will
now verify that such a function satisfies properties (a) and (b).
Let m = pa · b and n = pc · d such that a, c ∈ Z≥0 and p - b, d. Then mn = pa+c · bd and
p - bd. Therefore, f (mn) = a + c = f (m) + f (n). Hence, f satisfies (b). Finally, without loss of
generality, suppose a ≤ c. Then m + n = pa · b + pc · d = pa (b + pc−a d). Hence p divides into
m + n at least a times. Therefore, f (m + n) ≥ a = min{a, c} = min{f (m), f (n)}. Hence, f
satisfies (a). Note that p can be chosen to be any prime number.
Since f (2010) = 1 and 2010 = 2 · 3 · 5 · 67, we conclude that all of the possible values of p are
2, 3, 5, 67. We want to find the number of times each of these values of p divide into 2010!. The
maximum and minimum possible value of f (2010!) is obtained by taking p = 2 and p = 67,
respectively.
By Cauchy-Schwarz Inequality,
p
(a2 + b2 )(a2 + c2 ) ≥ a2 + bc.
Hence, our expression is greater than or equal to
P 2
(a + b2 ) + 2 (a2 + bc) (3 a2 ) + (a + b + c)2 (3 a2 ) + 9
P P P
cyc cyc cyc cyc 3
P 2 = P 2 = P 2 = ,
6+2 a 6+2 a 6+2 a 2
cyc cyc cyc
as desired.
Solution 2: (Mixing Variables - With The Cumbersome Steps Omitted) For x, y, z > 0, let
1 1 1
f (x, y, z) = 2 2
+ 2 2
+ .
2+x +y 2+y +z 2 + z 2 + x2
Let t = a+b
2 . We will prove that f (a, b, c) ≤ f (t, t, c) ≤
3
4 to solve the problem. Without loss of
generality, suppose a ≤ b ≤ c.
We prove each of these two terms is non-negative to prove f (a, b, c) ≤ f (t, t, c).
1 1 1 1 (a − b)2
2
− 2 2
= 2 − = ≥ 0.
2 + 2t 2+a +b 2+2 2a+b 2 + a2 + b2 4 1+
a+b 2
(2 + a2 + b2 )
2
In the second term, it suffices to show that the numerator is non-negative. The numerator is
equal to
1 2 3
2
+ 2 2
≤ .
2 + 2t 2+t +c 4
This simplifies to 3t2 c2 + 3t4 + c2 − t2 − 6 ≥ 0. By substituting c = 3 − 2t, we simplify this to
proving (t − 1)2 (5t2 − 2t + 1) ≥ 0. Since the latter term has negative discriminant, this term is
indeed non-negative and therefore the inequality is true. Hence, f (t, t, c) ≤ 43 , as desired.
Let h be the homothety with positive ratio that maps ω1 to ω2 . Then h(M ) = N and
h(O1 ) = O2 . We claim that h(B) = D and h(D) = C. Note that ∠O1 BC = 90 − ∠BAD and
∠O2 DC = 90 − ∠DAC. Since ∠BAD = ∠DAC, ∠O1 BC = ∠O2 DC. Therefore, BO1 ||DO2 .
Consequently, h(B) = D. Therefore, the centre of h lies on BD. Hence, h(D) is a point
(different from D) on ω2 that lies on BC. Therefore, h(D) = C.
Let E, X, Y be the midpoints of M N, BD, DC, respectively and ω the circumcircle of ∆EXY .
Since N = h(M ) and D = h(B), M B||N D. Therefore, EX||M B||N D. Similarly, EY ||M D||N C.
Hence, the homothety h1 (with positive ratio) that maps M B to EX has the same centre as h
and maps D to Y . Since the circumcircle of ∆M BD is tangent to ` and ` passes through the
centre of h1 and h1 maps ∆M BD to ∆EXY , the image of this circumcircle is also tangent
to `, i.e. the circumcircle of ∆EXY is tangent to h. This completes the problem.
We first consider the case when ` intersects DA on ray DA. Using the same argument as
in Solution 1, AD passes through the midpoint of M N ; we will call this point E. I claim
that M D||N C and M B||N D. Let α = ∠BAD = ∠DAC, θ1 = ∠N M D, θ2 = ∠M N D.
Note that ∠BM D = ∠BAD = α and ∠DN C = ∠DAC = α. Then by properties of
tangents, ∠M BD = θ1 , ∠N CD = θ2 . Looking at the quadrilateral M N CB, we have
2(θ1 + θ2 + α) = 360◦ . Hence, θ1 + θ2 + α = 180◦ . Since ∠N M D = θ1 and ∠M N C = θ2 + α
and θ1 + θ2 + α = 180◦ , M D||N C. Since ∠BM D = ∠DN C = α, M B||N D.
The case when ` intersects DA on ray AD is handled similarly. Using the same argument
as in Solution 1, AD passes through the midpoint of M N ; we will call this point E. I claim
that M D||N C and M B||N D. Let α = ∠BAD = ∠DAC, θ1 = ∠N M D, θ2 = ∠M N D.
Note that ∠BM D = 180 − ∠BAD = 180 − α and ∠DN C = 180◦ − ∠DAC = 180◦ − α.
Then by properties of tangents, ∠M BD = θ1 , ∠N CD = θ2 . Looking at the quadrilateral
M N CB, we have 2(θ1 + θ2 + 180◦ − α) = 360◦ . Hence, θ1 + θ2 + 180◦ − α = 180◦ . Since
∠N M D = θ1 and ∠M N C = 180 − α + θ2 and θ1 + θ2 + 180◦ − α = 180◦ , M D||N C. Since
∠BM D = ∠DN C = 180◦ − α, M B||N D.
2. Let P (x) be a non-constant polynomial with integer coefficients. Prove that there is no func-
tion T : Z → Z such that the number of integers x with T n (x) = x is equal to P (n) for every
n ≥ 1, where T n denotes the n-fold application of T .
Solution: Suppose such a function T exists. For each positive integer n, let
A(n) = {x ∈ Z | T n (x) = x}
and
B(n) = {x ∈ Z | T n (x) = x, and n is the smallest positive integer with this property.}
Then P (n) = |A(n)| for all positive integers n. This implies that A(n) is finite for all n ∈ N.
Note that [
A(n) = B(d)
d|n
We claim that d divides |B(d)|. We represent this as a direct graph: let the set of integers be
represented by vertices and there is a direct edge from a to b if and only if b = T (a). Then
an element x ∈ B(d) if and only if x is in a cycle of length d. Since the number of elements
in a given cycle of length d is a multiple of d, the number of elements in a cycle of length d is
a multiple of d. Therefore, d divides |B(d)|.
am + am−1 + · · · + a1 = 0 (1)
Finally, let p, q be any two distinct primes. Then P (pq) = |A(pq)| = |B(pq)|+|B(p)|+|B(q)|+
|B(1)| = |B(pq)|+(|A(p)|−B(1))+(|A(q)|−B(1))+|A(1)| = |B(pq)|+|A(p)|+|A(q)|−|A(1)| =
|B(pq)| + P (p) + P (q) − P (1). Therefore, |B(pq)| = P (pq) − P (p) − P (q) + P (1). Since pq
divides |B(pq)|, pq divides P (pq) − P (p) − P (q) + P (1). Therefore,
In particular, p divides this expression. Taking this equation modulo p and by (1), we have
Therefore, every prime q is a root of the equation am xm + am−1 xm−1 + · · · + a1 x (mod p).
We will show that this is impossible. Choose p such that p > m + 1. By Dirchlet’s theorem
for prime, for each i ∈ {1, · · · m + 1}, there exists a prime qi such that qi ≡ i (mod p).
Since p > m + 1, q1 , · · · , qm+1 are pairwise distinct modulo p. Then q1 , · · · , qm+1 are pairwise
distinct roots of the polynomial am xm + am−1 xm−1 + · · · + ax (mod p). But this polynomial
contains at most m roots. This is a contradiction.
3. On a 999 × 999 board a limp rook can move in the following way: From any square it can
move to any of its adjacent squares, i.e. a square having a common side with it, and every
move must be a turn, i.e. the directions of any two consecutive moves must be perpendicular.
A non-intersecting route of the limp rook consists of a sequence of pairwise different squares
that the limp rook can visit in that order by an admissible sequence of moves. Such a non-
intersecting route is called cyclic, if the limp rook can, after reaching the last square of the
route, move directly to the first square of the route and start over.
How many squares does the longest possible cyclic, non-intersecting route of a limp rook visit?
Colour the board with four colours A, B, C, D according to the following rules: Given a square
in the ith row and j th column, colour the square A if i, j are both odd, B if i is odd and j is
even, C if i is even and j is odd and D if i, j are both even. Then a limp rook encounters the
same colour exactly every four steps. More specifically, D appears once every four steps.
Since there are only 4992 squares coloured D, a limp rook cycle contains at most 4(4992 ). We
claim that a limp rook cycle cannot go through every square coloured D. This will prove that
a limp rook cycle passes through at most 4(4992 − 1) squares. We will then construct such a
cycle to prove that this length is indeed the maximum.
Colour the squares marked D in a ”chessboard” manner in black and white. Suppose that a
limp rook cycle passes through every square marked D. Note that two consecutive squares
of a limp rook cycle that are coloured D, are ”neighbours” of each other (i.e. distance 2
horizontally and/or vertically of each other.) Since there are an odd number of squares
marked D, there are two consecutive squares in the limp rook cycle coloured the same colour,
marked D. By symmetry, we may assume that these two squares are (a, b), (a + 2, b + 2),
where (i, j) is the square in the ith row and the j th column, for some positive integers a, b.
Without loss of generality, suppose (a, b) is going toward (a + 2, b + 2). Also, assume that the
path goes
It is easy to show that by parity, the limp rook cycle enters any square coloured D horizon-
tally and exits any square coloured D vertically. Consider the square (a, b + 2). This square
is coloured D. Then the limp rook cycle enters this square horizontally and exits this square
vertically. Therefore, the limp rook cycle goes from (a − 1, b + 2) → (a, b + 2) → (a, b + 3).
By the direction of the limp rook cycle, the limp rook goes from (a, b + 3) to (a, b) and from
(a + 2, b + 2) to (a − 1, b + 2). Let P1 , P2 be these two paths of the limp-rook cycle, respec-
tively. Note that P1 , P2 does not go through the interior of the quadrilateral whose vertices
are (a, b), (a + 2, b + 2), (a, b + 3), (a − 1, b + 2). Therefore, P1 and P2 must intersect. Hence,
the limp rook cycle must intersect. This is a contradiction.
Hence, a limp rook cycle contains at most 4(4992 − 1) squares. And now the construction.
2010 Mock 2
Time: 4.5 Hours
1. A positive integer N is called balanced, if N = 1 or if N can be written as a product of an
even number of not necessarily distinct primes. Given positive integers a and b, consider the
polynomial P defined by P (x) = (x + a)(x + b).
(a) Prove that there exist distinct positive integers a and b such that all the numbers
P (1), · · · , P (50) are balanced.
(b) Prove that if P (n) is balanced for all positive integers n, then a = b.
2. In a triangle ABC with AB 6= AC, the incircle touches the sides BC, CA, AB at D, E, F ,
respectively. Line AD meets the incircle again at P . The line EF and the line through P
perpendicular to AD meet at Q. Line AQ intersects DE at X and DF at Y . Prove that A
is the midpoint of XY .
Then, ∠AQP = ∠AM P = ∠ADI, and so ∠QAD = 90o −∠AQP = ∠BDI−∠ADI = ∠BDA,
and so BC||AQ. Therefore, ∠AXE = ∠EDC = ∠CED = ∠AEX. Therefore, AE = AX.
Similarly, AF = AY . Since AE = AF , AX = AY , as desired.
Solution 2: We do the same as Solution 1 to prove that AQ||BC.
for all x, y ∈ R.
Substituting x = 0 into the original equation yields f (0) = f (yf (0)), ∀y ∈ R. Suppose
f (0) 6= 0. Then yf (0) ranges over all reals as y varies over all reals. Hence, f (0) = f (z) for
all z ∈ R. Consequently, f is a constant. If f (x) = c, a constant, then substituting this into
the original equation yields c = c + x2 , or x2 = 0 for all x ∈ R. This is clearly absurd since
x = 1 violates this condition. Therefore,
f (0) = 0. (2)
f (z) = 0 ⇔ z = 0, ∀z ∈ R (3)
Substituting y = −x yields
Lemma 1: f is surjective.
Proof of Lemma 1: This follows from both (4) and (5). End Proof of Lemma 1
Lemma 2: f is injective.
Proof of Lemma 2: Suppose f (z) = f (z + r) for some z, r ∈ R. Substituting x = z, y = r
into the original equation yields f (zf (z + r)) = f (rf (z)) + z 2 . Since f (z) = f (z + r),
f (zf (z)) = f (rf (z)) + z 2 . By (4), f (zf (z)) = z 2 . Therefore, f (rf (z)) = 0. By (3), rf (z) = 0.
Therefore, f (z) = 0 or r = 0. If f (z) = 0, then z = 0, Hence, f (0) = f (0 + r). By (3), r = 0.
Hence, in either case, r = 0. Therefore, f is injective. End Proof of Lemma 2
Proof of Lemma 3: Let z ∈ R. If z = 0, then clearly, f (−z) = −zf (z), since both are equal to
0. Otherwise, suppose z 6= 0. Substituting x = −z yields f (−zf (−z + y)) = f (yf (−z)) + z 2 .
Since z 6= 0, f (−z) 6= 0. Therefore, yf (−z) ranges over all reals as y varies over all reals. Since
f is surjective, we can let w ∈ R such that f (wf (−z)) = −z 2 . Then f (−zf (−z + w)) = 0.
Hence, −zf (−z + w) = 0. Since z 6= 0, f (−z + w) = 0. Hence, w = z. Therefore, by the
definition of w, f (zf (−z)) = −z 2 . But we also know that f (−zf (z)) = −z 2 by (5). Since f
is injective, zf (−z) = −zf (z). Since z 6= 0, f (−z) = −f (z), as desired.
f (y + 4) − f (y) = 4, ∀y ∈ R. (8)
Substituting x = 2 into the original equation fields f (2f (y + 2)) = f (yf (2)) + 4. By (8), we
have f (yf (2) + 4) = f (yf (2)) + 4. Therefore, f (2f (y + 2)) = f (yf (2) + 4). Since f is injective,
2f (y + 2) = yf (2) + 4. Hence, f (y) = f (2) f (2)
2 (y − 2) + 2. Let c = 2 . Then f (y) = cy + (2 − 2c).
Since y = 0 and f (0) = 0, c = 1. Then f (y) = y, ∀y ∈ R, which can be verified as a solution
to the functional equation.
f (y) − f (y + 4) = 4, ∀y ∈ R. (11)
Substituting x = 2 into the original equation fields f (2f (y + 2)) = f (yf (2)) + 4. By (8), we
have f (yf (2) − 4) = f (yf (2)) + 4. Therefore, f (2f (y + 2)) = f (yf (2) − 4). Since f is injective,
2f (y + 2) = yf (2) − 4. Hence, f (y) = f (2) f (2)
2 (y − 2) − 2. Let c = 2 . Then f (y) = cy − (2 + 2c).
Since y = 0 and f (0) = 0, c = −1. Then f (y) = −y, ∀y ∈ R, which can be verified as a
solution to the functional equation. This completes the problem.
2010 Mock 3
Time: 4.5 Hours
1. For any n ≥ 2, let N (n) be the maximal number of triples (ai , bi , ci ), i = 1, · · · , N (n) consisting
of nonnegative integers ai , bi , ci such that the following two conditions are satisfied:
• ai + bi + ci = n for all i = 1, · · · , N (n)
• If i 6= j, then ai 6= aj , bi 6= bj , ci 6= cj .
Determine N (n) for n ≥ 2.
N
P N
P N
P
Note that ai , bi , ci ≥ N (N − 1)/2. Therefore,
i=1 i=1 i=1
n
3N (N − 1) X
≤ (ai + bi + ci ) = N n.
2
i=1
Therefore, N ≤ b2n/3c + 1.
It remains to construct such triples for each n. We will split this into 3 cases; n ≡ 0
(mod 3), n ≡ 1 (mod 3), n ≡ 2 (mod 3).
If n ≡ 0 (mod 3), let n = 3k for some positive integer k. Then N (n) = 2k + 1. The following
is a construction of triples satisfying the conditions given in the problem.
ai bi ci
2k 0 k
2k − 1 2 k−1
.. .. ..
. . .
k 2k 0
k−1 1 2k
k−2 3 2k − 1
.. .. ..
. . .
0 2k − 1 k+1
If n ≡ 1 (mod 3), let n = 3k + 1 for some positive integer k. Then N (n) = 2k + 1. The
following is a construction of triples satisfying the conditions given in the problem.
ai bi ci
2k 0 k+1
2k − 1 2 k
.. .. ..
. . .
k 2k 1
k−1 1 2k + 1
k−2 3 2k
.. .. ..
. . .
0 2k − 1 k+2
If n ≡ 2 (mod 3), let n = 3k + 2 for some positive integer k. Then N (n) = 2k + 2. The
following is a construction of triples satisfying the conditions given in the problem.
ai bi ci
2k + 1 0 k+1
2k 2 k
.. .. ..
. . .
k 2k + 2 0
k−1 1 2k + 2
k−2 3 2k + 1
.. .. ..
. . .
0 2k − 1 k + 3
Substituting x = f (y) into (12) and using (13) yields f (0) ≤ yf (f (y)) + f (y) ≤ yf (f (y)) +
y + f (0). Therefore,
Suppose there exists x ∈ R such that f (x) > 0. By (13), we have that f (y) approaches −∞
as y approaches −∞. Then we can choose a sufficiently small y such that f (y) < x and
yf (x) + x + f (0) < −1. This contradicts (15).
Therefore, f (x) ≤ 0 for all x ∈ R. Suppose there exists a positive real x such that f (x) < 0.
Then for all y ∈ R, f (y) ≤ 0 < x. Then by (15), we have −1 ≤ yf (x) + x + f (0). This is not
possible if y is sufficiently large. Therefore, f (x) = 0 for all positive real x.
Finally, choose y = 1 and any x < 0. Since f (y) = f (1) = 0, by (12) f (x) ≤ f (x) + x. Then
x ≥ 0. This contradicts the choice of x. This solves the problem.
3. Let P be a polygon that is convex and symmetric about some point O (i.e. if a point is on the
polygon, then its reflection in O is also on the polygon). Prove that for some parallelogram
R satisfying P ⊂ R we have
|R| √
≤ 2
|P |
where |R| and |P | denote the areas of R and P , respectively.
Solution: Let A, B be two vertices of P such that [OAB] is maximum, where [XY Z] denotes
the area of XY Z. Let A0 , B 0 be the reflections of A, B across O, respectively. Then A0 , B 0
are vertices of P , since P is symmetric about O. Let l1 , l3 be the lines passing through A, A0
respectively parallel to OB and l2 , l4 the lines passing through B, B 0 respectively parallel to
OA. Let P1 be the parallelogram formed by l1 , l2 , l3 , l4 . Clearly, P ⊆ P1 , since [OAB] has
maximum area. Note that A, B, A0 , B 0 are midpoints of the sides of P1 . Therefore, ABA0 B 0
is a parallelogram. Let P2 be this parallelogram. Then
|P1 | = 2|P2 |.
Note that P2 ⊆ P . Let P3 be the smallest parallelogram whose sides are parallel to that of
P2 such that P ⊆ P3 . Note that each side of P3 contains a vertex of P . Let X be a point
on the side of P3 “closest” to AB which is on P . Let X 0 be the image of the reflection of X
about O. Note that X 0 is on the side of P3 closest to A0 B 0 which is on P . Let Y be a point on
the side of P3 closest to BA0 which is on P . Let Y 0 be the image of the reflection of Y about O.
Let x be the ratio of the distance from X to AB to the distance from O to AB. Let y be the
ratio of the distance from Y to BA0 to the distance from O to BA0 . Note that the polygon
AXBY A0 X 0 B 0 Y 0 is contained in P , since P is convex. By symmetry, we have
|P2 | |P2 |
[AXBY A0 X 0 B 0 Y 0 ] = 2[OAXB] + 2[OBY A0 ] = 2(1 + x) + 2(1 + y)
4 4
|P2 | |P2 | |P2 |
= (1 + x) + (1 + y) = (2 + x + y).
2 2 2
Hence,
|P2 |
(2 + x + y) ≤ |P |.
2
2(1 + x)(1 + y)
|P3 | = (1 + x)(1 + y)|P2 | ≤ |P |
2+x+y
Therefore,
|P2 | 2 |P1 | 4
≤ ⇒ ≤
|P | 2+x+y |P | 2+x+y
√ √
If 2 + x + y ≥ 2 2, then |P 1| 4
|P | ≤ 2+x+y ≤ 2. Therefore, P1 satisfies the desired conditions.
√
Otherwise, we may assume that 2 + x + y < 2 2. Let a = 1 + x, b = 1 + y. Therefore,
|P3 | 2ab (a+b)2 a+b
√
|P | ≤ a+b ≤ 2(a+b) = 2 < 2. Thus, P3 satisfies the desired conditions. We are done.
2010 Mock 4
Time: 4.5 Hours
1. Find the largest possible integer k, such that the following statement is true:
Let 2009 arbitrary non-degenerated triangles be given. In every triangle the three sides are
colored, such that one is blue, one is red and one is white. Now, for every color separately,
let us sort the lengths of the sides. We obtain
Then there exist k indices j such that we can form a non-degenerated triangle with side
lengths bj , rj , wj .
We first prove that k > 0. Suppose k = 0. Then WLOG, suppose b2009 ≤ r2009 ≤ w2009 .
Since k = 0, these three numbers do not form the sides of a non-dengerate triangle. Therefore,
b2009 + r2009 ≤ w2009 . Since bi ≤ b2009 and rj ≤ r2009 for all i, j ∈ {1, · · · , 2009}, bi + rj ≤ w2009
for all i, j ∈ {1, · · · , 2009}. Therefore, w2009 cannot be the side length of any of the 2009 tri-
angles. This is a contradiction.
Therefore, k ≥ 1. The following is an example for which k = 1. For i ∈ {1, · · · , 2008}, let
bi = i, ri = 2i, wi = 3i.
Let
b2009 = 2009, r2009 = 2008 · 3, w2009 = 2008 · 3.
Then for all i ∈ {2, · · · , 2009}, it is easy to verify that (bi , ri−1 , wi−1 ) form the sides of a trian-
gle, since bi = i, ri−1 = 2i − 2 and wi−1 = 3i − 3 and i + (2i − 2) > 3i − 3, and (b1 , r2009 , w2009 )
form the sides of a triangle, since 1, a, a are the sides of a triangle for any positive integer a.
2. Given a cyclic quadrilateral ABCD, let the diagonals AC and BD meet at E and the lines
AD and BC meet at F . The midpoints of AB and CD are G and H, respectively. Show that
EF is tangent at E to the circle through the points E, G and H.
• Since reflection and dilation preserves the collinearity of any three points, ϕ preserves the
collinearity of any three collinear points.
Let C 0 = ϕ(C) and D0 = ϕ(D). Then by the similarity, C 0 lies on line AD and C 0 C||BD.
Similarly, D0 lies on BC and DD0 ||AC. Let E 0 = CC 0 ∩ DD0 . Since ϕ preserves collinear-
ity and (A, E, C) is a triple of collinear points, (ϕ(A), ϕ(E), ϕ(C)) = (C, ϕ(E), C 0 ) is a
triple of collinear points. Similarly, (D, ϕ(E), D0 ) is a triple of collinear points. Hence,
ϕ(E) = CC 0 ∩ DD0 = E 0 . Note that EDE 0 C is a parallelogram. Since H is the midpoint of
CD, EH = HE 0 and E, H, E 0 are collinear. Since ϕ preserves angles, ∠F EG = ∠F E 0 H. We
want to show this angle is equal to ∠GHE to solve the problem; i.e. GH||F E 0 .
a2k + 1
ak+1 = −1
ak−1 + 1
Since n ≥ 5,
(1) (a3 + 1)(a1 + 1) = a22 + 1.
(2) (a4 + 1)(a2 + 1) = a23 + 1.
(3) (a5 + 1)(a3 + 1) = a24 + 1.
We claim that each of a1 , a2 , a3 , a4 , a5 are even. If a1 is odd, then by (1), a2 is odd. Then by
(2), a3 is odd. Since a1 and a3 are both odd, in (1), a22 + 1 is divisible by 4. This is impossible
since no integer of the form a2 + 1 is divisible by 4. Therefore, a1 is even. If a2 is odd, then
similarly, a3 and a4 are odd. Sine a2 and a4 are both odd, in (2), a23 + 1 is divisible by 4. This
is again impossible. Therefore, a2 is even. If a3 is odd, then by (1), a2 is odd and by (3), a4
is odd. Again, a23 + 1 is divisible by 4, which is a contradiction. If a4 is odd, then by (2), a3 is
odd. By (1), a2 is odd. Again, a2 , a4 cannot both be odd. If a5 is odd, then a4 is odd, whose
case has already been handled. Therefore, each of a1 , · · · , a5 are even.
By (1) and (2), a3 + 1|a22 + 1 and a2 + 1|a23 + 1. It suffices to show that there are no pairs of
even positive integers x, y such that x + 1|y 2 + 1, y + 1|x2 + 1. Suppoe on the contrary that
there exist a pair of even numbers (x, y) such that x + 1|y 2 + 1 and y + 1|x2 + 1. Without
loss of generality, suppose x, y are chosen minimally and x ≥ y. If x = y, then x + 1|x2 + 1,
which implies that x + 1|2 ⇒ x = 1. This contradicts x being even. Therefore, x > y. The
conditions on x, y implies that x + 1|(y 2 + 1) + (x2 − 1) = x2 + y 2 . Similarly, y + 1|x2 + y 2 .
I claim that gcd(x + 1, y + 1) = 1. Let d = gcd(x + 1, y + 1). Then x, y ≡ −1 (mod d).
Hence, x2 + y 2 ≡ 2 (mod d). Since d|x + 1, d|x2 + y 2 . Hence, x2 + y 2 ≡ 0 (mod d). This
implies that 0 ≡ 2 (mod d). Therefore, d = 1 or d = 2. But d 6= 2 since x is even and
d|x + 1. Therefore, d = 1. Since x + 1|x2 + y 2 and y + 1|x2 + y 2 and gcd(x + 1, y + 1) = 1,
(x + 1)(y + 1)|x2 + y 2 . Let
x2 + y 2
= k. (16)
(x + 1)(y + 1)
for some positive integer k. This can be rewritten as x2 − k(y + 1)x + (y 2 − k(y + 1)) = 0.
Let x0 be the second root of t2 − k(y + 1)t + (y 2 − k(y + 1)) = 0, other than x. Note that x0
is an integer, since x is an integer and x + x0 = k(y + 1). Note that
x2 + y 2 y2 − x
k(y + 1) = =x+ .
x+1 x+1
2−x
If x < y 2 , then since x + x0 = k(y + 1) = x + yx+1 , x0 > 0. Since xx0 = (y 2 − k(y + 1)) and
x > y, x < y < x. Hence, x < x. We conclude that (x0 , y) is a smaller solution to (16),
0 0
contradicting the minimality of (x, y). If x > y 2 , then since x + 1|y 2 + 1, x + 1 ≤ y 2 + 1. Hence,
y 2 + 1 < x + 1 ≤ y 2 + 1. This is a contradiction. Finally, if x = y 2 , then since y + 1|x2 + 1,
y + 1|y 4 + 1. Since y + 1|y 4 − 1, y + 1|2. This implies that y = 1. This contradicts that y
is even. Therefore, there are no even numbers x, y such that x + 1|y 2 + 1 and y + 1|x2 + 1.
Hence, n 6≥ 5.
2010 Mock 5
Time: 4.5 Hours
1. Let O be the centre of the excircle of ∆ABC opposite A. Let M be the midpoint of AC, and
P the intersection of M O and BC. Prove that if ∠BAC = 2∠ACB, then AB = BP .
Solution 1: Let ∠ACB = θ. Then ∠BAC = 2θ and ∠ABC = 180 − 3θ. Note that O
lies on the angle bisector of ∠BAC. Therefore, ∠BAO = θ. Let D be the intersection
of AO and BC. To prove that BA = BP , it suffices to show that ∠BAP = 3θ/2, i.e.
∠DAP = ∠P AC = θ/2 ⇔ AP bisects ∠DAC ⇔ AD/AC = DP/P C. By Sine Law, we have
AD sin θ
= .
AC sin 2θ
Since M, P, O are collinear, by Menelaos Theorem on ∆ADC,
AO DP CM
· · = 1.
OD P C M A
Since CM = M A,
DP OD
= .
PC OA
Let r be the radius of the excircle of ∆ABC opposite A. Note ∠CDO = ∠BDA = 180 −
∠ABC − ∠BAO = 2θ. Let X, Y be the foot of the perpendicular from O on BC, AC,
r
respectively. Since OX = r, OD = sin ∠CDO = sinr2θ . Furthermore, OA = sin ∠CAO
r
= sinr θ .
Hence,
DP OA sin θ
= = .
PC OD sin 2θ
By looking at triangle ADC, by Sine Law,
AD sin ∠ACD sin ∠ACD sin θ DP
= = = = ,
AC sin ∠ADC sin(180 − ADC) sin 2θ PC
as desired.
Solution 2: Let AO and BC intersect at D. Since AO bisects ∠BAC and ∠BAC = 2∠ACB,
∠BAD = ∠CAD = ∠ACD. Therefore, AD = DC. Consider the triangles OAC and ODC.
Since they have equal altitudes from O and the same altitude from C,
[OAC] AC AO
= = .
[ODC] DC DO
Since M is the midpoint of AC, [OAM ] = [OCM ] and [P AM ] = [P CM ]. Therefore, [OAP ] =
[OCP ]. Then,
AC AC AO [OAP ] [OCP ] CP
= = = = = .
AD DC DO [ODP ] [ODP ] DP
Therefore, AP is the bisector of ∠DAC. It follows that ∠BAP = ∠BAD + ∠DAP =
∠ACP + ∠P AC = ∠AP B. Therefore, AB = BP , as desired.
2. Let f be a non-constant function from the set of positive integers into the set of positive
integers such that a − b divides f (a) − f (b) for all distinct positive integers a, b. Prove that
there exist infinitely many primes p such that p divides f (c) for some positive integer c.
Solution: Suppose the conclusion of the problem statement is false. Then there exist a finitely
number of primes p1 , · · · , pt such that the prime divisors of f (n) is a subset of {p1 , · · · , pt } for
every n ∈ N
First, I claim that there exists a positive integer N such that m ≥ N implies f (m) 6= f (1).
Suppose the contrary; i.e. there are infinitely many positive integers m such that f (m) = f (1).
Let k be the smallest positive integer such that f (k) 6= f (1). Such a positive integer exist
since f is a non-constant function. Let d = |f (k) − f (1)|. If f (m) = f (1) for some m > k + d.
Then m − k | |f (m) − f (k)| = |f (1) − f (k)| = d 6= 0 and m − k > d. This is a contradiction.
Let f (1) = pa11 · · · pat t for some non-negative integers a1 , · · · , at . Note that pm m m
1 p2 · · · pt di-
m m
vides f (p1 · · · pt + 1) − f (1). Consider this statement when m > max{a1 , · · · , at }. Then
pai i ||f (pm m m m a1 at
1 · · · pt +1) for all such m. This implies that f (p1 · · · pt +1) = p1 · · · pt = f (1), since
the only possible prime divisors of f (pm m m m
1 · · · pt + 1). But by our claim, f (p1 · · · pt + 1) > f (1)
when m gets sufficiently large. This is a contradiction.
Hence, there are indeed infinitely many primes p such that p divides f (c) for some positive
integer c.
3. A multiple choice contest with n questions was written by K students. The jury assigns the
difficulty to each question - a positive integer which is awarded to each student who solves the
question. If a student does not solve the question, the student gets 0 points. The student’s
score is the sum of the scores received for each question. It turns out that when the answer
sheets are submitted, the jury can assign the difficulty of each question in a way so that the
ranks of the students are in any pre-determined order. What is the maximum possible value
of K?
Label the students S1 , · · · , Sn . An example of n students where the rank of the students can
be pre-determined is by having each student solve a unique question Qi . If the ranking order
is S1 , · · · , Sn , then assign problem Qi a score of n + 1 − i for each i ∈ {1, · · · , n}.
Assume the result holds for K ≥ n + 1. Take n + 1 different students, and clone each of them
infinitely many times so that we get n + 1 types of students. Let us show that if we can find
two non-identical teams with finitely many students but with the same results, i.e. for every
question, the number of students on each team who solved the question is the same, we arrive
at a contradiction.
We can assume every type of students is present on at most one team (otherwise keep remov-
ing 1 student of the same type from both teams until this is no longer possible). Without
loss of generality, the first team has at least as many students as the second. The sum of the
scores of each of the two teams are the same. Then it is impossible to assign the difficulty
of questions so that each student on the first team is ranked higher than each student on the
second team.
It remains to find the two teams. We will suppose that it is possible and construct a system
of n linear equations on n + 1 variables. The non-zero solution of this system will correspond
to the assignment of the teams.
For j = 1, 2, · · · , n + 1, if the contestants that solved problem j are on team 1, then let xj be
the number of such contestants. If the contestants that solved problem j are on team 2, then
let xj be the negative number of such contestants.
For i = 1, 2, · · · , n let equation i state that the difference between the number of contestants
on the two teams who solved question i is 0; this is an equation involving only variables
x1 , x2 , · · · , xn+1 . We get a homogenous system of n equations and n + 1 unknowns. It has
a non-zero solution; it must be rational since all the coefficients in the equation are rational
numbers (namely, 0 or 1). Multiplying it by an appropriate constant still gives a solution
which contains only integers. This gives us the two teams.
Winter Camp 2011 Warmup Solutions
1 Problems
1.1 Algebra
A1. Prove that for any positive numbers a, b, c the following inequality holds:
p p p p
ab(a + b) + bc(b + c) + ca(c + a) > (a + b)(b + c)(c + a).
A2. Find all pairs of positive real numbers (x, y) that satisfy both
2 +y 2
2x + 2y +x = 8, and
√ √
x + y = 2.
A4. Find all functions f mapping rational numbers to real numbers, and satisfying
f (a + b + c) + f (a) + f (b) + f (c) = f (a + b) + f (b + c) + f (c + a) + f (0)
for rational numbers a, b, c.
1.2 Combinatorics
C1. There are eight rooks on a chessboard, no two attacking each other. Prove that some two of
the pairwise distances between the rooks are equal. (The distance between two rooks is the
distance between the centers of their cells.)
C2. Every vertex of the unit squares on an n × m grid is coloured either blue, green, or red, such
that all the vertices on the boundary of the board are coloured red. We say that a unit square
on the board is properly coloured if exactly one pair of adjacent vertices of the square are the
same colour. Show that the number of properly coloured squares is even.
C3. 3n points are marked on a circle, dividing it into 3n arcs, n of which have length 1, n others
have length 2, and n have length 3. Prove that it is possible to find two marked points
diametrically opposite to each other.
C4. In a round robin chess tournament each player faces every other player exactly once. For
every game, 1 point is awarded for a win, 0.5 points for a draw, and 0 points for a loss. Given
a positive integer m, a tournament is m−special if among every set S of m players, there is
one player who won all her games against the other m − 1 players in S and one player who
lost all her games against the other m − 1 players in S. For a given integer m ≥ 4, determine
the minimum value of n (as a function of m) such that in every m−special round robin chess
tournament with n players, the final scores of the n players are all distinct.
1 of 12
Winter Camp 2011 Warmup Solutions
1.3 Geometry
G1. Let ABCD be a fixed parallelogram with AB < BC, and let P be a variable point on side
CD. Let Q be the point on side BC so that P C = CQ. Prove that, as P moves, the
circumcircle of 4AP Q passes through a fixed point in addition to A.
G2. Let ABCD be a cyclic quadrilateral. The perpendiculars to AD and BC at A and C respec-
tively meet at M , and the perpendiculars to AD and BC at D and B meet at N . If the lines
AD and BC meet at E, prove that ∠DEN = ∠CEM .
G3. Let ABC be an acute-angled triangle with altitudes AD, BE, CF . Let the line tangent to the
circumcircle of 4ABC at A and the line tangent to the circumcircle of 4DEF at D intersect
at a point X. Define Y and Z analogously. Prove that X, Y, and Z are collinear.
G4. In an acute triangle ABC, the angle bisector of ∠A meets side BC at D. Let E and F be the
feet of the perpendiculars from D to AC and AB respectively. Lines BE and CF intersect
at H, and the circumcircle of 4AF H meets BE at H and G. Show that the triangle with
side lengths BG, GE, BF is right-angled.
N2. A sequence (ai ) of natural numbers has the property that for all i 6= j, gcd(ai , aj ) = gcd(i, j).
Show that ai = i for all i.
x2 −1
N3. Let’s say a natural number is Yorky if it can be expressed in the form y 2 −1
for integers
x, y > 1. Is is true that all but finitely many natural numbers are Yorky?
N4. Let a1 , a2 , a3 , . . . be a sequence of positive integers such that a1 > 1 and iteratively, for all
n ≥ 2, an is the smallest positive integer such that
Prove that the sequence contains every positive integer greater than one.
2 of 12
Winter Camp 2011 Warmup Solutions
2 Solutions
2.1 Algebra
A1.
p p p 2 X Xp
ab(a + b) + bc(b + c) + ca(c + a) = ab(a + b) + 2 ab2 c(a + b)(b + c)
cyc cyc
X
> ab(a + b) + 6abc
cyc
X
> ab(a + b) + 2abc
cyc
= (a + b)(b + c)(c + a).
Comment: Most Olympiad inequalities are tighter approximations, and steps like replacing
6abc with 2abc will not be good enough. Expanding can help though, and the cyclic sum
notation used here is a good way to write things down when you do expand.
Source: Russia 1990.
√ √ 1 √ √ 2
A2. Suppose x + y = 2. By the power-mean inequality, we have x + y ≥ 2 · x+ y ≥ 2,
and x2 + y 2 ≥ 12 · (x + y)2 = 2. Now, by the AM-GM inequality:
2 +y 2 +x
p
2x + 2y ≥ 2· 2x2 +y · 2y2 +x
x2 +y 2
+ x+y
= 21+ 2 2
≥ 8.
Equality holds if and only if x = y, so the only possible solution to the given pair of equations
is x = y = 1. Substituting this pair in, we see that it is indeed a valid solution.
Comment: Solving a system of equations like this requires doing two things: (a) proving
that nothing other than x = y = 1 works, and (b) proving that x = y = 1 does work. It is
always a good idea to check (b) separately. Your argument for (a) might take care of (b),
but it might not, and it’s better to be safe than sorry.
Source: Olymon 1997-1998.
A3. Let a = maxi∈{1,2,...,n} |xi |, let S0 denote the set of indices i such that xi = a, let S1 denote
the set of indices i such that xi = −a, and let S2 denote the remaining indices. Finally, let
b = maxi∈S2 |xi |. (If S2 is empty, then define b = 0.)
Consider replacing each xi in S0 with b and each xi in S1 with −b. This will have the following
effect on each |xi + xj | term:
• If i, j are both in S0 or both in S1 , then |xi + xj | will decrease from 2a to 2b. There are
exactly |S0 |2 + |S1 |2 ordered pairs (i, j) for which this case applies.
• If one of i, j is in S0 and the other is in S1 , then |xi + xj | will be 0 both before and after
the replacement, and therefore will be unchanged.
3 of 12
Winter Camp 2011 Warmup Solutions
= (a − b) · (|S0 | + |S1 |) · n.
On the other hand, n · ni=1 |xi | will decrease by exactly (a − b) · (|S0 | + |S1 |) · n. Therefore,
P
this replacement will always decrease the left-hand side by at least as much as it will decrease
the right-hand side by.
Performing this replacement repeatedly, we will eventually reach the configuration where
every xi is equal to 0, at which point it is obvious that the left-hand side is equal to the
right-hand side. Therefore, we must originally have had the left-hand side being greater than
or equal to the right-hand side, as required.
Solution #2: If {x1 , x2 , . . . , xn } contains only non-negative numbers or only non-positive
numbers, the claim is obvious.
Otherwise, assume without loss of generality that x1 , x2 , . . . , xm ≥ 0 and xm+1 , xm+2 , . . . , xn <
0. Let
x1 + . . . + xm xm+1 + . . . + xn
a= , b=− .
m n−m
Clearly, a, b ≥ 0. Returning to the original inequality, we can now write the left-hand side
minus the right-hand side in the following form:
m
X n
X m
X n
X m
X n
X
|xi + xj | + |xi + xj | + 2 |xi + xj | − n |xi | − n |xi |.
i,j=1 i,j=m+1 i=1 j=m+1 i=1 j=m+1
Now assume without loss of generality that a ≥ b. Since |x + c| is a convex function, Jensen’s
inequality implies LHS-RHS is greater than or equal to
n
2m2 a + 2(n − m)2 b + 2m
P
|a + xj | − nma − n(n − m)b
j=m+1
≥ 2m2 a + 2(n − m)2 b + 2m(n− m)(a − b) − nma − n(n − m)b
a 2m2 + 2m(n − m) − nm + b 2(n − m)2 − 2m(n − m) − n(n − m)
=
= amn + b(n2 − 5mn + 4m2 )
≥ bmn + b(n2 − 5mn + 4m2 )
= b(n2 − 4mn + 4n2 )
= b(n − 2m)2 ≥ 0.
4 of 12
Winter Camp 2011 Warmup Solutions
Comments: The technique of changing one or more variables, and seeing what happens to
the inequality is called “smoothing” or “mixing variables”. Usually, you make two variables
equal, or reduce one of them to zero. On some problems, smoothing is too messy to be useful,
but when the algebra is not too complicated, it is a very powerful technique.
Source: Iran 2006.
A4. Let us say a function f is “good” if it satisfies the given equation. Note that:
Fact1 : Fix real numbers a, b, c, t with t 6= 0. Then there exists a quadratic polynomial
P (x) such that P (−t) = a, P (0) = b, and P (t) = c.
Proof of lemma: Let P (x) be the quadratic polynomial that equals f (x) for x = −1 1
n , 0, n ,
−1 1
and let g(x) = f (x) − P (x). Then g is a good function satisfying g n = g(0) = g n = 0.
m
We need to show g n = 0 for all m.
Taking a = b = n1 and c = −1 1 −1 2 2
n , we have 3g n + g n = g n + 3g(0), and hence g n = 0.
Now, for any m, take a = b = n1 and c = m−2 n . Then
m 1 m−2 m−1 2
g + 2g +g = 2g +g + g(0),
n n n n n
Now pick an arbitrary good function f . By the lemma, there exists a quadratic polynomial P
t
suchthat f (m) = P (m) for all integers m. Suppose there exists a rational number n so that
t t
f n 6= P n . Applying the lemma again, there exists a quadratic polynomial P2 such that
f m m
n = P2 n for all m. But then P − P2 is a quadratic polynomial that has roots at -1,
0, and 1, but is non-zero at nt . However, this is impossible: a non-zero quadratic polynomial
can have at most 2 roots. Therefore it must be that f (x) ≡ P (x), and the problem is solved.
Comments: This problem generalizes Cauchy’s equation: f (a) + f (b) = f (a + b). Like
Cauchy’s equation, there can be all sorts of weird solutions if you let the domain of f be the
set of all real numbers. It depends on whether you assume the Axiom of Choice, and is far
beyond the scope of the IMO!
1
You can quote the following more general theorem: Given any k + 1 pairs (x1 , y1 ), (x2 , y2 ), . . . , (xk+1 , yk+1 ) with
xi 6= xj , there exists a unique degree-k polynomial P satisfying P (xi ) = yi for all i. In our case, the polynomial is
P (x) = a+c−2b
2t2
· x2 + c−a
2t
· x + b.
5 of 12
Winter Camp 2011 Warmup Solutions
2.2 Combinatorics
C1. Suppose that 8 rooks are placed on a chessboard with no two attacking each other. Then
there must be one rook in each row and one in each column. Consider a pair of rooks that
are
√ either √in adjacent√ rows or in √ adjacent columns. The distance between them is one of
12 + 12 , 12 + 22 , 12 + 32 , . . . , 12 + 72 . Note that there are exactly 7 distinct distances
of this form.
If all rook pairs are a different distance apart, it follows that there are at most 7 rook pairs
in adjacent rows or columns. However, there are exactly 7 pairs in adjacent rows (the rook
in row 1 paired with the rook in row 2, the rook in row 2 paired with the rook in row 3, etc.),
and exactly 7 pairs in adjacent columns. If these combine to give only 7 pairs altogether, it
must be that any time two rooks are in adjacent rows, they are also in adjacent √ columns. But
in this case, all 7 of these rook pairs are separated by a distance of exactly 2, and we have
a contradiction.
Solution #2: If there are 8 rooks on a board, this gives 82 = 28 pairs of rooks. The
p
distance between any pair must be of the form i2 + j 2 for i, j ∈ {1, 2, . . . , 7}. There are
7
exactly 2 +7 = 28 such distances. Therefore, if every pair of rooks is separated by a different
distance, each of these 28 distances must be used.
√
In particular, there are two rooks separated by a distance of 72 + 72 , meaning there are two
rooks in opposite corners. But then no more rooks can be placed on the boundary√without
attacking one of these two. Hence, no two rooks can be separated by a distance of 72 + 62 ,
and we have a contradiction.
Comments: With Pigeonhole Principle arguments like this, it is always a good idea to count
the number of possible distances and the number of rook pairs. In this
case, the total number
of rook pairs and the total number of possible distances are both 82 , which is not enough for
a contradiction. You need to tweak the argument a little bit, like we did here.
Source: Russia 2002.
C2. We will call an edge of a square good if both its endpoints are of the same colour, and bad
otherwise. Any square can have either no good edges (call such squares type A), one good
edge (type B), two good edges (type C), or four good edges (type D). Note that it cannot
have three good edges.
Let the number of squares of type A, B, C, D be a, b, c, d respectively. For each square, count
the number of good edges, and add the numbers. We must get an even number, since each
edge not on the boundary is counted twice, and all 2(m + n) edges on the boundary are good.
On the other hand, this number is b + 2c + 4d. Hence b is even and the result follows.
Comments: We are counting in two different ways the number of (square, good edge) pairs.
This kind of argument is very important in combinatorics problems!
Source: India 2001.
6 of 12
Winter Camp 2011 Warmup Solutions
number of blue points are both equal to 3n, all red points must be used in this way, and
therefore each red point is also diametrically opposite a blue point.
Let us call any arc between two adjacent blue points “elementary”. Consider an elementary
arc AC of length 2. Its endpoints are blue, and its midpoint B 0 is red. As argued above, the
point B diametrically opposite to B 0 must be blue.
Now suppose arc AB contains a1 elementary arcs of length 1, a2 elementary arcs of length
2, and a3 elementary arcs of length 3. Each length-1 elementary arc in AB is diametrically
opposite a length-3 elementary arc in BC: the two blue endpoints of the length-1 arc must
be opposite red points and no elementary arc can have length greater than 3. Conversely,
each length-3 elementary arc in BC is diametrically opposite a length-1 elementary arc in
AB. Therefore, the total number of length-3 elementary arcs in BC is exactly a1 . Since the
total number of length-3 elementary arcs on the whole circle is k, we have a1 = k − a3 .
Finally, arc AB is of length 3k − 1, so we have 3k − 1 = a1 + 2a2 + 3a3 = 2a2 + 2a2 + k.
Therefore, 2k − 1 = 2a1 + 2a2 , but this is impossible because the left-hand side is odd, and
the right-hand side is even.
Source: Russia 1982.
7 of 12
Winter Camp 2011 Warmup Solutions
tournament with no draws and with no 3-cycles. By the same argument, this tournament also
has a player Pn−1 who beats all remaining players. Proceeding recursively, we can label the
players P1 , P2 , . . . , Pn so that player Pi beats Pj whenever i > j. Notice that no two players
can have the same score for this kind of tournament.
Therefore, if two players have the same score, then the tournament must have a tie or a
3-cycle. In the case of m-special tournaments, we showed that this means n ≤ 2m − 4, and
the proof is complete.
Comments: In the final case, we proved one player had beaten all other players by starting
with the player who had the most wins and analyzing what properties she must have. This
is called an extremal argument, and it is very useful.
Source: China 2007.
2.3 Geometry
G1. Let E be the reflection of A through the angle bisector of ∠BCD. Since P is the reflection of
Q through the same line, it follows that P Q and AE are both perpendicular to the bisector,
and that AQ = EP . In particular, AQP E is an isosceles trapezoid, and is therefore cyclic.
It follows that the circumcircle of 4AP Q passes through E no matter where P is.
B Q C
A D
Comments: For many problems where you need a show a shape always passes through a
fixed point, the key is to first figure out where that point is, and then go from there. Always
have a straight-edge and compass ready to help with this kind of guessing!
Source: Russia 2005.
G2. Assume without loss of generality that E is closer to AB than to CD. Since ∠EBN =
∠EDN = 90◦ , the points E, B, D, N lie on a circle. (We do not know what order they lie in
however.) Similarly, the points E, A, C, M lie on a circle.
Now, suppose segments BN and DE cross. Then ∠EBD > ∠EBN = 90◦ , and ∠EAC =
180◦ − ∠CEA − ∠ECA = 180◦ − ∠DEB − ∠EDB = ∠EBD > 90◦ . It follows that segments
AM and CE also cross, as shown in the left diagram.
In this configuration, we have ∠DEN = 180◦ − ∠BED − ∠BDN = 90◦ − ∠BED − ∠BDA =
90◦ − ∠AEC − ∠ACB = 180◦ − ∠AEC − ∠ACM = ∠CEM .
8 of 12
Winter Camp 2011 Warmup Solutions
If segments AM and CE cross, then segments BN and DE also cross as argued above, and
we are in the same case as before. Otherwise, it must be that AM does not cross CE, and
BN does not cross DE, as shown in the right diagram.
In this configuration, we have ∠DEN = ∠DBN = ∠DBC −90◦ = ∠CAD −90◦ = ∠CAM =
∠CEM . Thus, ∠DEN = ∠CEM in all possible configurations.
N
D
A
M C
E
B
B
C E
A
N
M D
Solution #2: We know ∠ACE = ∠ACB = ∠BDA = ∠BDE and ∠CEA = ∠DEB, so
4EAC must be similar to 4EBD.
Also, E, A, C, M lie on a circle with diameter EM since ∠EAM = ∠ECM = 90◦ . In
particular, this means AM is the diameter of the circumcircle of 4EAC. Similarly, BN is
the diameter of the circumcircle of 4EBD.
Now consider the similarity transformation (rotation, scaling, translation, and/or reflection)
taking 4EAC to 4EBD. This transformation will also take the point on the circumcircle
of 4EAC opposite A to the point on the circumcircle of 4EBD opposite B; i.e., it will take
M to N . Thus, the entire figure EACM is similar to EBDN , and hence ∠DEN = ∠CEM .
Comments: Many students will just assume one configuration and ignore the possibility
of the other one, even when the argument needs to be tweaked. This will usually cost you
a point. In this problem, it would cost you multiple points. Always, always, always pay
attention to all possible configurations!
Source: Iran 2004.
G3. Let 4ABC have angles a, b, c, and assume without loss of generality that X lies on the same
side of AD as C. Then ∠XAD = ∠XAC + ∠CAD = b + (90◦ − c).
Now, ∠AF C = ∠ADC = 90◦ so AF DC is a cyclic quadrilateral. Similar arguments apply
for BDEA and CEF B. Therefore,
∠XDA = ∠XDE + ∠EDA = ∠DF E + ∠EDA = ∠DF C + ∠CF E + ∠EDA
= ∠DAC + ∠CBE + ∠EBA = (90◦ − c) + (90◦ − c) + (90◦ − a)
= 90◦ + b − c = ∠XAD.
Therefore, 4XAD is isosceles and XA = XD. Since XA and XD are both tangents, it
follows that X has equal power with respect to the circumcircles of 4ABC and 4DEF . In
9 of 12
Winter Camp 2011 Warmup Solutions
particular, X is on the radical axis of these circles. The same argument can be applied to Y
and Z, and so all three points lie on the one line as required.
C
X
D
A F B
Comments: As with G1, the trick here is to guess the line in advance. Once you know the
goal is to prove X, Y, Z are on the radical axis of the two circles, it is straightforward. If you
don’t know what power of a point and radical axes are, you should look them up. They are
important!
E
K
D
H
G
A F B
10 of 12
Winter Camp 2011 Warmup Solutions
2011 2011
X 1 1 X
= · xi .
3ai − 1 X
i=1 i=1
N2. For any integer n, we have n = gcd(n, 2n) = gcd(an , a2n ), which divides an . Therefore, n
divides an for all n.
Now suppose there exists n for which an 6= n. Then an = nm for some m > 1. Since nm
also divides anm , we have nm| gcd(an , anm ). However, nm does not divide n = gcd(n, nm),
so this is a contradiction.
It follows that an = n for all n.
Source: Russia 1995.
N3. Let p be an odd prime. We claim that p2 is not Yorky, which implies there are infinitely many
non-Yorky numbers.
x2 −1
Indeed, suppose y 2 −1
= p2 . Then:
x2 − 1 = y 2 p2 − p2
⇒ p2 − 1 = y 2 p2 − x2
⇒ p2 − 1 = (yp − x)(yp + x).
11 of 12
Winter Camp 2011 Warmup Solutions
expression. The advantage of trying to construct examples is you can restrict your attention
to whatever is easy to work with!
Source: Russia 2010.
N4. Lemma 1: Let n > 1 be a positive integer. If the sequence contains infinitely many multiples
of n, then it contains all multiples of n.
Lemma 2: Let n > 1 be a positive integer. If the sequence contains infinitely many multiples
of n, then it contains all positive integers greater than 1.
Proof of lemma 2: Let m > 1 be an arbitrary positive integer. By Lemma 1, the se-
quence contains all multiples of n, including all multiples of nm. Therefore, the sequence
contains infinitely many multiples of m, which means it contains all multiples of m, including
m itself. Since m was arbitrary, the proof is now complete.
We now show that the sequence must contain infinitely many prime numbers. Suppose
by way of contradiction that it contains only the primes p1 , p2 , . . . , pk . By Lemma 2, it can
contain only finitely many multiples of each of these primes. Therefore, it must eventually
reach a number m not divisible by any of them. Let q be the smallest prime factor of m.
Then q is the smallest number sharing a non-trivial divisor with m, so m will be immediately
followed by q if q has not already appeared in the sequence. Either way, q appears at some
point, and we have a contradiction.
Therefore, the sequence does indeed contain infinitely many prime numbers. For any prime p,
the smallest number sharing a non-trivial divisor with p is 2p. If 2p has not already appeared
in the sequence, it will therefore immediately follow p. Thus, having infinitely many primes
in the sequence guarantees that there will be infinitely many even numbers, and we are now
done by Lemma 2.
Comments: The devil here is in the details. Take a look at how the proof is made rigorous,
and make sure you could do the same thing!
Source: China 2010.
12 of 12
Winter Camp 2011 Buffet Problems
1 Problems
1.1 Algebra
A1. Positive real numbers x, y, z are given such that the difference between any two of them is
less than 2. Prove that:
p p √
xy + 1 + yz + 1 + zx + 1 > x + y + z
A2. Let M be a set of n ≥ 4 points in the plane, no three of which are collinear, and not all lying
on a circle. Suppose that f is a function assigning a real number to each point in M such
that for any circle C passing through at least three points of M ,
X
f (P ) = 0.
P ∈M ∩C
1.2 Combinaotircs
C1. All entries of an 8 × 8 matrix are positive integers. One may repeatedly transform the entries
of the matrix according to the following rules:
Prove that it is possible to transform the given matrix into the zero matrix.
C2. Let n, k be positive integers and I1 , I2 , . . . , In be n closed intervals on a line such that among
any k of the intervals, there are two with non-empty intersection. Prove that one can choose
k − 1 points on the line such that any of the intervals contains at least one of the chosen
points.
C3. In a certain country, every town is connected by a road to exactly three other towns. A
tourist traveling by roads visited each town exactly once and returned to the initial town.
Next year he comes back for a round trip different from the last years trip (not the original
path in either regular or reverse order), again visiting each town exactly once. Prove that he
can always do so.
1 of 8
Winter Camp 2011 Buffet Problems
1.3 Geometry
G1. Let ABC be a triangle. Let P be the point on line BC such that B is between P and C, and
BP = BA. Similarly, let Q be the point on line BC such that C is between Q and B, and
CQ = CA. If R is the second intersection of the circumcircles of 4ACP and 4ABQ, prove
that 4P QR is isosceles.
G2. Points K, L, M, N are respectively the midpoints of sides AB, BC, CD, DA in a convex quadri-
lateral. Line KM meets diagonals AC and BD at points P and Q, respectively. Line LN
meets diagonals AC and BD at points R and S, respectively. Prove that if AP ·P C = BQ·QD,
then AR · RC = BS · SD.
G3. Let ABCD be a convex quadrilateral whose opposite sides are not parallel. Let E, F be the
intersections of the opposite sides of ABCD, P the intersection of AC and BD, and J the
foot of the perpendicular from P to EF . Prove that ∠AJD = ∠BJC.
N2. Prove that for any positive integer n greater than 10000, there is a positive integer m that
√
can be written as a sum of two squares, such that 0 < m − n < 3 4 n.
N3. k is a given natural number. Find all functions f mapping natural numbers to natural
numbers, such that for all pairs of natural numbers m, n, we have
2 of 8
Winter Camp 2011 Buffet Problems
2 Solutions
2.1 Algebra
A1.
x+y 2
|x − y| < 2 ⇒ (x − y)2 < 4 ⇒ x2 + 2xy + y 2 < 4 + 4xy ⇒ ( ) < xy + 1
2
x+y p y+z p z+x √
Hence < xy + 1. Similarly < yz + 1 and < zx + 1. Adding these
2 2 2
three inequalities we get the result.
Source: Russia 2004.
n
A2. Label all possible pairs of points in M from 1 to K = . Look at an arbitrary pair i of
2
points Ai , Bi in M and consider the set Ci of all circles passing through the two points in the
pair and through at least one other point in M . The number of circles in Ci is mi ≥ 2, since
not all points lie on a circle. Label the circles from ω1i to ωmi .
i
X
Let Si = f (Ai ) + f (Bi ) and S = f (P ). We have:
P ∈M
mi
X X S
S=[ f (P )] − (mi − 1)Si = (1 − mi )Si ⇒ Si = (1)
1 − mi
j=1 P ∈ωm
i
j
K K
X Si 1
X
Assume S 6= 0. Equation (1) holds for all i hence n−1 = = . But 1−mi < 0
S 1 − mi
i=1 i=1
for all i which gives n − 1 < 0, a contradiction. Hence S = 0, and equation (1) implies Si = 0
for all i, hence f (P ) = 0 for all points P .
Source: Romania 1998.
A3. Let bi = a1 + ... + ai . Then b1 ≤ b2 ≤ ... ≤ bn . For any positive integers l < k with m = k − l:
al+1 + al+2 + . . . + ak bk − bl
=
m k−l
bk − bl
Consider points B0 (0, 0), B1 (1, b1 ), B2 (2, b2 )..., Bn (n, bn ) in the coordinate plane. Then
k−l
is equal to the tangent of the angle formed by line Bk Bl and the x−axis. The condition
mk > x is equivalent to the condition that the line lk passing through Bk with the slope angle
of tan−1 (x) lies above at least one point Bl for l < k. We will call such a point Bk good.
a1 + a2 + . . . + an bn
Also = , which is the distance between the point (n, 0) and the point of
x x
intersection of line ln with the x−axis.
Let us prove by induction on n that this distance is greater than the number of good points.
The base case is clear. If point Bn is not good, remove it; then the number of good points
does not change and the distance decreases, since bn−1 ≤ bn . If Bn is good, let k be the
largest integer such that Bk lies below ln . Remove points Bk+1 through Bn ; the number of
good points will then decrease by n − k, and the distance will decrease by more than n − k,
which finishes the induction step.
Source: Russia 2000.
3 of 8
Winter Camp 2011 Buffet Problems
2.2 Combinatorics
C1. Repeat the following procedure on the left-most column. If all integers in the left-most column
are greater than 1, subtract 1 from all entries in the column. Otherwise, multiply by 2 all
entries in every row containing a 1 in the column. Repeat this procedure until all entries in
the left-most column are 1. The process will eventually stop since the difference between the
smallest and the greatest entries in the column is non-decreasing, and it cannot stay constant
forever. Once all entries in the left-most column are 1, subtract 1 from every entry in the
column, so that the left-most column contains all zeroes. Once all entries in the left-most
column are 0, perform the same procedure on the next column, and so on.
Source: Netherlands 1999.
C2. Denote by P1 the left-most right endpoint of an interval. Throw away all intervals containing
P1 , and let P2 be the left-most right endpoint of the remaining intervals. Define P3 , P4 . . . in
the same way until all intervals have been thrown away. Let Pm be the last point defined. The
intervals with the right endpoints P1 , P2 , . . . , Pm cannot intersect, hence m ≤ k − 1. Every
interval that has been thrown away contains at least one of the chosen points, hence every
interval in the collection contains one of the chosen points.
C3. Consider a graph where the vertices are the towns, and edges are the roads. We call a directed
path P1 , P2 , . . . , Pn Hamiltonian if the path contains every vertex in the graph exactly once.
We call such a Hamiltonian path adjacent to another Hamiltonian path, if the latter path can
be obtained from the first one by inserting edge Pn Pi and deleting edge Pi Pi+1 for i > 1, so
that the latter path is P1 , P2 , . . . , Pi , Pn , Pn−1 , . . . , Pi+1 . Since Pn is adjacent to exactly 3 other
vertices, every Hamiltonian path is adjacent to two paths or to one path, the latter case hap-
pening only if Pn is adjacent to P1 . (1)
Consider a graph G where vertices are the paths, and edges join adjacent paths. Look at the
cycle along which the tourist traveled the first time. Remove one edge in the cycle to get
a Hamiltonian path X = S1 , S2 , . . . , Sn . Look at the subgraph of G which contains path X
and all paths that can be reached from X by following the edges of G. This subgraph must
be itself a path because of (1). Let Y = T1 , T2 , . . . , Tn be the endpoint of this path different
from X. Then Tn is adjacent to T1 , so path Y can be extended to a Hamiltonian cycle. By
definition of adjacency of Hamiltonian paths, T1 = S1 , T2 = S2 , hence the new cycle is not
the reverse of the old one. Since X 6= Y , the new cycle is different from the old one, hence it
satisfies the required conditions.
Source: Japan 2004
Comment: The result also holds if we only assume that all vertices in the graph have odd
degree. Think about how the above solution needs to be modified for this more general
problem.
4 of 8
Winter Camp 2011 Buffet Problems
2.3 Geometry
G1. Solution 1: Let S be the intersection of AR with the circumcircle of 4P AQ. Then:
1 1
∠P SA = ∠AQP = ∠ACP = ∠ARP ⇒ ∠P SA = ∠SP R ⇒ RP = RS
2 2
Similarly RQ = RS. Hence RP = RQ.
P Q
B C
R
BQ AP
(We use directed lengths). From equations (3), (4) it follows that QD = PC . Similarly it
AR
follows that RC = DS
SB .
5 of 8
Winter Camp 2011 Buffet Problems
D
M
C
Q
P
N
R S L
A K B
BQ AP
Since QD = P C , AP · P C = BQ · QD, we have |AP | = |BQ|, |P C| = |QD|, and |AC| = |BD|.
BQ AP
Since |AC| = |BD|, QD = PC we have |AR| = |DS|, |RC| = |SB|. Then AR · RC = BS · SD,
as required.
Source: Mathlinks.
G3. Lemma: Points A, C, B, D lie on a line in this order. P is a point not on on this line. Then
any two of the following conditions imply the third:
CA DB
1. · = −1.
CB DA
2. P B is the angle bisector of ∠CP D.
3. AP ⊥ P B.
Proof: Assume 2 and 3 hold. Then P A, P B are the external and internal angle bisectors of
∠CP D, hence:
|CA| |CP | |CB| CA DB
= = ⇒ · = −1
|DA| |DP | |DB| CB DA
Assume 1 and 2 hold. Let A0 be a point on line CD such that A0 C < A0 D and A0 P ⊥ P B.
0
Then CA DB 0
CB · DA0 = −1. Since 1 holds, it follows that A ≡ A and AP ⊥ P B.
Assume 1 and 3 hold. Let C 0 be on line segment AB such that P B is the angle bisector of
C 0 A DB
∠C 0 P D. Since 3 holds, it follows that C 0 B · DA = −1. Since 1 holds, it follows that C ≡ C
0
AQ AP BP BQ0
= , = 0
QC PC PD QD
6 of 8
Winter Camp 2011 Buffet Problems
Q’
F
J
Q
D
C
P
A B E
AP AQ BP BQ0
Hence, PC = QC . Similarly, PD = Q0 D . This completes the problem.
Source: Mathlinks.
Comment: Consider four points A, C, B, D, occurring on a line in this order. These points
CA DB
are called harmonic iff (A, B; C, D) = · = −1. Let P be a point not collinear with
CB DA
A, B, C, D; we define the pencil P (A, B, C, D) to be made up of 4 lines P A, P B, P C, P D.
There are a few useful results involving harmonic points that make them a powerful geometry
tool.
Fact: A pencil P (A, B, C, D) is given. The lines P A, P B, P C, P D intersect a line l at A0 , B 0 ,
C 0 , D0 respectively. Then (A0 , B 0 ; C 0 , D0 ) = (A, B; C, D).
Fact: In 4ABC, points D, E, F are on sides BC, CA, AB. Let F E intersect BC at G. Then
(B, C; D, G) is harmonic iff AD, BE, CF are concurrent.
Fact: The lemma used in the solution. Condition 1 is equivalent to (A, B; C, D) = −1.
√
a2 ≤ n < (a + 1)2 ⇒ n − a2 ≤ 2a ≤ 2 n
√
p q
2 2 2
(b − 1) ≤ n − a ⇒ b − 1 ≤ n − a ≤ 2 n
√ √
(b − 1)2 ≤ n − a2 < b2 ⇒ m − n = b2 − (n − a2 ) ≤ 2b − 1 ≤ 2 2 4 n + 1
√ √ √
It is clear that 2 2 4 n + 1 < 3 4 n for n > 10000, and we are done.
Source: Russia 2002.
7 of 8
Winter Camp 2011 Buffet Problems
N3. We first prove that f is injective. Assume f (a) = f (b) for a 6= b, then for every positive
integer n, we have f (a) + f (n)|(a + n)k , f (b) + f (n)|(b + n)k . Since f (a) + f (n) = f (b) + f (n),
it follows that f (a) + f (n) is a common divisor of (a + n)k and (b + n)k . Since gcd(a + n, b +
n) = gcd(a + n, b − a), we can take n such that a + n is a prime greater than b − a, then
gcd(a + n, b + n) = 1, and f (a) + f (n) cannot be a common divisor of (a + n)k and (b + n)k .
Fix a positive integer m. For every n, f (n) + f (m)|(n + m)k , f (n) + f (m + 1)|(n + m + 1)k .
Since gcd(m + n, m + n + 1) = 1, it follows that
gcd(f (n) + f (m), f (n) + f (m + 1)) = gcd(f (n) + f (m), f (m + 1) − f (m)) = 1 (4)
Assume there is a prime p dividing f (m + 1) − f (m). Let a be a positive integer such that
pa > m. Let n = pa − m, then f (n) + f (m)|(n + m)k = pak , hence p|f (n) + f (m). But then
p| gcd(f (n) + f (m), f (m + 1) − f (m)), contradicting (4). Hence f (m + 1) − f (m) = ±1. Since
f is injective, f (m + 1) − f (m) is always 1 or always -1 for all m. Since f only takes positive
values, f (m + 1) − f (m) = 1 for all positive integers m.
Therefore for some non-negative integer c, f (n) = n + c, for all n. If c > 0, let p be a prime
greater than 2c. Then f (1) + f (p − 1)|pk , hence p|f (1) + f (p − 1) = 2c + p, which is impossible
since 0 < 2c < p. Hence c = 0 and the only solution is f (n) = n for all positive integers n.
Source: Iran 2008.
8 of 8
Winter Camp 2011 Mock Olympiad Solutions
1 Problems
1. Sequences (an ), (bn ), (cn ), and (dn ) satisfy the following conditions:
an+1 = an + bn , bn+1 = bn + cn ,
cn+1 = cn + dn , dn+1 = dn + an ,
2. Let m, n be positive integers. Show that the number of positive integers x such that
n j k
X x
=x−m
2i
i=1
is at most 2n .
3. In a cyclic quadrilateral ABCD with AB = AD, points M, N lie on the sides BC and CD
respectively so that BM + N D = M N . Lines AM and AN meet the circumcircle of ABCD
again at points P and Q respectively. Prove that the orthocenter of the triangle AP Q lies on
the segment M N .
4. Let n ≥ 3 be an odd integer. Amy has coloured the squares in an n × n grid white and black.
We will call a sequence of squares S1 , S2 , . . . , Sm a “path” if all these squares are the same
colour, if Si and Si+1 share an edge for all i ∈ {1, 2, . . . , m − 1}, and if no other squares in
the sequence share an edge. Prove that if both the white squares and black squares form a
single path, then one of these paths must begin or end at the center of the grid.
For example, the grid on the left is a valid colouring, but the other two are invalid. In the
middle grid, the white squares loop back and touch themselves, and in the right grid, the
black squares are disconnected.
1
A sequence (tn ) is said to be “periodic” if there exists a positive integer p for which tn+p = tp for all n.
1 of 5
Winter Camp 2011 Mock Olympiad Solutions
2 Solutions
1. If a sequences (tn ) is periodic, note that |tn | must be upper-bounded by some constant2 .
Specifically, if the sequence has period p, then an ∈ {a1 , a2 , . . . , ap } for all n, and hence
|an | ≤ max(a1 , a2 , . . . , ap ).
In our case, this means the sequence (an + bn + cn + dn ) is bounded (since (an ), (bn ), (cn ), and
(dn ) are all bounded). Substituting the given recurrence in for n ≥ 1, we have an+1 + bn+1 +
cn+1 + dn+1 = 2(an + bn + cn + dn ). To prevent |an + bn + cn + dn | from getting arbitrarily
large, an + bn + cn + dn must be 0 for all n.
If n ≥ 2, we then have:
an+2 = an+1 + bn+1
= an + 2bn + cn
= an−1 + 3bn−1 + 3cn−1 + dn−1
= 2bn−1 + 2cn−1
= 2bn .
Similarly bn+2 = 2cn , cn+2 = dn , and dn+2 = 2an . Therefore, an+8 = 2bn+6 = 4cn+4 =
8dn+2 = 16an , and so |an | will become arbitrarily large unless a2 = 0. Thus, a2 = 0. The
same reasoning also implies b2 = c2 = d2 = 0.
Source: Yugoslavia 1992.
2. Let f (x) = x − ni=1 2xi .
P
Any integer x can be written uniquely in the form a · 2n + b for non-negative integers a, b with
b < 2n , Using this notation, we have:
n
a · 2n + b
X
n n
f (x) = f (a · 2 + b) = a · 2 + b −
2i
i=1
n n
n
X
n−i
X b
= a·2 +b− a·2 −
2i
i=1 i=1
n
X
= a · 2n + f (b) − a · 2n−i
i=1
= a + f (b).
Now fix a positive integer m. For any b, there is at most one a such that f (x) = a + f (b) = m.
Since there are only 2n legal values of b, there are at most 2n values of x for which f (x) = m.
Comment: You can think of 2xi as stripping off the last i binary digits from x. Given this,
it is natural to consider a binary representation of x. The main insight is to group the last n
digits, and handle everything else separately.
Source: Adapted from Iran 2008.
3. Let H be the point on N M such that N H = N D and M H = M B.
2
This is not true for most aperiodic sequences. For example, if tn = n for all n, there is no such thing as maxn |tn |.
For any constant C you choose, I can find an n such that |tn | > C.
2 of 5
Winter Camp 2011 Mock Olympiad Solutions
M’
D
A
N
Q
H B
M
C P
3 of 5
Winter Camp 2011 Mock Olympiad Solutions
Lemma: Let A be an origin point. Then there exist two line segments `1 and `2 connecting
A to either a boundary point or a corner point, and satisfying the following conditions: (1)
`1 and `2 form a 45◦ angle with the gridlines, (2) `1 and `2 are perpendicular to each other,
and (3) `1 and `2 do not pass through any other origin points.
Proof of Lemma: First note that no 2 × 2 square can form a checkerboard pattern with two
opposite squares being white, and the other two being black. If this happened, then the two
white squares would have to be connected by a single path, but this path would disconnect
the two black squares, which is not allowed.
Now consider the endpoint A of a single path. Assume without loss of generality that A is
white, and let B be the one white square adjacent to A:
X
@
@
4@ 3 3 4
3 @2 1 2 3
1 @A 1
B
In the picture above, the squares labeled 1 must all be black since A is assumed to be an
endpoint. The squares labeled 2 must also be black to avoid a 2 × 2 checkerboard. But now
the squares labeled 2 already have two black neighbours, so the squares labeled 3 must be
white. And then the squares labeled 4 must be white to avoid a 2 × 2 checkerboard. This
same argument can be extended until we reach squares on the boundary, say X and Y .
Since X and Y already have two neighbours of the same colour, they must either be at a
corner of the grid, or the other square they are adjacent to must be of the opposite colour.
Thus, the lines through the center of A and the centers of X, Y must end at either a corner
point or a transition point. Note that this is true even if A itself is on the boundary.
It is clear that these two lines are perpendicular and form a 45◦ angle with the gridlines.
Finally, they cannot pass through another origin point because each square between A and
X, Y must have two neighbours of the same colour.
Next, let us consider the boundary of the grid. Suppose it has two disjoint white components
W1 and W2 . Then there must also be two disjoint black components B1 and B2 . W1 and W2
must be part of the same path, but this path would have to disconnect B1 and B2 , which is
not allowed. Therefore, the boundary contains only one white component and only one black
component, and hence there are exactly two transition points.
Consider the following eight 45◦ diagonals: one emerging onto the grid from each corner point
(call these corner diagonals), and two emerging onto the grid from each transition point (call
these transition diagonals). By the Lemma, each origin point must be at the intersection of
two perpendicular diagonals. Since there are only eight diagonals altogether, each diagonal
can be used by only one origin point, and there are four origin points, each diagonal must be
used. If two transition diagonals are used by the same origin point, then another origin point
must use two corner diagonals to compensate, and hence be in the center of the grid. The
problem is solved in that case.
4 of 5
Winter Camp 2011 Mock Olympiad Solutions
Otherwise, every transition diagonal must intersect a corner diagonal at an origin point. Let
A, B, C, D be the four corners of the grid, and let P be a transition point. Without loss of
generality, we can assume that P lies on side AB. If each grid square has side length 1, then
AB has odd length, so either AP or P B is even. Assume further without loss of generality
that AP is even. But then the diagonal through P parallel to BD never intersects BD, and it
intersects AC only on the corner of a square. Therefore, it cannot possibly intersect a corner
diagonal at an origin point, and we have the desired contradiction.
Source: Adapted from Google Code Jam Finals 2010.
5 of 5
Winter Camp 2014 Pre-Camp Problem Set Solutions
1 Algebra
1. Find all real numbers x, y and z which satisfy the simultaneous equations x2 − 4y + 7 = 0,
y 2 − 6z + 14 = 0 and z 2 − 2x − 7 = 0.
2. Let a1 , a2 , . . . , an and b1 , b2 , . . . , bn be positive integers such that a1 < a2 < . . . < an , b1 >
b2 > . . . > bn , and (a1 , a2 , . . . , an , b1 , b2 , . . . , bn ) is a permutation of (1, 2, . . . , 2n). Prove that
is a perfect square.
3. Prove that if a, b, c, d are non-negative real numbers, no two of which are zero, then
ab ac ad bc bd cd
+ 2 + 2 + 2 + 2 + 2 ≥ 3.
c2 +d 2 b +d 2 b +c 2 a +d 2 a +c 2 a + b2
When does equality hold?
2 Combinatorics
1. 100 queens are placed on a 100 × 100 chessboard so that no two attack each other. Prove
that each of the four 50 × 50 corners of the board contains at least one queen.
2. Consider a planar region of area 1 which is the union of circular disks. Prove that from these
disks we can select some that are mutually disjoint and have total area of at least 91 .
3. On each of 12 points around a circle we place a disk with one white side and one black side.
We may perform the following move: select a black disk, and reverse its two neighbors. Find
all initial configurations from which some sequence of such moves leads to a position where
all disks but one are white.
1 of 9
Winter Camp 2014 Pre-Camp Problem Set Solutions
3 Geometry
1. Let A, B, C and D be points on a line in that order, and let P be a point not on the line such
that AB = BP and P C = CD. If the circumcircles of ACP and BDP intersect at P and Q,
prove that Q is equidistant from A and D.
3. Five points are given on a circle. A perpendicular is drawn through the centroid of the
triangle formed by any three of them to the chord connecting the remaining two. Such a
perpendicular is drawn for each triplet of points. Prove that the ten lines obtained in this
way have a common point.
4 Number Theory
1. Natural numbers a and b are such that a+1 b +
b+1
a is an integer. If d is the greatest common
2
divisor of a and b, prove that d ≤ a + b.
4. Prove that infinitely many primes divide at least one number of the form 1! + 2! + . . . + n!.
2 of 9
Winter Camp 2014 Pre-Camp Problem Set Solutions
5 Algebra Solutions
1. If we add the three given equations, we get
x2 − 4y + 7 + y 2 − 6z + 14 + z 2 − 2x − 7 = 0
=⇒ (x − 1)2 + (y − 2)2 + (z − 3)2 = 0
Therefore, we can only have x = 1, y = 2, z = 3. Checking, we see these values do indeed
satisfy the given equations. (England 2013)
Comment: You have to check your answer for questions like this (even if you do not show
much work). We first showed that if (x, y, z) satisfy the three given equations, then they must
be (1, 2, 3). It could still be that there are no (x, y, z) which satisfy the given equations. For
example, think about what happens if we replace the first equation with x2 − 4y + 6 = 0 and
the second equation with y 2 − 6z + 15 = 0.
2. Suppose ai > bi for some i. Then ai > a1 , a2 , . . . , ai−1 , bi , bi+1 , . . . , bn . Therefore, ai is greater
than n different terms from {1, 2, . . . , 2n}, and so must be at least n + 1. Similarly if bi > ai ,
then bi > a1 , a2 , . . . , ai , bi+1 , bi+2 , . . . , bn , and so must also at least n + 1.
Therefore, {max(a1 , b1 ), max(a2 , b2 ), . . . , max(an , bn )} is a subset of {n + 1, n + 2, . . . , 2n}.
Since both sets are of size n, it follows that
{max(a1 , b1 ), max(a2 , b2 ), . . . , max(an , bn )} = {n + 1, n + 2, . . . , 2n}.
This leaves {min(a1 , b1 ), min(a2 , b2 ), . . . , min(ab , bn )} on the one side and {1, 2, . . . , n} on the
other side, which implies
{min(a1 , b1 ), min(a2 , b2 ), . . . , min(an , bn )} = {1, 2, . . . , n}.
Finally,
n
X n
X
|ai − bi | = max(ai , bi ) − min(ai , bi )
i=1 i=1
n
X n
X
= max(ai , bi ) − min(ai , bi )
i=1 i=1
= (n + 1 + n + 2 + . . . + 2n) − (1 + 2 + . . . + n)
= n2 .
(Mathlinks)
3. Let u = a2 b2 + c2 d2 , v = a2 c2 + b2 d2 , w = a2 d2 + b2 c2 . Combining opposite terms and applying
the AM-GM inequality1 , we have
ab cd a3 b + ab3 + c3 d + cd3
+ =
c2 + d2 a2 + b2 a2 c2 + b2 c2 + a2 d2 + b2 d2
2a2 b2 + 2c2 d2
≥
a2 c2 + b2 c2 + a2 d2 + b2 d2
2u
= .
v+w
1 x1 +x2 +...+xn √
For non-negative real numbers x1 , x2 , . . . , xn , the AM-GM inequality states that n
≥ n x1 x2 . . . xn .
3 of 9
Winter Camp 2014 Pre-Camp Problem Set Solutions
6 Combinatorics Solutions
1. Suppose the top-left quadrant has no queens. There are 100 queens that must fit in 100 rows,
so each row must contain a queen. Since the top-left quadrant is empty, this means there
must be at least 50 queens in the top-right quadrant. Similarly, by looking at columns, we
see there must be at least 50 queens in the bottom-left quadrant.
However, any square in the bottom-left or top-right quadrant lies on one of 99 different
diagonals going from bottom-left to top-right. Therefore, there must be two queens on the
same diagonal, which is impossible. (Tournament of the Towns 2008)
2. We prove by induction on n that if n disks cover an area of size k, we can choose a subset of
disjoint disks that cover an area of size at least k9 .
If n = 1, the claim is trivial. Now let’s prove the result for n = m, assuming the result for
n < m. Let d be the disk with maximal radius r, let S denote the set of disks intersecting d,
4 of 9
Winter Camp 2014 Pre-Camp Problem Set Solutions
and let T denote the remaining set of disks. Consider a disk s ∈ S. By assumption, it has
radius at most r, and so the centers of c and s differ by distance at most 2r. It follows that S
is contained entirely inside the disk centered at c with radius 3r. This means that the total
area covered by all disks in T is at least k − 9πr2 .
By the inductive hypothesis, there exists a subset of disjoint disks in T that cover an area of
size at least k9 − πr2 . By assumptions, these disks are all also disjoint from c, so we can add
in c to get a total area of at least k9 , as required. (Putnam 1998)
3. Every move, we change the colour of two disks. Therefore, the parity of the number of black
disks never changes, so if there are an even number of black disks to start with, the task is
impossible.
Otherwise, we claim the task is possible. We first describe how to transform the circle to have
11 black disks. Since the total number of black disks is odd, there must exist some contiguous
sequence of black disks of odd length: D1 , D2 , . . . , D2n+1 (n ≤ 4). Apply the move to black
disks D1 , D3 , . . . , D2n+1 . When everything is done, D1 , D2 , . . . , D2n+1 will all still be black,
and so will the disks on either side. Thus, we can keep increasing the number of black disks
until all but one disk is black.
This leaves one contiguous sequence of black disks and it is of odd length: D1 , D2 , . . . , D2n+1 .
Apply the move to black disks D2 , D4 , . . . , D2n . When everything is done, D2 , D3 , . . . , D2n
will all still be black, but D1 and D2n+1 will be white. Thus, we can keep decreasing the
number of black disks until only one disk is black, as required. (Japan 1998)
4. Pick a permutation π of {1, 2, . . . , n} uniformly at random. Let us say π matches Si if the first
|Si | elements of π are the elements of Si in some order. Let Pi denote the probability that π
matches Si , and let P denote the probability that π matches at least one of {S1 , S2 , . . . , Sk }.
Note that π cannot simultaneously match both Si and Sj for distinct i, j, because this would
imply one of the two sets contained the others. Therefore, P = P1 +P2 +. . .+Pk . Additionally,
Pi = n1 since the first |Si | elements of π are equally likely to be any of the |Si |-element
(|Si |)
subsets of {1, 2, . . . , 2n}.
Therefore, ki=1 n1 = ki=1 Pi = P ≤ 1.
P P
(|Si |)
7 Geometry Solutions
1. Let O be the circumcircle of 4AP D. Then ∠P OA = 2∠P DA = ∠P CA, so O lies on the
circumcircle of 4ACP . Similarly, it lies on the circumcircle of 4BDP so O = Q and the
problem is solved.
Comment: A slicker solution is to extend P Q to hit the circumcircle of 4AP D at R, and
angle chase from there. However, the solution presented here is better motivated.
2. Since AB and CD are parallel, we have ∠DCA = ∠BAC, which implies DA = BC. Addi-
tionally, we have ∠EDA = ∠DCA = ∠P DC = ∠P BC = ∠QBC and ∠DAE = ∠BCD =
∠BCQ. Therefore 4EDA is congruent to 4QBC by angle-side-angle. It follows that
EA = QC, and hence EACQ is a parallelogram. This gives EQ = AC, as required. (Nordic
2002)
5 of 9
Winter Camp 2014 Pre-Camp Problem Set Solutions
3. Let the 5 points on the circle be A, B, C, D, E. Set up vectors with the center of the circle
being at 0. We claim the lines meet at X = A+B+C+D+E
3 .
Let CABC denote the centroid of 4ABC. We know CABC lies at vector A+B+C 3 , so the line
through CABC and X travels along the vector D+E 3 . This is parallel to the line between
the center of the circle and the midpoint of segment DE, and that line is orthogonal to DE
because the center of a circle lies on the perpendicular bisector of any chord. It follows that
CABC X is also orthogonal to segment DE.
The same argument can be applied for any triple of points, so the lines concur at X, as
required.
Comment: The main challenge with a problem like this is to figure out where X has to
be in the first place. One approach is to start with just two lines. Let M be the midpoint
of AB, and let CABC and CABE denote the centroids of triangles ABC and ABE. Let `C
denote the line through CABC perpendicular to DE and let `E denote the line through CABE
perpendicular to CD. If we perform a homothety about M with factor 3, then `C becomes
the altitude from C to DE, and `E becomes the altitude from E to CD. Therefore, the point
X we are looking for is mapped to the orthocenter of 4CDE by a homothety with factor
3 about M . Now it is natural to introduce vectors since, if the vectors are centered at the
circumcenter of 4CDE, then the orthocenter of 4CDE is at precisely C + D + E. (Do you
see why this is true? It is a consequence of the Euler line.)
4. First suppose ` is parallel to AB. Then ∠A2 P A1 = ∠BAA1 = ∠BCA1 = ∠A2 CA1 , so
6 of 9
Winter Camp 2014 Pre-Camp Problem Set Solutions
A2 P CA1 is cyclic. Similarly, B2 P CB1 is cyclic, so the two circles meet at P , which is on `
as required.
Otherwise, let X denote the intersection of AB and `. We will assume the configuration
shown in the diagram; the remaining configurations are similar2 .
Let Q be the point on ` so that (using directed lengths) P Q · P X = P B · P B1 = P A · P A1 .
Then XQBB1 and XQAA1 are cyclic by Power of a Point. Therefore, ∠B2 QB1 = ∠XQB1 =
∠XBB1 = 180◦ − ∠ABB1 = 180◦ − ∠ACB1 = 180◦ − ∠B2 CB1 . It follows that Q lies on
the circumcircle of 4B1 B2 C. A similar argument shows it also lies on the circumcircle of
4A1 A2 C, which solves the problem since Q is on `. (Mathlinks)
a1 − 1 2 a2 − 1 2 ak − 1 2
2 3 n
· · ... · · · · ... ·
1 2 n−1 a1 a2 ak
2
(a1 − 1)(a2 − 1) . . . (ak − 1)
= n· .
a1 a2 . . . ak
2
a1 a2 ...ak
If this equals 1, then n = (a1 −1)(a2 −1)...(ak −1) , and in particular n is a perfect square.
2
Saying other configurations are “similar” on a contest is not usually a good idea. You should either explicitly
go through every possible configuration or use something like directed angles. You can find more on directed angles
here: https://sites.google.com/site/imocanada/2011-winter-camp.
7 of 9
Winter Camp 2014 Pre-Camp Problem Set Solutions
m2 m2 − 1
m+1 m−1 m−2 1
2
· 2
· ... · · · · ... ·
m −1 m −2 m m m−1 2
1
= m·
m
= 1.
Therefore, the product can be made equal to 1 if and only if n is a perfect square. (Bay Area
2013)
3. For each pair of integers n, i, let sn,i denote the number that is 1 if i ∈ Sn and 0 otherwise.
For n ≥ 0, we are given that sn+1,i ≡ sn,i−1 + sn,i mod 2.
We are now going to work with polynomials mod 2. Specifically, given two polynomials Q and
R with integer coefficients, we will write Q ≡ R mod 2 if every coefficient of Q is congruent
to the corresponding coefficient of R mod 2. You can add, subtract, and multiply polynomials
mod 2, and they work just like integers3 .
Now, for each n, define a polynomial Pn (x) = i sn,i xi . The given conditions can be rewritten
P
to state that Pn+1 (x) ≡ Pn (x) · (x + 1) mod 2, and hence Pn (x) ≡ P0 (x) · (x + 1)n mod 2.
k k
Lemma: (x + 1)2 ≡ x2 + 1 mod 2 for all non-negative integers k.
k
Proof: Using the binomial theorem, it suffices to show that 2i is even for
all i satisfying
0 < i < 2k . Recall that the largest power of 2 dividing n! is exactly n2 + n4 +. . .. Therefore,
k
the largest power of 2 dividing 2i is precisely
k k
2k − i
X 2 i
− j −
2j 2 2j
j=0
k−1 k
2k − i
X 2 i
= 1+ − j −
2j 2 2j
j=0
k−1 k
i + (2k − i)
X 2
≥ 1+ −
2j 2j
j=0
= 1.
2k
This shows i is indeed even, which completes the proof of the lemma.
Finally, let R denote the difference between the largest and smallest elements of S0 , and choose
n to be a power of 2 larger than R. Then Pn (x) ≡ xn P (x) + P (x) mod 2 by our lemma. It
follows that Sn (x) consists of the integers that are in exactly one of S0 and {n + a : a ∈ S0 }.
Since n > R, these two sets are disjoint, and hence Sn (x) = S0 ∪ {n + a : a ∈ S0 }. (Putnam
2000)
3
What does this really mean? The key property you want is this: if Q ≡ Q0 mod 2 and R ≡ R0 mod 2, then
Q + R ≡ Q0 + R0 mod 2, Q − R ≡ Q0 − R0 mod 2, and Q · R ≡ Q0 · R0 mod 2. Abstract algebra is very interested
in these kinds of questions.
8 of 9
Winter Camp 2014 Pre-Camp Problem Set Solutions
9 of 9
Winter Camp 2014 Buffet Contest Solutions
1 Algebra
1. Find all functions f from the set of non-zero real numbers to itself satisfying
2. Consider the sequence (an )n≥1 of real numbers where a1 ∈ (1, 2) and an+1 = an + ann for all
n ≥ 1. Prove that there is at most one pair of terms in this sequence with integer sum.
Prove that there exist indices i, j such that 1 < |i − j| < n − 1 and |ai bj − aj b1 | = 1.
2 Combinatorics
1. One or more powers of 2 are written on each of n sheets of paper. The sum of the numbers
on each sheet are the same. If each number appears at most 5 times among the n sheets,
what is the largest possible value for n?
2. The numbers 1 and -1 are written in the cells of a 2000 × 2000 grid. If the sum of the entries
in the grid is positive, show that one can select 1000 rows and 1000 columns such that the
sum of the numbers written in cells of their intersections is at least 1000.
3. In a country, every pair of cities is either joined by a two-way road or not joined by any road.
The country has the property that for any pair of cities there is a sequence of roads one can
take to get from the first city to the second city. However, if any sequence of an odd number
of distinct roads beginning and ending at the same city is closed down, the country no longer
has this property. Prove that the cities in the country can be partitioned into 4 districts such
that there is no road between two cities in the same district.
1 of 8
Winter Camp 2014 Buffet Contest Solutions
3 Geometry
1. ABC is an isosceles triangle with AC = BC. Let O be its circumcenter, let I be its incenter,
and let D be the point on BC such that lines OD and BI are perpendicular. Prove that lines
ID and AC are parallel.
2. Consider the parallelogram ABCD with obtuse angle A. Let H be the foot of the perpen-
dicular from A to the side BC. The median from C in triangle ABC meets the circumcircle
of triangle ABC at the point K. Prove that points K, H, C, D lie on the same circle.
3. Let M be the midpoint of the internal bisector AD of 4ABC. Circle ω1 with diameter AC
intersects segment BM at E and circle ω2 with diameter AB intersects segment CM at F .
Show that B, E, F, C lie on a circle.
4 Number Theory
1. A perfect number greater than 28 is divisible by 7. Prove that it is also divisible by 49.
(Note: A number n is “perfect” if the sum of the divisors of n is equal to 2n. For example,
28 is perfect since 1 + 2 + 4 + 7 + 14 + 28 = 2 · 28.)
2. Find (with proof) all monic polynomials P (x) with integer coefficients that satisfy the fol-
lowing two conditions.
3. Let an be the leftmost digit of 2n and let bn be the leftmost digit of 5n . Prove that the reverse
of any k consecutive terms in the sequence an appears as k consecutive terms in the sequence
bn .
2 of 8
Winter Camp 2014 Buffet Contest Solutions
5 Algebra Solutions
1. Suppose f (z) 6= z1 for some z 6= 0. Let x = f (z)
1
and y = z − x. Clearly x 6= 0; we also know
1
y 6= 0 since we assumed f (z) 6= z , and we know x + y = z 6= 0. Therefore,
which is a contradiction. It follows that f (x) = x1 is the only possible solution. Checking, we
see it does indeed satisfy the conditions of the problem. (Nordic 2003)
n (an − n)(an − 1)
an+1 = an + = + n + 1 > n + 1.
an an
It is given that an > n for n = 1, so it follows from induction that an > n for all n. Therefore,
n
an+1 = an + < an + 1.
an
Define bn = an − n. We have just shown that the sequence (bn ) is always positive and
decreasing. Furthermore, ai + aj is an integer if and only if bi + bj is an integer, so it suffices
to prove there exists at most pair (i, j) such that bi + bj is an integer.
a2 +1
Now, a2 = 1a1 = (a1 −2)(a a1
1 −0.5)
+ 2.5 < 2.5, so b2 < 0.5. Therefore, if i, j > 1, we have
bi + bj > 0 and bi + bj ≤ 2b2 < 1, so bi + bj cannot possibly be an integer.
It remains to consider b1 + bi . In this case, we know b1 + bi > 0 and b1 + bi ≤ 2b1 < 2, so this
is an integer if and only if bi = 1 − b1 . However, (bn ) is decreasing, so there is at most one i
with this property. (Russia 2009)
3 of 8
Winter Camp 2014 Buffet Contest Solutions
real numbers u, v. Using the base times height formula for the area of a triangle again, we
have 12 = Ki,i+1 = |u| · Ki−1,i = |u|
2 , which implies u = ±1.
Now, gcd(ai , bi ) = 1 or else |ai bi+1 −ai+1 bi | could not possibly be 1. Since vPi = −Pi+1 ±Pi−1
has integer coordinates, it then follows that v must also be an integer. If v = 0, then Pi−1
and Pi+1 are collinear, which we already covered. If |v| ≥ 2, then |Pi+1 | = |uPi−1 + vPi | ≥
|v| · |Pi | − |u| · |Pi−1 | ≥ |Pi |. Equality holds only if Pi−1 , Pi are collinear, which is impossible.
Thus, |Pi+1 | > |Pi | in this case, and that contradicts the choice of i. The only remaining
possibility is v = ±1, but then Ki−1,i+1 = |v| · Ki−1,i = 21 and the problem is solved. (Korea
2001)
6 Combinatorics Solutions
1. Suppose that we have n sheets with the same sum S. There exists k such that 2k ≤ S < 2k+1 .
The sum of all numbers must then be at least n · 2k . On the other hand, we cannot have any
power of 2 larger than 2k or the sum on that sheet would be too large. Since each number
appears at most 5 times, the sum of all numbers must be at most 5 · (1 + 2 + . . . + 2k ) < 10 · 2k .
Therefore, n · 2k < 10 · 2k and hence n < 10.
Conversely, if n = 9, we can use the following arrangement:
(a) 2 sheets containing 1, 1, 2, 4.
(b) 1 sheet containing 2, 2, 4.
(c) 1 sheet containing 4, 4.
(d) 5 sheets containing 8.
Therefore, the maximum value of n is 9. (St. Petersburg 1998)
2. Lemma: The numbers 1 and -1 are written in the cells of a grid with n rows and m columns.
Let S denote the sum of the entries in the grid. If m is odd, it is possible to delete one row
and end up with sum at least min(S + 1, n − 1). If m is even, it is possible to delete one row
and end up with sum at least min(S, n − 1).
Proof: If every row has positive sum, then we can drop an arbitrary row, and there will
remain n − 1 rows, each with sum at least 1, and the claim is proven.
Otherwise, there exists some row with non-positive sum. If m is odd, then this row actually
has sum at most -1, and therefore if we drop it, the remaining sum will be at least S + 1. If
m is even, then we still know this row has sum at most 0, and therefore if we drop it, the
remaining sum will be at least S. This covers all cases, and the lemma is proven.
Note that the same argument can be applied to removing columns as well. Now we use the
lemma repeatedly to solve the problem:
4 of 8
Winter Camp 2014 Buffet Contest Solutions
(Russia 1995)
Comment: Can you solve the problem if 2000 × 2000 is replaced with 1500 × 1500? The
same approach works.
3. Let G be the graph with vertices corresponding to cities and with edges corresponding to
roads between cities. In graph theoretic terms, we are given that G is connected (there is
sequence of edges joining any two vertices) and that removing any odd cycle will disconnect
G. We need to show it is 4-colourable (the vertices can be assigned colours 0, 1, 2, 3 such
that no two vertices with the same colour have an edge between them).
Suppose we remove edges from G one by one without disconnecting it, stopping only when
it is impossible to remove any more edges. Let G1 be the subgraph of G containing only the
edges we removed, and let G2 be the subgraph of G containing only the edges we did not
remove. Since we can remove all of the edges in G1 without disconnecting the graph, the
problem conditions guarantee that G1 has no odd cycles. Furthermore, G2 has no cycles at
all. If it did have a cycle, we could remove one of the edges in the cycle without disconnecting
the graph, which contradicts the definition of G2 .
We will now use the following very useful theorem from graph theory: if a graph has no odd
cycles, then it is 2-colourable.1
It follows that G1 and G2 are both 2-colourable. In particular, there are functions f1 , f2
mapping cities to {0, 1} such that f1 (u) = f1 (v) only if there is no edge between u and v in G1 ,
and f2 (u) = f2 (v) only if there is no edge between u and v in G2 . Let f (v) = 2 · f1 (v) + f2 (v).
This assigns every vertex a colour in {0, 1, 2, 3}, and if f (u) = f (v), then f1 (u) = f1 (v) and
f2 (u) = f2 (v), which means u and v are not connected by an edge in either G1 or G2 . It
follows that f is a valid 4-colouring of G, and the problem is solved. (Russia 2010)
7 Geometry Solutions
1. Since ABC is isosceles, I and O both lie on the perpendicular bisector of AB. Therefore,
1
You can prove this as follows. First assume the graph is connected. If it is not, we can apply the proof
independently to each connected component. Pick an arbitrary vertex u and colour it 0. Now, consider another
vertex v. If there is both an even-length path from u to v and an odd length path from u to v, we could combine
them to get an odd-length cycle. Therefore, we have either that all paths from u to v are even length, or all paths
are odd length. In the former case, we colour v with 0; in the latter case, we colour it with 1. Do you see why this is
a valid 2-colouring?
5 of 8
Winter Camp 2014 Buffet Contest Solutions
∠DOI = 90◦ − ∠BIO = ∠ABI = ∠DBI, from which it follows that DBOI is a cyclic
quadrilateral.
This means ∠BDI = 180◦ − ∠BOI = 180◦ − ∠BOC = 2∠BCI = ∠BCA, and hence ID and
AC are parallel. (Russia 1996)
2. Let M be the midpoint of AB and let E be the point such that AHBE is a rectangle. Note
that, since AHBE is a rectangle, H, M, E are collinear.
Now, ∠DEH = ∠AEH = ∠ABH = ∠ABC = 180◦ − ∠DCB = 180◦ − ∠DCH, so DCHE is
cyclic. Consider the circumcircles of DCHE, AHBE, and ABC. The radical axis of DCHE
and AHBE is HE, and the radical axis of ABC and AHBE is AB, so the radical center of
the three circles is M . In particular, M is on the radical axis of DCHE and ABC.
Since C is also on the radical axis, we know the radical axis is equal to line CM , and so K is
also on the radical axis. Since K is on circle ABC, it follows that K is also on circle DCHE,
and the problem is solved. (Russia 2012)
3. Let C 0 be the point on line AB such that C 0 C is parallel to AD, and let P be the midpoint
of C 0 C. The homothety about B that takes AD to C 0 C also takes M to P . This implies that
P lies on line BEM .
6 of 8
Winter Camp 2014 Buffet Contest Solutions
8 2k+1
The right-hand side must be a multiple of 8, so k ≥ 2. Then 7 ≥ 2k+1 −1
, which implies
e +1
pi i −1
(2k+1 − 1) · 8 ≥ 2 · 2k · 7. Furthermore pi −1 > pei i for all i. This means that if k ≥ 2,
!
Y pei i +1 − 1 Y e
(2k+1 − 1) · 8 · ≥ 2 · 2k · 7 · pi i
pi − 1
with equality only if k = 2 and there are no primes pi . However, this occurs only when
n = 28, so the problem is solved. (Russia 2000)
pd + ad−1 pd−1 + . . . + a1 p − p = 0.
e +1
pi i −1
2
pei i , then the sum of the divisors of n is
Q Q
The general formula is that if n = pi −1
. To prove this, note
e +1
pi i −1
that pi −1
= 1 + pi + p2i + . . . + pei i , and see what monomials appear when you multiply everything out.
7 of 8
Winter Camp 2014 Buffet Contest Solutions
However,
3. Lemma: For every real number > 0 and every integer A, there exist integers n and m with
n > A and 10m ≤ 5n < 10m · (1 + ).
1
Proof: Let B be a positive integer such that (1 + )B > 10. Then 10 B < 1 + .
Consider the numbers {0 · log10 5}, {1 · log10 5}, . . . , {B · log10 5}, where {x} denotes the frac-
tional part of x. There are B + 1 such numbers and all lie in the range [0, 1]. Furthermore,
they are all distinct: if {u · log10 5} = {v · log10 5}, then 5u−v is a power of 10, but that is
impossible for u 6= v. Therefore, there must exist non-negative integers u, v ≤ B such that
0 < {u · log10 5} − {v · log10 5} ≤ B1 .
Let x = {u · log10 5} − {v · log10 5}, and let C be a positive integer larger than A + Bx . Then
there must exist some non-negative integer k such that kx ≤ {C · log10 5} < (k + 1)x. Let
n = C + k(v − u). Note that n > A since |k(v − u)| ≤ x1 · B < C − A. Furthermore,
and similarly
8 of 8
Winter Camp 2014 Mock Olympiad Solutions
1 Problems
1. Let a1 be a natural number not divisible by 5. The sequence a1 , a2 , a3 , . . . is defined by
an+1 = an + bn , where bn is the last digit of an . Prove that the sequence contains infinitely
many powers of 2.
2. Let ABC be a triangle and let D and E be points on the sides AB and AC respectively
such that DE is parallel to BC. Let P be any point interior to triangle ADE and let F and
G be the intersections of DE with the lines BP and CP respectively. Let Q be the second
intersection point of the circumcircles of triangles P DG and P F E. Prove that the points
A, P, and Q lie on a straight line.
f (x − y) = f (x) + xy + f (y).
4. There are a number of markets in a city, joined by one-way streets. Every market has exactly
two one-way streets going out of it, and there is at most one street joining any two markets.
Prove that the city may be partitioned into 2014 districts such that:
1 of 4
Winter Camp 2014 Mock Olympiad Solutions
2 Solutions
1. Checking each possible last digit for a1 , we can see there must always exist some k such that
ak has last digit equal to 2.
Focusing on that k, we have ak = 10m + 2 for some non-negative integer m, and hence
• ak+1 = 10m + 4
• ak+2 = 10m + 8
• ak+3 = 10(m + 1) + 6
• ak+4 = 10(m + 2) + 2.
At this point, the process repeats.
If m is even, then (an ) includes every sufficiently large number that is 4 mod 20. In particular,
it includes 4 · 16t = 24t+2 for all sufficiently large t.
If m is odd, then (an ) includes every sufficiently large number that is 12 mod 20. In particular,
it includes 32 · 16t = 24t+5 for all sufficiently large t.
Therefore, (an ) includes infinitely many powers of 2 in either case. (Russia 1994)
2. Let P 0 be the image of P under the dilation about A that takes BC to DE. Then A lies on
line P P 0 .
Now,
∠DQE = ∠DQP + ∠EQP
= ∠DGP + ∠EF P since DQGP and EQF P are cyclic
= ∠DEP 0 + ∠EDP 0 since DP 0 ||F P and EP 0 ||GP by the definition of P 0
= 180◦ − ∠DP 0 E.
Therefore, DP 0 EQ is cyclic. It follows that ∠DQP 0 = ∠DEP 0 = ∠DGP = ∠DQP , and so
Q lies on line P P 0 .
We have now shown A and Q both lie on line P P 0 , which implies A, P, Q must be collinear.
(India 1995)
2 of 4
Winter Camp 2014 Mock Olympiad Solutions
3. It is easy to check that f (x) = 0 for all x is a solution. We focus on the other case, in which
there exists some a ∈ f (R) with a 6= 0.
Setting x = y, we have f (0) = 2f (y) + y 2 for all y ∈ f (R). (*) Let z be an arbitrary real
number, and set x = z−fa(a) , y = a. Then we have
z − f (a) z − f (a)
f −a =f + z.
a a
Therefore, there exist real numbers u, v such that f (u)−f (v) = z. Setting x = f (u), y = f (v),
we have
4. Let G be the graph with vertices corresponding to markets and directed edges corresponding
to one-way streets. We are given that every vertex has out-degree equal to 2.
Lemma: Let us say two vertices are “separated” if there is no path from one to the other
of length 1 or 2. Then we can assign every vertex a “colour” in {1, 2, . . . , 13} such that if
vertices u, v have the same colour (possibly with u = v), then they are separated.
Proof: We prove this by induction on the number of vertices, relaxing the conditions to say
each vertex has out-degree at most two.
If there is only one vertex, the claim is trivial. Now suppose it has been shown for n − 1
vertices, and consider a graph with n vertices. From every vertex, there are 2 paths going
out of length 1 and 4 paths going out of length 2. Therefore, the total number of unordered
vertex pairs (u, v) that are not separated is at most 6n. By the pigeonhole principle, there
exists a vertex v that belongs to at most 12 of these pairs.
Using the inductive hypothesis, all vertices except v can be coloured in the required manner.
Then there are up to 12 vertices that v cannot be the same colour as, so we can choose a
valid colour for v as well, completing the inductive proof.
Using the colouring described above, associate to each vertex a triple (a, b, c) where a is the
colour of the vertex, and b ≤ c are the colours of the vertices that the vertex has an edge
leading out to. We create one district for each possible value of this triple, and use that to
assign vertices into districts. There are 13 possible values for a. Once a is fixed, (b, c) can be
any pair chosen from the remaining colours, so there are 12 + 12 2 = 78 possibilities for them,
giving a total of 13 · 78 < 2014 districts.
3 of 4
Winter Camp 2014 Mock Olympiad Solutions
If two vertices are in the same district, then they have the same colour and the colouring
guarantees there is no edge between them.
Next suppose there is an edge from vertex u in district (a1 , b1 , c1 ) to vertex v in district
(a2 , b2 , c2 ). If b2 or c2 equals a1 , then we can follow a second edge from v to another vertex
w with colour a1 . However, u and w then have the same colour but are not separated, which
is a contradiction. Thus, b2 and c2 are both distinct from a1 , and so there is no edge from
district (a2 , b2 , c2 ) to district (a1 , b1 , c1 ).
Therefore, our district division satisfies the required conditions, and the problem is solved.
(Russia 2002)
4 of 4